You are on page 1of 217

Corporate

Office
DISHA PUBLICATION
45, 2nd Floor, Maharishi Dayanand Marg,
Corner Market, Malviya Nagar, New Delhi - 110017
Tel : 49842349 / 49842350

© Copyright
No part of this publication may be reproduced in
any form without prior permission of the publisher.

Disha
The author and the publisher do not take any legal
responsibility for any errors or misrepresentations that
might have crept in. We have tried and made our best
efforts to provide accurate up-to-date information in
this book.
All Right Reserved

Typeset by Disha DTP Team

www.dishapublication.com www.mylearninggraph.com
Books & Etests
ebooks for
School & for
Competitive Competitive
Exams Exams

Write to us at feedback_disha@aiets.co.in
Contents
S.no. Chapter Name Page no.
Questions Solutions
1. SETS 1-2 75-77

2. RELATIONS & FUNCTIONS-1 3-4 77-81

3. TRIGONOMETRIC FUNCTIONS 5-6 81-84

4. PRINCIPLE OF MATHEMATICAL INDUCTION 7-8 84-88

5. COMPLEX NUMBERS AND QUADRATIC EQUATIONS 9-11 88-92

6. LINEAR INEQUALITIES 12-13 92-96

7. PERMUTATIONS AND COMBINATIONS 14-15 96-98

8. BINOMIAL THEOREM 16-17 99-101

9. SEQUENCES AND SERIES 18-19 102-106

10. STRAIGHT LINES 20-22 107-112

11. CONIC SECTIONS 23-25 113-118

12. LIMITS & DERIVATIVES 26-27 118-123

13. MATHEMATICAL REASONING 28-29 123-124

14. STATISTICS 30-31 124-127

15. PROBABILITY-1 32-33 128-131

16. RELATIONS & FUNCTIONS-2 34-35 132-135

17. INVERSE TRIGONOMETRIC FUNCTIONS 36-38 136-140


18. MATRICES 39-41 141-145

19. DETERMINANTS 42-44 146-152

20. CONTINUITY AND DIFFERENTIABILITY 45-47 153-158

21. APPLICATION OF DERIVATIVES 48-50 159-165

22. INDEFINITE INTEGRATION 51-53 165-170

23. DEFINITE INTEGRATION 54-56 171-175

24. APPLICATION OF INTEGRALS 57-59 176-182

25. DIFFERENTIAL EQUATION AND ITS APPLICATIONS 60-62 182-187

26. VECTOR ALGEBRA 63-65 188-193

27. THREE DIMENSIONAL GEOMETRY 66-68 194-200

28. PROBABILITY-2 69-71 201-207

29. PROPERTIES OF TRIANGLE 72-74 208-212


1
SETS

MCQs with One Correct Answer (a) { x : 1< x £ 2} (b) {x : 1£ x < 2}


1. If A and B are non-empty sets such that A É B, (c) {x : – 2 £ x £ 2} (d) None of these
then 8. If aN = {ax : x Î N} and bN Ç cN = dN, where b,
(a) B' – A' = A – B (b) B' – A' = B – A c Î N are relatively prime, then
(c) A' – B' = A – B (d) A' Ç B' = B – A (a) d = bc (b) c = bd
2. Let A = {q : sin(q) = tan(q)} and B = {q : cos(q) = 1} (c) b = cd (d) None of these
be two sets. Then : 9. The set (A È B È C) È (A Ç B' Ç C')' Ç C' is
(a) A = B equal to
(a) B Ç C' (b) A Ç C
(b) A Ë B
(c) B' Ç C' (d) None of these
(c) B Ë A 10. A dinner party is to be fixed for a group of 100
(d) A Ì B and B - A ¹ f persons. In this party, 50 persons do not prefer
fish, 60 prefer chicken and 10 do not prefer either
3. A set S contains 3 elements, the number of
chicken or fish. The number of persons who
subsets of which of following sets is 256. prefer both fish and chicken is
(a) S (b) P(S)
(a) 20 (b) 22
(c) P(P(S)) (d) None of these
(c) 25 (d) None of these
4. In a college of 300 students every student reads
5 newspapers and every newspaper is read by 11. Let U be the universal set and A È B È C = U.
60 students. The number of newpapers is Then {(A – B) È (B – C) È (C – A)}¢ is equal to:
(a) at least 30 (b) at most 20 (a) A È B È C (b) A È (B Ç C)
(c) exactly 25 (d) None of these (c) A Ç B Ç C (d) A Ç (B È C)
5. Let A and B be two sets then 12. If n(A) = 1000, n(B) = 500 and if n(A Ç B) ³ 1 and
n(A È B) = p, then
(A È B)'È (A 'Ç B) is equal to
(a) 500 £ p £ 1000 (b) 1001 £ p £ 1498
(a) A¢ (b) A (c) 1000 £ p £ 1498 (d) 1000 £ p £ 1499
(c) B¢ (d) None of these
6. The set (A \ B) È (B \ A) is equal to 13. In a battle 70% of the combatants lost one eye,
80% an ear, 75% an arm, 85% a leg, x % lost all
(a) [ A \ ( A Ç B)] Ç [ B \ ( A Ç B)] the four limbs. The minimum value of x is
(b) ( A È B) \ ( A Ç B) (a) 10 (b) 12
(c) A \ ( A Ç B ) (c) 15 (d) None of these
(d) A Ç B \ A È B 14. The number of students who take both the
subjects mathematics and chemistry is 30. This
7. Let A = { x : x ÎR, | x | < 1} represents 10% of the enrolment in mathematics
B = { x : x ÎR, | x – 1| ³1} and and 12% of the enrolment in chemistry. How
AÈB = R – D, then the set D is :
2 MATHEMATICS
many students take at least one of these two monthly magazine. Summarized information is
subjects? presented below:
(a) 520 (b) 490 Only September: 18
(c) 560 (d) 480 September but not August: 23
15. At a certain conference of 100 people, there are September and July: 8
29 Indian women and 23 Indian men. Of these September: 28
Indian people 4 are doctors and 24 are either July: 48
men or doctors. There are no foreign doctors. July and August:10
How many foreigners and women doctors are None of the three months: 24
attending the conference? What is the number of surveyed people who
(a) 48, 1 (b) 34, 3 have read exactly for two consecutive months?
(c) 46, 4 (d) 42, 2 (a) 7 (b) 9
16. Each student in a class of 40, studies at least (c) 12 (d) 14
one of the subjects English, Mathematics and
Economics. 16 study English, 22 Economics and Numeric Value Answer
26 Mathematics, 5 study English and Economics, 21. An investigator interviewed 100 students to
14 Mathematics and Economics and 2 study all determine their preferences for the three drinks :
the three subjects. The number of students who milk (M), coffee (C) and tea (T). He reported the
study English an d Math ematics but not following : 10 students had all the three drinks
Economics is M, C and T; 20 had M and C; 30 had C and T; 25
(a) 7 (b) 5 had M and T; 12 had M only; 5 had C only; and
(c) 10 (d) 4 8 had T only. If number of students who did not
17. In a market research project, 20% opted for n
'Nirma' detergent whereas 60% opted for 'Surf take any of the three drinks is n, then is
blue' detergent. The remaining individuals were 5
not certain. If the difference between those who 22. In a class of 80 students numbered 1 to 80, all
opted for 'Surf blue' and those who were odd numbered students opt of Cricket, students
uncertain was 720, how many respondents were whose numbers are divisible by 5 opt for
covered in the survey Football and those whose numbers are divisible
(a) 1100 (b) 1150 by 7 opt for Hockey. If the number of students
(c) 1800 (d) None of these who do not opt any of the three games is n, then
18. In a school 80 students like chocolate, 40 like n
coffee if the number of students doesn’t like any is equal to
4
of them is equal to the number of students who 23. A survey shows that 61%, 46% and 29% of the
like both of them then what is the total number people watched “3 idiots”, “Rajneeti” and
of students in the school? “Avatar” respectively. 25% people watched
(a) 115 (b) 90 exactly two of the three movies and 3% watched
(c) 120 (d) None of these none. What percentage of people watched all
19. In a school there are 100 students 60 of them the three movies?
don’t like Chocolate and 50 don’t like Biscuit 24. If n(A) = 4 and n(B) = 7, then the difference
and 10 of them like none then how many of them between maximum and minimum value of
like both? n(A È B) is
(a) 20 (b) 30 25. Two finite sets have m and n elements. The
(c) 40 (d) None of these number of subsets of the first set is 112 more
20. A survey was conducted of 100 people whether than that of the second set. The value of m – n is
they have read recent issues of ‘Golmal', a

ANSW ER KEY
1 (a) 4 (c) 7 (b) 10 (a) 13 (a) 16 (b) 19 (d) 22 (7) 25 (3)
2 (b) 5 (a) 8 (a) 11 (c) 14 (a) 17 (c) 20 (b) 23 (7)
3 (c) 6 (b) 9 (a) 12 (d) 15 (a) 18 (c) 21 (4) 24 (4)
2
RELATIONS &
FUNCTIONS-1

MCQs with One Correct Answer 2a + b a


(a) (b)
( ) a - b2
2
1. If f (x) . f (y) = f (x) + f (y) + f (xy) – 2 x, y Î R 2 a2 - b 2
and if f (x) is not a constant function, then the
value of f (a) is a + 2b
(a) 1 (b) 2 (c) 0 (d) – 1 (c) (d) None of these
a 2 - b2
2. The domain of the function
7. The domain of f(x) = cos(sin x ) + log x {x} ;
f ( x ) = x14 - x11 + x 6 - x3 + x 2 + 1 is {.} denote the fractional part, is
(a) (-¥, ¥) (b) [0, ¥) (a) [1, p) (b) ( 0, 2p ) - [1, p)
(c) (- ¥, 0] (d) R - [0,1]
æ pö
3. If f(x) = cos (log x) then (c) çè 0, ÷ø - {1} (d) (0, 1)
2
1 ìï æ x ö üï
f (x)f (y) – í f ç ÷ + f ( xy ) ý is equal to : 1 æp x ö
2 ïî è y ø ïþ 8. Let f ( x) = - tan ç ÷ , -1< x <1 &
2 è 2 ø
(a) 0 (b) 1
(c) –1 (d) None of these g ( x) = 3 + 4 x - 4 x 2 , then dom (f + g) is given
4. The domain of the function by
é1 ù é1 ö
f ( x ) = 10 - x 4 - 21x 2 is (a) êë 2 ,1úû (b) ê 2 , -1 ÷
(a) [5, ¥) ë ø

é - 21, 21 ù é 1 ö é 1 ù
(b) ë û (c) ê - 2 ,1÷ (d) ê - , -1ú
ë ø ë 2 û
(c) éë -5, - 21 ùû È éë 21, 5 ùû È {0} 9. The domain of function
(d) (–¥, –5) f(x) = log4[log5{log3 (18x – x2 – 77)}] is:
5. The function f satisfies the functional equation (a) (7, 11) (b) (8, 10)
(c) (8, 11) (d) (7, 10)
æ x + 59 ö 10. Let f(x) = ax(a > 0) be written as f(x) = g(x) + h(x),
3f(x) + 2f ç ÷ = 10x + 30 for all real x ¹ 1.
è x -1 ø where g(x) is an even function and h(x) is an
The value of f(7) is odd function. Then the value of g(x + y) + g(x – y)
(a) 8 (b) 4 (c) –8 (d) 11 is
(a) 2g(x) g(y)
æ 1 ö
6. If af(x + 1) + bf ç ÷ = x , x ¹ –1, a ¹ b, then (b) 2g(x + y) g(x – y)
è x +1 ø (c) 2g(x)
f(2) is equal to (d) None of these
4 MATHEMATICS
11. Let f be a real valued function such that for any 18. Let f(x) = sin x – cos x and g(x) = log 5x ; then
real x, f (l + x) = f (l – x) and f (2l + x) = –f (2l – x)
for some l > 0. Then the range of g ( 2 f ( x) + 3) is
(a) f is even and non-periodic (a) [0, 1] (b) [0, 2]
(b) f is odd and periodic
(c) f is odd and non-periodic (c) é 3ù
(d) None of these
(d) f is even and periodic êë 0, 2 úû
12. Let f(x) = ([a]2 – 5[a] + 4)x3 – (6{a}2 – 5{a} + 1) x 1
– (tan x) sgn x, be an even function for all x Î R, 19. Let f(x) = 1 + and g(x, y) = log y, then the
then sum of all possible values of 'a' is (where 4 x
[ ] and { } denote greatest integer function and æ1 ö
fractional part functions respectively) domain of g ç , - g (2, f ( x)) - 1÷ is
è2 ø
17 53 31 35 (a) 0 < x < 1 (b) 0 < x £ 1
(a) (b) (c) (d) (c) x ³ 1 (d) Null set
6 6 3 3
13. The set of all integer values of n for which the 20. Let f (x) be defined as
5x ì |x| 0 £ x <1
function f(x) = cos nx . sin is periodic with ï
n f ( x) = n í| x - 1| + | x - 2 | 1 £ x < 2
period 2p is equal to ï | x -3| 2£ x<3
î
(a) {1, 5, 10} (b) {1, 5}
The range of function g(x) = sin (7(f (x)) is :
(c) {±1, ±5} (d) None of these (a) [0, 1] (b) [–1, 0]
14. If f : ¡ ® ¡ & g: ¡ ® ¡ be two given
é 1 1 ù
functions, then 2 min {f (x) – g(x), 0} equals (c) ê - , ú (d) [–1, 1]
(a) f(x) + g(x) – |g(x) – f(x)| ë 2 2û
(b) f(x) + g(x) + |g(x) – f(x)|
(c) f(x) – g(x) + |g(x) – f(x)| Numeric Value Answer
(d) f(x) – g(x) – |g(x) – f(x)|
15. The domain of f(x) is (0, 1), therefore the domain 1
21. If f ( x ) = and S = f(5) + f(4) + f(3) +....+
of y = f(ex) + f(ln | x |) is : 1 + e- x
æ1 ö f(–3)+ f(–4) + f(–5), then the value of S is
(a) ç , 1÷ (b) (–e, –1) 22. If the period of f(x) satisfying the condition:
èe ø
æ 1ö f(x + p) = 1 + {1 – 3f(x) + 3f 2(x) – f 3(x)}1/3 is lp,
(c) ç -1, - ÷ (d) (–e, –1) È (1, e)
è eø then evaluate l.
16. Suppose that f is a periodic function with period 23. If f(x) is an odd function, f(1) = 3, and f(x + 2)
1 æ 9ö = f(x) + f(2), then the value of f(3) is
and that f(2) = 5 and f ç ÷ = 2 then 24. Let f(x, y) be a function satisfying the functional
2 è 4ø equation: f(x, y) = f(2x + 2y, 2y – 2x) for all real
f(–3) – f æç 1 ö÷ has the value equal to numbers x, y. Define g(x) by g(x) = f(2x, 0). Also
è 4ø given that g(x) is a periodic function with period
(a) 2 (b) 3 (c) 5 (d) 7 k
k, then find value of .
é 2 x + 3 ; x £ 1 2
17. Let f ( x ) = ê 2 . If the range of f(x) 25. Number of elements in the range set of
êë a x + 1 ; x > 1
= R (set of real numbers) then number of integral é x ù é 15 ù
f ( x) = ê ú ê - ú " x Î (0, 90) ; (where [.]
value(s), which a may take is ë15 û ë x û
(a) 2 (b) 3 (c) 4 (d) 5 denotes greatest integer function)
ANSW ER KEY
1 (b) 4 (c) 7 (d) 10 (a) 13 (c) 16 (b) 19 (d) 22 (2) 25 (6)
2 (a) 5 (b) 8 (c) 11 (b) 14 (d) 17 (c) 20 (d) 23 (9)
3 (a) 6 (a) 9 (b) 12 (d) 15 (b) 18 (b) 21 (5.5 24 (6)
3
TRIGONOMETRIC
FUNCTIONS

MCQs with One Correct Answer 6. The value of the expression


sin 7a + 6sin 5a + 17 sin 3a + 12 sin a
1 where
1.
16
If sin a = , then the value of sin 6a + 5 sin 4a + 12 sin 2a
5
p
1 1 2 4 a= is equal to:
+ + + 5
cos2 a 1 + sin 2 a 1 + sin 4 a 1 + sin8 a
is equal to 5 -1 5 +1
(a) 2 (b) 4 (c) 6 (d) 10 (a) (b)
4 4
2. If a, b, g, d are the smallest positive angles in
ascending order of magnitude which have their 5 +1 5 -1
sines equal to the positive quantity k, then the (c) (d)
2 2
a b g d
value of 4 sin + 3 sin + 2 sin + sin is sin q sin (3q) sin(9q) sin(27q)
2 2 2 2 7. + + + =
equal to cos (3q) cos(9q) cos(27q) cos(81q)
(a) 2 1 - k (b) 2 1 + k sin(81 q) sin(80 q)
(a) (b)
(c) (d) none of these 2 cos(80 q) cos q 2 cos(81 q) cos q
2 k
2cos b - 1 sin(81 q) sin(80 q)
3. If cos a = (0 < a < b < p) , then (c) (d)
2 - cos b cos(80 q) cos q cos(81 q) cos q
tan a / 2 p+q æp qö
is equal to 8. If cosec q = , then cot ç + ÷ =
tan b / 2 p-q è4 2ø
1
(a) 1 (b) (c) (d) p q
2 3 3 (a) (b) (c) pq (d) pq
q p
4. Let 0 £ a, b, g, d £ p whrere b and g are not
complementary such that 9. If sin 2q + sin 2f = 1/2 and cos 2q + cos 2f = 3/2,
2 cos a + 6 cos b + 7 cos g + 9 cos d = 0 then cos2 (q – f) =
and 2 sin a – 6 sin b + 7 sin g – 9 sin d = 0 (a) 3/8 (b) 5/8 (c) 3/4 (d) 5/4
cos (a + d ) m 10. If u = (1 + cos q) (1 + cos 2q) – sin q × sin 2q,
If = where m and n are v = sin q (1 + cos 2q) + sin 2q (1 + cos q), then
cos (b + g ) n
relatively prime positive numbers, then the u2 + v2 =
value of (m + n) is equal to: (a) 4(1 + cos q) (1 + cos 2q)
(a) 11 (b) 10 (c) 9 (d) 7 (b) 4(1 + sin q) (1 + sin 2q)
5. If sin q + sin 2q + sin 3q = sin a and cos q (c) 4(1 – cos q) (1 – cos 2q)
+ cos 2q + cos 3q = cos a, then q is equal to (d) 4(1 – sin q) (1 – sin 2q)
(a) a/2 (b) a (c) 2a (c) a/6
6 MATHEMATICS
11. The product 19. The number of solutions of the equation
ì 2p ü ìï 2 p üï ìï 2 p üï
2 64 æ æ sin x ö ö æ æ sin x ö ö
cos í 64 ý cos í 64 ý ...cos í 64 ý çè 2 sin çè 2 ÷ø ÷ø çè cos çè 2 ÷ø ÷ø
î 2 - 1þ îï 2 - 1 ïþ ïî 2 - 1ïþ
1 1 1 æ æ x 2 xö ö
(a) (b) 1 (c)(d) çè sin çè 2 tan 2 cos 2 ÷ø - 3÷ø + 2 = 0 in [0, 2p] is :
32
16 16
88 32 6464
12. Let x, y Î R satisfy the condition such that (a) 0 (b) 1 (c) 2 (d) 4
20. If the equation x2 + 12 + 3 sin (a + bx) + 6x = 0 has
sin x sin y + 3 cos y + 4 sin y cos x = 26. atleast one real solution, where a, b Î [0, 2p], then
The value of tan2 x + cot2 y is equal to the value of a – 3b is (n Î Z)
(a) 9 × 17 (b) 205 (a) 2np (b) (2n + 1) p
1 9 p p
(c) + (d) None of these (c) (4n – 1) (d) (4n + 1)
16 17 2 2
13. The product
æ xö æ xö æ xö æ x ö Numeric Value Answer
ç cos ÷ × ç cos ÷ × ç cos ÷ ........ × ç cos ÷ 21. The number of solutions of the equation
è 2ø è 4ø è 8ø è 256 ø
is equal to: 1
+ log15 cos x)
51/2 + 51/2+ log5 (sinx) = 15 2 for
sin x sin x
(a) (b) x Î [0, 100 p] is
x x
128 sin 256 sin 22. Number of solution(s) of th e equation
256 256
sin x sin 3x sin 9x
sin x sin x + + = 0 in the interval
(c) (d) cos 3x cos 9x cos 27x
x x
128 sin 512 sin æ pö
128 512
ç 0, 4 ÷ is _______.
14. The maximum value of log20 (3 sin x – 4 cos x + 15) è ø
is equal to: 23. Number of integral value(s) of m for which the
(a) 1 (b) 2 (c) 3 (d) 4 4m - 6
15. The value of equation sin x – 3 cos x = has
4-m
cos 25° cos 70° cos 85°
+ + solutions, x Î [0, 2p], is _______.
sin 70° sin 85° sin 25° sin 85° sin 25° sin 70° 24. The number of solutions of the equation
is
æ pö
(a) 1/2 (b) 1 (c) 2 (d) 3/2 cos 2 ç x + ÷ + cos2 x – 2
16. The number of solutions of the equation è 6ø
2 2
32sec x + 1 = 10.3tan x in the interval [0, 2p] is æ pö p p
cos ç x + ÷ × cos = sin 2 in interval
(a) 8 (b) 6 (c) 4 (d) 2 è 6ø 6 6
17. Number of solutions of the equation æ -p p ö
4 (cos2 2x + cos 2x + 1) + tan x (tan x – 2 3 ) = 0 ç 2 , 2 ÷ is _______.
è ø
in [0, 2p] is 25. The number of solutions of the equation
(a) 0 (b) 1 (c) 2 (d) 3 1 + cos x + cos 2x + sin x + sin 2x + sin 3x = 0,
18. The number of solutions of the equation
p p
2cos 2 x - 3cos x + 1 which satisfy the condition < 3x - £ p is
2 sin x - 3 = 1 in [0, p] is 2 2
(a) 2 (b) 3 (c) 4 (d) 5 ___________.
ANSW ER KEY
1 (d) 4 (b) 7 (b) 10 (a) 13 (b) 16 (c) 19 (a) 22 (6) 25 (2)
2 (b) 5 (a) 8 (b) 11 (a) 14 (a) 17 (c) 20 (c) 23 (4)
3 (c) 6 (c) 9 (b) 12 (c) 15 (c) 18 (b) 21 (50) 24 (2)
PRINCIPLE OF 4
MATHEMATICAL INDUCTION

MCQs with One Correct Answer 5. If n Î N , then the result

1. If P(n) = 2 + 4 + 6 + .....+ 2n, n Î N , then 1 1 1 1


+ + + ...... +
n n +1 n + 2 2n - 1
P(k) = k (k + 1) + 2
1 1 1 1
Þ P(k + 1) = (k + 1)(k + 2) + 2 for all k Î N . So = 1- + - + ...... + holds for
2 3 4 2n - 1
we can conclude that P(n ) = n(n + 1) + 2 for (a) all n Î N
(a) all n Î N (b) for even values of n
(b) n > 1 (c) for odd values of n
(d) not true for any n
(c) n > 2
6. For all n ³ 1, find
(d) nothing can be said
1 1 1 1
2. The greatest positive integer, which divides + + + ..... +
1.2 2.3 3.4 n( n + 1)
n (n + 1)(n + 2)(n + 3) for all n Î N , is
(a) 2 (b) 6 (c) 24 (d) 120 n 1
(a) (b)
n +1 n +1
3. For any n Î N , the value of the expression
1
2 + 2 + ..... + 2 is (c) n(n + 1)
(d) None of these
n - roots
7. For all natural numbers n, find
æ p ö æ p ö æ 3 ö æ 5 ö æ 7 ö æ 2n + 1 ö
(a) 2 cos çç ÷÷ (b) 2 sin çç ÷÷ ç1 + ÷ ç1 + ÷ ç1 + ÷ ..... ç1 + 2 ÷
è 2 n +1 ø è 2 n +1 ø è 1 øè 4 øè 9 ø è n ø
(a) (n + 1)2 (b) (n – 1)2
(c) 2 cos( 2 n +1 p) (d) none of these (c) n(n + 1) (d) None of these
4. For a positive integer n, 8. 2n > n2 when n Î N such that
(a) n > 2 (b) n > 3 (c) n < 5 (d) n ³ 5
1 1 1 1
Let a(n) = 1 + + + +…+ n . Then
2 3 4 (2 ) - 1 4n (2n)!
9. If < , then P(n) is true for
(a) a(100) £ 100 (b) a(100) > 100 n + 1 (n!)2
(c) a(200) £ 100 (d) a(200) < 100 (a) n ³ 1 (b) n > 0 (c) n < 0 (d) n ³ 2
8 MATHEMATICS

10. If P(n) : 3n < n!, n Î N, then P(n) is true


(2n + 1) 3n + 1 + 3 (2n – 1) 3n + 1 + 3
(a) for n ³ 6 (b) for n ³ 7 (a) (b)
4 4
(c) for n ³ 3 (d) for all n
11. If n is a positive integer, then 2 . 42n + 1 + 33n + 1 is (2n + 1) 3n +3 (2n – 1) 3n + 1 + 1
(c) (d)
divisible by : 4 4
(a) 2 (b) 7 (c) 11 (d) 27 18. For all n Î N,
12. For every natural number n, n(n2 – 1) is divisible
by 1 1 1
1+ + + ..... +
(a) 4 (b) 6 1+ 2 1+ 2 + 3 1 + 2 + 3 + ..... + n
(c) 10 (d) None of these is equal to
13. If p is a prime number, then n p – n is divisible by p
when n is a 3n n 2n 2n
(a) (b) (c) (d)
(a) Natural number greater than 1 n +1 n +1 n –1 n +1
(b) Irrational number 19. When 2301 is divided by 5, the least positive
(c) Complex number remainder is
(d) Odd number (a) 4 (b) 8 (c) 2 (d) 6
14. If 49n + 16n + l is divisible by 64 for all n Î N,
20. By the principle of induction " n Î N, 32n
then the least negative value of l is
when divided by 8, leaves remainder
(a) –2 (b) –1 (c) –3 (d) – 4 (a) 2 (b) 3 (c) 7 (d) 1
15. By mathematical induction,
Numeric Value Answer
1 1 1
+ + ..... + is 21. If n Î N, then 11n + 2 + 122n+1 is divisible by
1× 2 ×3 2 × 3× 4 n (n + 1)(n + 2)
___________.
equal to
22. For all n Î N, 41n – 14n is a multiple of
n (n + 1) n (n + 3) ___________.
(a) (b)
4 ( n + 2 ) ( n + 3) 4 (n + 1) (n + 2) 23. If m, n are any two odd positive integers with
n < m, then the largest positive integer which
n (n + 2) divides all the numbers of the type m2 – n2 is
(c) (d) None of these ___________.
4 (n + 1) (n + 3)
24. For every natural number n, 32n + 2 – 8n – 9 is
16. For all n Î N, 3.52n + 1 + 23n + 1 is divisible by divisible by ___________.
(a) 19 (b) 17 (c) 23 (d) 25 25. The remainder when 599 is divided by 13, is
17. For all n Î N, 1.3 + 2.32 + 3.33 + ..... + n.3n is ___________.
equal to

ANSW ER KEY
1 (d) 4 (a) 7 (a) 10 (b) 13 (a) 16 (b) 19 (c) 22 (27) 25 (8)
2 (c) 5 (a) 8 (d) 11 (c) 14 (b) 17 (b) 20 (d) 23 (8)
3 (a) 6 (a) 9 (d) 12 (b) 15 (b) 18 (d) 21 (133) 24 (16)
COMPLEX NUMBERS AND 5
QUADRATIC EQUATIONS

MCQs with One Correct Answer the point in the complex plane given by wk =

1. Let x1 and y1 be real numbers. If z1 and z2 are cos 2a k + i sin 2a k


for k = 1, 2, 3. The origin, O
complex numbers such that |z1| = |z2| = 4, then zk
|x1z1 – y1z2|2 + |y1z1 + x1z2|2 = is the
(a) 32(x12 + y12) (b) 16(x12 + y12) (a) incentre of DA1A2A3
(c) 4(x12 + y12) (d) 32 (b) orthocentre of DA1A2A3
(c) circumcentre of DA1A2A3
2. Let z and w be two non-zero complex numbers
(d) centroid of DA1A2A3
such that | z | = | w | and Arg z + Argg w = p, then
7. If a point z1 is the reflection of a point z2 through
z equals
(a) w (b) – w (c) w (d) – w the line b z + b z = c, b ¹ 0 in the argand plane,
3. If | z - 1| + | z + 3 | £ 8, then the range of values then b z2 + b z1 is equal to
(a) 4c (b) 2c
of | z - 4 | is
(c) c (d) None of these
(a) (0, 7) (b) (1, 8) (c) [1, 9] (d) [2, 5] 8. z1 and z2 lie on a circle with centre at the origin.
4. Which of the following is/are value of The point of intersection z3 of the tangents at z1
2 2 and z2 is given by
sin ln (ii )i + cos ln (ii )i ?
1 2z1 z2
(a) – 1 (b) 1 (a) ( z1 + z2 ) (b)
2 z1 + z2
(c) 0 (d) None of these
1æ 1 1 ö z1 + z2
5. If n1, n2 are positive integers, then (1+ i ) n1 + (c) + (d)
2 çè z1 z2 ÷ø z1 + z2
(1 + i 3 )n1 + (1+ i 5 )n2 + (1+ i 7 )n2 is real number 9. If z1, z2 are two complex numbers such that
if and only if:
(a) n1 = n2 + 1 z1 – z2
= 1 and i z1 = Kz2, where K Î R, then
(b) n1 + 1 = n2 z1 + z2
(c) n1 = n2
the angle between z1 – z2 and z1 + z2 is
(d) n1,n2 are any positive integers
6. Let rk > 0 and zk = rk (cos ak + i sin ak) for k = 1, æ 2K ö æ 2K ö
(a) tan– 1 ç 2 ÷ (b) tan– 1 ç ÷
1 1 1 è K + 1ø è 1– K 2 ø
2, 3 be such that + + = 0 Let Ak be
z1 z2 z3 (c) – 2 tan– 1 K (d) 2 tan– 1 K
10 MATHEMATICS

10. Let 'z' be a complex number and 'a' be a real 16. If the roots of the equation x2 + px + c = 0 are 2,
parameter such that z2 + az + a2 = 0, then which –2 and the roots of the equation x2 + bx + q = 0
is of the following is not true? are –1, –2, then the roots of the equation x2 + bx
(a) locus of z is a pair of staight lines + c = 0 are
(b) |z| = |a| (a) –3, –2 (b) –3, 2 (c) 1, – 4 (d) –5, 1
17. If x Î R, then the maximum value of
2p
(c) arg (z) = ±
3 y = 2(a - x) ( x + x 2 + b2 ) is
(d) None of these
11. Let P denotes a complex number z = r(cos q + i (a) a 2 + b2 (b) a2 - b2
sin q) on the Argand's plane, and Q denotes a
(c) a 2 + 2b 2 (d) None of these
æ p pö
complex number 2| z |2 çcosæçq+ ö÷ +i sinæçq+ ö÷÷ . 18. If a, b are the roots of x 2 + px + q = 0, and
è è 4ø è 4øø
If 'O' is the origin, then DOPQ is a
x 2 n + p n x n + q n = 0 and is a root of
(a) isosceles but not right angled b
(b) right angled but not isosceles
(c) right isosceles x n + 1 + ( x + 1)n = 0, a n ¹ bn , the n must be
(d) equilateral (a) any integer (b) an even integer
12. If w ¹ 1 and w3 = 1, then (c) an odd integer (d) None of these
a w + b + c w2 a w2 + b + cw 19. If 0 < a < b < g < p/2, then the equation (x – sin b)
+ is equal to (x – sin g) + (x – sin a) (x – sin g) + (x – sin a)
a w2 + b w + c a + b w + c w2 (x – sin b) = 0 has
(a) 2 (b) w (c) 2w (d) 2w2 (a) real and unequal roots.
1 1 (b) non-real roots.
13. If z + = 1 and a = z2017 + 2017 and b is (c) real and equal roots.
z z
2 n (d) real and unequal roots greater than 2.
the last digit of the number 2 – 1, when the 20. The set of values of a for which inequation
integer n > 1, the value of a2 + b2 is (a – 1) x2 – (a + 1)x + a – 1 ³ 0 is true for all x ³ 2
(a) 23 (b) 24 (c) 26 (d) 27
14. If y1 = max ||z – w| – |z – w2||, where |z| = 2 and é 7ù
(a) ê1, 3 ú (b) (– ¥, 1)
1 ë û
y2 = max ||z – w| – |z – w2||, where |z| = and w and
2 é7 ö
w2 are complex cube roots of unity, then (c) ê 3 , ¥÷ø (d) None of these
ë
(a) y1 = 3 ; y2 = 3
Numeric Value Answer
(b) y1 < 3 ; y2 = 3
21. a, b, c are integers, not all simultaneously equal
(c) y1 = 3 ; y2 < 3 and w is cube root of unity (w ¹ 1), then minimum
(d) y1 > 3; y2 < 3 value of |a + bw + cw2| is
15. If a and b are the roots of the equation ax2 + bx
22. If 2 – i is a root of the equation ax2 + 12x + b = 0
+ c = 0, (c ¹ 0) , then the equation whose roots (where a and b are real), then the value of ab is
1 1 equal to
are and is
aa + b a b +b 23. If the equations ax 2 + bx + c = 0 and
(a) acx2 – bx + 1 = 0 (b) x2 – acx + bc + 1 = 0
cx2 + bx + a = 0, a ¹ c have a negative
(c) acx2 + bx – 1 = 0 (d) x2 + acx – bc + 11 = 0
common root, then the value of a - b + c is
Complex Numbers and Quadratic Equations 11
24. If a, b are the roots the quadratic equation 28. If z and w are two complex numbers having
non-negative imaginary parts such that
x2 – (3 + 2 log2 3 –3 log 3 2 ) x –2(3log3 2 – 2log2 3 )
æ z - 2ö æ w - 1ö
= 0, then the value of a2 + ab + b2 is equal to arg ç = arg ç = p / 2, then
è z + 2 ÷ø è w + 1÷ø
25. Let f (x) = x4 + ax3 + bx2 + cx + d be a polynomial
with real coefficients and real zeroes. If |f (i)| = 1, | w- z |< k; evaluate k. (Here k is least upper
(where i = -1 ) then a + b + c + d is equal to bond)
26. If w and w2 be the non-real cube roots of unity p æ 1 - pi p -i ö
29. If z = (1 + i )4 çç + ÷÷ , where
1 1 1 4 è p + i 1 + pi ø
and + + = 2w 2 and
a+w b+w c+w
æ |z| ö
1 1 1 i = -1, then ç amp( z ) ÷ equals to
+ + = 2w, where a, b, c è ø
a + w2 b + w2 c + w2 30. If p, q, r are positive and are in A.P., then the
1 1 1 roots of the quadratic equation px2 + qx + r = 0
are real then the value of + + is
a + 1 b +1 c + 1 r
are real for - 7 ³ k 3 then find the value
equal to : p
27. For any real x, the maximum value of
of k.
2
k2
(x - k) x +( x +k2 2
) is equal to

ANSW ER KEY
1 (a) 5 (d) 9 (d) 13 (c) 17 (a) 21 (1) 25 (0) 29 (4)
2 (d) 6 (d) 10 (d) 14 (c) 18 (b) 22 (45) 26 (2) 30 (4)
3 (c) 7 (c) 11 (c) 15 (a) 19 (a) 23 (0) 27 (2)
4 (b) 8 (b) 12 (d) 16 (c) 20 (c) 24 (7) 28 (3)
12 MATHEMATICS

6
LINEAR INEQUALITIES

MCQs with One Correct Answer æ9 ö æ 3ö


(a) ç , ¥÷ (b) ç - ¥, ÷
1. The set of all x satisfying the inequality è 2 ø è 2ø
2 1 2x - 1 æ 3ö æ 3ö
- - 3 ³0 (c) ç - 2, - ÷ (d) ç - 1, ÷
x - x +1 x +1 x +1
2 è 2ø è 2ø
(a) (–¥, 2] (b) [1, 2] 12x
(c) (–¥, –1) È (–1, 2] (d) (2, ¥] 7. If ³ 1 for all real values of x, the
2. The set of all x satisfying the inequality 4x 2 + 9
(2x + 1) (x – 3) (x + 7) < 0 inequality being satisfied only if | x | is equal to
3 2 1 1
æ 1 ö (a) (b) (c) (d)
(a) (–¥, 7) (b) ç - ,3 ÷ 2 3 3 2
è 2 ø
8. The equation | x - 1| +a = 4 can have real
æ 1 ö solutions for x if ‘a’ belongs to the interval
(c) (–¥, 7) È ç - ,3 ÷ (d) (–¥, 3)
è 2 ø (a) (-¥, + ¥) (b) (-¥, 4]
3. The set of real values of x satisfying
(c) (4, + ¥) (d) [–4, 4]
| x - 1| £ 3 and | x - 1| ³ 1 is 9. The interval(s) that satisfy the equation
(a) [2, 4]
x 2 - 8x + 12 x 2 - 8x + 12
(b) (-¥, 2] È [4, + ¥) =- is /are
x 2 - 10x + 21 x 2 - 10x + 21
(c) [-2, 0] È [2, 4]
(d) none of these (a) (-¥, 2] (b) [2, ¥)
4. The system of equation (c) [6, 7) (d) [3, 6] È [7, ¥)
| x - 1 | +3y = 4, x - | y - 1 |= 2 has 10. The set of all real x satisfying the inequality
(a) No solution 3- | x |
³ 0 is
(b) A unique solution 4- | x |
(c) Two solutions (a) [– 3, 3] È (– ¥, – 4) È (4, ¥)
(d) More than two solutions (b) (– ¥, – 4) È (4, ¥)
5. The number of real roots of the equation (c) (– ¥, – 3) È (4, ¥)
| 2- | 1- | x |||= 1 is (d) (– ¥, – 3) È (3, ¥)
11. If x, y, z be any three positive real number and
(a) 1 (b) 3 (c) 5 (d) 6
x+y y+z x +z
6. The solution set of the inequality A= 2 2
+ 2 2
+ 21 2
x +y y +z x +z
3
| x + 2 | - | x - 1 |< x - is 1 1 1
2 and B = + + ;
x y z
Linear Inequalities 13
then which of the following is true? 18. Solution set of the in equality log102x
(a) A ³ B (b) A £ B (c) A = B (d) A < B – 3 (log10 x) (log10 (x – 2)) + 2log102 (x – 2) < 0, is :
12. If x, y, z are arbitrary real numbers satisfiying the (a) (0, 4) (b) (– ¥, 1 )
condition xy + yz + zx < 0 and if (c) (4, ¥) (d) (2, 4)
19. The solution set of log|sin x| (x2 – 8x + 23)
x 2 + y2 + z 2
u = then only one of the 3
xy + yz + zx > contains
following statements is always correct. log 2 | sin x |
Which one is it? æ 3p ö æ 3p ö
(a) – 1 £ u < 0 (a) x Î (3, p) È ç p, ÷ È ç , 5 ÷
(b) u takes all negative real values è 2 ø è 2 ø
(c) – 2 < u £ – 1 (b) x Î (3, p) È (p, 5)
(d) u £ – 2 æ 5p ö
(c) x Î ç 3, ÷
13. If a1,a2.....,an are positive real numbers whose è 2 ø
product is a fixed number c, then the minimum (d) None of these
value of a1 + a2 + .......+an-1 + 2an is 20. If x is real, the maximum value of
(a) n(2c)1/n (b) (n + 1)c1/n
(c) 2nc 1/n (d) (n + 1)(2c)1/n 3x 2 + 9 x + 17
is
14. The set of real values of x for which 3x 2 + 9 x + 7
x+2 1 17
log 0.2 £ 1 is (a) (b) 41 (c) 1 (d)
x 4 7
æ 5ù é5 ö
(a) ç - ¥, - ú È (0, ¥) (b) ê 2 , ¥÷ Numeric Value Answer
è 2û ë ø
21. If ‘k’ any integer such that set of all real values
(c) (-¥, - 2) È [0, ¥) (d) None of these of x, satisfy the equation (sin3 x ) (cos3 x )
x 2 + 6x + 9 3x + 3- x
15. If log1 / 2 < - log 2 ( x + 1), then x lies £ ; then find the minimum value k.
2(x + 1) k
in the interval 22. For any x, y Î R, xy > 0. Then the minimum value
(a) (-1, - 1 + 2 2 ) (b) (1 - 2 2 , 2) 2x x 3 y 4y 2
of + + 4 is.
(c) (-1, ¥) (d) None of these y3 3 9x
16. The least integer a, for which
23. Number of integeral solution satisfying
1 + log 5 ( x 2 + 1) £ log 5 (ax 2 + 4x + a ) is true for
x + 3 - x ³ 3 - x + 3 is
all x Î R is
(a) 6 (b) 7 (c) 10 (d) 1 x+3 + x
17. The set of all real numbers x for which 24. Solve for x, > 1 and then find
x+2
x2 – [x + 2] + x > 0, is modulus of smallest integral member of the
(a) (- ¥,-2) È (2, ¥ ) interval.
(b) (- ¥,- 2 )È ( 2,¥ ) 25. The number of integers satisfying the equation
4 - x2 4 - x2
(c) (- ¥,-1) È (1, ¥ ) |x|+ = x+ is
x x
(d) ( 2, ¥ )
ANSW ER KEY
1 (c) 4 (b) 7 (a) 10 (a) 13 (a) 16 (b) 19 (a) 22 (2) 25 (4)
2 (c) 5 (c) 8 (b) 11 (b) 14 (a) 17 (b) 20 (b) 23 (1)
3 (c) 6 (a) 9 (c) 12 (d) 15 (a) 18 (c) 21 (4) 24 (4)
14 MATHEMATICS

PERMUTATIONS AND 7
COMBINATIONS

MCQs with One Correct Answer words always begin with B and end with T. Then
m/n is
1. Ten different letters of an alphabet are given (a) 10 (b) 20 (c) 1 (d) 2
words with five letters are formed from three 7. Anil have tiled his square bathroom wall with
given letters. Then the number of words which congruent square tiles. All the tiles are red,
have at least one letter repeated are except those along the two diagonals, which are
(a) 69760 (b) 30240 all blue. If he used 121 blue tiles, then the
(c) 99748 (d) None of these number of red tiles used are
2. How many different nine digit numbers can be (a) 900 (b) 1800 (c) 3600 (d) 7200
formed from th e number 223355888 by 8. The number of distinct natural numbers up to a
rearranging its digits so that the odd digits maximum of four digits and divisible by 5, which
occupy even positions ? can be formed with the digits 0, 1, 2, 3, 4, 5, 6, 7,
(a) 16 (b) 36 (c) 60 (d) 180 8, 9, each digit not occurring more than once in
3. The letters of the word COCHIN are permuted each number, is
and all the permutations are arranged in an (a) 1246 (b) 952
alphabetical order as in an English dictionary. (c) 1106 (d) None of these
The number of words that appear before the word 9. 6 white and 6 black balls are distributed among
COCHIN is ten identical urns, so that there is atleast one
(a) 360 (b) 192 (c) 96 (d) 48 ball in each urn. Balls are all alike except for the
4. The total number of 5-digit numbers of different colour and each box can hold any number of
digits in which the digit in the middle is the balls. The number of different distributions of
largest is the balls is:
9 (a) 26250 (b) 132 (c) 12 (d) 10
(a) å n
p4 (b) 4563 10. Number of ways in which two Americans, two
n =4
British, one Chinese, one Dutch and one Egyp-
(c) 2688 (d) 5292 tian can sit on a round table so that persons of
5. All possible 120 permutations of WDSMC are the same nationality are separated is
arranged in dictionary order, as if each were an (a) 48 (b) 240
ordinary five-letter word. The last letter of the (c) 336 (d) None of these
86th word in the list, is :
11. In an examination of 9 papers a candidate has to
(a) W (b) D (c) M (d) C
pass in more papers than the number of papers
6. If m be the number of different words that can be
in which he fails in order to be successful. The
formed with the letters of the word BHARAT in
number of ways in which he can be unsuccessful
which B and H are never together and n be
is
number of different words that can be formed
with the letters of the words BHARAT in which (a) 255 (b) 256 (c) 193 (d) 319
Permutations and Combinations 15
12. During a draw of lottery, tickets bearing numbers 18. There are three coplanar parallel lines. If any p
1, 2, 3, ..., 40, 6 tickets are drawn out and then points are taken on each the lines, the maximum
arranged in the descending order of their number of triangles with vertices at these points is
numbers. In how many ways, it is possible to (a) 3p2(p – 1) + 1 (b) 3p2(p – 1)
have 4th ticket bearing number 25? 2
(c) p (4p – 3) (d) None of these
(a) 15C3 × 24C2 (b) 12C3 × 20C2
(c) 15C3 + 24C2 (d) None of these 19. From the vertices of a regular polygon of 10 sides,
13. A teacher takes 3 children from her class to the the number of ways of selecting three vertices
zoo at a time as often as she can, but she does such that no two vertices are consecutive is
not take the same three children to the zoo more (a) 10 (b) 30 (c) 50 (d) 40
than once he finds than she goes to the zoo 84 20. 5 different objects are to be distributed among 3
times more than a particular child goes to the persons such that no two persons get the same
zoo. The number of children in her class is number of objects. Number of ways this can be done, is
(a) 12 (b) 10 (a) 60 (b) 90 (c) 120 (d) 150
(c) 60 (d) None of these
14. Messages are conveyed by arranging four white, Numeric Value Answer
one blue, and three red flags on a pole. Flags of
the same colour are alike. If a message is 21. If a, b, c are three natural numbers in AP such that
transmitted by the order in which the colours a + b + c = 21 and if possible number of ordered
are arranged, the total number of messages that triplet (a, b, c) is then the value of (l – 5) is
can be transmitted if exactly six flags are used is 22. In a single correct match the column question,
(a) 45 (b) 65 (c) 125 (d) 185 column I contain 10 questions and Column II
15. There were two women participating in a chess
tournament. Every participant played two games contain 10 answers written in some arbitrary order.
with the other participants. The number of games If the number ways a student can answer this
that the men played between themselves proved question so that exactly 6 of his matching are
to exceed by 66 the number of games that the men correct is k, then (sum of digits of k)/2 is equal to
played with the women. The number of partici- 23. There are 720 permutations of the digits 1, 2, 3, 4,
pants is 5, 6. Suppose these permutations are arranged
(a) 6 (b) 11 from smallest to largest numerical values,
(c) 13 (d) None of these beginning from 1 2 3 4 5 6 and ending with 6 5 4
16. In a class tournament, all participants were to 3 2 1. Then the digit in unit place of number at
play different games with one another. Two
players fell ill after having played three games 267th position is ...... .
each. If the total number of games played in the 24. If N is the number of ways in which a person can
tournament is equal to 84, the total number of walk up a stairway which has 7 steps if he can
participants in the beginning was equal to take 1 or 2 steps up the stairs at a time, then the
(a) 10 (b) 15 (c) 12 (d) 14 value of N/3 is ...... .
17. The number of ways in which 5 X's can be placed 25. In an international convention participants from
in the squares of the figure so that no row 10 different countries were arranged in a row such
remains empty is that all the participants from the same country
were together. Each country has different number
of participants with maximum 10 participants from
a country. If K is the number of ways that they
can be arranged in a row then find the highest
power of 10 in K.
(a) 97 (b) 44 (c) 100 (d) 126
ANSWER KEY
1 (a) 4 (d) 7 (c) 10 (c) 13 (b) 16 (b) 19 (c) 22 (9) 25 (9)
2 (c) 5 (b) 8 (c) 11 (b) 14 (d) 17 (b) 20 (b) 23 (6)
3 (c) 6 (b) 9 (d) 12 (a) 15 (c) 18 (c) 21 (8) 24 (7)
16 MATHEMATICS

8
BINOMIAL THEOREM

MCQs with One Correct Answer 8. Let N = 21224 – 1, a = 2153 + 277 + 1 and b = 2408
– 2204 + 1. Then which of the following
1. 2 60 when divided by 7 leaves the remainder statement is correct ?
(a) 1 (b) 6 (c) 5 (d) 2 (a) a divides N but b does not
32 (b) b divides N but a does not
2. If 7 divides 32 32 , the remainder is
(c) a and b both divides N
(a) 1 (b) 0 (c) 4 (d) 6
(d) neither a nor b divides N
n æ r -1 ö 9. The number N = 20C7 – 20C8 + 20C9 – 20C10 +
3. å ç ç å n r
Cr C p 2 p÷
÷
is equal to ..... – 20C20 is not divisible by :
r=1 è p =1 ø
(a) 3 (b) 7 (c) 11 (d) 19
(a) 4n – 3n + 1 (b) 4n – 3n – 1
10. Let x be the 7th term from the beginning and y be
n
(c) 4 – 3 + 2 n (d) 4n – 3n the 7th term from the end in the expansion of
4. If (1 + x – 2x ) = 1+ a1x + a2x2 + ..... + a12 x 12,
2 6
n
then the value of a2 + a4 + ..... + a12 is : æ 1/3 1 ö
ç 3 + 1/3 ÷ . If y = 12x then the value of n is :
(a) 1024 (b) 64 è 4 ø
(c) 32 (d) 31 (a) 9 (b) 8 (c) 10 (d) 11
5. The last term in the binomial expansion of 11. The coefficient of the term independent of x in
1/ 3 n 1/ 3 10
(2 - 1/ 2) is (1/ 3.9 ) log 3 8. Then the æ ö
5th term from the beginning is ç x +1 x –1 ÷
the expansion of ç 2 – is
1 1÷
(a) 10 C6 (b) 2.10C4 çè 3 ÷
10
(c) 1/2. C4 (d) None x – x3 +1 x – x2 ø
(a) 70 (b) 112 (c) 105 (d) 210
If (1 + ax) = 1+ 8x +24x2 + ......... ; than a – n is
n
6.
a+n 12. If the middle term of (1 + x)2n (x > 0, n Î N) is the
equal to (n being a positive integer) greatest term of the expansion. Then the interval
(a) 3 (b) – 3 in which x lies, is

1 1 én +1 n + 2ù é n –1 n + 1ù
(a) êë n , n úû (b) êë n , n úû
(c) – (d)
3 3

7. The unit digit of 17 2009 + 112009 – 7 2009 is é n n + 1ù


(c) êë n + 1 , n úû (d) None of these
(a) 1 (b) 2 (c) 3 (d) 0
Binomial Theorem 17
13. The number of irrational terms in the expansion
C1 C 3 C 5
+ + + ....... is equal to
of ( 8
)
5 + 6 2 100 is
19. The value of
2 4 6
(a) 97 (b) 98 (c) 96 (d) 99
2n + 1 2n
14. The greatest value of the term independent of x in (a) (b)
–1
n +1 n +1
the expansion of ( x sin a + x cos a )10 , a Î R, is
10!
2n + 1 2n - 1
(a) 25
(b) (c) (d)
n -1 n +1
(5!)2
(c) 1 10!
(d) None of these
. n
2 5 (5!) 2 20. If I is integral part of (2 + 3) and f is its
25
15. The largest term in the expansion of (2 + 3x ) fractional part. Then (I + f ) (1 – f ) is
where x = 2 is its.
(a) 13th term (b) 19th term (a) I + 1 (b) 1 (c) n (d) 2n
th
(c) 20 term (d) 26th term
16. If the middle term in the expansion of Numeric Value Answer
10
æ1 ö 7 21. The largest real value for x such that
ç + x sin x ÷ equals to 7 then x is equal to
è x ø 8 4 æ 34 - k öæ x k ö 32
(n Î I)
å çç ÷ç ÷ =
÷ç ÷ 3 is
p p k = 0 è ( 4 - k )! øè k! ø
(a) 2np ± (b) n p +
6 6
22. Find the coefficient of x13 in the expansion of
np n 5p
(c) n p + (-1) (d) n p + ( -1) (1 – x)5 (1 + x + x2 + x3 )4 is
6 6
17. The sum of the coefficients of all the integral 23. If (1 + x)n = C0 + C1x + C2x2 + ..... + Cnxn, then
40
find the value of 3C0 – 5C1 + 7C2 + ...+ (–1)n
powers of x in the expansion of (1 + 2 x ) is (2n+3)Cn
(a) 3 40 + 1 (b) 3 40 – 1 1 1 1 1 2k
1 40 1 40 24. If + + + ... + = then
(c) (3 – 1) (d) (3 + 1) 1!10! 3!8! 5!6! 11!1! 11!
2 2
18. If p (n) denotes product of all binomial coefficients k
is equal to
2
in (1+ x ) n , then ratio of p (2002) to p (2001) is
25. (1 – 2x + 5x2 – 10x3) (1 + x)n = 1 + a1x + a2x2 + ...
2001
(2002)
(a) 2002 (b) and that a21 = 2a2, then the value of n is____
(2001) !
(2001) 2002
(c) (d) 2001
(2002)!

ANSWER KEY
1 (a) 4 (d) 7 (a) 10 (a) 13 (a) 16 (c) 19 (d) 22 (4) 25 (6)
2 (c) 5 (a) 8 (c) 11 (d) 14 (c) 17 (d) 20 (b) 23 (0)
3 (d) 6 (c) 9 (c) 12 (c) 15 (c) 18 (b) 21 (1) 24 (5)
18 MATHEMATICS

9
SEQUENCES AND SERIES

MCQs with One Correct Answer y x z


(a) x, y, z are in A.P. (b) , , are in A.P..
1. Sum to n terms of the series 6 3 2
3
13 + 3. 23 + 33 + 3. 43 + 5 + ................ is y x z
(c) , , are in A.P.. (d) 6y, 3x, 2z are in A.P.
(n is even) 6 3 2
n(n 2 + 1) (2 n + 1) a +b b+c
(a) 6. For a, b, c Î R – {0}, let , b, are in
3 1– ab 1– bc
A.P. If a, b are the roots of the quadratic
(b) n(n3 + 4n2 + 10n + 8) equation 2ac x2 + 2abc x + (a + c) = 0, then the
8 value of (1 + a)(1 + b) is
(a) 0 (b) 1 (c) – 1 (d) 2
n(n3 + 1) 7. An A.P. consist of even number of terms 2n
(c)
8 having middle terms equal to 1 and 7 respec-
n2 (2n 2 + 6 n + 5) tively. If n is the maximum value which satisfy
(d) t1t2n + 713 ³ 0, then the value of the first term
4
2. If 1, log9 (31–x + 2), log3 (4.3x – 1) are in A.P., then of the series is
x equals (a) 17 (b) – 15 (c) 21 (d) – 23
(a) log3 4 (b) 1 – log3 4 8. If a, b, c are in G. P., x and y be the A. M.’s between
(c) 1 – log4 3 (d) log4 3
3. In the sum of first n terms of an A.P. is cn2, then æ a cö æ b bö
a, b and b, c respectively, then ç + ÷ ç + ÷
the sum of squares of these n terms is è x yø è x yø
is equal to
n (4n2 –1) c 2 n (4n2 +1) c 2 (a) – 2 (b) – 4 (c) 2 (d) 4
(a) (b)
6 3 9. Suppose a, b, c are in A.P. and a2, b2, c2 are in G.P.
n (4n2 +1) c 2 3
n (4n2 –1) c 2 if a < b < c and a + b + c = , then the value of a is
(c) (d) 2
3 6 1 1
4. If a1, a2, ..., an are in A.P. with common difference (a) (b)
d ¹ 0, then (sin d) [sec a1 seca2 + sec a2 sec a3 + 2 2 2 3
... + sec an–1 sec an] is equal to 1 1 1 1
(a) cot an – cot a1 (b) cot a1 – cot an (c) 2 - (d) 2 -
3 2
(c) tan an – tan a1 (d) tan an – tan an–1 10. An infinite G.P. has first term ‘x’ and sum ‘5’,
1 1 1 1 3 1 3 then x belongs to
5. If x = 2 + 2 + 2 + ..., y = 2 + 2 + 2 + 2 (a) x < – 10 (b) – 10 < x < 0
1 3 5 1 2 3 4 (c) 0 < x < 10 (d) x > 10
1 1 1 1 11. In the quadratic equation ax2 + bx + c = 0, D = b2
+ ... and z = 2 – 2 + 2 – 2 +..., then – 4ac and a + b, a2 + b2, a3 + b3, are in G.P. where
1 2 3 4 a, b are the root of ax2 + bx + c = 0, then
(a) D ¹ 0 (b) bD = 0 (c) cD = 0 (d) D = 0
Sequences and Series 19
12. The sum of an infinite geometric series is 2 and the 1
sum of the geometric series made from the cubes 1 n -
(a) 1 – (b) n +1
of this infinite sereis is 24. Then the series is ( n + 1)2 n 2
3 3 3 3 3 3 1
(a) 3 + - + - .... (b) 3 + + + + .... (c) 1- (d) None of these
2 4 8 2 4 8 n.2n+1
3 3 3 b
(c) 3 - + - + ... (d) None of these 20. Let ax2 + ³ c for all positive x, where a < 0
2 4 8 x
13. If a, b, c are in G. P. and log a – log 2b, log 2b – and b < 0. The value of the expression 27ab2
log 3c and log 3c – log a are in A. P., then a, b, c cannot be less than
are the sides of a triangle which is (a) 4c3 (b) 4c2 (c) 8c3 (d) c3
(a) Acute angled (b) Obtuse angled
(c) Right angled (d) None of these Numeric Value Answer
14. Ar ; r = 1, 2, 3, ........... , n are n points on the
21. Sum of infinite number of terms of GP is 20 and sum
parabola y 2 = 4x in the first quadrant. of their square is 100. The common ratio of GP is
22. Three numbers a, b, c are in GP. If a, b, c – 64 are
If Ar = ( xr , yr ) , where x1 , x2 , x3 , ..............., in AP and a, b – 8, c – 64 are in GP, then the sum
xn are in G. P. and x1 = 1, x2 = 2, then yn is equal to of the numbers may be
23. a, b, c are positive integers forming an
n +1 n
n +1 n +1 increasing G.P. and b – a is a perfect cube and
(a) –2 2 (b) 2 (c) ( 2) (d) 22 log6 a + log6 b + log6 c = 6, then a + b + c =
15. If three successive terms of a G..P. with common 24. The sum to infinite term of the series
ratio r (r > 1) form the sides of a D ABC and [r]
denotes greatest integer function, then [r] + [– r] = 2 6 10 14
1 + + 2 + 3 + 4 + ... is
(a) 0 (b) 1 3 3 3 3
(c) – 1 (d) None of these 25. The 20th term of the series 2 + 3 + 5 + 9 + 16
16. If a, b, c, are in A.P. and p, p¢ are respectively +.......is
A.M. and G.M. between a and b while q, q¢ are 26. Two consecutive numbers from 1, 2, 3,.........., n
respectively AM.and G.M. between b and c, then are removed. If the arithmetic mean of the
(a) p 2 + q 2 = p '2 + q '2 n
(b) pq = p ' q ' remaining numbers is 105/4 then is equal to
10
(c) p 2 - q 2 = p '2 - q '2 27. Let a, b, c, d be four distinct real numbers in
(d) p 2 + p¢2 = q 2 + q '2 A.P. Then half of the smallest positive value of
17. The sum of the series k satisfying 2(a – b) + k(b – c)2 + (c – a)3 = 2(a
– d) + (b – d)2 + (c – d)3 is ..... .
1 + 2.2 + 3.2 2 + 4.2 3 + 5.2 4 + ... + 100.2 99 is 28. Let x1, x2, ... Î (0, p) denote the of values of x
(a) 99.2100 – 1 (b) 100.2100 satisfying the equation
(c) 99.2100 (d) 99.2100 + 1 2 3
18. If a, b, c, d are positive real number such that 27(1 + |cos x| + cos x + |cos x| + ....upto ¥) = 93, find
a + b + c + d = 2, then M = (a + b) (c + d) satisfies
the relation: 1
the value of ( x1 + x2 + ...)
(a) 0 < M £ 1 (b) 1 £ M £ 2 p
29. For a, b > 0, let 5a – b, 2a + b, a + 2b be
(c) 2 £ M £ 3 (d) 3 £ M £ 4 in A.P. and (b + 1)2, ab + 1, (a – 1)2 are in
2 3 G.P., then the value of (a–1 + b– 1) is ..... .
3 1 4 æ1ö 5 æ1ö
19. The sum of . + .ç ÷ + .ç ÷ 30. If one geomteric mean G and two Arithmetic
1 .2 2 2.3 è 2 ø 3.4 è 2 ø means P and q be inserted between two quanti-
+ ......... to n terms is equal to ties, then G2 = (kp – q)(kq – p) then find k.
ANSWER KEY
1 (d) 4 (c) 7 (d) 10 (c) 13 (b) 16 (c) 19 (a) 22 (124) 25 (990) 28 (1)
2 (b) 5 (b) 8 (d) 11 (c) 14 (c) 17 (d) 20 (a) 23 (189) 26 (5) 29 (6)
3 (c) 6 (b) 9 (d) 12 (c) 15 (c) 18 (a) 21 (0.60) 24 (3) 27 (8) 30 (2)
20 MATHEMATICS

10
STRAIGHT LINES

MCQs with One Correct Answer x y


5. The line + = 1 meets the axis of x and y at A
a b
1. Through the point P (a, b), where ab > 0 , the
and B respectively and the line y = x at C so that
x y area of the trinagle AOC is twice the area of the
straight line + = 1 is drawn so as to form triangle BOC, O being the origin, then one of the
a b
with coordinate axes a triangle of area S. If positions of C is
ab > 0, then least value of S is
æ 2a 2a ö
(a) (a, a) (b) ç ,
1 è 3 3 ÷ø
(a) 2ab (b) ab
2 æ b bö æ 2b 2b ö
(c) çè 3 , 3 ÷ø (d) ç ,
(c) ab (d) None of these è 3 3 ÷ø
6. The range of values of b such that (0, b ) lie
2.
æ 1ö
The vertices of a triangle are ç ab, ÷ , on or inside the triangle formed by the lines
è abø y + 3x + 2 = 0, 3y – 2x – 5 = 0, 4y + x – 14 = 0 is
æ 1ö æ 1ö 1
çè bc, bc ÷ø and ç ca, ca ÷ where a, b, c are the (a) 5 < b £ 7 (b) £ b£1
è ø 2
roots of the equation x3 – 3x2 + 6x + 1 = 0. The 5 7
coordinates of its centroid are. (c) £ b £ (d) None of these
3 2
(a) (1, 2) (b) (2, – 1) (c) (1, – 1) (d) (2, 3) 7. The intercepts on the straight line y = mx by the
3. Consider points A (3, 4) and B (7, 13). If P be a lines y = 2 and y = 6 is less than 5, then m belongs
point on the line y = x such that PA + PB is to
minimum, then coordinates of P are
æ 4 4ö
æ 12 12 ö æ 13 13 ö (a) çè - , ÷ø
(a) ç , ÷ (b) ç , ÷ 3 3
è7 7ø è7 7ø
æ 4 3ö
(b) çè , ÷ø
æ 31 31 ö 3 8
(c) ç , ÷ (d) (0, 0)
è 7 7ø æ 4ö æ4 ö
4. If the straight lines 2x + 3y – 1 = 0, x + 2y – 1 = 0 (c) ç -¥, - ÷ È ç , ¥ ÷
è 3ø è3 ø
and ax + by – 1 = 0 form a triangle with origin as
orthocentre, then (a, b) is given by æ4 ö
(a) (6, 4) (b) (– 3, 3) (c) (– 8, 8) (d) (0, 7)
(d) çè , ¥÷ø
3
Straight Lines 21
8. If three distinct points A, B, C are given in the (a) ax + by + 2a = 0 (b) ax – by – 2a = 0
2-dimensional coordinate plane such that the ratio (c) bx + ay – 2b = 0 (d) ay – bx + 2b = 0
of the distance of each one of them from the
13. If the point (a, 2) lies between the lines x – y – 1
point (1, 0) to the distance from (– 1, 0) is equal
= 0 and 2 (x – y) + 5 = 0, then the set of values of
1
to , then the circumcentre of the triangle ABC ‘a’ is
2
is at the point æ9 ö
(a) (– ¥ , 3) È ç , ¥ ÷
è2 ø
æ5 ö
(a) ç ,0÷ (b) (0, 0)
è3 ø
æ 9ö
(b) ç 3, ÷
æ1 ö è 2ø
(c) ç ,0÷ (d) (3, 0)
è3 ø (c) (– ¥ , 3)
9. Let A (–3, 2) and B (–2, 1) be the vertices of a
(d) æ 1 ö
triangle ABC. If the centroid of this triangle lies çè - , 3÷ø
2
on the line 3x + 4y + 2 = 0, then the vertex C lies
on the line : 14. If two vertices of a triangle are (5, –1) and (–2, 3)
(a) 4x + 3y + 5 = 0 (b) 3x + 4y + 3 = 0 and its orthocentre is at (0, 0), then the third
vertex is
(c) 4x + 3y + 3 = 0 (d) 3x + 4y + 5 = 0
(a) (4, – 7) (b) (– 4, – 7)
10. The circumcentre of a triangle lies at the origin
(c) (– 4, 7) (d) (4, 7)
and its centroid is the mid point of the line
15. The base of an equilateral triangle is along the
segment joining the points (a2 + 1, a2 + 1) and
(2a, – 2a), a ¹ 0. Then for any a, the orthocentre line given by 3x + 4y = 9. If a vertex of the triangle
of this triangle lies on the line: is (1, 2), then the length of a side of the triangle
is:
(a) y – 2ax = 0
(b) y – (a2 + 1)x = 0 2 3 4 3
(a) (b)
(c) y + x = 0 15 15
(d) (a – 1)2x – (a + 1)2y = 0
4 3 2 3
11. The line parallel to the x- axis and passing through (c) (d)
the intersection of the lines ax + 2by + 3b = 0 and 5 5
bx – 2ay – 3a = 0, where (a, b) ¹ (0, 0) is 16. The equation of bisector of that angle between
the lines x + y + 1 = 0 and 2x – 3y – 5 = 0 which
3 contains the point (10, – 20) is
(a) below the x - axis at a distance of from it
2
(a) x ( 13 + 2 2 ) + y ( 13 – 3 2 )
2
(b) below the x - axis at a distance of from it
3 + ( 13 – 5 2 ) = 0
3 (b) x ( 13 – 2 2 ) + y ( 13 + 3 2 )
(c) above the x - axis at a distance of from it
2
2 + ( 13 + 5 2 ) = 0
(d) above the x - axis at a distance of from it
3
12. The straight line y = x – 2 rotates about a point (c) x ( 13 + 2 2 ) + y ( 13 + 3 2 )
where it cuts the x-axis and becomes
+ ( 13 + 5 2 ) = 0
perpendicular to the straight line
ax + by + c = 0. Then its equation is (d) None of these
22 MATHEMATICS

17. The bisector of the acute angle formed always pass through a fixed point P for all
between the lines 4 x - 3 y + 7 = 0 and possible values of q. If the maximum value of the
difference of distances of P and B (3, 4) from a
3x - 4 y + 14 = 0 has the equation :
x+ y+3= 0 x - y -3 = 0 k2
(a) (b) point on the line x - y + 3 = 0 is k then is
10
(c) x- y+3 = 0 (d) 3x + y - 7 = 0
equal to .
Numeric Value Answer 22. The straight line L º x + y + 1 = 0 and L1 º x + 2y
+ 3 = 0 are intersecting. m is the slope of the
18. A straight line through the origin O meets the straight line L2 such that L is the bisector of the
parallel lines 4x + 2y = 9 and 2x + y + 6 = 0 at anlge between L1 and L2. The unit digit of
points P and Q respectively. If the point 812m2 + 3 is equal to
m 23. If tana, tanb, tanl are the roots of the equation
O divides the segment PQ in the ratio , then t3 – 12t2 + 15t – 1 = 0; then the centroid of
n
triangle having vertices (tana, cota); (tanb,
m + n is ________.
cotbb); (tanl, cotl) is given by G(h, k); then
19. The vertex of an equilateral triangle is (2, –1),
evaluate (h + k)/(k – h).
and the equation of its base is x + 2y = 1. If the
24. Consider a DABC whose sides AB, BC, and CA
length of its sides is 2 / K , then value of K is are represented by the straight lines 2x + y = 0,
____. x + py = q, and x – y = 3, respectively. The point
20. If (sin q, cos q), q Î [0, 2p] and (1, 4) lie on the P(2, 3) is the orthocenter. The value of (p + q)/10
is ..... .
same side or on the line 3x – y + 1 = 0, then the 25. In DABC, the vertex A = (1, 2), y = x is the perpen-
maximum value of sin q will be ______. dicular bisector of the side AB and x – 2y + 1 = 0
21. The straight lines (3sec q + 5cosec q)x is the equation of the internal angle bisector of

+ (7 sec q - 3cosec q) y + 11(sec q - cosec q) = 0 L . If the equation of the side BC is ax + by – 5


= 0, then the value of a – b is ..... .

ANSW ER KEY
1 (a) 4 (c) 7 (c) 10 (d) 13 (d) 16 (a) 19 (15) 22 (1) 25 (4)
2 (b) 5 (d) 8 (a) 11 (a) 14 (b) 17 (c) 20 (0) 23 (9)
3 (c) 6 (c) 9 (b) 12 (d) 15 (b) 18 (7) 21 (4) 24 (5)
CONIC 11
SECTIONS

MCQs with One Correct Answer 5. Tangents are drawn from O (origin) to touch the
circle x2 + y2 + 2gx + 2fy + c = 0 at points P and Q.
1. The line 4 x + 3 y - 4 = 0 divides the The equation of the circle circumscribing triangle
circumference of the circle centered at (5, 3), in OPQ is
the ratio 1 : 2. Then the equation of the circle is (a) 2x2 + 2 y 2 + gx + fy = 0
2
(a) x 2 + y 2 - 10 x - 6 y - 66 = 0 (b) x + y 2 + gx + fy = 0
(c) x 2 + y 2 + 2gx + 2fy = 0
(b) x 2 + y 2 - 10 x - 6 y + 100 = 0
(d) None of these
(c) x 2 + y 2 - 10 x - 6 y + 66 = 0 6. A ray of light incident at the point (– 2, – 1) gets
reflected from the tangent at (0, –1) to the circle
(d) x 2 + y 2 - 10 x - 6 y –100 = 0
x 2 + y 2 = 1. The reflected ray touches the circle.
2. Let A(– 4, 0) and B(4, 0). Then the number of
points C = (x, y) on the circle x2 + y2 = 16 lying in The equation the line along which the incident
first quadrant such that the area of the triangle ray moved, is
whose vertices are A, B and C is a integer is (a) 4 x - 3 y + 11 = 0 (b) 4 x + 3 y + 11 = 0
(a) 14 (b) 15
(c) 3 x + 4 y + 11 = 0 (d) 4x + 3 y + 7 = 0
(c) 16 (d) None of these
3. If (a, b) is a point on the circle whose centre is 7. If the line y = mx + 1 meets the circle x2 + y2 + 3x
on the x-axis and which touches the line x + y = 0 = 0 in two points equidistant from and on
at (2, –2), then the greatest value of a is opposite sides of x-axis, then
(a) 3m + 2 = 0 (b) 3m – 2 = 0
(a) 4 – 2 (b) 6 (c) 2m + 3 = 0 (d) 2m – 3 = 0
(c) 4 + 2 2 (d) 4 + 2 8. If a circle passes through the point (a, b) and
4. The set of values of ‘c’ so that the equations cuts the circle x 2 + y 2 = 4 orthogonally, then
2 the locus of its centre is
y = | x | + c and x + y 2 – 8| x | – 9 = 0 have no
solution is (a) 2ax - 2by - (a 2 + b2 + 4) = 0
(a) (– ¥ , – 3) È (3, ¥ ) (b) 2ax + 2by - (a 2 + b2 + 4) = 0
(b) (– 3, 3)
(c) 2ax - 2by + (a 2 + b2 + 4) = 0
(c) (–¥, – 2 ) È (5 2 , ¥)
(d) (5 2 –4, ¥) (d) 2ax + 2by + (a 2 + b2 + 4) = 0
24 MATHEMATICS

9. The set of all real values of l for which exactly


( x - 2)2 ( y - 1)2
two common tangents can be drawn to the circles (c) + =1
x2 + y2 – 4x – 4y + 6 = 0 and 16 25
x2 + y2 – 10x – 10y + l = 0 is the interval: ( x - 2)2 ( y - 1)2
(d) + =1
(a) (12, 32) (b) (18, 42) 9 25
(c) (12, 24) (d) (18, 48) 16. The radius of the circle passing through the foci
10. A circle bisects the circumference of the circle 2 2
x2 + y2 – 2y – 3 = 0 and touches the line x = y of the ellipse x + y = 1 , and having its
and the point (1, 1). Its radius is : 16 9
centre at (0, 3) is
3 9
(a) (b) (c) 4 2 (d) 3 2 1 7
2 2
11. Let L1 be the length of the common chord of the (a) 4 (b) 3 (c) (d)
2 2
curves x2 + y2 = 9 and y2 = 8x, and L2 be the 17. Equation of the line passing through the points
length of the latus rectum of y2 = 8x, then: of intersection of the parabola x2 = 8y and the
(a) L1 > L2 (b) L1 = L2
x2
L1 ellipse + y 2 = 1 is :
(c) L1 < L2 (d) = 2 3
L2
(a) y – 3 = 0 (b) y + 3 = 0
12. If the tangent at the point P (x1, y1) to the parabola
(c) 3y + 1 = 0 (d) 3y – 1 = 0
y2 = 4ax meets the parabola y2 = 4a (x + b) at Q
and R, then the mid-point of QR is 18. Equation of the largest circle with centre (1, 0)
that can be inscribed in the ellipse x2 + 4y2 = 16,
(a) (x1 + b, y1 + b) (b) (x1 – b, y1 – b) is
(c) (x1, y1) (d) (x1 + b, y1 – b)
2 2
13. Tangent to the curve y = x2 + 6 at a point (1, 7) (a) 2 x + 2 y – 4x + 7 = 0
touches the circle x2 + y2 + 16x + 12y + c = 0 at a
(b) x 2 + y 2 – 2x + 5 = 0
point Q. Then the coordinates of Q are
(a) (–6, –11) (b) (–9, –13) 2
(c) 3 x + 3 y 2 – 6x – 8 = 0
(c) (–10, –15) (d) (–6, –7) (d) None of these
14. A circle is drawn with centre at the focus S of the
parabola y2 = 4x so that a common chord of the æ 3ö
19. The normal at ç 2, ÷ to the ellipse,
parabola and the circle is equidistant from the è 2ø
focus and the vertex. Then the equation of the
x2 y 2
circle is + = 1 touch es a par abola, whose
16 3
9 9
(a) (x – 1)2 + y2 = (b) (x – 1)2 = – y2 equation is
4 16 (a) y2 = – 104 x (b) y2 = 14 x
9 9 (c) y = 26x
2
(d) y2 = – 14x
(c) (x – 1)2 + x2 = (d) (y – 1)2 + x2 = 20. The angle subtended by the common tangent of
4 16
15. Locus of all such points so that sum of its
( x - 4)2 y 2
distances from (2, – 3) and (2, 5) is always 10, is the two ellipse + = 1 and
25 4
( x - 2)2 ( y - 1)2 ( x + 1)2 y 2
(a) + =1 + = 1 at the origin is
25 9 1 4
( x - 2)2 ( y - 1)2 p p p p
(b) + =1
25 16 (a) (b) (c) (d)
2 4 3 6
Conic Sections 25

Numeric Value Answer 27. S1 and S2 be the foci of the hyperbola whose
transverse axis length is 4 and conjugate axis
21. Two equal chords AB and AC of the circle x2 + y2 length is 6, S3 and S4 be the foci of the conjugate
– 6x – 8y – 24 = 0 are drawn from the point hyperbola. If the area of the quadrilateral S1 S3
A ( )
33 + 3, 0 . Another chord PQ is drawn
S2 S4 is A, then find
A
.
intersecting AB and AC at points R and S, 13
respectively given that AR = SC = 7 and RB = AS 28. If the ratio of the area of equilateral triangles
= 3. The value of PR/QS is made of the common chord of the circles x2 + y2
22. If p and q be the longest and the shortest distance = 4 and x2 + y2 – 8x + 4 = 0 and their respective
respectively of the point (–7, 2) from any point pairs of tangents drawn from points on the
(a, b) on the curve whose equation
positive x- axis is 57 + 24 3 : k then k is
is x 2 + y 2 - 10 x - 14 y - 51 = 0 and G.M. of p
________.
and q is 2 k , then value k is _______. 29. P(a , b) is a points in the first quadrant. Circles
23. The straight line y = mx + c (m > 0) touches the are drawn through P touching the coordinate
parabolas y 2 = 8 (x + 2) then the minimum value axes, such that the length of common chord of
taken by c is these circle is maximum. If possible values of
24. Two tangents are drawn from a point (–2, –1) to a/b is k1 ± k2 2 then k1 + k2 is equal
the curve, y2 = 4x. If a is the angle between them,
to______.
then |tan a| is equal to:
2 x2 y 2
x2 y 30. C is the centre of the hyperbola - = 1,
25. Tangents are drawn to the ellipse + =1 4 1
9 5
and ' A ' is any point on it. The tangent at A to
at ends of latus rectum. The area of quadrilateral
so formed is the hyperbola meets the line x - 2 y = 0 and
26. A trapezium is inscribed in the parabola y2 = 4x x + 2 y = 0 at Q and R respectively. The value
such that its diagonal pass through the point of CQ.CR is equal to
25
(1, 0) and each has length . If the area of
4
éPù
trapezium be P then ê ú is equal to
ë4û

ANSW ER KEY
1 (a) 5 (b) 9 (b) 13 (d) 17 (d) 21 (1) 25 (27) 29 (5)
2 (b) 6 (b) 10 (b) 14 (a) 18 (c) 22 (11) 26 (4) 30 (5)
3 (c) 7 (b) 11 (c) 15 (d) 19 (a) 23 (4) 27 (2)
4 (d) 8 (b) 12 (c) 16 (a) 20 (a) 24 (3) 28 (9)
26 MATHEMATICS

12
LIMITS & DERIVATIVES

MCQs with One Correct Answer 5. The integer n for which


1. Given a real valued function 'f ' such that (cos x - 1)(cos x - e x )
lim is a finite non-zero
ì tan 2 {x} x®0 xn
ï 2 for x > 0 number is
2
ï x - [ x] (a) 1 (b) 2 (c) 3 (d) 4
ï
f ( x) = í
ï
1 for x = 0
6. The value of lim (sin x)
x ®0
( 1/ x
)
+ (1/ x)sin x ,
ï {x} cot{x} for x < 0 where x > 0 is
ï (a) 0 (b) –1 (c) 1 (d) 2
î
then æ px ö
tan
a ö èç 2a ø÷
(a) LHL = 1 7. The limit lim æç 2 - ÷ is equal to
(b) RHL = cot 1 x ®a è xø
lim f ( x ) exist
a 2a 2
(c) - - -
x®0 (a) e p (b)
e ep (c)(d) 1 p
(d) lim f ( x ) does not exists 8. If a and b are the roots of the quadratic
x®0
x equation ax 2 + bx + c = 0 , then
æ x 2 + 5x + 3 ö
2. lim ç ÷
x ®¥ çè x 2 + x + 2 ÷ø 1 - cos(cx 2 + bx + a )
lim =
(a) e4 (b) e2 (c) e3 (d) 1 1 2(1 - ax )2
2 x®
x x3 a
3. If 3 – £ f ( x) £ 3 + for all x ¹ 0, then the c æ 1 1ö c æ 1 1ö
12 9 (a) - (b) -
value of lim f ( x ) is equal to 2a èç a bø÷ 2b èç a bø÷
h®0
(a) 1/3 (b) 3 (c) –3 (d) –1/3 c æ 1 1ö
4. Given that (c) - (d) None of these
ab èç a bø÷
log ( n + r ) - log n
n
æ 1ö
lim
n®¥
å n
= 2 ç log 2 - ÷ ,
è 2ø cos2 (1- cos2 (1- cos2(............ - cos2(x))))..........)
r =1 9. lim
1 k k k 1/ n x®0 ìï æ x + 4 - 2ö üï
then lim k [(n + 1) (n + 2) ...(n + n) ] is sin ípç ÷ý
n®¥ n x
ïî è ø þï
equal to
is equal to
1/ k k k
4k æ 4ö æ 4ö æ eö p 4 p 2
(a) (b) çè ÷ø (c) ç ÷ (d) çè ÷ø (a) (b) (c) (d)
e e è eø 4 4 p 2 p
Limits & Derivatives 27
10. The value of 17. Let f(x) be a polynomial function satisfying
lim [1 1/ cos2 x
+2 1/ cos 2 x
+ ..... + n 1/ cos 2 x cos 2 x
] æ1ö æ1ö
f ( x ) × f ç ÷ = f ( x ) + f ç ÷ . If f(4) = 65 and
x®p / 2 è xø èxø
is
l1, l2, l3 are in GP, then f '(l1 ), f '(l2 ), f '(l3 )
n(n+1) are in
(a) 0 (b) n (c) ¥ (d)
2 (a) AP (b) GP
11. lim {log n -1 (n) log n (n + 1)log n +1( n + 2)... (c) HP (d) None of these
n®¥

...log (n x )} is equal to Numeric Value Answer


n k -1
(a) n (b) k 18. If f (a ) = 2, f ' (a) = 1 , g (a) = -1 , g ' (a) = 2 ,
(c) ¥ (d) None of these
12. If [x] denotes the greatest integer £ x , then g ( x) f ( a ) - g ( a ) f ( x )
then the value of lim is
lim
1
{[12 x] + [22 x] + [32 x] + ... + [n 2 x]} equals x®a x-a
n®¥ n 3 19. The value of lim (sinx)tanx is
(a) p
x / 2 (b) x/3 (c) x/6 (d) 0 x®
2
ì x
ï (e( x+ 3) ln 27 ) 27 - 9 é ù
20. The value of lim ê x + x + x - x ú is
ï ; x<3 x ®¥ ë û
13. If f ( x) = í 3x - 27
ï 1 - cos( x - 3) f (x)
21. Let f (x) be a function such that lim =1
ïl ; x>3 x®0 x
î ( x - 3) tan( x - 3) x (1 + a cos x ) - b sin x
and lim = 1, then b – 3a
If lim f ( x ) exist, then l = x ®0 { f ( x )3}
x ®3
is equal to
9 2 2
(a) (b) (c) (d) None 22. The largest value of non-negative integer a for
2 9 3
1- x
14. The value of
sec 2
p ìï - ax + sin ( x - 1) + a üï1- x 1
ì æ p öü 2 -bx which lim í ý = is
lim ísin 2 ç ÷ý is equal to x ®1 îï x + sin ( x - 1) - 1 þï 4
x ®0 î è 2 - ax ø þ
23. The value of lim ì x ü is
2
/ b2
(a) e-a / b (b) e-a ï 3 ï
x ®¥ ï x ï
íx+
4a / b
(c) a 2a / b (d) e 3 ý
ï x
f ( x) f (1 - cos x ) x+ ....¥ ï
15. If lim 2 = a and lim =b ïî x+3 x ïþ
x ®0 x x ®0 g ( x ) sin 2 x
24. If a, b are two distinct real roots of the
g (1 - cos 2 x)
(where b ¹ 0 ), then lim is equation ax3 + x - 1 - a = 0, (a ¹ -1, 0), none
x ®0 x4 of which is equal to unity, then the value of
4a a a
(a) (b) (c) (d) None (1 + a) x 3 - x 2 - a al ( k a - b)
b 4b b lim is . Find
16. Let f (x) be a polynomial function of second (e1-ax - 1)( x - 1)
x ®(1/ a ) a
degree. If f (1) = f ( -1) and a1, a2 , a3 are in A.P.. the value of kl.
then f '(a1 ), f '( a2 ), f '( a3 ) are in 3x + 33- x - 12
(a) A. P. (b) G. P. 25. lim is equal to _______.
x ®2 3- x /2 - 31- x
(c) H. P. (d) None of these

ANSW ER KEY
1 (d) 4 (c) 7 (c) 10 (b) 13 (c) 16 (a) 19 (1) 22 (2) 25 (36)
2 (a) 5 (c) 8 (a) 11 (b) 14 (a) 17 (b) 20 (0.50) 23 (1)
3 (b) 6 (c) 9 (b) 12 (b) 15 (c) 18 (5) 21 (6) 24 (1)
28 MATHEMATICS

13
MATHEMATICAL
REASONING

MCQs with One Correct Answer 7. (p Ù ~ q) Ù (~ p Ù q) is


1. For the statement “17 is a real number or a (a) A tautology
positive integer”, the “or” is (b) A contradiction
(a) Inclusive (b) Exclusive (c) Both a tautology and a contradiction
(c) Only (a) (d) None of these (d) Neither a tautology nor a contradiction
2. Let p and q be any two logical statements and 8. The false statement in the following is
r : p ® (: p Ú q) . If r has a truth value F, then (a) p Ù (~ p) is contradiction
the truth values of p and q are respectively : (b) (p Þ q) Û (~ q Þ ~ p) is a contradiction
(a) F, F (b) T, T (c) T, F (d) F, T (c) ~ (~ p) Û p is a tautology
3. If p : Ashok works hard (d) p Ú (~ p) Û p is a tautology
q : Ashok gets good grade 9. The conditional ( p Ù q) Þ p is
The verbal form for (~ p ® q) is (a) A tautology
(b) A fallacy i.e., contradiction
(a) If Ashok works hard then gets good grade
(c) Neither tautology nor fallacy
(b) If Ashok does not work hard then he gets (d) None of these
good grade 10. If p and q are two statements, then
(c) If Ashok does not work hard then he does
not get good grade ( p Þ q) Û (-q Þ~ p) is a
(d) Ashok works hard if and only if he gets (a) contradiction (b) tautology
grade (c) neither (a) nor (b) (d) None of these
4. If p is false and q is true, then 11. Which of the following is false?
(a) p Ù q is true (b) p Ú ~ q is true (a) p Ú ~ p is a tautology
(c) q Ù p is true (d) p Þ q is true (b) ~ (~p) « p is a tautology
5. ~ p Ù q is logically equivalent to (c) p Ù ~ p is a contradiction
(a) p ® q (b) q ® p (d) ((p Ù q) ® q) ® p is a tautology
12. In the truth table for the statement ( p ® q) «
(c) ~ (p ® q) (d) ~ (q ® p)
(~ p Ú q), the last column has the truth value in
6. Which of the following is a contradiction?
the following order is
(a) (p Ù q)Ù ~ (p Ú q) (b) p Ú (~ p Ù q) (a) TTFF (b) FFFF
(c) (p Þ q) Þ p (d) None of these (c) TTTT (d) FTFT
Mathematical Reasoning 29

13. If p Þ (~ p Ú q) is false, then truth values of p 21. The negation of (p Ú q)Ù (p Ú ~ r) is


(a) (~ p Ù ~ q) Ú (q Ù ~ r)
and q are respectively
(b) (~ p Ù ~ q) Ú (~ q Ù r)
(a) F. T (b) F, F
(c) (~ p Ù ~ q) Ú (~ q Ù r)
(c) T, T (d) T, F (d) (p Ù q) Ú (~ q Ù ~ r)
14. Negation of “2 + 3 = 5 and 8 < 10” is 22. Let p, q and r be any three logical statements.
(a) 2 + 3 ¹ 5 and < 10 (b) 2 + 3 = 5 and 8 </ 10 Which of the following is true?
(c) 2 + 3 ¹ 5 or 8 </ 10(d) None of these (a) ~ [ p Ù (~ q)] º (~ p) Ù q
15. If the compound statement p ® (~ p Ú q) is false
(b) ~ [( p Ú q ) Ù (~ r ) º (~ p) Ú (~ q) Ú (~ r )
then the truth value of p and q are respectively
(a) T, T (b) T, F (c) F, T (d) F, F (c) ~ [ p Ú (~ q)] º (~ p) Ù q
16. The contrapositive of p ® (~q ® ~r) is (d) ~ [ p Ú (~ q)] º (~ p)Ù ~ q
(a) (~ q Ù r) ® ~ p (b) (q ® r) ® ~p 23. Identify the false statements
(c) (q Ú ~r) ® ~ p (d) None of these (a) ~ [p Ú (~ q)] º (~ p) Ú q
17. The negation of the compound proposition (b) [p Ú q] Ú (~ p) is a tautology
p Ú (~ p Ú q) is (c) [p Ù q) Ù (~ p) is a contradiction
(d) ~ [p Ú q] º (~ p) Ú (~ q)
(a) ( p Ù ~ q ) Ù ~ p (b) ( p Ù ~ q) Ú ~ p
24. Negation of the statement (p Ù r) ® (r Ú q) is
(c) ( p Ú ~ q ) Ú ~ p (d) None of these (a) ~ (p Ù r) ® ~ (r Ú q)
18. The inverse of the statement (p Ù ~ q) ® r is (b) (~p Ú ~r) Ú (r Ú q)
(a) ~ (p Ú ~q) ® ~ r (b) (~p Ù q) ® ~ r (c) (p Ù r) Ù (r Ù q)
(c) (~p Ú q) ® ~ r (d) None of these (d) (p Ù r) Ù (~ r Ù ~q)
19. ~ ( (~ p) Ù q ) is equal to 25. Let A, B, C and D be four non-empty sets. The
(a) p Ú (~ q) (b) p Ú q contrapositive statement of “If A Í B and B Í D,
(c) p Ù (~ q) (d) ~ p Ù ~ q then A Í C ” is:
20. Which of the following is true? (a) If A Í C, then A Í B and B Í D
(a) p Þ q º ~ p Þ ~ q (b) If A Í C, then B Ì A or D Ì B
(b) ~ ( p Þ ~ q) º ~ p Ù q
~ (~ p Þ ~ q) º ~ p Ù q (c) If A Í C, then A Í B and B Í D
(c)
(d) ~ (~ p Û q) º [~ ( p Þ q)Ù ~ (q Þ p)] (d) If A Í C, then A Í B or B Í D

ANSW ER KEY
1 (a) 4 (d) 7 (b) 10 (b) 13 (d) 16 (a) 19 (a) 22 (c) 25 (d)
2 (c) 5 (d) 8 (b) 11 (b) 14 (c) 17 (a) 20 (c) 23 (d)
3 (b) 6 (a) 9 (a) 12 (c) 15 (b) 18 (c) 21 (c) 24 (d)
30 MATHEMATICS

14
STATISTICS

MCQs with One Correct Answer 5. The mean of n items is X . If the first item is
1. The weighted mean of first n natural numbers increased by 1, second by 2 and so on, the new
whose weights are equal to the number of mean is :
selections out of n n atural numbers of x
(a) X + (b) X + x
corresponding numbers is 2
n +1
n.2n-1 3n (n + 1) (c) X + (d) none of these
(a) (b) 2 (2n + 1) 2
2n - 1
6. The median of 100 observations grouped in
(n + 1) (2 n + 1) n (n + 1)
(c) (d) classes of equal width is 25. If the median class
6 2 interval is 20 - 30 and the number of observations
2. The A.M. of n observations is M. If the sum of less than 20 is 45, then the frequency of median
n – 4 observation is a, then the mean of remaining class is
4 observation is (a) 10 (b) 20 (c) 15 (d) 12
7. If the mean deviation of the numbers 1, 1 + d,
nM - a nM + a
(a) (b) 1 + 2d, .... 1 + 100d from their mean is 255, then d
4 2 is equal to :
nM – a (a) 20.0 (b) 10.1 (c) 20.2 (d) 10.0
(c) (d) nM + a 8. In a series of 2 n observations, half of them equal
2
a and remaining half equal –a. If the standard
3. The mean income of a group of persons is ` 400. deviation of the observations is 2, then | a |
Another group of persons has mean income equals
` 480. If the mean income of all the persons in
the two groups together is ` 430, then ratio of 2 1
(a) (b) 2 (c) 2 (d)
the number of persons in the groups is n n
n1 5 n1 2 9. The standard deviations of two sets containing
(a) = (b) = 10 and 20 members are 2 and 3 respectively
n2 3 n2 5 measured from their common mean 5. The SD for
n1 7 the whole set of 30 members is
(c) = (d) None of these
n2 4 2
(a) (b) 6
4. The mean of six numbers is 30. If one number is 3
excluded, the mean of the remaining numbers is
29. The excluded number is æ 22 ö
(a) 29 (b) 30 (c) 35 (d) 45
(c) ç 3 ÷ (d) 3
è ø
Statistics 31
10. If M. D. is 12, the value of S.D. will be the following statistical measures will not change
(a) 15 (b) 12 even after the grace marks were given ?
(c) 24 (d) None of these (a) mean (b) median
11. Suppose values taken by a variable x are such (c) mode (d) variance
that a £ xi £ b, where xi denotes the value of x in 17. Coefficient of variation of two distribution are
ith case for i = 1, 2, ... n. Then 60 and 70, and their standard deviations are 21
(a) a £ Var(x) £ b (b) a2 £ Var(x) £ b2 and 16, respectively. What are their arithmetic
means?
a2 (a) 35, 22.85 (b) 22.85, 35.28
(c) £ Var( x ) (d) (b – a)2 ³ Var(x)
4 (c) 36, 22.85 (d) 35.28, 23.85
12. If the median and the range of four numbers
{x, y, 2x + y, x – y}, where 0 < y < x < 2y, are 10 Numeric Value Answer
and 28 respectively, then the mean of the 18. Variance of the data 2, 4, 5, 6, 8, 17 is 23.33.
numbers is : Then, variance of 4, 8, 10, 12, 16, 34 will be
(a) 18 (b) 10 (c) 5 (d) 14 19. Coefficient of variation of two distributions are
13. Let X and M.D. be the mean and the mean 50 and 60 and their arithmetic means are 30 and
25, respectively. Then, difference of their
deviation about X of n observations xi, i = 1, 2, standard deviations is
........, n. If each of the observations is increased 20. In an experiment with 15 observations on x, the
by 5, then the new mean and the mean deviation following results were available:
about the new mean, respectively, are :
(a) X, M.D. (b) X + 5, M.D.
å x2 = 2830, å x = 170
One observation that was 20 was found to be
(c) X, M.D. + 5 (d) X + 5, M.D. + 5 wrong and was replaced by the correct value
14. The mean and variance for first n natural 30. The corrected variance is
numbers are respectively 21. Consider the following data
2 1, 2, 3, 4, 5, 6, 7, 8, 9, 10
n +1 n -1
(a) mean = , variance = If 1 is added to each number, then variance of
2 12 the numbers so obtained is
n -1 2
n +1 22. Coefficient of variation of two distribution are
(b) mean = , variance =
2 12 50% and 60% and their standard deviation are
10 and 15, respectively. Then, difference of their
n2 - 1 n +1
(c) mean = , variance = arithmetic means is
12 2 23. The mean of 5 observation is 4.4 and their
2 n -1
n +1 variance is 8.24. If three of the observations are
(d) mean = , variance =
2 2 1, 2 and 6, then difference of the other two
15. The variance of 20 observations is 5. If each observations is
observation is multiplied by 2, then the new 24. If the variance of the first n natural numbers is
variance of the resulting observation is 10 and the variance of the first m even natural
(a) 23 × 5 (b) 22 × 5 (c) 2 × 5 (d) 24 × 5 numbers is 16, then m + n is equal to _______.
16. All the students of a class performed poorly in 25. If the mean and variance of eight numbers 3, 7, 9,
Mathematics. The teacher decided to give grace 12, 13, 20, x and y be 10 and 25 respectively, then
marks of 10 to each of the students. Which of x × y is equal to ____.

ANSW ER KEY
1 (a) 4 (c) 7 (b) 10 (a) 13 (b) 16 (d) 19 (0) 21 (8.25) 24 (18)
2 (a) 5 (c) 8 (c) 11 (d) 14 (a) 17 (a) 20 (78) 22 (5) 25 (52)
3 (a) 6 (a) 9 (c) 12 (d) 15 (b) 18 (93.32) 23 (5)
32 MATHEMATICS

15
PROBABILITY-1

MCQs with One Correct Answer 2 1 1 2


1. If two numbers p and q are choosen randomly (a) (b) (c) (d)
9 5 9 5
from the set {1, 2, 3, 4} with replacement, then 6. 5 different games are to be distributed among 4
the probability that p 2 ³ 4q is equal to children randomly. The probability that each child
gets atleast one game is
1 3 1 7
(a) (b) (c) (d) 1 15 21 1
4 16 2 16 (a) (b) (c) (d)
2. A natural number x is chosen at random from the 4 64 64 4
first 100 natural numbers. Then the probability, 7. If three squares are chosen at random on a chess
board, then find the probability that they should
100 be in a diagonal line
for the equation x + > 50 is
x (a) 9/743 (b) 11/743
1 11 1 3 (c) 13/743 (d) None of these
(a) (b) (c) (d) 8. If the letters of the word MATHEMATICS are
20 20 3 20 arranged arbitrarily, the probability that C comes
3. A person throws two dice, one the common cube before E, E before H, H before I and I before S is
and the other regular tetrahedron with numbers
1, 2, 3, 4 on its faces, the number on the lowest 1 1 1 1
face being taken in the case of a tetrahedron.The (a) (b) (c) (d)
75 24 120 720
chance that the sum of numbers thrown is not 9. In a game of chance a player throws a pair of dice
less than 5 is and scores points equal to the difference between
1 3 4 5 the numbers on the two dice. Winner is the person
(a) (b) (c) (d) who scores exactly 5 points more than his opponent.
4 4 5 6
If two players are playing this game only one time,
4. A is a set containing n element. A subset P of A is
then the probability that neither of them wins is
chosen at random, and the set A is reconstruced
by replacing the elements of P. Another subset 1 1 53 107
Q of A is now chosen at random. The probability (a) (b) (c) (d)
54 108 54 108
that P È Q contains exactly r elements, 1 £ r £ n, 10. A box contains 2 fifty paise coins, 5 twenty five
is paise coins and 15 ten paise coins. Five coins
n
Cr 3r 3n are taken out of the box at random. Probability
(a) (b) that the value of these five coins is less than
4n 4n one rupee and fifity paise is
n
Cr 3n - r n
C r 3n 170 150
(c) n
(d) (a) 22 (b) 1 - 22
4 4n C5 C5
5. 10 persons sit around a circular table with 10
numbered chairs. The probability that the two 170
(c) 1- 22 (d) None of these
particular persons A and B are always together is C5
Probability-1 33
11. Two distinct numbers a and b are chosen 17. From a well-shuffled pack of 52 cards, four cards
randomly from the set {2, 22, 23, .... 225}. Then the are selected at random. The probability of draw-
probability that logab is an integer is ing exactly 2 spades and exactly 2 aces is
131 31 1494 1594
(a) (b) (a) (b)
300 300 270725 270725
21 62 1296 1396
(c) (d) (c) (d)
200 300 270725 270725
12. There are two vans each having numbered seats, Numeric Value Answer
3 in the front and 4 at the back. There are 3 girls
and 9 boys to be seated in the vans. The prob- 18. The probability that in the random arrangement
ability of 3 girls sitting together in a back row on of the letters of the word ‘UNIVERSITY’, the
adjacent seats, is two I’s does not come together is
19. A four digit number is formed by the digits 1, 2,
1 1 1 1 3, 4 with no repetition. The probability that the
(a) (b) (c) (d)
13 39 65 91 number is odd, is
13. For the three events A, B, and C, P (exactly one 20. Two numbers x and y are chosen at random
of the events A or B occurs) = P (exactly one of without replacement from amongst the numbers
the two events B or C occurs) = P(exactly one of 1, 2, 3, .........,3n. The probability that x3 + y3 is
the events C or A occurs) = p and P (all the three divisible by 3 is
events occur simultaneously) = p2, where 21. 5 girls and 10 boys sit at random in a row having
0 < p < 1/2. Then the probability of at least one 15 chairs numbered as 1 to 15. If the probability
of the three events A, B and C occurring is that the end seats are occupied by the girls and
2 2 between any two girls odd number of boys take
3p + 2p p + 3p
(a) (b) 20 n
2 4 seat is , then is equal to
n 1001
p + 3 p2 3 p + 2 p2 22. A quadratic equation is chosen from the set of
(c) (d) all quadratic equations which are unchanged by
2 4
14. There are three events E1, E2 and E3. one of which squaring their roots. If the chance that the chosen
must, and only one can happen. The odds are 7 p
to 4 against E1 and 5 to 3 against E2. The odds equation has equal roots, is , then p + q =
against E3 is q
(a) 4 : 11 (b) 3 : 8 (c) 23 : 88 (d) 65 : 23 23. In a multiple choice question, there are five
15. The chance of an event happening is the square alternative answers of which one or more than
of the chance of a second event but the odds one are correct. A candidate will get marks on the
against the first are the cube of the odds against question, if he ticks all the correct answers. If he
the second. The chances of the events are decides to tick answers at random, then the least
number of choice should he be allowed so that
1 1 1 1 the probability of his getting marks on the
(a) , (b) ,
9 3 16 4 1
question exceeds is
1 1 8
(c) , (d) None of these 24. A die is rolled three times, if p be the probability
4 2 of getting a large number than the previous
16. Ashmit and Bishmit are two students who number, then the value of 54p is
appeared in Class X exam, Probability that 25. A class contains 20 boys and 20 girls of which
Ashmit will pass is 0.5 while probability that half the boys and half the girls have cat eyes. If
both of them will pass is at least 0.1 and at most one student is selected from the class & if the
0.3, and probability that Bishmit will pass is P(B) probability that either the student is a boy or
then find the range of P(B)
(a) (0.2, 0.8) (b) (0.15, 0.25) a
has cat eyes is , then a 2 + b2 =
(c) (0.1, 0.8) (d) None of these b
ANSWER KEY
1 (d) 4 (b) 7 (d) 10 (c) 13 (a) 16 (c) 19 (0.50) 22 (3) 25 (5)
2 (b) 5 (a) 8 (c) 11 (b) 14 (d) 17 (a) 20 (0.33) 23 (4)
3 (b) 6 (b) 9 (c) 12 (d) 15 (a) 18 (0.80) 21 (3) 24 (5)
RELATIONS 16
& FUNCTIONS-2

MCQs with One Correct Answer 6. Let f : f : {x, y, z} ® {1, 2, 3} be a one-one


mapping such that only one of the following
1. Let P be the relation defined on the set of all real three statements is true and remaining two are
numbers such that P = {(a, b) : sec2a – tan2b false : f (x) ¹ 2, f (y) = 3, f (z) ¹ 1, then –
= 1}. Then P is: (a) f (x) > f (y) > f (z) (b) f (x) < f (y) < f (z)
(a) reflexive and symmetric but not transitive. (c) f (y) < f (x) < f (z) (d) f (y) < f (z) < f (x)
(b) reflexive and transitive but not symmetric. 7. Let A º {1,2,3,4} B º {a, b, c}, then number of
(c) symmetric and transitive but not reflexive. functions from A ® B, which are not onto is
(d) an equivalence relation. (a) 8 (b) 24 (c) 45 (d) 6
2. Consider the following relations. R = {(x, y) | x, y 8. A quadratic polynomial maps from [–2, 3] onto
are real numbers and x = wy for some rational [0, 3] and touches X-axis at x = 3, then the
polynomial is
æm pö
number w} S = {ç , ÷ | m, n, p and q are 3 2 3 2
èn qø (a) ( x - 6 x + 16) (b) ( x - 6 x + 9)
16 25
integers such that, q ¹ 0, n ¹ 0 and qm = pn}.
Then 3 2 3 2
(c) ( x - 6 x + 16) (d) ( x - 6 x + 9)
(a) Neither R nor S is an equivalence relation 25 16
(b) S is an equivalence relation but R is not an 9. Let f : R + ® {-1, 0, 1} defined by
equivalence relation
(c) R and S both are equivalence relations f ( x ) = sgn( x - x 4 + x 7 - x8 - 1) where sgn
(d) R is an equivalence relation but S is not an denotes signum function. Then f(x) is
equivalence relation (a) many-one and onto
3. For real numbers x and y, we define xRy iff (b) many-one and into
x - y + 5 is an irrational number. Then, (c) one-one and onto
relation R is (d) one-one and into
(a) reflexive (b) symmetric 10. Let f : R ® R be a function defined by
(c) transitive (d) None of these
4. If f : R ® S , defined by f ( x) = x + x2 , then f is
(a) injective (b) surjective
f ( x) = sin x - 3 cos x + 1, is onto, then the (c) bijective (d) None of these
interval of S is 11. If f (x) = sin x + cos x, g (x) = x2 – 1, then g (f (x))
(a) [ –1, 3] (b) [–1, 1] is invertible in the domain
(c) [ 0, 1] (d) [0, 3] é p pù
é pù
5. If the function f : (– ¥ ,¥) ® B defined by (a) ê0 , 2 ú (b) ê- 4 , 4 ú
f (x) = – x2 + 6x – 8 is bijective, then B = ë û ë û
(a) [1,¥) (b) (– ¥, 1] é p pù
(c) (– ¥, ¥) (d) None of these (c) ê- 2 , 2 ú (d) [0, p]
ë û
Relations & Functions-2 35
12. Let g : R ® R be given by g (x) = 3 + 4x. 17. Let f : [0, 1] ® R be an injective continuous
function that satisfies the condition
If g n ( x) = gogo....... og (x), then g-n (x) = –1 < f (0) < f (1) < 1.
Then, the number of functions g : [–1, 1] ® [0, 1]
(where g–n (x) denotes inverse of g n ( x) ) such that (gof ) (x) = x for all x Î [0, 1] is
(a) (4n - 1) + 4n x (b) ( x + 1)4- n - 1 (a) 0
(b) 1
(c) ( x + 1)4n - 1 (d) (4- n - 1) x + 4n (c) more than 1, but finite
(d) infinite
13. If f (x) = 3 | x | - x - 2 and g (x) = sin x, then
domain of definition of fog (x) is Numeric Value Answer
ì pü 18. Let E = {1, 2, 3, 4} and F = {1, 2}. Then the
(a) í2np + ý number of onto functions from E to F is
î 2 þ n ÎI
7p 11p ö 19. If for x > 0, f ( x ) = (a - x n )1/ n ,
æ
(b) U çè 2np +
6
, 2np +
6 ø
÷ g ( x) = x 2 + px + q; p, q Î R
n ÎI
and the equation g (x) – x = 0 has imaginary
ì 7p ü roots, then number of real roots of equation
(c) í 2 np + ý g (g (x)) – f (f (x)) = 0 is
î 6 þ n ÎI
ì -1, x < 0
p é 7p 11p ù ï
(d) {(4m + 1) : m ÎI} U ê2np + , 2np + 20. Let g(x) = 1 + x - [x] and f ( x ) = í 0, x = 0 .
2 nÎI ë
6 6 úû
ï 1, x > 0
æ pö æ pö î
14. If f ( x) = sin 2 x + sin 2 ç x + ÷ + cos x cos ç x + ÷ Then for all x, f (g(x)) is equal to
è 3ø è 3ø
æ5ö e x - e- x
and g ç ÷ = 1, then graph of y = g [f (x)] is 21. Let g ( x) = and g(f(x)) = x, then
è4ø 2
(a) a circle (b) a straight line æ e22 -1ö
f ç 11 ÷ - 5 =.
(c) a parabola (d) None of these ç 2e ÷
15. Let f (x) = x2 and g(x) = sin x for all x Î R. Then è ø
the set of all x satisfying (f o g o g o f) (x) = ìï - x + 1, x£0
(g o g o f) (x), where (f o g) (x) = f (g(x)), is 22. If f ( x) = í 2
, then the number
(a) ± np , n Î{0,1, 2,....} ïî-( x - 1) , x ³ 1
of solutions of f ( x) - f -1 ( x) = 0 is
(b) ± np , n Î{1, 2,....}
p æp ö æp ö
23. If f (x) = cos2 x + cos2 ç + x ÷ - cos x cos ç + x ÷
(c) + 2np, n Î{... - 2, -1, 0,1, 2....} è3 ø è3 ø
2
and g(3/4) = 2, then find the value of (gof) (1).
(d) 2np, n Î{... - 2, -1, 0,1, 2,....} 24. If a non-zero function f(x) is symmetrical about
16. Let f : [0, 1] ® [–1, 1] and g : [–1, 1] ® [0, 2] be y = x, then the value of p (constant) such that
two functions such that g is injective and
gof : [0, 1] ® [0, 2] is surjective. Then, f 2 ( x) = ( f -1 (x))2 - px × f ( x) × f -1( x) + 2x2 f ( x)
(a) f must be injective but need not be
surjective for all x Î R + is ....... .
(b) f must be surjective but need not be 25. Let f be defined on the natural numbers as follows :
injective f (1) = 1 and for n > 1, f (n) = f [f (n – 1)]
(c) f must be bijective 1 20
(d) f must be a constant function + f [n – f (n – 1)], the value of å f (r ) is
30 r =1
ANSWER KEY
1 (d) 4 (a) 7 (c) 10 (d) 13 (d) 16 (b) 19 (0) 22 (4) 25 (7)
2 (b) 5 (b) 8 (b) 11 (b) 14 (b) 17 (d) 20 (1) 23 (2)
3 (a) 6 (c) 9 (b) 12 (b) 15 (a) 18 (14) 21 (6) 24 (2)
17
INVERSE TRIGONOMETRIC
FUNCTIONS

MCQs with One Correct Answer 4. The range of the function


1. If ax + b (sec (tan–1 x)) = c and f (x) = sin -1 (log[ x ]) + log(sin -1[ x]); (where [.]
x+ y denotes the greatest integer function) is
ay + b (sec (tan–1 y)) = c, then = (a) R (b) [1, 2)
1 - xy
ì pü
(c) ílog ý (d) {– sin 1}
ac 2 ac î 2þ
(a) (b)
a 2 + c2 a-c 5. If x1, x2, x3, x4 are roots of the equation
2 ac a+c x4 – x3 sin 2b + x2 cos 2b – x cos b – sin b = 0, then
(c) 2 2 (d) 4
a -c 1 - ac
2. The range of the function å tan -1 xi is equal to
i =1
-1 é 2
1ù -1 é 2 1 ù
f (x) = sin êë x + 2 úû + cos êë x - 2 úû , where (a) p – b (b) p – 2b
[.] is the greatest integer function, is p p
(c) -b (d) - 2b
ìp ü ì 1ü 2 2
(a) í , pý (b) í0, - ý 6. The maximum value of f(x)
î2 þ î 2þ
æ pö ( )
æ 12 - 2 x 2 ö
(c) {p} (d) ç 0, ÷ = tan çç 4
–1 ÷ is
è 2ø x + 2x2 + 3 ÷
è ø
k
kp
3. If å cos-1 br = 2
for any k ³ 1 where (a) 18º (b) 36º
r =1 (c) 22.5º (d) 15º
k
7. The number of solutions of the equation
br ³ 0"r and A = å (b r ) r . Then
( x2 + 1)
r =1 2
tan -1 x = - 4 x 2 is
2 1/ 3 1/ 4
(1 + x ) - (1 - 2 x )
lim = (a) 2 (b) 3
x® A x + x2 (c) 4 (d) none of these
1 A p
(a) (b) 0 (c) (d)
2 2 2
Inverse Trigonometric Functions 37

æ ö when
1 -1 æ 1 ö
8. S = tan-1 ç ÷ + tan ç 2 ÷ + ...
2
è n + n +1ø è n + 3n + 3 ø 1
(a) a = –3 & b = 1 (b) a = 1 & b = -
æ ö 3
1
+ tan -1 ç ÷ , then tan S is
è 1 + ( n + 19)( n + 20) ø 1 1
equal to : (c) a= &b= (d) None of these
20
6 2
n
(a) (b)
401 + 20 n 2
n + 20n + 1 14. If Sn denotes the sum to n terms of the series
20 n
(c) (d) 7 19 39
2
n + 20n + 1 401 + 20 n cot–1 + cot –1 + cot –1 + ..... then
4 4 4
9. The number of roots of the equation n
(a) S n = tan –1
-1 1 -1 1 2n + 5
sin x- = cos is x-
sin -1 x cos -1 x n+5
(a) 0 (b) 1 (b) Sn = cot –1
2n
(c) 2 (d) 3
10. The minimum integral value of a for which the 4n
quadratic equation (cot–1 a)x2 – (tan–1 a)3/2 x + (c) S n = cot –1
2n + 5
2(cot–1 a)2 = 0 has both positive roots
(a) 1 (b) 2 1
(d) S¥ = cot –1
(c) 3 (d) 4 2
11. The sum of series 15. Find : tan (1/3) + tan–1(1/7) + tan–1 (1/13)
–1

10 50
sec -1 2 + sec-1 + sec -1 + . . . . + æ 1ö
3 7 +..... + tan -1 ç ÷ +....¥
2
è n + n +1ø

sec -1
( n2 + 1)( n2 - 2n + 2) is (a)
p
(b)
p
(c)
p
(d)
p
2 3 4 6
( n2 - n + 1)
2
16. If the equation x3 + bx2 + cx + 1 = 0 (b < c) has
only one real root a.
(a) tan–1 1 (b) tan–1 n Then the value of 2 tan –1 (cosec a) + tan –1
(c) tan–1 (n + 1) (d) tan–1 (n – 1) (2 sin a sec2 a) is:
12. The value of x satisfying the equation
p
(sin -1 x)3 - (cos-1 x)3 + (a) - (b) –p
2
p3 p
(sin -1 x)(sin -1 x - cos -1 x ) = (c) (d) p
16 2
is: p
17. Solution of equation cos -1 x 3 + cos -1 x =
p p 2
(a) cos (b) cos
5 4 is
p p 1 1
(c) cos (d) cos (a) x= (b) x= -
8 12 2 2
æ ö 1
a 2 a3 (c) x= ± (d) None of these
13. sin -1 ç a - + + .... ÷ 2
ç 3 9 ÷
è ø
p
+ cos -1 (1 + b + b2 + ...) =
2
38 MATHEMATICS
18. The sum of infinite series Numeric Value Answer
æ 1 ö -1 æ 2 - 1 ö -1 æ 3 - 2 ö
sin -1 ç ÷ + sin çç ÷ + sin çç ÷÷
1/ x
è 2ø ÷ lim æ π - tan -1 x ö
è 6 ø è 2 2 ø 21. equals
x ®¥ çè 2 ÷ø
æ n - n -1 ö 22. Find the value of
+... + sin -1 ç ÷ + ...
is
ç n ( n + 1) ÷
è ø æ1 ö
tan -1 ç (tan 2 A) + tan -1 (cot A) + tan -1 (cot 3 A) ÷ ,
è 2 ø
p p p
(a) (b) p (c) (d) p p
2 3 4 < A<
when
19. The value of 4 2
23. The number of ordered triplets (x, y, z) that satisfy
3 3 the equation
a æ1 aö b æ1 æ b öö
cosec2 ç tan-1 ÷ + sec2 ç tan -1 ç ÷ ÷
2 è 2 b ø 2 è 2 è a øø p2
(sin–1 x)2 = + (sec–1 y)2 + (tan–1 z)2 is
is equal to 4
(a) ( a - b ) ( a 2 + b2 ) (b) ( a + b ) ( a2 - b2 ) 24. Find the number of solution of
cos (2 sin–1(cot(tan–1(sec (6 cosec–1x))))) + 1 = 0;
( a + b ) ( a2 + b2 ) (d)
where x > 0.
(c) None of these
25. If the domain of the function
20. cot–1(2.12) + cot–1(2.22) + cot–1(2.32) +... is equal
to f(x) = 3cos -1 ( 4x ) - p is [a, b], then the value
p p p p of (4a + 64b) is _______.
(a) (b) (c) (d)
4 3 2 5

ANSWER KEY
1 (c) 4 (c) 7 (c) 10 (b) 13 (b) 16 (b) 19 (c) 22 (0) 25 (7)
2 (c) 5 (c) 8 (c) 11 (b) 14 (d) 17 (a) 20 (a) 23 (2)
3 (a) 6 (d) 9 (c) 12 (c) 15 (c) 18 (a) 21 (1) 24 (3)
Matrices 39

18
MATRICES

MCQs with One Correct Answer (a) I + 2p (b) 2I + P


1. The product of the matrices (c) I + P (d) None of these
4. For each real x : – 1 < x < 1. Let A (x) be the matrix
é cos 2 q cos q sin q ù
A=ê ú and
êë cos q sin q sin 2 q úû é 1 – xù x+ y
(1 – x)–1 ê ú and z = then
ë–x 1 û 1 + xy
é cos2 f cos f sin f ù
B=ê ú is a null matrix if (a) A (z) = A (x) + A(y)
êëcos f sin f sin 2 f úû
(b) A (z) = A (x) [A (y)]–1
q-f = (c) A (z) = A (x) A (y)
p (d) A (z) = A (x) – A (y)
(a) (2n + 1) (b) np
2
é1 0 ù
p 5. If A = ê1/ 2 1ú , then A50 is
n ë û
(c) 2np (d)
2
é - 3 2 ù é1 0 ù é1 0 ù é1 0 ù
é2 1ù (a) ê0 50 ú (b) ê50 1 ú
2. If ê ú A ê ú= ê ú , then the ë û ë û
ë 3 2û ë 5 - 3û ë0 1 û
matrix A equals
é1 25 ù
é1 1 ù é1 1ù (c) ê0 1 ú (d) None of these
(a) ê1 0ú (b) ê ú ë û
ë û ë0 1û
6. If A = [aij]m × n and aij = (i2 + j2 – ij) (j – i), n is odd,
é1 0ù é0 1ù then which of the following is not the value of
(c) ê1 1 ú (d) ê1 1ú
ë û ë û Tr(A)
(a) 0 (b) | A |
é 0 - tan (q / 2) ù (c) 2| A | (d) None of these
3. If matrix P = ê ú , then find
ë tan q / 2 0 û

é cos q - sin q ù
(I – P) ê ú
ë sin q cos q û
40 MATHEMATICS

10. Matrix A is such that A2 = 2A – I when I is unit


é 3 1 ù matrix then An is equal to
ê ú é1 1ù (a) nA – (n – 1) I (b) nA – I
2 2 ú
7. If P = ê and A = ê ú and Q = (c) 2n–1 A – I (d) none of these
ê 1 3ú ë 0 1û
ê- ú 11. The matrices
ë 2 2 û
é u1 v1 w1 ù é 2 2 1ù
PAPT and x = PTQ2005P then x is equal to ê ú 1 ê ú
P = ê u2 v2 w 2 ú ; Q = ê 13 – 5 m ú
é1 2005ù 9
êë u 3 v3 w 3 úû êë – 8 1 5 úû
(a) ê0 1 úû
ë
are such that PQ = I, an identity matrix. Solving
é 4 + 2005 3 6015 ù
(b) ê ú é u1 v1 w1 ù é x ù é1ù
ëê 2005 4 - 2005 3 úû ê ú êyú ê ú
the equation ê u 2 v 2 w 2 ú ê ú = ê 1 ú ,
êë u 3 v 3 w 3 úû êë z úû êë 5 úû
1 é2 + 3 1 ù
(c) ê ú the value of y comes out to be –3. Then the
4 êë - 1 2 - 3 úû
value of m is equal to
1 é 2005 2 - 3 ù (a) 27 (b) 7 (c) – 27 (d) – 7
(d) ê ú 12. If A, B are two square matrices such that AB = A
4 ëê2 + 3 2005 ûú
and BA = B, then
(a) only B is idempotent
é 0 2b c ù (b) A, B are idempotent
8. The value of a, b, c when êê a b - c úú is (c) only A is idempotent
êë a - b c úû (d) None of these
orthogonal, are 13. If A1, A3, ..., A2n–1 are n skew – symmetric matrices
of same order, then
1 1 1
(a) ± ,± ,± X = A1 + 3 A33 + ...(2 n - 1)(A 2n -1 )2 n -1 will be
2 6 2
(a) symmetric
1 1 1 (b) skew-symmetric
(b) ± ,± ,±
3 2 6 (c) neither symmetric nor skew symmetric
1 1 1 (d) depends on ‘n’ is even or odd
(c) ± ,± ,±
2 6 3 éa b c ù
ê ú
14. For A = ê d e f ú , a, b, c, d, e, f, g, h, i Î C, we
1 1 1
(d) ± ,± ,± ëê g h i úû
2 2 2

é 0 1 0ù éa d g ù
ê ú
9. If A = ê 0 0 1ú then A3 – rA2 – qA = q
say A = êb e h ú and we say that A is the
ê ú êc f i ú
êë p q r úû ë û
(a) p I (b) q I Hermitian matrix if A = Aq. Suppose A is the
(c) r I (d) None of these Hermitian matrix such that A2 = O then
(a) A = – A' (b) A = A'
(c) A = O (d) A = I
Matrices 41
15. A, B, C are three matrices of the same order such
that any two are symmetric and the 3rd one is éa b c ù
skew symmetric. If X = ABC + CBA and Y = 21. If matrix A = ê b c a ú where a, b, c are real
ê ú
ABC – CBA, then (XY)T is êë c a b úû
(a) symmetric (b) skew symmetric
(c) I – XY (d) – YX positive numbers, abc = 1 and ATA = I, then find
the value of a3 + b3 + c3.
écos q sin q ù é 1 0ù
16. If A = ê ú , B = ê –1 1ú , C = ABA ,
T
-1 + 3i
ë sin q – cos q û ë û 22. Let z = , where i = -1 , and r, s Î
2
then ATCnA equals to (n Î I+)
é (- z)r z 2s ù
é – n 1ù é1 – n ù
(a)
ê 1 0ú
(b) ê0 1 ú {1, 2, 3}. Let P = ê 2s ú and I be the
ë û ë û êë z z r úû
identity matrix of order 2. Then the total number
é0 1 ù é 1 0ù
(c) ê1 – n ú (d) ê – n 1ú of ordered pairs (r, s) for which P2 = –I is
ë û ë û
17. Let A be a 3 × 3 matrix given by A = (aij)3 × 3. If for é 1 0 0ù
ê ú
every column vector X satisfies X 'AX = 0 and a12 23. Let P = ê 4 1 0 ú and I be the identity matrix
= 2008, a13 = 1010 and a23 = – 2012. Then the êë16 4 1 úû
value of a21 + a31 + a32 = of order 3. If Q = [qij] is a matrix such that P50 – Q
(a) – 6 (b) 2006
(c) – 2006 (d) 0 q 31 + q32
=I, then equals
103 q 21
Numeric Value Answer
24. Let X be the solution set of the equation Ax = I;
18. If B, C are square matrices of order n and if é2 –1 ù
where A = ê and I is the corresponding
A = B + C, BC = CB, C2 = 0, then for any positive
ë3 – 2 úû
integer N, A N + 1 = BK[B + ( N + 1) C], then K/N is
unit matrix and x Î ¥ , then find the minimum
19. A and B are two square matrices such that A2B =
BA and if (AB)10 = AkB10, then k is ì np ü
20. If the product of n matrices value of S(cos x q + sin x q), qΡ – í ; n ΢ ý .
î2 þ
é1 1 ù é1 2 ù é1 3 ù é1 n ù
ê 0 1ú ê 0 1ú ê 0 1ú ... ê 0 1ú is equal to the é0 a ù éa b ù
ë ûë ûë û ë û 25. Let A = ê ú and (A + I) – 50A =
50
êc d ú ,
ë0 0 û ë û
é1 378ù find a + b + c + d.
matrix ê ú , then the value of n is equal to
ë0 1 û

ANSWER KEY
1 (a) 4 (c) 7 (a) 10 (a) 13 (b) 16 (d) 19 (1023) 22 (1) 25 (2)
2 (a) 5 (d) 8 (c) 11 (d) 14 (c) 17 (c) 20 (27) 23 (1)
3 (c) 6 (d) 9 (a) 12 (b) 15 (d) 18 (1) 21 (4) 24 (2)
19
DETERMINANTS

MCQs with One Correct Answer (a) 0 (b) positive


(c) negative (d) can not be determined
1. The value of the determinant 5. Consider three points
1 a a2 P = ( - sin(b - a ), - cos b),
cos (n –1) x cos nx cos (n + 1) x Q = (cos(b - a ),sin b) and
is zero, if
sin (n –1) x sin nx sin (n + 1) x
R = (cos(b - a + q),sin(b - q)) , where
(a) sin x = 0 (b) cos x = 0
p
1 + a2 0 < a, b, q < . Then,
(c) a = 0 (d) cos x = 4
2a (a) P lies on the line segment RQ
2. If a, b, c, are the sides of a triangle ABC such (b) Q lies on the line segment PR
(c) R lies on the line segment QP
a2 b2 c2 (d) P, Q, R are non-collinear
that (a + 1) 2 (b + 1)2 (c + 1) 2 = 0 , then
(a - 1)2 (b - 1)2 (c - 1)2 r 2r - 1 3r - 2
(a) DABC is non-isosceles right-angled triangle n
6. If D r = n -1 a
(b) D ABC is an equilateral 2
(c) D ABC is an acute angled triangle with no 1 1
two angles being equal n(n - 1) (n - 1)2 (n - 1)(3n - 4)
2 2
(d) D ABC is an isosceles
3. If a, b, c, are the sides of a triangle A B C and A, n -1
B, C are angles opposite to a, b, c, and then the value of å Dr
2 r =1
a b sin A c sin A (a) depends only on a
D = b sin A 1 cos A (b) depends only on n
then, (c) depends both on a and n
c sin A cos A 1
(d) is independent of both a and n
(a) D = area of triangle 7. If a2 + b2 + c2 + ab + bc + ca £ 0 " a, b,
(b) D = perimeter of the triangle c Î R, then value of the determinant,
(c) D = Sa2
(d) None of these ( a + b + 2)2 a 2 + b2 1
4. If P, Q and R represent the angles of an acute 2
angled triangle, no two of them being equal then 1 (b + c + 2) b + c2
2

1 1 + sin P sin P(1 + sin P) c2 + a2 1 (c + a + 2) 2


1 1 + sin Q sin Q(1 + sin Q) equals
the value of is (a) 65 (b) a2 + b2 + c2 + 31
1 1 + sin R sin R(1 + sin R) (c) 4(a2 + b2 + c2) (d) 0
Determinants 43
8. If A is matrix of order 3 such that |A| = 5 and B = (a + a) x + ay + az = 0; ax + (a + b) y + az = 0;
adj A, then the value of |A –1 T
| ( AB) is equal ax + ay + (a + c ) z = 0 has a non-trivial
solution if :
to (where |A| denotes determinant of matrix A.AT
denotes transpose of matrix A, A– 1 denotes inverse (a) a -1 = -(a -1 + b-1 + c -1 )
of matrix A. adj A denotes adjoint of matrix A) (b) a -1 = a + b + c
1 (c) a + a + b + c = 1
(a) 5 (b) 1 (c) 25 (d)
25 (d) None of these
9. If f(x), h(x) are polynomials of degree 4 and 14. If the system of equations ax + y + z = 0,
x + by + z = 0 and x + y + cz = 0 (a, b, c ¹ 1) has a
f ( x ) g ( x ) h( x ) non-trivial solution, then the value of
a b c = mx4 + nx3 + rx2 + sx + t be 1 1 1
+ + is
p q r 1– a 1– b 1– c
an identity in x, then (a) –1 (b) 0
f "'(0) – f "(0) g "'(0) – g "(0) h "'(0) – h "(0) (c) 1 (d) None of these
a b c is 15. If a, b, c are non-zeros, then the system of equa-
p q r tions (a + a)x + ay + az = 0, ax + (a + b)y + az
= 0, ax + ay + (a + c)z = 0 has a non-trivial
(a) 2(3n – r) (b) 2(2n – 3r) solution if
(c) 3(n – 2r) (d) None of these
(a) a– 1 = – (a– 1 + b– 1 + c– 1)
1 1 1 (b) a– 1 = a + b + c
10. If a b c = (a – b) (b – c) (c – a) (a + b + c), (c) a + a + b + c = 1
(d) None of these
a 3 b3 c 3
16. If c < 1 and the system of equations x + y – 1 = 0,
where a, b, c are all different, then the determinant 2x – y – c = 0, and –bx + 3by – c = 0 is consistent,
1 1 1 then the possible real values of b are
æ 3ö
(b) b Î æç – , 4 ö÷
( x – a)2 ( x – b) 2 ( x – c) 2 3
(a) b Î ç – 3, ÷
è 4ø è 2 ø
( x – b)( x – c ) ( x – c )( x – a) ( x – a)( x – b)
(c) b Î æç – ,3ö÷
3
vanishes when (d) None of these
è 4 ø
1
(a) a + b + c = 0 (b) x = (a + b + c) 1
3 x n -1 cos x
1 x+3
(c) x = (a + b + c) (d) x = a + b + c 17. If f(x) = np ( -1)n n ! then
2 0 cos
2 2 3n +1
11. If t is real and l = t - 3t + 4 , then the number 3
t 2 + 3t + 4 a a a5
dn
of solutions of the system of equations [f ( x )]x =0 =
3x – y + 4z = 3, x + 2y – 3z = – 2, 6x + 5y +lz = –3 is : dx n
(a) one (b) two (c) zero (d) infinite (a) 1 (b) –1
12. The set of homoegeneous equations : (c) 0 (d) None of these
tx + (t + 1) y + (t - 1) z = 0; a1 b1 c1
(t + 1) x + ty + (t + 2) z = 0; 18. Suppose D = a2 b2 c2 and
(t - 1) x + (t + 2) y + tz = 0 a3 b3 c3
has non-trivial solution for
(a) three values of t (b) two values of t a1 + pb1 b1 + qc1 c1 + ra1
(c) one value of t (d) no value of t D¢ = a2 + pb2 b2 + qc2 c2 + ra2 . Then
13. If a, b, c are non-zeros, then the system of
equations : a3 + pb3 b3 + qc3 c3 + ra3
44 MATHEMATICS
(a) D¢ = D (b) D¢ = D(1 – pqr) 24. If a1, a2, a3, ..., an are in G.P. and ai > 0 for each i,
(c) D¢ = D(1+p+q+r) (d) D¢ = D(1 + pqr) then the determinant
é x 2 –1ù log a n log an + 2 log an + 4
ê ú æ –11ö
19. If A = ê –1 1 2 ú ; çè x ¹ ÷ and det (adj(adj D = log an + 6 log an + 8 log an + 10 is equal to:
3 ø
êë 2 –1 1úû
log an + 12 log an + 14 log an + 16
A)) = (14)4. Then the value of x is
(a) 2 (b) – 2 25. Area of triangle whose vertices are (a, a2) (b, b2)
(c) 0 (d) None of these 1
(c, c2) is , and area of another triangle whose
é x 3 2 ù 2
ê1 y 4 ú vertices are (p, p2) , (q, q2) and (r, r2) is 4, then the
20. Matrix A = ê ú , if x y z = 60 and 8x + 4y + 3z
êë 2 2 z úû (1 + ap)2 (1 + bp)2 (1+ cp)2
= 20, then A(adj A) is equal to value of (1 + aq)2 (1 + bq)2 (1+ cq)2 is
é 64 0 0ù é88 0 0ù
ê ú (1 + ar )2 (1 + br )2 (1 + cr )2
(a) ê 0 64 0 ú (b) ê 0 88 0ú
ê ú
êë 0 0 88úû x sin x cos x
ëê 0 0 64ûú f ¢( x)
2
é 68 0 0 ù é34 0 0 ù 26. If f ( x) = x - tan x - x3 then lim
x ®0 x
ê 0 68 0 ú ê 0 34 0 ú 2 x sin 2 x 5x
(c) ê ú (d) ê ú
êë 0 0 68úû êë 0 0 34 úû is equal to
27. If a, b are the roots of the equation x2 – 2x + 4 = 0,
find the value of
Numeric Value Answer
21. The value of the determinant a+b a2 + b2 a 3 + b3
æ 3p ö æ 5p ö æ 7p ö a 2 + b2 a 3 + b3 a4 + b4
sin2 ç x + ÷ sin2 ç x + ÷ sin2 ç x + ÷
è 2ø è 2ø è 2ø
a 3 + b3 a 4 + b 4 a 5 + b5
æ 3p ö æ 5p ö æ 7p ö 28. Find the co-efficient of x in the expansion of the
sin ç x + ÷ sin ç x + ÷ sin ç x + ÷ is
è 2ø è 2ø è 2ø determinant
æ 3p ö æ 5p ö æ 7p ö (1 + x) 2 (1 + x )4 (1 + x)6
sin ç x - ÷ sin ç x - ÷ sin ç x - ÷
è 2ø è 2ø è 2ø
(1 + x )3 (1+ x)6 (1 + x)9 .
ìæ a a ö ü
22. Let S = íç 11 12 ÷ : aij Î{0,1, 2}, a11 = a22 ý (1 + x) 4 (1 + x )8 (1 + x)12
îè a21 a22 ø þ
Then the number of non-singular matrices in the éa b ù
set S is : 29. Suppose A = ê ú is a real matrix with non-
ëc d û
é3; when $i = $j zero entries, ad – bc = 0 and A2 = A. Then, a + d
23. Let A = [aij]3 × 3 be such that aij = ê $i ¹ $j ,
equal to
ëê 0; 30. The number of integers x satisfying
ì det (adj(adj A)) ü é1 x x2 ù
then í ý equals: (where {×}
î 5 þ -3x 4 + det ê1 x 2 x 4 ú = 0 is equal to
ê1 x3 x 6 úû
denotes fractional part function) ë
ANSWER KEY
1 (a) 5 (d) 9 (a) 13 (a) 17 (c) 21 (0) 25 (16) 29 (1)
2 (d) 6 (d) 10 (b) 14 (c) 18 (d) 22 (20) 26 (4) 30 (2)
3 (d) 7 (a) 11 (a) 15 (a) 19 (d) 23 (0.20) 27 (0)
4 (d) 8 (b) 12 (c) 16 (c) 20 (c) 24 (0) 28 (0)
CONTINUITY AND 20
DIFFERENTIABILITY

MCQs with One Correct Answer continuous satisfying f '(1) = f (–1), then g (x) is
(a) (1 + sin 1) x +1 (b) (1– sin1) x + 1
2 (c) (1– sin 1) x –1 (d) (1 + sin 1) x – 1
1. The function f ( x ) = (sin 2 x ) tan 2x
is not
5. Let f (x) be defined as follows :
p æ pö
defined at x = . The value of f ç ÷ so that f æ
4 è 4ø p
ç (cos x - sin x)cosec x , - <x<0
ç 2
p
is continuous at x = is f ( x) = ç a , x=0
4 ç
ç 1/ x 2/ x 3/ x
e +e +e p
1 ç , 0< x<
(a) e (b) è ae 2/ x
+ be 3/ x 2
e
(c) 2 (d) None of these If f (x) is continuous at x = 0, then (a, b) =

ì1 - [ x] æ 1ö æ1 ö
ï , x ¹ -1 (a) çè e, ÷ø (b) ç , e÷
2. If f (x) = í 1 + x , then the value of e è ø e
ïî1 , x = -1
(c) (e, e) (d) ( e -1 , e -1 )
f ( 2 k ) will be (where [] shows the greatest
6. The function f defined by
integer function]
ìï (1 + sin p x )t - 1 üï
(a) Continuous at x = –1 f ( x) = lim í
t ý is
(b) Continuous at x = 0 1 t ®¥ ïî (1 + sin p x ) + 1 þï
(c) Discontinuous at x =
(d) All of these 2 (a) every where continuous
3. Suppose 'f ' is continuous function from R to R (b) discontinuous at all integer values of x
and f ( f (a)) = a for some a Î R then the equation (c) continuous at x = 0
f (x) = x has (d) None of these
(a) no solution 7. Let f : R ® R be a function defined by
(b) exactly one solution
(c) at most one solution f (x) = min {x + 1, x + 1} ,then which of the
(d) atleast three solutions following is true?
4. Let g (x) be a polynomial of degree one and f (x) (a) f (x) is differentiable everywhere
(b) f (x) is not differentiable at x = 0
ìï g ( x), x £ 0
be defined by f ( x) = í sin x . If f (x) is (c) f (x) ³ 1 for all x Î R
ïî x ,x>0 (d) f (x) is not differentiable at x = 1
46 MATHEMATICS

dy
( x – 1) n 14. If 3 3
(1 - x 6 ) + (1 - y 6 ) = a(x – y ), and dx
8. Let g ( x) = ; 0 < x < 2, m and n
log cos m ( x – 1)
æ 6ö
are integers, m ¹ 0, n > 0 , and let p be the left = f (x, y) ç 1 - y6 ÷ , then
è1- x ø
hand derivative of |x – 1| at x = 1. If lim g(x) = p,
x ®1+ (a) f (x, y) = y /x (b) f (x, y) = y2 / x2
then (c) f (x, y) = 2y / x (d) f (x, y) = x2 / y2
2 2
(a) n = 1, m = 1 (b) n = 1, m = – 1 15. If f (x) = (1 + x)n, then the value of
(c) n = 2, m = 2 (d) n > 2, m = n
f ''(0) f n (0)
9. Let f: R ® R be a function such that f ( x ) £ x 2 , f (0) + f '(0) + + .... + is
2! n!
for all x Î R . Then, at x = 0, f is: (a) n (b) 2n
(a) continuous but not differentiable. (c) 2n– 1 (d) None of these
(b) continuous as well as differentiable. 16. If a and b are any two roots of equation ex cos
(c) neither continuous nor differentiable. x = 1, then the equation ex sin x – 1 = 0 has
(d) differentiable but not continuous. (a ) exactly one root in (a, b)
æ xx – x – x ö (b) exactly two roots in (a, b)
10. If f (x) = cot–1 ç
2 ÷ , then f ¢ (1) is
è ø (c) at least one root in (a, b)
(a) – 1 (b) 1
(d) no root in (a, b)
(c) log 2 (d) – log 2
17. Let f(x) = x |sin x|, x Î R. Then
1 (a) f is differentiable for all x, except at
11. If g is the inverse of f and f ' (x) = , then
1 + x3 x = np, n = 1, 2, 3, ....,
find g' (x). (b) f is differentiable for all x, except at
(a) 1 + [g(x)]2 (b) 1 – [g(x)]2 x = np, n = ±1, ±2, ±3, ...
(c) 1 + [g(x)] 3 (d) 1 – [g(x)]3 (c) f is differentiable for all x, except at
x = np, n = 0, 1, 2, 3
æp 2ö (d) f is differentiable for all x, except at
12. If f ( x) = sin ç [ x] - x ÷ , where 2 < x < 3 and
è 3 ø x = np, n = 0, ± 1, ± 2, ± 3, ...
[ . ] represetns greatest integer function, then 18. f : [–1, 1] ® R be a function defined by
æ 5p ö ì 2 æp ö
f ¢ç is equal to ï x cos ç ÷ for x ¹ 0
ç 3 ÷÷ f ( x) = í è xø
è ø
ï 0 for x = 0
5p 5p î
(a) (b) 2 The set of points where f is not differentiable is
3 3
(a) {x Î [–1, 1] : x ¹ 0}
5p 5p
(c) -2 (d) - ì 2 ü
3 3 (b) í x Î[-1, 1] : x = 0or x = , n ÎZý
î 2 n + 1 þ
13. If 1 - x 2n + 1 - y 2n = a( x n - y n ) , then
ì 2 ü
(c) í x Î[ -1,1] : x = , n ÎZ ý
î 2n + 1 þ
1 - x 2 n dy
is equal to (d) [–1, 1]
1 - y 2 n dx
19. Let f : R ® R be a continuous function such that
(a) 1 (b) x/y f (x2) = f (x3) for all x Î R. Consider the following
statements.
x n -1
(c) (d) None of these
y n -1
Continuity and Differentiability 47
I. f is an odd function.
II. f is an even function. 26. If y = (1 + 1/ x ) x then find the value of
III. f is differentiable everywhere 2 y2 (2) + 1/ 8
Then
(a) I is true and III is false log 3/ 2 - 1/ 3
(b) II is true and III is false
(c) both I and III are true 27. If y = ( x - sin x ) + ( x - sin x ) + ..., then
(d) both II and III are true
20. Let f : R ® R be function defined by
2
ì sin x 2
ï ( )
if x ¹ 0,
dx
dy p
- 2p = .........
f (x) = í x x=
2
ï
î 0 if x = 0,
ìa cot x b
Then, at x = 0, f is ïï x + x 2 , 0 <| x |£ 1
(a) not continuous 28. Let f ( x) = í
(b) continuous but not differentiable ï1 x=0
(c) differentiable and the derivative is not ïî 3
continuous If f (x) is continuous at x = 0, then the value of
(d) differentiable and the derivative is a2 + b2 is
continuous
29. Number of points where function f (x) defined
Numeric Value Answer as f :[0, 2p ] ® R, f ( x)
21. If f is a real valued differentiable function
ì 1 1
satisfying | f (x) – f (y) | £ ( x - y )2 , x, y Î R and ï3 - cos x - , | sin x |<
f (0) = 0, then f (1) is equal to ï 2 2
=í is
22. If x = cosec q - sin q ; y = cosecn q- sin n q , ï 2 + cos x + 1 1
, | sin x |³
ï 2 2
æ dy ö
2 î
and ( x2 + 4) ç ÷ - n 2 y 2 = kn2, then value of
è dx ø non-differentiable is
k is 30. If the derivative of the function
23. If f (x) = cos x cos 2x cos 22 x cos 23 x ..... ì 1 ü
æ pö
f ( x ) = cos-1 í (2 cos x - 3sin x) ý
cos 2 n–1 x and n > 1, then f ' çè 2 ÷ø is î 13 þ
24. If f '' (x) = – f (x) and g (x) = f ' (x) and ì 1 ü
+ sin -1 í (2 cos x + 3sin x) ý
2 2 î 13 þ
æ æ x öö æ æ x öö
F ( x) = ç f ç ÷ ÷ + ç g ç ÷ ÷ and given that 3 10
è è 2 øø è è 2 øø w.r.t. 1 + x 2 at x = is , then b =
F (5) = 5, then F (10) is equal to 4 b
25. Number of functions f : [0, 1] ® [0, 1] satisfying |
f (x) – f (y) | = |x – y| for all x, y in [0, 1] is

ANSWER KEY
1 (b) 5 (b) 9 (b) 13 (c) 17 (b) 21 (0) 25 (2) 29 (4)
2 (d) 6 (b) 10 (a) 14 (d) 18 (c) 22 (4) 26 (3) 30 (3)
3 (d) 7 (a) 11 (c) 15 (b) 19 (d) 23 (1) 27 (3)
4 (b) 8 (c) 12 (b) 16 (c) 20 (d) 24 (5) 28 (2)
APPLICATION OF 21
DERIVATIVES

MCQs with One Correct Answer x x


5. If f (x ) = and g(x) = , where 0 < x £ 1,
1. The function f (x) = tan–1(sin x + cos x) is an sin x tan x
then in this interval
increasing function in
(a) both f(x) and g(x) are increasing functions
æ pö æ p pö (b) both f(x) and g(x) are decreasing functions
(a) çè 0, ÷ø (b) ç - , ÷ (c) f(x) is an increasing function
2 è 2 2ø
(d) g(x) is an increasing function.
æ p pö p p æ p pö
(c) çè , ÷ø (d) æç - , ö÷ 6. Let the function g : ( -¥, ¥) ® ç - , ÷ be
4 2 è 2 4ø è 2 2ø
-1 u p
2. The greatest of the numbers given by g (u ) = 2 tan (e ) - . Then, g is
2
1, 21 / 2 , 31 / 3 , 41 / 4 , 51 / 5 , 61 / 6 and 71 / 7 is (a) even and is strictly increasing in (0, ¥ )
(b) odd and is strictly decreasing in ( -¥, ¥)
(a) 21 / 2 (b) 31 / 3
(c) odd and is strictly increasing in ( -¥, ¥)
(c) 71 / 4 (d) All but 1 are equal (d) neither even nor odd, but is strictly
3. The interval of decrease of the function increasing in ( -¥, ¥)
7. The largest term in the sequence,
f ( x) = x 2 log 27 - 6 x log 27
n2
2 2
+ (3x - 18 x + 24) log( x - 6 x + 8) is an = is
n3 + 200
(a) (3 - 1 + 1/ 3e , 2) È (4,3 + 1 + 1/ 3e ) (a) a 6 (b) a 7
(c) a 8 (d) None of these
(b) (3 - 1 + 1/ 3e , 3 + 1 + 1/ 3e )
x2 + 1
(c) (-¥, 3 - 1 + 1/ 3e ) È (3, 4 + 1 + 1/ 3e ) 8. Let f (x) be a function given by f ( x) =
[ x]
(d) None of these. for all x Î [1, 4], where [.] denotes the greatest
4. The set of positive values of the parameter ‘a’ integer function. then, f (x) is monotonically.
for each of which the function (a) increasing on [1, 4)
(b) decreasing on [1, 4)
f ( x) = sin 2 x - 8(a + 1)sin x - (4a 2 + 8a - 14) x (c) increasing on [1, 2)
is monotonic increasing in R and has no (d) decreasing on [2, 3)
critical points are 9. If A > 0, B > 0 and A + B = p/3, then the maximum
value of tan A tan B is
(a) (0, 6 - 2) (b) (-2 - 6, 6 - 2)
1 1
(c) (-2 - 6, 0) (d) None of these (a) (b) (c) 3 (d) 3
3 3
Application of Derivatives 49
10. The set of values for which the function 15. Two men P and Q start with velocities v at the
x x same time from the junction of two roads inclined
f ( x) = (4a - 3)( x + ln 5) +2( a - 7) cot sin 2 at 45o to each other. If they travel by different
2 2
does not possess critical points is roads, the rate at which they are being separated
is
æ 4ö
(a) ç -¥, - ÷ È (2, ¥) (a) v 2 (b) v 2+ 2
è 3ø
(b) (-¥, 2) (c) v 2- 2 (d) v/ 2
(c) [1, ¥) 16. The general value of a such that the line x cos a+
(d) (1, ¥) y sin a = p is a normal to the curve (x + a) y = c2
11. A given right circular cone has a volume p, and is
the largest right circular cylinder that can be
p 3p
inscribed in the cone has a volume q. then p : q is (a) æç 2np + ,(2n +1)p ö÷ È æç 2np + ,(2n + 2)p ö÷
(a) 9 : 4 (b) 8 : 3 è 2 ø è 2 ø
(c) 7 : 2 (d) None of these
ìï 2 - | x 2 + 5 x + 6 |; æ 3p ö
12. f ( x) = í
x ¹ -2 (b) ç 2np + p , 2np + ÷
è 2 ø
2
ïî a + 1 ; x = -2
then the range of a is so, that f(x) has maxima (c) æ 3p ö
at x = – 2 is ç 2np + ,(2n + 2)p ÷
è 2 ø
(a) | a |³ 1 (b) | a |< 1
(c) a >1 (d) a <1 æ p ö
(d) ç 2np , 2np + , ÷ È
p p è 2 ø
13. Let f ( x) = sin 3 x + l sin 2 x , - <x< .
2 2
æ 3p ö
In order that f ( x) has exactly one minimum, ç (2n + 1)p , 2np + , ÷ (n Î I )
è 2 ø
l should belong to
æ 3 ö æ 3ö 17. Let P be a point on the hyperbola x 2 - y 2 = a 2 ,
(a) (–1, 1) (b) ç - , 0 ÷ È ç 0, ÷
è 2 ø è 2ø where a is a parameter, such that P is nearest to
æ 3 1 ö æ 1 3ö the line y = 2x. Then the locus of P is
(c) (0, ¥) (d) ç - , - ÷ Èç , ÷
è 2 2 ø è 2 2ø (a) y = 2x (b) y = x
p (c) 2y = x (d) x + y = 0
14. The function f (x) = 1 + x (sin x) [cos x], 0 < x £
2 18. For the curve y = 3 sinq cosq, x = eq sin q,
(where [ . ] is G.I.F.) 0 £ q £ p, the tangent is parallel to x-axis when q is:

(a) is continuous on æç 0, pö
÷ 3p p p p
è 2ø (a) (b) (c) (d)
4 2 4 6
æ pö
(b) is strictly increasing in ç 0, ÷ 19. Let C be the curve y3 – 3xy + 2 = 0. If H is the set
è 2ø
of points on the curve C, where the tangent is
(c) is strictly decreasing in æç 0, p ö÷ parallel to x-axis and V is the set of points on
è 2ø
C where the tangent is parallel to y-axis, then
(d) has global maximum value 2
50 MATHEMATICS

(a) H = {(x, y): y = 0, x Î R}, V = {(1, 1)} 25. Let P be a non-zero polynomial such that
(b) H = {(x, y) : x = 0, y Î R}, V = {(1, 1)} P(1 + x) = P(1- x) for all real x, and P(l) = 0. Let m
(c) H = f, V = {(1, 1)} be the largest integer such that (x - l)m divides
(d) H = {(1, 1)} , V = {(x, y) : y = 0, x Î R} P(x) for all such P(x). Then m is equal to
26. The integral value of b for which the function
20. The number of polynomials p : R ® R satisfying
f ( x ) = (b2 - 3b + 2)(cos 2 x - sin 2 x)
1
p(0) = 0, p(x) > x2 for all x ¹ 0 and p¢¢(0) = is +(b - 1) x + sin(b + 1)
2 does not possess stationary point is
(a) 0 27. If the curves ax2 + by2 = 1 and a1x2 + b1y2 = 1
(b) 1 may cut each other orthogonally such that
(c) more than 1, but finite 1 1 æ1 1 ö
- = l ç - ÷ then l is equal to
(d) infinite a a1 è b b1 ø
28. Tangent at P1 (2,3) on the curve 3 y = x3 + 1
Numeric Value Answer
meets the curve again at P2 . The tangent at P2
21. The fuel charges for running a train are meets the curve at P3 and so on. If the sum of
proportional to th e square of the speed the ordinates for P1, P2 , P3 ,.....P60 be S then
generated at 16 miles per hour and costs ` 48 per æ 2183 - 8 ö
hour. The most economical speed if the fixed S +ç ÷ is equal to 5k, when k is equal to
charges i.e. salaries etc. amount to ` 300 per hour è 27 ø
29. A conical vessel is to be prepared out of a
is
circular sheet of metal of unit radius. In order
22. The curve x + y – ln (x + y) = 2 x +5 has a vertical that the vessel has maximum volume, the
tangent at the point (a, b). Then a + b is equal to sectorial area that must be removed from the
23. Two ships A and B are sailing straightaway from sheet is A1 and the area of the given sheet is A. If
a fixed point O along routes such that ÐAOB is A
= m + n , then m + n is equal to
always 120°. At a certain instance, OA = 8 km, A1
OB = 6 km and the ship A is sailing at the rate of 30. An arch way is in the shape of semi-ellipse, the
20 km/hr while the ship B sailing at the rate of 30 road level being the major axis. If the breadth of
km/hr. If the distance between A and B is the road is 30 m and the height of the arch is 6 m
k at a distance of 2 m from the side, the greatest
changing at the rate km/hr, then value of k height of the arch in metres, is
37
is _______.
24. A rectangle with its sides parallel to the X-axis
and Y-axis is inscribed in the region bounded
by the curves y = x2 – 4 and 2y = 4 – x2. The
maximum possible area of such a rectangle is

ANSWER KEY
1 (d) 4 (a) 7 (b) 10 (a) 13 (b) 16 (a) 19 (c) 22 (1) 25 (2) 28 (4)
2 (b) 5 (c) 8 (c) 11 (a) 14 (a) 17 (c) 20 (a) 23 (260) 26 (2) 29 (9)
3 (a) 6 (c) 9 (b) 12 (a) 15 (c) 18 (c) 21 (40) 24 (9.22) 27 (1) 30 (6)
22
INDEFINITE INTEGRATION

MCQs with One Correct Answer


1 + nx n -1 - x 2 n
òe
4. x
sin x dx =
1. If ò sin( x - a) dx = Ax + B log sin( x - a) + C , (1– x n ) 1 - x 2 n

then value of (A, B) is æ 1 - xn ö æ 1 + xn ö


(a) ex ç ÷ +C (b) ex ç ÷ +C
(a) (- cos a, sin a) (b) (cos a, sin a) è 1 - xn ø
è 1 + xn ø
(c) (- sin a, cos a) (d) (sin a, cos a)
æ 1+ xn ö æ 1 - xn ö
2. If f ( x ) = lim [2 x + 4 x3 + ......... + 2nx 2 n -1 ] ; (c) ex ç ÷ + C (d) ex ç ÷ +C
n®¥ çè 1 - x n ÷ø çè 1 + x n ÷ø
(0 < x < 1), then ò ( f ( x )) dx is equal to
5. If f ( x ) = lim n 2 ( x1/ n - x1/( n +1) ) , x > 0 then
n ®¥
1
(a) - 1 - x2 + c (b) +c
1 - x2 ò xf ( x) dx is equal to
1 1 x2 x2 x2
(c) +c (d) +c (a) + ln x + C (b) - ln x + +C
x2 - 1 1 - x2 2 4 2
æ f ( x ) g '( x ) - f '( x ) g ( x ) ö x3 x2 x2
3. ò çè f ( x) g ( x)
÷ (log ( g ( x ))
ø
(c)
3
+ xl n x + C (d)
2
ln x -
4
+C

- log ( f ( x))) dx is equal to xn - x -n


6. If f ( x ) = lim , 0 < x < 1 , n Î N then
æ g ( x) ö n®¥ x n + x -n
(a) log ç ÷+C
è f ( x) ø -1
2
ò (sin x ) f ( x)dx is equal to
1 æ g ( x) ö
(b) ç ÷ +C (a) -[ x sin -1 x + 1 - x 2 ] + C
2 è f ( x) ø
2 (b) x sin -1 x + 1 - x 2 + C
1æ æ g ( x) ö ö
(c) ç log ç ÷÷ +C
2è x2
è f ( x) ø ø (c) +C
2
æ æ g ( x ) ö2 ö
1
(d) log ç ç ÷ ÷+C (d) (sin -1 x )2 + C
ç è f ( x) ø ÷ 2
è ø
52 MATHEMATICS

æ ln a a x / 2 x
ö 1
ln bb (c) log{log e ex} - log{log e e 2 x 2 }
7. ò ç 3a5 x / 2b3 x 2a 2 x b 4 x
x ç + ÷ dx (where
÷ 2
è ø
+ log{log e e 3 x 3 } + C
+
a, b Î R ) is equal to 1
(d) log{loge ex} - log{log e e 2 x}
1 2 x 3x a 2 x b3 x 2
(a) a b ln +k
6 ln a 2b3 e 1
+ log{log e e 3 x} + C
1 1 1 2
(b) ln +k
2 3
6 ln a b a b 2 x 3x 2x 3x
ea b dx
11. ò ( x - b) ( x - a )(b - x )
is
1 1
(c) ln(a 2 x b 3 x ) + k
6 ln a 2b 3 a 2 x b3 x 2 x -a
(a) +C
1 1 a -b b - x
(d) - 2 3 2 x 3x
ln(a 2 x b3 x ) + k
6 ln a b a b 2
(b) ( x - a) (b - x) + C
8. If I n = ò (sin x + cos x ) dxn a -b
a-b
(sin x + cos x )n -1 (n - 1) (c) ( x - a) b - x + C
= (sin x – cos x) +2 I n -2 , 2
n n (d) None of these
then I5 equals (t = (sin x + cos x))
æp ö
(sin x - cos x) 5 tan ç - x ÷
(a) [t - 3t 3 + 8t - 32] + C è4 ø
15 12. If ò cos2 x
tan 3 x + tan 2 x + tan x
dx.
(sin x - cos x ) 5
(b) [t + 3t 4 + 8t 3 + 32t + 1] + C = –2 tan–1 u + c then u is equal to
15
( a) 1 + tan x + cot x (b) 1 + tan x + tan 2 x
(sin x - cos x) 4
(c) [3t + 8t 2 + 32] + C
15 (c) tan x + cot x (d) tan -1 (tan x + cot x)
(d) None of these
dx
9.
æ cos q + sin q ö
ò cos 2q loge çè cos q - sin q ÷ø d q
13. If ò (a + bx2 )
b - ax 2
= K tan–1 ( L tan q) + M ,

M being constant of integration then KL is equal to


sin 2q æ cos q + sin q ö 1
= ´ loge ç ÷ - f ( x) + c 1 1
2 è cos q - sin q ø 2 (a) 1 (b) (c)
a
(d) a
where f (x) is a a
(a) sec 2q (b) log e (sec 2q) 1
14. If ò f ( x)sin x cos xdx = l n f ( x) + c,
(c) 2 tan–1q (d) tan 2q 2(b - a 2 )
2
1 then f (x) is
10. ò x{logex e × log e2 x e × loge3 x e} dx is equal to 1
(a)
1 a sin x + b cos x
(a) log{log e e 2 x} - log{log e e3 x}
2 1
(b)
+ log{loge e 4 x} + C a 2 sin 2 x + b2 cos2 x
1 1
(b) log{loge x} - log{log e x} (c)
2 a 2 sin x + b2 cos x
+ log{loge x} + C 1
(d)
a sin 2 x + b cos2 x
Indefinite Integration 53
15. The integral cos x
æ sec 6α
= + ax + b l n | 2 cos x - sin x | + c,
sec18α sec54α ö 2 cos x - sin x
ò çè cosec 2α + cosec 6α + cosec18α ÷ø dα then value of |a + b| is ______.
is equal to 1 - (cot x )2010
ln | sec 54a | ln | sec 2a |
20. If ò tan x + (cot x)2011 dx
(a) - +c
108 4
1
ln | sec 6a | ln | sec18a | ln | sec54a | = log e | (sin x ) k + (cos x ) k | + c
(b) + + +c k
6 18 54 then k is equal to
1 sec 6α 1 sec18α q
(c) ln + ln sin3 dq
6 cosec 2α 18 cosec 6α
1 sec 54α
21. If òcos
q
2
cos q + cos 2 q + cos q
3
+ ln +c 2
54 cosec18α
(d) None of these = tan -1 f (q) + c
then the least value of f(q) for allowable values
16. If I n = ò (sin x + cos x)n dx (n ³ 2), then of q is equal to
nI n - 2(n - 1) I n- 2 is equal to 22. Let f(x) be a function such that f ( xy ) = f ( x) × f ( x )
(a) n
(sin x + cos x) (sin x - cos x ) " x, y Î R. If f (1 + x) = 1 + x(1 + g(x)) where
(b) (sin x + cos x) n -1 (sin x - cos x) f ( x) xk
lim g ( x) = 0. Further if
t ®0
ò f '( x) dx = k
+ C,
(c) (sin x + cos x)n +1 (sin x - cos x)
then evaluate the numerical quantity k.
(d) (sin x - cos x )n -1 (sin x + cos x ) sin(100 x)(sin x)l
99
17. Let ln x denote the logarithm of x with respect to 23. ò {sin (101x) × sin x}dx = m
,
the base e. Let S Ì R be the set of all points
where the function ln (x2 –1) is well-defined. l
Then, the number of functions f : S ® R that then is equal to ..........
m
are differentiable, satisfy f ¢ (x) = ln (x 2 – 1)
for all x Î S and f (2) = 0, is dx
(a) 0 (b) 1 24. If ò cos3 x - sin3 x
(c) 2 (d) infinite
2 + f ( x)
Numeric Value Answer = A tan -1 f ( x) + B ln +C
2 - f ( x)
f ( x) where f (x) = sin x + cos x find the value of
18. If lim exists finitely
x ®0 x2 (12 A + 9 2 B) - 3.
1/ x
æ f ( x) ö 3
lim ç1 + x +
x ÷ø
= e , and ò f ( x) log e x dx sin8 x sin 7 x sin 5 x
x ®0 è 25. If ò cos x dx = - a
+
b
is equal to ax 3 ( log e x - b ) + c , then value of
sin 3 x sin x æp xö
a + b is _______. - + + ln tan ç + ÷ + C ,
c d è 4 2ø
cos 2 x + sin 2 x then evaluate (a + b + c + d).
19. If ò (2 cos x - sin x)2 dx

ANSWER KEY
1 (b) 4 (c) 7 (b) 10 (d) 13 (c) 16 (b) 19 (0.60) 22 (2) 25 (4)
2 (d) 5 (d) 8 (c) 11 (a) 14 (b) 17 (d) 20 (2012) 23 (1)
3 (c) 6 (a) 9 (b) 12 (a) 15 (a) 18 (1) 21 (3) 24 (8)
23
DEFINITE INTEGRATION

MCQs with One Correct Answer 1


(a) ±1 (b)
1. Let p (x) be a polynomial of least degree whose 2
graph has three points of inflection (–1, –1), (c) ±2 (d) None of these
(1, 1) and a point with abscissa 0 at which the p/2
curve is inclined to the axis of abscissa at an dx p
1
5. If
ò 2 2
a cos x + b sin x 2 2
=
16
. Then
0
angle of 60°. Then ò p ( x)dx is equal to
minimum value of a cos x + b sin x is
0
(a) – 4 (b) – 8
3 3 +4 3 3
(a) (b) (c) – 2 (d) -2 2
14 7
p/3
3+ 7 3+2 sin x
(c) (d) 6. Let I = ò x
dx , then I belongs to
14 7 p/4
p æ 3 2ö æ 2
2 ç ÷ 3ö
2. ò (cos px - sin qx) dx; where p, q are integers; (a) ,
ç 8 6 ÷
(b) ç
ç 2
, ÷
2 ÷ø
-p è ø è
is equal to
æ1 2ö
(a) – p (b) 0 (c) p (d) 2p (c) çç , ÷ (d) None of these
3. The value of the definite integral è2 2 ÷ø
2p 7. Let f(x) be positive, continuous and
ò ecos q cos q(sin q)d q is differentiable on the interval (a, b) and
0 lim f ( x ) = 1 , lim f ( x ) = 31/ 4 . If
x®a + x ®b –
(a) 0 (b) p (c) 2p (d) ep
4. If p, q, r, s are in arithmetic progression and 1
f '( x ) ³ f 3 ( x ) + then the greatest value
p + sin x q + sin x p - r + sin x f ( x)
of b – a is
f ( x) = q + sin x r + sin x -1 + sin x
r + sin x s + sin x s - q + sin x (a) 1 (b) 31 / 4
p p
2 (c) (31/ 4 - 1 ) (d)
24 24
such that ò f ( x)dx = -4 , then the common
0
difference of the progession is
Definite Integration 55
8. Let a, b, c be non-zero real numbers such that 1 1
1 13. Suppose the limit L = lim nò dx
n®¥ 0 (1 + x 2 ) n
ò (1 + cos x)(ax + bx + c ) dx
8 2

0 1
exists and is larger than . Then,
2 2
= ò (1 + cos8 x)(ax2 + bx + c) dx .
1
0 (a) <L<2 (b) 2 < L < 3
Then the quadratic equation 2
(c) 3 < L < 4 (d) L > 4
ax 2 + bx + c = 0 has 14. Suppose a continuous function f : [0, ¥) ® R
(a) no root in (0, 2) satisfies
(b) at least one root in (0, 2) x
(c) a double root in (0, 2) f (x) = 2ò tf (t )dt + 1 for all x ³ 0.
(d) two imaginary roots 0

x 2 Then f (l) equals


(a) e (b) e 2 (c) e 4 (d) e 6
9. Let f ( x ) = ò (t - 1)dt . Then the value of
1
x x
15. Let J = ò 8
dx.
| f '(w) | where w is a complex cube root of unity 0 1+ x
is Consider the following assertions:
(a) 3 (b) 2 3 (c) 3 3 (d) 4 3 1 p
I. J> II. J<
4 8
sec 2 x
Then
ò f (t )dt
(a) only I is true
10. 2 equals
lim (b) only II is true

p p2
2 (c) both I and II are true
4 x -
16 (d) neither I nor II is true
8 2 x
(a) f (2) (b) f (2)
p p 16. For x ÎR, let f (x) = | sin x | and g (x) = ò f (t ) dt.
0
2 æ 1ö 2
(c) fç ÷ (d) 4 f (2) Let p(x) = g(x) - x. Then
p è 2ø p
p (a) p(x + p) = p(x) for all x
11. The value of the integral ò0
(1- | sin8 x |)dx is (b) p(x + p) ¹ p(x) for at least one but finitely
(a) 0 (b) p –1 many x
(c) p –2 (d) p –3 (c) p(x + p) ¹ p(x) for infinitely many x
12. Let S be the set of real numbers p such that there (d) p is a one-one function
is no non-zero continuous function f : R ® R 17. Let f: [0, 1] ® [0, 1] be a continuous function
such that
ò f ( t )dt = p f (x) for all x Î R. Then,
x
satisfying 1
p
x2 + (f(x))2 £ 1 for all x Î [0, 1] and ò f ( x ) dx =
0
.
S is 0
4
(a) the empty set 1
2
(b) the set of all rational numbers f ( x)
(c) the set of all irrational numbers Then ò 1 - x2 dx equals
1
(d) the whole set R. 2
56 MATHEMATICS

p p 22. Let f : (0, ¥) ® R be a differentiable function


(a) (b)
12 15 such that
x x
2 -1 p
(c)
2
p (d)
10 ò ò
x (1 - t ) f (t )dt = t f (t )dt "x Î (0, ¥) and
0 0
Numeric Value Answer f(1) = 1. The value of the limit lim f ( x ) is
x ®¥
18. Let f : R ® R be a differentiable function and equal to
f (x) 23. If p and q are different roots of the equation
ò 2t dt 1
f (1) = 4. Then the value of lim
x ®1
4
x -1
, if tan x = x then ò 2 sin( pt )sin(qt )dt is equal to
0
f ' (1) = 2 is 3

sin x sec x 2
x -1 24. Find the value ò é x ù dx , when [x] is the
ë û
0
19. If f ( x ) = cosec x x sin x cos x then
greatest integral value of x.
tan x x tan x x2 + 1 25. Let f : ¡ ® ¡ be a continuous odd function,
which vanishes exactly at one point and
p/3 x
1
ò f (x)dx= f (1) = . Suppose that F(x) = ò f (t )dt for all
2
-p / 3 –1
x
é1 x 2 1 1 ù
lim ê 5 ò e-t dt - 4 + 2 ú =
x Î [–1, 2] and G(x) = ò t | f ( f (t)) | dt for all
20. –1
x®0 ê x x 3x úû
ë 0 F ( x) 1
xÎ[–1, 2]. If lim = , then the value of
p/2 x ®1 G ( x ) 14
21. If I (n) = ò q sin n qd q, n Î N , n > 3, then
æ 1ö
f ç ÷ is
0
è 2ø
1 -1
[2010 I (2010) -2009 I (2008)] is
1005
equal to

ANSWER KEY
1 (a) 4 (a) 7 (d) 10 (a) 13 (a) 16 (a) 19 (0) 22 0 25 (7)
2 (d) 5 (a) 8 (b) 11 (c) 14 (a) 17 (a) 20 (0.30) 23 0
3 (c) 6 (a) 9 (c) 12 (d) 15 (a) 18 (16) 21 (2) 24 (2)
24
APPLICATION OF
INTEGRALS

MCQs with One Correct Answer 5. If the line y = mx + 2 cuts the parabola 2y = x2 at
1. The area bounded by the x-axis, the curve points ( x1, y1 ) and (x2, y2) where (x1 < x2), then
y = f(x) and the lines x = 1, x = b, is equal to x2
æ x2 ö
b 2 + 1 - 2 for all b > 1, then f(x) is
the value of m for which ò ç
è
mx + 2 -

÷ dx is
x1
(a) x -1 (b) x +1 minimum, is
x 8 1
(c) 2 (d) (a) 2 (b) (c) (d) 0
x +1 3 3
1+ x 2
6. The area bounded by the curve
2. 2
Area of region bounded by [ x ] = [ y ] 2
f ( x) = x + sin x and its inverse function
if x Î [1, 5] (where [.] represents the greatest between the ordinates x = 0 to x = 2p is
integer function), is (a) 4p square units (b) 8p square units
(a) 10 sq. unit (b) 8 sq. unit (c) 4 square units (d) 2 square units
(c) 6 sq. unit (d) 5 sq. unit 7. The area bounded by the curve y2 (2a – x) = x3
3. If a point P moves such that its distance from and the line x = 2a is
line y = 3x - 7 is same as its distance from 3pa 2
(a) 3pa2 sq. unit (b) sq. unit
(2 3, -1), then area of the curve, described by 2
P, enclosed between the coordinate axes is 3pa 2 6pa 2
(c) sq. unit (d) sq. unit
3 4 5
(a) (b) 2 3 (c) 6 (d) 3
2 8. The area bounded by the curves y = xe x ,
4. The area bounded by the curve f (x) = ||tan x +
cot x| – |tan x – cot x|| between the lines x = 0, y = xe - x and the line x = 1 is
p 2 2 1 1
x= and the x-axis, is (a) (b) 1 - (c) (d) 1 -
2 e e e e
(a) ln 2 (b) 2 ln 2 9. Area enclosed by the curve
| x + y – 1| + |2x + y – 1| = 1 is
(c) ln 4 (d) 2ln 2 (a) 2 sq. units (b) 3 sq. units
(c) 6 sq. units (d) 7 sq. units
58 MATHEMATICS

10. The area bounded by the curve


1æ 1ö 1æ 1ö
f ( x) = max.{4 - x 2 ,| x - 2|, ( x - 2)1/ 3} (a) ç log 2 - ÷ (b) ç log 2 + ÷
2è 2ø 2è 2ø
for x Î[-2, 4] and x-axis is 1 1
(c) (1 - log 2 ) (d) (1 + log 2 )
45 23 2 2
(a) sq. units (b) sq. units 15. Area of the region
4 4

(c)
59
sq. units (d)
53
sq. units {( x, y ) Î ¡ : y ³
2
x + 3 , 5y £ x + 9 £ 15 }
4 4
is equal to
11. The area of the region between the curves
1 4 3 5
1 + sin x 1 - sin x (a) (b) (c) (d)
y= and y = bounded by 6 3 2 3
cos x cos x 16. The maximum possible area bounded by the
p parabola y = x2 + x + 10 and a chord of the
the lines x = 0 and x = is parabola of length 1 is
4
1 1 1 1
2 -1 (a) (b) (c) (d)
t 12 6 3 2
(a) ò dt
17. The area of the region bounded by the lines
0 (1 + t ) 1 - t 2
2
x = l, x = 2 and the curves x(y - ex) = sin x and
2 -1
2xy = 2sin x + x3 is
4t
(b) ò dt
(a) e2 - e -
1
(b) e2 - e -
7
0 (1 + t 2 ) 1 - t 2 6 6
2 +1 1 7
4t (c) e2 - e + (d) e2 - e +
(c) ò dt 6 6
0 (1 + t 2 ) 1 - t 2
Numeric Value Answer
2 +1
t 18. If [x] is the greatest integrer £ x , then
(d) ò dt
0 (1 + t 2 ) 1 - t 2 2

12. The area enclosed by the curves y = x , y = x , 2 3 ò min{x - [ x], - x - [ - x ]} dx =


-2
x = 0 and x = p, where p > 1, is 1/6. Then p equals
(a) 8/3 (b) 16/3 (c) 2 (d) 4/3 19. Let for all n Î N , n - 2 ³ 2 log e n then the
13. If a straight line y – x = 2 divides the region minimum value of the area bounded by the curve
x2 + y 2 £ 4 into two parts, then the ratio of the 1
| y|+ £ e -| x| is
area of the smaller part to the area of the greater n
part is
(a) 3p – 8 : p + 8 (b) p – 3 : 3p + 3 ïì {x}, x ÏI 2
20. If f ( x ) = í and g ( x ) = {x} ,
(c) 3p – 4 : p + 4 (d) p – 2 : 3p + 2 ïî1, x ÎI
14. The area of the region above the x-axis bounded
(where {x} is fractional part of x), then area
p
by the curve y = tan x, 0 £ x £ and the tangent to bounded between f (x) and g(x) "x Î[0,10] is .
2
p
the curve at x = is:
4
Application of Integrals 59
21. P (x, y) is a point, which moves in the xy plane 24. Let f ( x) be a polynomial of degree 3 such that
such that 2 [ y ] = 3 [ x ] where [ . ] denotes the
greatest integer function and -2 £ x £ 5 and the curve y = f ( x) has relative extremes at
-3 £ y £ 6 . The area of the region containing x = ±2 / 3 and passes through (0, 0) and
the point P (x, y) is equal to (1, –2) dividing the circle x2 + y2 = 4 in two parts.
22. If the value of a ( a ³ 1) for which the area of the Find out the difference of areas of these two
figure bounded by pair of straight lines parts.
25. Area of the region bounded by the curve
y 2 - 3 y + 2 = 0 and the curves y = [ a ] x ,
2
y = {x 2 }, where {.} denotes fractional part
1 æ 7ö
y=
2
[ a ] x2 is greatest, is [a, b), then function x Î [–2, 2] is p ç q + r - ÷ . Find
è 3ø
(b – a) = where [.] denotes the greatest integer the value of p + q + r.
function.
23. If the area bounded by y = f (x) and the curve
2
y= where f is a continuous function
1 + x2
satisfying the conditions f (x) · f (y) = f (xy). "
æ aö
x, y, Î R and f ¢ (1) = 2, f (1) = 1is ç p - ÷ , then
è 3ø
a=

ANSWER KEY
1 (d) 4 (c) 7 (b) 10 (c) 13 (d) 16 (b) 19 (2) 22 (1) 25 (7)
2 (b) 5 (d) 8 (a) 11 (b) 14 (a) 17 (b) 20 (3.33) 23 (2)
3 (a) 6 (d) 9 (a) 12 (d) 15 (c) 18 (1) 21 (4) 24 (0)
DIFFERENTIAL EQUATION 25
AND ITS APPLICATIONS

MCQs with One Correct Answer 4. The solution of the differential equation
dy
1. Through any point (x, y) of a curve which passes x3 = y 3 + y 2 y 2 - x 2 is
dx
through the origin, lines are drawn parallel to
the co-ordinate axes. The curve, given that it (a) y + y 2 - x2 = cxy
divides the rectangle formed by the two lines
(b) y - y 2 - x 2 = cxy
and the axes into two areas, one of which is twice
the other, represents a family of (c) y y 2 - x2 = cx + y
(a) circles
(d) x y 2 - x 2 = cx + y
(b) pair of straight lines
(c) parabolas 5. The solution of the differential equation,
(d) rectangular hyperbolas dy p
2x2 y = tan( x 2 y 2 ) – 2 xy 2 given y (1) =
2. Lef f (x) be a positive, continuous and dx 2
differentiable function on the interval (a, b). If is
(a) sinx2y2 = ex–1 (b) sin(x2y2) = x
lim f ( x ) = 1 an d lim f ( x) = 31/ 4 . Also (c) cosx y + x = 0 (d) sin(x2y2) = e.ex
2 2
x®a + x ®b - 6. Solution of the differental equation
1
f ¢ (x) ³ (x) +f3 then æ yö æ yö
f ( x) x cos ç ÷ ( ydx + xdy ) = y sin ç ÷ ( xdy - ydx)
p p x
è ø èxø
(a) b – a ³ (b) b – a £ is
4 4
æxö
(b) sec æç ö÷ = cxy
p y
(c) b – a £ (d) None of these (a) y = cx cos ç ÷
24 è yø èxø
3. Let f be a non-negative function defined on the
æ yö æ yö
interval [0, 1]. (c) ç ÷ sec ç ÷ = c (d) None of these
è xø èxø
x x 7. The family of curves whose tangents form an
If ò0 1 - ( f '(t )) 2 dt = ò f (t ) dt , 0 £ x £ 1 and
0
p
f (0) = 0, then angle of with the hyperbolas xy = C are
4
(a) f (1/ 2) < 1/ 2 and f (1/ 3) > 1/ 3 (a) pair of straight lines
(b) f (1/ 2) > 1/ 2 and f (1/ 3) > 1/ 3 (b) y 2 - xy - x 2 = C
(c) f (1/ 2) < 1/ 2 and f (1/ 3) < 1/ 3
(c) y 2 - 2 xy - x 2 = C
(d) f (1/ 2) > 1/ 2 and f (1/ 3) < 1/ 3
(d) y 2 + 2 xy - x 2 = C
Differential Equation and its Applications 61
8. Solution of the differential equation (b) e2 x y 2 - 2 ln | y |= c
ìï 1 y 2 üï ìï x 2 1 üï
í - 2ý
dx + í 2
- ý dy = 0 is (c) e x + ln | y |= c
îï x ( x - y ) þï îï ( x - y ) y þï
(d) None of these
x xy
(a) ln + =c
y x- y 12. The orthogonal trajectory of the curve
xy an–1 y = xn is
(b) ln | xy | + =c (a) n2 + x2 = constant (b) ny2 + x2 = constant
( x - y)
(c) x2 + y2 = constant (d) y2 + nx2 = constant
xy
(c) = ce x / y 13. A curve f (x) passes through the point P (1, 1).
( x - y) The normal to the curve at point P is
xy a( y - 1) + ( x - 1) = 0 . If the slope of the tangent
(d) = c e xy
( x - y) at any point on the curve is proportional to the
(where c is arbitrary constant)
9. The equation of the curve passing through the ordinate at that point, then the equation of the
points (3a, a) (a > 0) in the form x = f(y) which curve is
satisfy the differential equation; (a) y = e ax - 1 (b) y - 1 = eax
a 2 dx x y (c) y = e a ( x -1) (d) y - a = e ax
× = + - 2, is
xy dy y x 14. A tangent and a normal to a curve at any point P
æ 1 + e y -k ö meet the x and y axes at A, B and C, D respectively.
(a) x = y + a çç y-k ÷ If the centre of circle through O, C, P and B lies
÷
è 1 - 2e ø on the line y = x (O is the origin) then the
æ1+ e y - k ö differential equation of all such curves is :
(b) x = y + a çç y-k ÷ ÷
è 1- e ø dy y - x dy y 2 - x 2
- (a) = (b) =
æ1+ e y k ö dx y + x dx y 2 + x 2
(c) y = x + a çç y-k ÷ ÷ dy x - y
è 1- e ø (c) = (d) None of these
(d) None of these dx xy
10. The solution of the differential equation dy
15. If - y log e 2 = 2sin x (cos x - 1) log e 2, then y =
( y + x xy ( x + y ))dx + ( y xy ( x + y) - x)dy = 0, dx
is (a) 2sin x + c 2 x (b) 2cos x + c 2 x
x2 + y 2 x
(a) + 2 tan -1 =C (c) 2sin x + c 2- x (d) 2cos x + c 2- x
2 2y
16. Solution of differential equation
x2 + y 2 x
(b) + 2 tan -1 =C æ d ö
t ç ( g ( x )) ÷ - t 2
2 y dt dx
= è ø is
x2 + y 2 x dx g ( x)
(c) + 2 tan -1 =C
2 y g ( x) g ( x)
(d) None of these (a) t= +c (b) t = +c
x x2
dy y3
11. Solution of dx = 2 x is t=
g ( x)
e + y2 (c) (d) t = g ( x) + x + c
x+c
-2 x 2
(a) e y + 2 ln | y |= c
62 MATHEMATICS

17. Let f(x) be continuously differentiable on the x x

interval (0, ¥ ) such that f(1) = 1, and x ò y (t )dt = ( x + 1) ò ty (t )dt , x > 0 , and y(1) = e,
0 0
t 2 f ( x ) - x 2 f (t ) æ 1ö
lim = 1 for each x > 0, then f(x)
then y ç ÷ is equal to
è 2ø
t®x t-x
22. If the solution of the differential equation
is
x dx - y dy æ 1 + x 2 - y2 ö
1 2 x2 1 4 x2 = ç ÷ be
(a) + (b) - + x dy - y dx è x2 - y 2 ø
3x 3 3x 3

1 2 1 æ x+ y ö
(c) - + 2 (d) f ( x, y ) + 1 + f ( x, y ) = c ç ÷
x x x è f ( x, y ) ø
where c is an arbitrary constant then f (3, 2) is
Numeric Value Answer equal to
18. Find the sum of the order ‘O’ and degree D of 23. A curve y = f (x) is such that f ( x) ³ 0 and
the differential equation
f (0) = 0 and bounds a curvilinear trapezoid
2 2 n
æ dy ö x æ dy ö xn æ dy ö with the base [0, x] whose area is proportional to
y = 1 + x ç ÷ + ç ÷ + ..... + ç ÷
è dx ø 2! è dx ø n! è dx ø (n + 1)th power of f (x). If f (1) = 1, then {f(10)}n is
+........¥ . equal to 10 k, where k equals
19. Let y = f (x) be a curve passing through (e, ee), 24. Let y = f(x) is a polynomial function satisfying
which satisfy the differential equation (2ny + xy
logex) dx – x logex dy = 0, x > 0, y > 0. If æ1ö æ1ö æ1ö
x2 f '( x) f ç ÷ - f ( x) f ' ç ÷ = x2 f '( x) - f ' ç ÷ .
e x
è ø x
è ø èxø
g ( x) = lim f ( x), then
n®¥ ò g ( x) dx equals to. If f(1) = 2 and f(5) = 26, then find f(6) – 30.
1/ e
1
20. If the solution of the differential equation 25. If the ar ea bounded by y = f ( x), x = ,
2
dy 1
= is 3
dx xy[ x sin y 2 + 1]
2 x= and the X-axis is A sq. units where
2
2
x 2 (cos y 2 - sin y 2 - 2Ce - y ) = k , then value 2 2 4 2 4 6
f ( x ) = x + x3 + × x 5 + × × x 7 +
of k is ______. 3 3 5 3 5 7
21. If the equation of a curve y = y(x) satisfies the ....¥, | x |< 1, then the value of [4A] is (where
differential equation
[.] is G..I.F)

ANSWER KEY
1 (c) 4 (a) 7 (c) 10 (b) 13 (c) 16 (c) 19 (0) 22 (5) 25 (1)
2 (c) 5 (a) 8 (a) 11 (a) 14 (a) 17 (a) 20 (2) 23 (1)
3 (c) 6 (b) 9 (b) 12 (b) 15 (a) 18 (2) 21 (8) 24 (7)
26
VECTOR ALGEBRA

uuur uuur
MCQs with One Correct Answer 5. In a parallelogram ABCD, | AB |= a,| AD |= b and
r uuur uuuur uuur
1. The vectors a ( x) = cos xiˆ + sin xjˆ and | AC |= c , the value of DB . AB is
r
b ( x) = xiˆ + sin xjˆ are collinear for 3a 2 + b2 - c 2 a 2 + 3b2 - c 2
p (a) (b)
(a) Unique value of x, 0 < x < 2 2
6
p p a 2 - b2 + 3c 2 a 2 + 3b2 + c 2
(b) Unique value of < x < (c) (d)
6 3 2 2
(c) No value of x r r r
6. a and c are unit vectors and | b | = 4 . The angle
p
(d) Infinitely many values of x, 0 < x < r r æ 1ö r r r
2 between a and c is cos -1 ç ÷ . If b - 2c = la ,
2. If cos a ¹ 1, cos b ¹ 1 and cos g ¹ 1, then the è 4ø
r r then l is equal to
vector a = iˆ cos a + ˆj + kˆ, b = iˆ + ˆj cos b + kˆ ,
r 1 3
c = iˆ + ˆj + kˆ cos g are (a) 3, – 4 (b) ,
4 4
(a) coplanar vectors
(b) coplanar vectors if cos a = cos b = cos g ¹ 1 1 3
(c) - 3,4 (d) - ,
(c) coplanar vectors if os a ¹ cos b ¹ cos g 4 4
(d) never coplanar 7. The angles of a triangle, two of whose side are
3. Let a, b, c be distinct non-negative numbers. If r
represented by the vectors 3(aˆ ´ b) and
the vectors aiˆ + ajˆ + ckˆ, iˆ + kˆ and ciˆ + cjˆ + bkˆ r r r
lie in a plane, then c is b - (a.b).a
ˆ ˆ where b is a non-zero vector and â
(a) the Arithmetic Mean of a and b is a unit vector, are
(b) the Geometric Mean of a and b
(c) the Harmonic Mean of a and b æ 1 ö æ 1ö æ 3+2ö
(d) equal tor zero (a) tan -1 ç ÷ ; tan -1 ç ÷ ; tan -1 ç ÷
r è 3ø è 2ø è1- 2 3ø
4. Let a and b are two vectors making angle q
æ 1ö
with each other, then which of the following (b) tan-1( 3); tan -1 ç ÷ ;cot -1(0)
r
represents unit vectors along bisector of a and è 3ø
r
b is: æ 3+2ö
(c) tan -1 ( 3); tan -1 (2); tan -1 ç ÷
aˆ + bˆ aˆ + bˆ è 2 3 - 1ø
(a) ± (b) ±
2 2 cos q æ 2 +3ö
ˆ
aˆ + b (aˆ + bˆ) (d) tan -1 ( 3); tan -1 (2); tan -1 ç ÷
(c) ± (d) è 3 2 - 1ø
2cos q / 2 | aˆ + bˆ |
64 MATHEMATICS
uur 12. A unit vector which is perpendicular to the
8. If the vectors a = (c log 2 x)iˆ - 6 ˆj + 3kˆ and
uur vector 2iˆ - ˆj + 2kˆ and is coplanar with the
b = (log 2 x)iˆ + 2 ˆj + (2c log 2 x)kˆ make an
obtuse angle for any x Î(0, ¥) , then the vectors iˆ + ˆj - kˆ and 2iˆ + 2 ˆj - kˆ is
interval to which ‘c’ belongs is
2 ˆj + kˆ 3iˆ + 2 ˆj - 2 kˆ
(a) (0, ¥) (b) (-¥, 0) (a) (b)
5 17
4
(c) (- , 0) (d) ( -1, 0) È (0, 2 ) 3i + 2 ˆj + 2kˆ
ˆ 2i + 2 ˆj - 2kˆ
ˆ
3 3 (c) (d)
ur r 17 3
9. Let p = ai$+ b$j + ck$ and q = bi$+ c$j + ak$, 13. If a, b and c are pth, qth, rth terms of HP and
ur r
where a, b, c Î R. If 'q' be the angle between p u = (q - r )iˆ + (r - p) ˆj + ( p - q)kˆ,
r
and q then, r iˆ ˆj kˆ
v = + + , then
(a) q Î (0, p / 2) (b) q Î[0, 2p / 3] a b c
r r
(c) q Î (2p / 3, p] (d) q Î[p / 2, p] (a) u and v are parallel vectors
r r
10. The value of ‘a’ so that th e volume of (b) u and v are orthogonal vectors
parallelopiped formed by iˆ + ajˆ + kˆ, ˆj + akˆ and r r
(c) u × v = 1
r r
aiˆ + kˆ becomes minimum is (d) u ´ v = iˆ + ˆj + kˆ
(a) –3 (b) 3 (c) 1 3 (d) 3 14. If the two adjacent sides of two rectangles are
® ® ® represented by vectors
11. If a , b , c are three non-zero, non-coplanar r r r r
r r r r r
vectors and p = 5a - 3b ; q = -a - 2b an d r = -4a - b ;
® ® ® ® r r r
® ® b× a ® ® ® b× a ® s = - a + b , r espectively, then the angle
b1 = b- a , b2 = b + a, r 1 r r r
® ® between the vectors x = ( p + r + s ) and
2 2
|a| |a| 3
® ® ® ® r 1 r r
® ® y = ( r + s ) is
c× a ® b× c ® 5
c1 = c- a+ b1 ,
® ®
2 2 æ 19 ö
|a| |c| (a) - cos -1 ç ÷
® ® ® ® è 5 43 ø
® ® c × a ® b1× c ®
c2 = c- ® a- ® b1 , æ 19 ö
(b) cos -1 ç ÷
2 2
|a| | b1 | è 5 43 ø
® ® ® ® æ 19 ö
® ® c× a ® b× c ® (c) p cos -1 ç ÷
c3 = c- a+ b1 , è 5 43 ø
® ®
2 2
|c| |c| (d) Cannot be evaluated
uur uur uur uur r r
uur uur c . a uur b . c uur 15. r
c4 = c - uur a = uur b1 , a, b and c be three non-coplanar vectors and
r
| c |2 | b |2 d be a non-zero vector, which is perpendicular
then the set of orthogonal vectors is r r r
® ® ® ® ® ®
to (a + b + c ). Now if
(a) ( a , b1, c3 ) (b) ( a , b , c ) r r r r r r r
1 2 d = sin x(a ´ b ) + cos y (b ´ c ) + 2(c ´ a ), then
® ® ® ® ® ®
(c) ( a , b1, c1) (d) ( a , b , c ) minimum value of x + y is equal to
2 2
2 2
(a) p2 (b) 0 (c) p2/4 (d) 5p2/4
Vector Algebra 65
r where a, b, g are scalars. If
16. The vectors a = 2l 2iˆ + 4lˆj + kˆ and
r a= k1 ( Fˆ × aˆ ) - k2 ( Fˆ × bˆ), then the value of
b = 7iˆ - 2 ˆj + lkˆ make an obtuse angle
r 2(k1 + k2) is
whereas the angle between b and k̂ is acute r r
and less than p/6, 21. Let a = $i + $j + $k , b = x1$i + x2 $j + x3 $
k,
1 where x1, x2, x3 Î {– 3, – 2, – 1, 0, 1, 2}.
(a) 0 < l < (b) l > 159 r r
2
If the number of possible vectors b such that a
1 r
(c) - < l < 0 (d) null set and b are mutually perpendicular is t, then t /5 =
2
r 22. Find the absolute value of parameter t for which
17. The angle q between two non-zero vectors a
r the area of the triangle whose vertices are
and b satisfies the relation A(–1, 1, 2); B(1, 2, 3) and C(t, 1, 1) is minimum.
r r r r
cos q = (a ´ iˆ) × (b ´ iˆ) + (a ´ ˆj ) × (b ´ ˆj ) r
r r 23. Given a vector A defined as
+(a ´ kˆ) × (b ´ kˆ), r r r r r r r
A = (a ´ b ) ´ (c ´ d ) + (a ´ c )
then the least value of |a| + |b| is equal to (where r r r r r r
q ¹ 90°) ´(b ´ d ) + (a ´ d ) ´ (b ´ c ) , then find the value
r r
1 of | A ´ a | .
(a) (b) 2
2
(c) (d) 4 r r é1ù
2 24. Let v0 be a fixed vector and v0 = ê ú . Then for
ë0û
Numeric Value Answer
n ³ 0 a sequence is defined
18. If the sum of two unit vectors is a unit vector, n +1 n +1
r r æ1ö é0 -1ù r
and the magnitude of their difference is k , vn +1 = vn + ç ÷ ê ú v0 then
è2ø ë1 0 û
then value of k is
r éa ù a
ˆ yˆ and ẑ are three unit vectors in three-
19. If x, lim vn = ê ú . Find .
n ®¥ b
ë û b
dimensional space, then the minimum value of
25. If A1, A2, ...., Ag are vertices of a regular octagon,
2 2 2
xˆ + yˆ + yˆ + zˆ + zˆ + xˆ 7 uuur uuur uuur uuur
then å (OA j ´ OA j +1 ) = K (OA1 ´ OA2 )
20. Let â and b̂ be two unit vectors such that j =1

1 r where the value of K is ........


aˆ × bˆ = and aˆ ´ bˆ = cˆ. Also F = aaˆ + bbˆ + gcˆ,
3
ANSWER KEY
1 (b) 4 (c) 7 (b) 10 (c) 13 (b) 16 (d) 19 (3) 22 (2) 25 (7)
2 (d) 5 (a) 8 (c) 11 (b) 14 (b) 17 (c) 20 (3) 23 (0)
3 (b) 6 (a) 9 (b) 12 (d) 15 (d) 18 (3) 21 (5) 24 (2)
THREE DIMENSIONAL 27
GEOMETRY

MCQs with One Correct Answer l1 - l2 m - m2 n -n


(c) , 1 , 1 2
1. The angle between the lines whose direction 2 sin q / 2 2 sin q / 2 2 sin q / 2
cosines are given by the equations
l1 - l2 m - m2 n -n
(d) , 1 , 1 2
3l + m + 5n = 0 , 6nm - 2nl + 5lm = 0 is 2 cos q / 2 2 cos q / 2 2 cos q / 2
4. If the straight lines
æ1ö æ 1ö
(a) cos -1 ç ÷ (b) cos -1 ç - ÷ x = 1 + s , y = -3 - l s , z = 1 + l s
è6ø è 6ø
t
and x = , y = 1 + t , z = 2 - t , with parameters
æ2ö æ 5ö 2
(c) cos -1 ç ÷ (d) cos -1 ç - ÷
è3ø è 6ø s and t respectively, are co-planar, then l equals.
1
2. A line in the 3-dimensional space makes an angle (a) 0 (b) –1 (c) -
(d) –2
2
æ pö 5. L1 and L2 are two lines whose vector equations
q çè 0 < q £ ÷ø with both the x and y axes. Then
2 r
the set of all values of q is the interval: are L1 : r = l[(cos q + 3)ˆi +

æ pù é p pù ( 2 sin q)ˆj + (cos q - 3)kˆ ]


(a) ç 0, ú (b) ê6 , 3ú
è 4û ë û r
and L2 : r = m(aˆi + bˆj + ckˆ ) where, l and m are
é p pù æ p pù scalars and a is the acute angle between L1 and
(c) ê 4 , 2ú (d) ç , ú L2. If the angle a is independent of q, then the
ë û è 3 2û
3. If l1, m1, n1 and l2, m2, n2 are DCs of the two value of a is
lines inclined to each other at an angle q, then p p p p
the DCs of the internal bisector of the angle (a) (b) (c) (d)
6 4 3 2
between these lines are
l1 + l2 m + m2 n +n x + 6 y + 10 z + 14
(a) , 1 , 1 2 6. The line = = is the
2 sin q / 2 2 sin q / 2 2 sin q / 2 5 3 8
hypotenuse of an isosceles right angled triangle
l1 + l2 m + m2 n +n whose opposite vertex is (7, 2, 4). Then which of
(b) , 1 , 1 2
2 cos q / 2 2 cos q / 2 2 cos q / 2 the following is not the side of the triangle?
Three Dimensional Geometry 67

x-7 y-2 z-4 ABC satisfies the relation 1 + 1 + 1 = k ,


(a) = =
2 -3 6 x2 y 2 z 2
x-7 y-2 z-4 then the value k is
(b) = = 1
3 6 2 (a) 3 (b) 1 (c) (d) 9
3
x-7 y-2 z-4 12. The plane containing the line
(c) = =
3 5 -1
x -1 y - 2 z - 3
(d) None of these = = and parallel to the line
1 2 3
7. The vertex A of the triangle ABC is on the line
r
r = iˆ + ˆj + lkˆ and the vertices B and C have x
=
y z
= passes through the point:
1 1 4
respective position vectors iˆ and ĵ . Let D be
(a) (1, – 2, 5) (b) (1, 0, 5)
é 3 33 ù (c) (0, 3, –5) (d) (– 1, – 3, 0)
the area of the triangle and D Î ê 2 , 2 ú , then 13. The position vectors of points a and b are
ë û
the range of values l corresponding to 'A' is iˆ - ˆj + 3kˆ and 3iˆ + 3 ˆj + 3kˆ respectively. The
(a) [-8, - 4] È [4, 8] (b) [–4, 4]
equation of a plane is r × (5iˆ + 2 ˆj - 7 kˆ) + 9 = 0.
(c) [–2, 2] (d) [-4, - 2] È [2, 4] The points a and b
8. A line with positive direction cosines passes (a) lie on the plane
through the point P(2, –1, 2) and makes equal (b) are on the same side of the plane
angles with the coordinate axes. The line meets
(c) are on the opposite side of the plane
the plane 2x + y + z = 9 at point Q. The length of
(d) None of the above
the line segment PQ equals
14. The angle between the pair of planes repre-
(a) 1 (b) 2 (c) 3 (d) 2 sented by equation
9. Through a point P (h, k, l) a plane is drawn at
right angles to OP to meet the co-ordinate axes 2 x 2 - 2 y 2 + 4 z 2 + 6 xz + 2 yz + 3xy = 0 is
in A, B and C. If OP = p, then the area of DABC
æ1ö æ 4ö
is : (a) cos -1 ç ÷ (b) cos -1 ç ÷
è 3ø è 21 ø
p 2 hk p 3l
(a) (b) æ 7 ö
æ4ö
l2 3hk (c) cos -1 ç ÷ (d) cos -1 ç ÷
è9ø è 84 ø
p 2t 2 p5
(c) (d) 15. Let P = (–3, 1, 1) and Q = (3, 4, 2). R divides PQ
2hk 2hkl
10. Projection of the line x + y + z – 3 = 0 = 2x + 3y + in the ratio PR : PQ = 1 : 3. Then, the equation of
4z – 6 on the plane z = 0 is uuur
the plane perpendicular to PQ at R is
x y-6 z x y-6 z (a) 18x + 9y + 3z = 8 (b) 18x + 9y + 3z = 4
(a) = = (b) = =
-2 1 0 1 -2 0 (c) 9x + 18y + 3z = 4 (d) 3x + 9y + 18z = 8
x y-6 z 16. Let s1, s2, s3 be planes passing through the
(c) = = (d) none of these origin. Assume that s1 is perpendicular to the
1 2 0
vector (1, 1, 1), s2 is perpendicular to a vector
11. A variable plane at a distance of the one unit
(a, b, c), and s3 is perpendicular to the vector
from the origin cuts the coordinates axes at
(a2, b2, c2).
A, B and C. If the centroid D (x, y, z) of triangle
68 MATHEMATICS

What are all the positive values of a, b and 21. Let the equation of the plane containing line
c so that s1 Ç s2 Ç s3 is a single point? x – y – z – 4 = 0 = x + y + 2z – 4 and parallel to the
(a) Any positive value of a, b, and c other than 1 line of intersection of the planes 2x + 3y + z = 1
(b) Any positive values of a, b and c where and x + 3y + 2z = 2 be x + Ay + Bz + C = 0. Then
either a ¹ b, b ¹ c or a ¹ c the values of |A + B + C – 4| is ........ .
(c) Any three distinct positive values of a, b, 22. Let f be a one-one function with domain {–2, 1, 0}
and c and range {1, 2, 3} such that exactly one of the
following statements is true :
(d) There exist no such positive real numbers
a, b and c f (-2) = 1, f (1) ¹ 1, f (0) ¹ 2 and the remaining
r ˆ ˆ ˆ r r two are false. If the area of the triangle formed
17. Let a = i + j + k , b = 2iˆ + 2 ˆj + kˆ , and c = 5iˆ + ˆj - kˆ
be three vectors. The area of the region formed by (–2, 1, 0) and (f(–2), f(1), f(0)) and the origin is
by the set of points whose position vectors k
r r r given by ; then sum of digits of k is.
r s art i sfy t h e eq u a t i on s r . a = 5 an d
r r r 2
| r - b | + | r - c | = 4 is closest to the integer
23. If the equation of the plane through the
(a) 4 (b) 9 intersection of the planes x + 2y + 3z – 4 = 0 and
(c) 14 (d) 19 2x + y – z + 5 = 0 and perpendicular to the plane
5x + 3y + 6z + 8 = 0 is ax + by + cz + 173 = 0, then
Numeric Value Answer b – 9(a + c) is equal to ........
18. A line with direction ratios (2, 1, 2) intersects 24. If a line is passing through (a, b, c) and
r intersecting y = 0, z2 = 4ax lies on the surface
the lines r = - ˆj + l(iˆ + ˆj + kˆ) and
(bz - cy)2 = k a(b - y)(bx - ay); then find the
r
r = -iˆ + m(2iˆ + ˆj + kˆ) at A and B, respectively,, value of k.
then length of AB is equal to 25. If the volume enclosed by the equation
19. The shortest distance between the z-axis and the | x |£ 8, | y |£ 8, | z |£ 8 and | x + y + z |£ 8 is t,
line, x + y + 2z - 3 = 0 , 2x + 3y + 4z - 4 = 0 is
t
20. The distance of the point (1, 0, –3) from the plane then =
512
x - y - z = 9 measured parallel to the line
x-2 y+2 z -6
= = is
2 3 -6

ANSW ER KEY
1 (b) 4 (d) 7 (d) 10 (b) 13 (c) 16 (c) 19 (2) 22 (7) 25 (4)
2 (c) 5 (a) 8 (c) 11 (d) 14 (c) 17 (a) 20 (b) 23 (6)
3 (b) 6 (c) 9 (d) 12 (b) 15 (b) 18 (3) 21 (7) 24 (4)
28
PROBABILITY-2

1. Let A and B be the sets {1, 2, …10} and {1, 2, 5. The odds in favour of a book reviewed by three
…20} respectively. A function is selected independent critics are, respectively, 5 : 2, 4 : 3
randomly from A to B the probability that the and 3 : 4. The probability that majority of the
function is non-decreasing is critics give favourable remark is
29 29
C10 C20 210 209 211 205
(a) 10
(b) (a) (b) (c) (d)
(20) (20)10 343 343 343 343
29
C19 6. Matrices of order 3 × 3 are formed by using the
(c) (d) None of these elements of the set A = {–3, –2, –1, 0, 1, 2, 3},
(20)10 then probability that matrix is either symmetric
2. Raj and Sanchita are playing game in which they or skew symmetric is
throw two dice alternately till one of them gets 9.
Which one of the following could be the 1 1 1 1 1
probability that Sanchita win the game? (a) + 3 (b) 9
+ 3- 6
6 7 7 7
(a) 7/15 or 8/15 (b) 6/11 or 5/11 7 7
(c) 8/17 or 9/17 (d) None of these 1 1 1 1 1
3. A boy whose hobby is tossing a fair coin is to (c) 3
+ 9 (d) + -
3 6
score one point for every tail and 2 points for 7 7 7 7 79
every head. The boy goes on tossing the coin, 7. If a Î[– 20, 0], then probability that the graph of
till his score reaches n or exceeds n where n > 2. the function y =16x2 + 8(a + 5) x – 7a – 5 is
If pn is the probability that his score attains strictly above the x-axis is
exactly n, then pn is equal to
1 7 13 17 3
(a) pn–1 + pn–2 (b) p n -1 + pn -2 (a) (b) (c) (d)
2 20 20 20 20
n n 8. In a Competitive test, a candidate guesses,
2 ( -1) 2n +1 + ( -1) copies or knows the answer to a multiple choice
(c) + (d)
3 2n 3.2 n question with four choices. The probability that
4. Entries of a 2 × 2 determinant are chosen from he makes a guess is 1/3 and the probability that
the set {–1, 1}. The probability that determinant he copies the answer is 1/6. The probability that
has zero value is his answer is correct given that he copied it is 1/
1 1 8. Find the probability that he knew the answer
(a) (b) to the question, given that he answered it
4 3
correctly.
1 (a) 24/29 (b) 26/29
(c) (d) None of these
2 (c) 22/29 (d) None of these
70 MATHEMATICS

9. A wire of length l is cut into three pieces. Then 2n + 1 1


the probability that the three pieces form a triangle (a) (b) less than
4n + 3 2
is
1
1 1 (c) greater than (d) None of these
(a) (b) 2
2 4 14. A book writer writes a good book with probability
2 1
(c) (d) None of these . If it is a good book, the probability that it will
3 2
10. Given that the sum of two non-negative 2 1
quantities is 200, the probability that their be published is , otherwise it is . If he writes
3 4
3 2 books, the probability that at least one book
product is not less than times their greatest
4 will be published is
product value is 407 411 405 307
(a) (b) (c) (d)
7 101 9 10 576 576 576 576
(a) (b) (c) (d) 15. If X follows a binomial distribution with
16 201 16 16
11. On each evening a boy either watches 1
DOORDARSHAN channel or TEN SPORTS. The parameters n = 8 and p = , then p(|x – 4| < 2) is
2
4 equal to
probability that he watches TEN SPORTS is
.
5 121 119 117 115
(a) (b) (c) (d)
If he watches DOORDARSHAN, there is a 128 128 128 128
3 1 16. Ravi and Rashmi are each holding 2 red cards
chance of that he will be asleep, while it is and 2 black cards (all four red and all four black
4 4
cards are identical). Ravi picks a card at random
when he watches TEN SPORTS. On one day, the from Rashmi, and then Rashmi picks a card at
boy is found to be asleep. The probability that random from Ravi. This process is repeated a
the boy watched DOORDARSHAN is second time. Let p be the probability that both
5 2 3 4 have all 4 cards of the same colour. Then p
(a) (b) (c) (d) satisfies
7 7 7 7
(a) p £ 5% (b) 5% < p £ 10%
12. Let the probability Pn that a family has exactly n (c) 10% < p £ 15% (d) 15% < p
children be apn, when n ³ 1 and P0 = 1 – ap 17. The probability of men getting a certain disease
(1 + p + p2 + ...). Suppose that all sex distributions 1
is and that of women getting the same
of n children have the same probability. If k ³ 1 , 2
then the probability that a family contains exactly 1
k boys is disease is . The blood test that identifies the
5
2a pk 4
disease gives the correct result with probability .
(a) (b) 5
( 2 - p )k +1 ( 2 - p )k +1
Suppose a person is chosen at random from a
k
2a . p group of 30 males and 20 females, and the blood
(c) k +1 (d) None of these test of the person is found to be positive. What
(2 - p) is the probability that the chosen person is a
13. A die is thrown 2n + 1 times, n Î N. The man?
probability that faces with even numbers show
75 3 15 3
odd numbers of times, is (a) (b) (c) (d)
107 5 19 10
Probability-2 71

Numeric Value Answer 22. A number x is selected from the set of first 9
natural numbers (i.e., x = 1, 2, 3, ......., 9). If the
18. A and B play a game of tennis. The situation of probability that f(f(x)) = x where f(x) = x2 – 3x + 3
the game is as follows : if one scores two
consecutive points after a deuce, he wins, if loss m
is , then m is equal to _________.
of a point is followed by win of a point, it is 9
deuce. The chance of a server to win a point is
23. A special die is so constructed that the
2 probabilities of throwing 1, 2, 3, 4, 5 and 6 are
. The game is at deuce and A is serving.
3 (1 – k) / 6, (1 + 2k) / 6, (1 – k) / 6, (1 + k) / 6,
Probability that A will win the match is, (serves (1 – 2k) / 6 and (1 + k) / 6, respectively. If two
are changed after each game). such dice are thrown and the probability of
19. The probablilty that the length of a randomly 1 2
getting a sum equal to 9 lies between and ,
9 9
2 5
chosen chord of a circle lies between and then the integral value of k is.
3 6
of its diameter is 24. A sportsman's chance of shooting an animal at a
distance r > a ("a" is constant) is given to be
20. In a hurdle race, a runner has probability p of
jumping over a specific hurdle. Given that in 5 a2
trials, the runner succeeded 3 times, the . He fires at r = 2a and if he misses, then
r2
conditional probability that the runner had
again fires at r = 3a. He repeats the same process
succeeded in the first trial, is
at r = 4a, 5a and 6a. When he misses at r = 6a, the
21. An artillery target may be either at point I with
animal escapes into the jungle. If the odds
8 against the sportsman are p : q, then q – p is
probability or at the point (II) with probability
9 _______.,
25. A determinant of the second order is made with
1
. There are 21 shells each of which can be
9 m
the elements 0 and 1. If be the probability
n
fired either at point I or II. Each shell may hit the
target independently of the other shell with that the determinant made is non-negative, where
m and n are relative primes, then the value of
1 n – m is
probability . Minimum number of shells that
2
must be fired at point I to hit the target with
maximum probability is equal to 2k. Then value
of k is.
ANSW ER KEY
1 (c) 4 (c) 7 (b) 10 (b) 13 (d) 16 (a) 19 (0.25) 22 (2) 25 (3)
2 (c) 5 (b) 8 (a) 11 (c) 14 (a) 17 (a) 20 (0.60) 23 (0)
3 (d) 6 (d) 9 (b) 12 (c) 15 (b) 18 (0.50) 21 (6) 24 (5)
PROPERTIES OF 29
TRIANGLE

MCQs with One Correct Answer 5. If a, b, c be the sides of a triangle and

1. If the sines of the angles A and B of a triangle (a + b + c )2


ABC satisfy the equation c2x2 – c(a + b) x + P= , then
ab + bc + ca
ab = 0, then the triangle is (a,b,c, are sides of D) (a) P Î [1, 2] (b) P Î [3, 4)
(a) acute angled (c) P Î (2, 4] (d) None of these
(b) obtuse angled 6. If a, b, c are the sides of a triangle such that
(c) right angled b. c = l2, for some positive l, then
(d) no such triangle is possible
A B
2. In DABC, a ³ b ³ c . If (a) a ³ 2l sin (b) b ³ 2l sin
2 2
a 3 + b 3 + c3
= 7, then the C
sin 3 A + sin 3 B + sin 3 C (c) c ³ 2l sin (d) all are correct
2
maximum possible value of a is
7. In DABC, a2 (s – a) + b2 (s – b) + c2 (s – c) =
1 (a) 4R(cos A + cos B + cos C)
(a) 7 (b) 49 (c) 3 (d) (b) 4RD(sin A + sin B + sin C)
7 7
3. If in a triangle ABC, sin A, sin B, sin C are in (c) æ A B Cö
4RD ç 1 + 4 sin sin sin ÷
A.P., then è 2 2 2ø
(a) the altitudes are in A.P. (d) None of these
(b) the reciprocals of altitudes are in A.P.
8. In an obtuse angled triangle, the obtuse angle is
(c) the altitudes are in G.P.
(d) the medians are in A.P. 3p
and the other two angles are equal to two
4. Let a, b and A are given and c1 and c2 are two 4
values of c, then the value of c12 + c22 – 2c1c2 values of q satisfying a tan q + b sec q = c,
cos 2A =
2 2
(a) a2sin2A (b) a2 cos2 A where | b | £ a + c , then a2 – c2 is equal to
2
(c) 2(a + b ) 2 (d) 4a2cos2 A (a) ac (b) 2ac
a
(c) (d) None of these
c
Properties of Triangle 73

1 (a) 6 (b) 6 6
9. In a triangle ABC, 2ac sin ( A - B + C ) =
2 (c) 6 (d) (216)1/4
(a) a2 + b2 – c2 (b) c2 + a2 – b2 16. In triangle ABC if A : B : C = 1 : 2 : 4, then (a2 – b2)
(b2 – c2) (c2 – a2) = l a2b2c2, where l =
(c) b2 – c2 – a2 (d) c2 – a2 – b2
(where notations have their usual meaning)
10. The sides of a triangle are sin a, cos a and (a) 1 (b) 2 (c) 4 (d) 9
17. If in a right angle triangle ABC, 4 sinA cosB – 1 = 0
p
1 + sin a cos a for some 0 < a < . Then the and tan A is real, then
2
(a) angles are in A.P. (b) B2 = AC
greatest angle of the triangle is
(a) 150° (b) 90° (c) 120° (d) 60° 2 AC
(c) B= (d) None of these
11. Given a triangle DABC such that sin2 A + sin2 C A+ C
=1001 × sin 2 B. Then the value of 18. A tower of height b subtends an angle at a point
2 (tan A + tan C) × tan 2 B O on the level of the foot of the tower and at a
tan A + tan B + tan C distance a from the foot of the tower. If a pole
1 1 1 1 mounted on the tower also subtends an equal
(a) (b) (c) (d)
2000 1000 500 250 angle at O, the height of the pole is
12. In DABC, if æ a 2 - b2 ö æ a2 + b2 ö
(a) bç ÷ (b) b ç ÷
sin A sin B sin C c b a ç a 2 + b2 ÷ ç a 2 - b2 ÷
+ + = + + , then è ø è ø
c sin B c b ab ac bc
the value of angle A is æ a 2 - b2 ö æ a2 + b2 ö
(c) aç ÷ (d) aç ÷
(a) 120° (b) 90° (c) 60° (d) 30° ç a2 + b2 ÷ ç a 2 - b2 ÷
è ø è ø
13. If the hypotenuse of a right-angled triangle is
four times the length of the perpendicular drawn 19. An observer on the top of a tree, finds the angle
from the opposite vertex to it, then the differ- of depression of a car moving towards the tree
ence of the two acute angles will be to be 30°. After 3 minutes this angle becomes
(a) 60° (b) 15° (c) 75° (d) 30° 60°. After how much more time, the car will reach
the tree?
p p (a) 4 min. (b) 4.5 m
14. In a DABC, ÐB = and ÐC = let D divide
3 4 (c) 1.5 min (d) 2 min.
20. Two flagstaffs stand on a horizontal plane. A
sin (ÐBAD)
BC internally in the ratio 1 : 3, then and B are two points on the line joining their feet
sin (ÐCAD)
and between them. The angles of elevation of
is equal to: the tops of the flagstaffs as seen from A are 30°
and 60° and as seen from B are 60º and 45°. If AB
1 1 1 2
(a) (b) (c) (d) is 30m, then the distance between the flagstaffs
6 3 3 3
in metres is
15. In a triangle ABC, a = 5, b = 4 and cos (A – B)
(a) 30 + 15 3 (b) 45 + 15 3
31
= , then the third side is equal to:
32 (c) 60 - 15 3 (d) 60 + 15 3
(where symbols used have usual meanings)
74 MATHEMATICS

Numeric Value Answer 24. AD, BE, CF are internal angular bisectors of
A
21. Points D, E are taken on the side BC of an acute DABC and I is the incentre. If a (b + c) sec
2
angled triangle ABC, such that BD = DE = EC. If B C
ÐBAD = x, ÐDAE = y and ÐEAC = z then the ID + b(a + c) sec IE + c(a + b) sec
2 2
sin (x + y) sin (y +z)
value of is _________. IF = kabc, then the value of k is
sin x sin z
25. A man from the top of a 100 metres high tower
22. In triangle ABC, sin A sin B + sin B sin C + sin C
see a car moving towards the tower at an angle
sin A = 9/4 and a = 2, then the value of 3D, of depression of 30°. After some time, the angle
where D is the area of triangle, is _________. of depression becomes 60°. If the distance (in
23. Triangle ABC is right angled at A. The points P metres) travelled by the car during this time is
and Q are on hypotenuse BC such that BP = PQ a b æ a ö
, then ç - b ÷ =
= QC. If AP = 3 and AQ = 4, if BC = a b then 3 è 20 ø

| a - b |=

ANSWER KEY
1 (c) 4 (d) 7 (c) 10 (c) 13 (a) 16 (a) 19 (c) 22 (3) 25 (7)
2 (c) 5 (b) 8 (b) 11 (d) 14 (a) 17 (a) 20 (d) 23 (2)
3 (b) 6 (a) 9 (b) 12 (b) 15 (c) 18 (b) 21 (4) 24 (2)
Hints & Solutions
CHAPTER
1 Sets
8. (a) bN = {bx : x ÎN}; cN = {cx : x ÎN}
1. (a) B A \ bN Ç cN = {x : x is multiple of b and c both}
= { x: x is multiple of l.c.m. of b and c }
B¢ – A¢ and A – B
= { x : x is multiple of b c}
[given b and c are relatively prime \ l.c.m. of b
and c = bc]
2. (b) Let A = {q : sin q = tan q} \ bN Ç cN = {bc x : x ÎN} = dN (Given)
and B = {q : cos q = 1} \ d = bc.
ì sin q ü 9. (a) (A È B È C) È (A Ç B' Ç C' )' Ç C'
\ A = íq : sin q = ý = (A È B È C) Ç (A' È B È C ) Ç C'
î cos qþ
= [(A Ç A') È (B È C)] Ç C' = (f È B È C) Ç C'
= {q : sin q (cos q – 1) = 0} = {q = 0, p, 2p, 3p,.....} = (B È C) Ç C' = (B Ç C') È (C Ç C’)
For B : cos q = 1 Þ q = p, 2p, 4p,...... = (B Ç C') È f = B Ç C'
This shows that A is not contained in B. i.e. A Ë B. 10. (a) Total number of persons = a + b + c + n
but B Ì A. = 100
3. (c) n(P(S)) = 23 = 8 elements.
Total 100
n(P(P(S))) = 28 = 256 elements.
4. (c) Let n be the number of newspapers which Fish (50)
Chicken (60)

are read. Then 60 n = (300) (5) Þ n = 25


a c b
5. (a) From Venn-Euler’s Diagram.

(A È B) '
U n

(A 'Ç B) Do not prefer fish b + n = 50


60 prefer chicken hence b + c = 60
A B Do not like fish and chicken is n = 10
On solving these equations we will get a = 30,
b = 40, c = 20
\ (A È B) ' È (A 'Ç B) = A ' The number of persons who prefer both fish
6. (b) Given set can be written as and chicken is = c = 20
(A – B) È (B – A) = (A È B) – (A Ç B) 11. (c) {(A – B) È (B – C) È (C – A)}¢
(By definition of symmetric difference) = (A – B)¢ Ç (B – C)¢ (C – A)¢
Hence, (A \ B) È (B \ A) = (A È B) \ (A Ç B) = [(U – (A – B)) Ç (U – (B – C) Ç (U – (C – A))]
7. (b) A = {x : |x| < 1} = (–1, 1) A B
Since, |x| < 1 Þ –1< x < 1
B = {x : |x–1| ³ 1} = (– ¥, 0] È [2, ¥) A– B
Since, |x –1| ³ 1 Þ x–1£ –1 or x – 1³ 1
Þ x £ 0 or x ³ 2 C –A C
\ A È B = (– ¥ , 0] È [2, ¥ ) È (–1, 1)
= (– ¥, 1) È [2, ¥ ) = R – [1, 2) By Venn - diagram = (A Ç B Ç C)
\ D = [1, 2) = { x : 1 £ x < 2}
76 MATHEMATICS
12. (d) n(A) = 1000, n(B) = 500, Hence a = 20k and b = 60k, then
n(A Ç B) ³ 1 & n(A È B) = p n = 100k – 20k – 60k = 20k
Also, n(A È B) = n(A) + n(B) – n(A Ç B) The difference between those who opted for
Þ p = 1000 + 500 – n (A Ç B) ‘Surf blue’ and those who were uncertain
Q 1 £ n(A Ç B) £ 500 = 60k – 20k = 40k = 720 hence, k = 18,
Hence p £ 1499 and p ³ 1000 Þ 1000 £ p £ 1499 Hence total number of persons covered in
13. (a) Minimum value of n survey
= 100 – (30 + 20 + 25 +15) = 100 – 90 = 10 = 100k = 1800
14. (a) Let the number of students who take only 18. (c) From the given information (C) = 80,
Math be x and only Chemistry be y. (F) = 40, and (C Ç F) = (n)
Hence (U) = (C) + (F) – (C Ç F) + (n)
M C
Or 80 + 40 – x + x = 120
x 30
y
19. (d) If number of students who like chocolate
=a+c
Number of students who like Biscuit = b + c
So, from the Venn diagram, we have total number Number of students who like Both = c
of students who take Math = x + 30 and take Number of students who like none = n = 10
Chemistry = y + 30. From the given condition 100 = a + b + c + n
According to question, we have Since 60 of them don’t like Chocolate, hence
10 b + n = 60 or b = 50
30 = ( x + 30) And 50 of them don’t like Biscuit hence a + n
100
= 50, a = 40
12
Þ x = 270 and 30 = ( 30 + y ) Hence 100 = 40 + 50 + c + 10 or c = 0
20. (b) The given information can be represented
100
Þ y = 220 as follows.
x + y + 30= 270 + 220 + 30 = 520. U = 100
15. (a) See the following Venn diagram
I M F
23
4–x

x
29
W
n (I) = 29 + 23 = 52; n (F) = 100 – 52 = 48
n (m È D ) = n ( m ) + n ( D ) - n ( m Ç D )
24 = 23 + 4 - n (m Ç D) \ n (m Ç D) = 3
\ n( W Ç D) = 4 - 3 = 1 Now from the given information we can frame
16. (b) C stands for set of students taking following equations-
economics c = 18, f + c = 23, f + g = 8,
E C c + f + g + e = 28, a + d + f + g = 48,
a b c d + g = 10, n = 24
g
People who read exactly two consecutive
d e months is represented by d and e.
f f + c = 23 and c = 18 \ f= 5
f + g = 8 and f = 5 \ g=3
M d + g = 10 and g = 3 \ d=7
a + b + c + d + e + f + g = 40; a + b + d + g = 16 c + f + g + e = 28, c = 18, f = 5 and g = 3
b + c + e + g = 22; d + e + f + g = 26 \ e = 2 or \ d + e = 9
b + g = 5; e + g = 14; g = 2 21. (4) We have
Go by backward substitution
e = 12, b = 3, d + f = 12, c + e = 17 Þ c = 5; n (U) = 100, where U stands for universal set
a + d = 11 n (M Ç C Ç T) = 10; n (M Ç C) = 20;
a +d + f = 18 Þ f = 7 \ d = 12 – 7 = 5 n (C Ç T) = 30; n (M Ç T) = 25;
17. (c) Let those who opted for Nirma = a and n (only M ) = 12; n (only C) = 5; n (only T) = 8
those who opted Surf Blue = b and those who Filling all the entries we obtain the Venn diagram
opted for none is n. as shown below :
Solutions 77

C U = Total students – n(C È F È H)


M
12 10 5 n 28
n(C¢ Ç F¢ Ç H¢)= 80 – 52 = 28, So = =7
10 4 4
15 20
23. (7) The given condition is as follows-
8 T

\ n (M È C È T)
= 12 + 10 + 5 + 15 + 10 + 20 + 8 = 80
\ n (M È C È T)' = 100 – 80 = 20 = n.
n 20
So, = =4
5 5
22. (7) Numbers which are divisible by 5 are 5,
10, 15, 20, 25, 30, 35, 40, 45, 50, 55, 60, 65, 70,
75, 80 they are 16 in numbers. Now, Numbers
which are divisible by 7 are 7, 14, 21, 28, 35, 42, We know that {(a + d + e + g) + ( b + d + f + g)
49, 56, 63, 70, 77 they are 11 in numbers. + (c + e + f + g)} – ( d + e + f) – 2g
Also, total odd numbers = 40 =a+b+c+d+e+f+g
Let C represents the students who opt. for or 61x + 46x + 29x – 25x – 2g = 97x
cricket, F for football and H for hockey. or 2g = 14x or g = 7x. So 7% of people
\ we have n(C) = 40, n(F) = 16, n(H) = 11 watched all the three movies)
Now, C Ç F = Odd numbers which are divisible 24. (4) n (A È B) is minimum when A Í B. In this
by 5. case, (A È B) = B and hence minimum value of
CÇ H = Odd numbers which are divisible by 7. n (A È B) = n(B) = 7.
F Ç H = Numbers which are divisible by both 5 n (A È B) is maximum when A and B are disjoint.
and 7. \ Maximum value of n (A È B) = 4 + 7 = 11.
n(C Ç F), 8, n(C Ç H) = 6, So 11 – 7 = 4
n(FÇ H) = 2, n (C Ç F Ç H) = 1
We Know 25. (3) 2m - 2n = 112 Þ 2 n (2 m- n - 1) = 16.7
n(CÈFÈH) = n(C) + n(F) + n(H) \ 2n (2m -n - 1) = 24 (23 - 1)
– n(C Ç F) – n(C Ç H)
– n(F Ç H) + n(C Ç H Ç F) Comparing, we get n = 4 and m – n = 3
n(CÈFÈH) = 67 – 16 + 1 = 52 Þ n = 4 and m = 7
\ n(C¢ Ç F¢ Ç H¢) So m – n = 7 – 4 = 3
CHAPTER
2 Relations & Functions-1
1. (b) Put x = y = 1, (f (1))2 = 3 f (1) – 2
= ( x14 - x11 ) + ( x 6 - x3 ) + ( x 2 + 1) > 0
Þ f (1) = 1 or 2
Case 2 : 0 £ x £ 1
Let f (1) = 1, then put y = 1
x14 - x11 + x 6 - x3 + x 2 + 1
f (x) . f (1) = f (x) + f (1) + f (x) – 2
= x14 - {( x11 - x6 ) + ( x3 - x 2 )} + 1 > 0
Þ f (x) = 1 constant function \ f (1) ¹ 1,
{Q x11 - x6 £ 0, x 3 - x 2 £ 0 }
hence f (1) = 2
Case 3 : x < 0
2. (a) Given
x14 - x11 + x 6 - x3 + x 2 + 1 > 0
f ( x) = x14 - x11 + x 6 - x3 + x 2 + 1
for f (x) to be defined, (Q x11 < 0, x3 < 0, x14 , x 6 , x 2 > 0)
Thus, for all real x,
x14 - x11 + x6 - x 3 + x 2 + 1 ³ 0
Case 1 : x ³ 1 x14 - x11 + x6 - x 3 + x 2 + 1 ³ 0
x14 - x11 + x 6 - x3 + x 2 + 1 Hence, the domain of f (x) = R = (-¥, ¥)
78 MATHEMATICS
3. (a) Given f (x) = cos (log x) 1
\ f (xy) = cos (log xy) Replacing x + 1 by , we get
x +1
f (xy) = cos [log x + log y] ....(1)
æ 1 ö 1
æ xö æ ö af ç ÷ + bf ( x + 1) = -1
And f ç ÷ = cos log x è x +1ø x +1
...(2)
è yø çè y ÷ø Eq. (1) × a – Eq. (2) × b
æ xö b
f ç ÷ = cos (log x – log y) ....(2) Þ ( a 2 - b 2 ) f ( x + 1) = a ( x + 1) - a - +b
è yø x +1
Adding (1) and (2), we get
2 2 b
æ xö Putting x = 1, (a - b ) f (2) = 2a - a - +b
f (xy) + f ç ÷ 2
è yø
b 2a + b
= cos (log x + log y) + cos (log x – logy) =a+ =
= 2 cos (log x). cos (log y) 2 2
7. (d) f (x) = cos(sin x ) + log x {x}
æ xö
Þ f (xy) + f ç ÷ = 2 f (x). f (y) Domain cos(sin x) ³ 0 {x} > 0, x > 0, x ¹ 1,
è yø
logx {x} ³ 0
Then the value of f (x)f (y) (i) cos(sin x) ³ 0 for all x, x Î R [–1, 1]
1 ì æ xö ü (ii) {x} > 0, x Ï Int. (iii) x > 0, x Î(0, ¥)
– í f ç ÷ + f ( xy)ý (iv) x ¹ 1
2 î è yø þ
1 (v) log x {x} ³ 0 Þ 1 > f ( x ) ³ 0 ,
= f ( x) f ( y) - .2 { f ( x). f ( y)} = 0
2 so 1 > x ³ 0 logx f(x) is positive x Î[0, 1)
4. (c) We must x 4 - 21x 2 ³ 0 and Þ x Î(0, 1)
8. (c) Given that,
10 - x 4 - 21x 2 ³ 0
æ px ö
( )
1
Þ x 2 x 2 - 21 ³ 0 ...(1) f ( x ) = - tan ç ÷ , -1 < x < 1
2 è 2ø
4 2 Here, domain of f(x) = (– 1, 1) and
and 100 ³ x - 21x ...(2)
Eq. (1) gives x = 0 or x £ - 21 or x ³ 21
g ( x ) = 3 + 4 x - 4 x 2 = -(2 x - 3)(2 x + 1)
Eq. (2) Þ x 4 - 21x 2 - 100 £ 0
1 3
Þ ( )(
x 2 - 25 x 2 + 4 £ 0) Þ (2x – 3 ) (2x +1) £ 0 Þ -
2
£x£
2
Þ x 2 - 25 £ 0 (as x2 + 4 > 0 always) Domain of g(x) = éê- , ùú
1 3
\
Þ -5 £ x £ 5 ë 2 2û
Domain is given by Hence, domain of (f + g) = intersection of their
é -5, - 21 ù È é 21,5 ù È {0} é 1 ö
ë û ë û domains = ê - ,1÷ .
ë 2 ø
æ x + 59 ö 9. (b) We have f(x)
5. (b) 3 f ( x) + 2 f ç ÷ = 10 x + 30
è x -1 ø = log4[log5{log3 (18x – x2 – 77)}]
For x = 7, 3f (7) + 2f (11) = 70 + 30 = 100 Since, loga x is defined for all x > 0, f(x) is defined if
For x = 11, 3f (11) + 2f (7) = 140 log5 {log3 (18x – x2 – 77)} > 0 and 18x – x2 – 77 > 0
f (7) f (11) -1 or log3 (18x – x2 – 77) > 50 and x2 – 18x + 77 < 0
= = or f(7) = 4 or log3 (18x – x2 – 77) > 1 and (x – 11)(x – 7) < 0
-20 -220 9 - 4 or 18x – x2 – 77 > 31 and 7 < x < 11
æ 1 ö or 18x – x2 – 80 > 0 and 7 < x < 11
6. (a) af ( x + 1) + bf ç ÷ = ( x + 1) - 1 ...(1) or x2 – 18x + 80 < 0 and 7 < x < 11
è x +1 ø
Solutions 79
or (x – 10) (x – 8) < 0 and 7 < x < 11 1 1 1 1
or 8 < x < 10 and 7 < x < 11 or 8 < x < 10 \ a = 1+ , 1+ , 4 + , 4 +
or x Î (8, 10) 3 2 3 2
Hence, the domain of f(x) is (8, 10). 35
1 x Sum of all possible values of a =
-x 3
10. (a) Clearly, g(x) = (a + a ) and
2 æ 5x ö
13. (c) f (x) = cos nx . sin ç ÷ ;
1 è nø
h( x ) = (a x - a - x ) . Now g(x + y) + g(x – y)
2
2p
1 x+ y 1 Period of cos nx =
= (a + a - ( x + y ) ) + (a x - y + a - x + y ) |n|
2 2
5x 2p 2 | n | p
1 x y x -y -x y -x -y Period of sin = =
= (a a + a a + a a + a a ) n 5 5
2
n
1 x y -y -x y -y
= (a (a + a ) + a (a + a )) æ 2p 2 | n | p ö
2 \ Period of f (x) = L.C.M. ç , = 2p
æ1 x öæ1 ö è|n| 5 ÷ø
= 2 ç ( a + a - x )÷ ç ( a y + a - y ) ÷ = 2 g ( x ) g ( y ) (given)
è2 øè2 ø
11. (b) Given f (l + x) = f (l – x) ...(1)
f (2l + x) = – f (2l – x) ...(2) æ 1 | n |ö L.C.M. (1, | n |)
Þ L.C.M. ç , =1= =1
for l > 0 è | n | 5 ÷ø H.C.F. (| n |, 5)
Replacing x by l – x in l – x in (1), we get
f(2l – x) = f(x) ...(3) |n|
Þ = 1 Þ H.C.F. (|n|, 5) = |n|
\ From (2) and (3), f(x) = –f(2l + x) H.C.F. (| n |, 5)
Þ f(x) = –[–f (2l + 2l + x)] If g.c.d. (|n|, 5) = 1 Þ |n| = 1 Þ n = 1
Þ f(x) = f(x + 4l) ...(4) If g.c.d. (|n|, 5) ¹ 1 Þ |n| = 5m; m Î ¥
Þ f(x) is periodic with period 4l. Þ g.c.d. (5m, 5) = 1
Further from (3), replacing x by –x, we get Þ |n| = 5 Þ n = ±5 \ n Î {±1, ±5}
f (2l + x) = f (–x) ...(5)
From (2), (3) and (5), we have 14. (d) f : ¡ ® ¡ , g; ¡ ® ¡
f (–x) = f (2l + x) = –f (2l – x) = –f (x) We know that min. {f1(x), f2(x)}
i.e. f (–x) = –f (x) ( f1 ( x) + f 2 ( x )) - | f1 ( x) - f 2 ( x) |
Þ f(x) is odd function =
Thus, f is odd and periodic function. 2
\ min {f(x) – g(x), 0}
12. (d) f ( x) = ax3 - bx - (tan x )sgn x ( f ( x ) - g ( x ) + 0) - | f ( x) - g ( x) - 0 |
Since f (- x) = f ( x) =
2
Þ -ax3 + bx - tan x sgn x ( f ( x ) - g ( x)) - | f ( x ) - g ( x) |
=
2
= ax 3 - bx - (tan x ) (sgn x ) 15. (b) y = f(ex) + f(|ln| |x|) Domain f(x) = (0, 1)
Þ ( )
2 ax 2 - b x = 0 " x Î R Þa = 0 and b = 0
Þ 0 < ex < 1 Þ x < 0
and 0 < ln |x| < 1 Þ 1 < |x| < e
...(1)

Þ x Î {–e, –1} È (1, e) ... (2)


\ [ a ]2 - 5 [ a ] + 4 = 0 Taking intersection x Î (–e, –1)
æ 1ö
and 6 {a}2 - 5{a} + 1 = 0 16. (b) f ç x + ÷ = f ( x ); f (2) =5;
è 2ø
Þ ([a] – 1)([a] – 4) = 0 and
(3{x} – 1) (2{x} – 1) = 0 æ9ö æ1ö
f ç ÷ = 2, f (-3) - f ç ÷ =?
1 1 è4ø è4ø
Þ [a] = 1, 4 and {a} = ,
3 2
80 MATHEMATICS

1 1
Q f(x) is periodic with period Þ 0 <1+ < 2- 1
2 4 x
æ nö 1 1
Þ f(x) = f ç x + ÷ " x Î ¥ Þ -1 < <- Þ x Î f (null set)
è 2ø 4 x 2
æ 10 ö 20. (d) y
Þ f(–3) = f çè - 3 + ÷ø = f (2) = 5
2
1
æ 1ö æ1 æ 1ö ö æ 9ö
and f ç ÷ = f ç + 4 ç ÷ ÷ = f ç ÷ = 2
è 4ø è4 è 2ø ø è 4ø
x
0 1 2 3
æ 1ö 0 £ f (x) £ 1 Þ 0 £ 7 f(x) £ 7
\ f (-3) - f çè ÷ø = 5 - 2 = 3
4 Þ – 1 £ sin (7 f(x)) £ 1
é 2x + 3 x £ 1 1
17. (c) f(x) = ê 2 21. (5.5) Here f ( - x ) =
êë a x + 1 x > 1 1 + ex
For x £ 1 ; f(x) £ 5 1 1
So for range of f(x) to be R. So, f ( x ) + f ( - x ) = -x
+
1+ e 1 + ex
Þ a2 + 1 £ 5 and a ¹ 0 Þ a Î [–2, 2]
Hence, a = {–2, – 1, 1, 2} ex 1 ex +1
= + = =1
18. (b) g ( 2 f ( x) + 3) ex + 1 1+ ex ex +1
= log 5 ( 2 (sin x - cos x) + 3) \ S = {f ( 5 ) + f ( -5 )} + ... + {f (1) + f ( -1)} + f ( 0 )
We know that 1 1 11
= 1 + 1 + 1 + 1 + 1 + f(0) = 5 + = = 5+
- 2 £ (sin x - cos x) £ 2 " x Î ¡ 1+ e 0
2 2
22. (2) f(x + p) = 1 + {1 – 3f (x) + 3f 2(x) – f 3(x)}1/3
éQ - a 2 + b2 £ a sin x + b cos x £ a 2 + b 2 ù Þ f(x + p) = 1 + (1 – f(x)) = 2 – f(x)
ëê ûú Þ f(x + p) = 2 – [2 – f(x – p)]
Þ - 2 £ 2(sin x - cos x) £ 2 Þ f(x + p) = f(x – p)
Þ f(x) = f(x + 2p)
Þ 1 £ 2(sin x - cos x) + 3 £ 5 Þ Period of f(x) = 2p = lp (given)
Þ l=2
Þ 0 £ log 5
( 2 (sin x - cos x ) + 3) £ 2 23. (9) Given f(x + 2) = f(x) + f(2)
(Q logax is increasing for a > 1) Put x = –1. Then f(1) = f(–1) + f(2)
or f(1) = –f(1) + f(2) [as f(x) is an odd function]
Hence, range of g ( 2 f ( x) + 3) is [0, 2].
or f(2) = 2f(1) = 6
æ 1 ö Now, put x = 1
19. (d) –g(2, f(x)) = - log 2 ç 1 + ÷ We have f(3) = f(1) + f(2) = 3 + 6 = 9
è 4 xø 24. (6) Given f(x, y) = f(2x + 2y, 2y – 2x) " x, y Î ¡ ,
æ 1 ö f(x) = f(2x, 0) and f(x) is periodic with period k.
Þ –g(2, f(x)) – 1 = - log 2 ç1 + ÷ -1 Þ f(x) = f(2x, 0) = f(2.2x + 2(0), 2(0) – 2.2x)
è 4 xø = f(2x + 1, 2x +1)
æ1 ö = f(2.2x + 1 – 2.2x + 1, – 2.2x + 1 – 2.2x + 1)
\ g ç , - g (2, f ( x )) - 1÷ = f (0, – 2x + 3)
è2 ø = f(2.(–2x + 3), –2.2x + 3) = f(– 2x + 4, – 2x + 4)
æ æ 1 ö ö = f(–2x + 6, 0)
= log1/2 ç - log 2 ç 1 + ÷ - 1÷ = f(–2x +7, 2x + 7) = f(0, 2x + 9)
è è 4 xø ø = f(2x + 10, 20x + 10) = f(2x + 12, 0) = f(x + 12)
æ 1 ö Þ f(x) is periodic with period 12 Þ k = 12.
Þ log 2 ç1 + ÷ +1< 0
è 4 xø
Solutions 81
45 £ x < 60 f(x) = – 3
é x ù é 15 ù
25. (6) f(x) = ê ú ê - ú x Î (0, 90) 60 £ x < 75 f(x) = – 4
ë15 û ë x û 75 £ x < 90 f(x) = – 5
0 £ x < 15 f(x) = 0 Total integers in range f(x) = {0, –1, –2, –3, –4, –5}
15 £ x < 30 f(x) = – 1
30 £ x < 45 f(x) = – 2
CHAPTER
3 Trigonometric Functions
1 1 2 4 b
1. (d) + + + 3 sin 2
2
cos a 1 + sin a 2 4
1 + sin a 1 + sin a 8 a 2
Þ sin 2 = …(ii)
2 b
2 2 4 1 + 2 sin 2
= + + 2
1 - sin 4 a 1 + sin 4 a 1 + sin8 a Divide equation (ii) by (i)
4 4 8 a
= + = tan
1 - sin 8 a 1 + sin 8 a 1 - sin16 a a b 2 = 3
tan 2 = 3 tan 2 Þ
8 40 2 2 b
= = = 10 æ 16 1ö tan
1 4 çQ sin a = ÷ 2
1- è 5ø 4. (b) Gives equations can be written as
5
2. (b) a < b < g < d and 2 cos a + 9 cos d = – 6 cos b – 7 cos g ..(i)
sin a = sin b = sin g = sin d = k 2 sin a – 9 sin d = 6 sin b – 7 sin g ...(ii)
Square and add equations (i) and (ii),
Þ b = p - a , g = 2p + a, d = 3p - a .
Þ 4 + 36 + 36 [cos a cos d – sin d sin a]
So that the given expression is equal to
= 36 + 49 + 84 [cos b cos g – sin b sin g]
a æ p-a ö 2p + a 3p - a
4 sin + 3 sinç ÷ + 2 sin + sin Þ 36 [cos(a + d)] = 84 [cos (b + g)]
2 è 2 ø 2 2
cos (a + d) 84 7 m
a a a a = = = ; m + n = 10
= 4sin + 3cos - 2sin - cos cos (b + g ) 36 3 n
2 2 2 2
5. (a) sin q + sin 3q + sin 2q = sin a
æ a aö a a Þ 2 sin 2q cos q + sin 2q = sin a
= 2çsin + cos ÷ = 2 1+ 2sin cos = 2 1+ k
è 2 2ø 2 2 Þ sin 2q(2 cos q + 1) = sin a ...(i)
3. (c) 1 + cos a = 1 +
2 cos b - 1 2 - cos b + 2cos b - 1
= Now cos q + cos 3q + cos 2q = cos a
2 - cos b 2 - cos b 2 cos 2q cos q + cos 2q = cos a
b cos 2q (2 cos q + 1) = cos a ...(ii)
cos 2
1 + cos b 2 a 2 From (i) and (ii),
= Þ cos = … (i)
2 - cos b 2 b tan 2q = tan a Þ 2q = a Þ q = a/2
1 + 2 sin 2
2 6. (c) (sin 7a + sin 5a ) + 5(sin 5a + sin 3a)
b
cos 2 +12(sin 3a + sin a )
2a 2
Þ 1 - cos =1- sin 6a + 5 sin 4a + 12 sin 2a
2 b
1 + 2sin 2 2sin 6a cos a + 5(2sin 4a cos a)
2
b b b b + 12(2sin 2a cos a )
1 + 2sin 2 - cos 2 1 + 2 sin 2 - [1 - sin 2 ] =
2 2= 2 2 sin 6a + 5sin 4a + 12sin 2a
=
2b 2b 5 +1
1 + 2 sin 1 + 2 sin = 2 cos a =
2 2 2
82 MATHEMATICS
7. (b) 2
æ qö q
é 2sin q cos q 2sin 3q cos3q 2sin9q cos9q ù = 4 ç 2 cos q× cos ÷ = 16 cos 2 q× cos 2
+ + è 2 ø 2
1 êê cos q cos3q cos9q cos3q cos9q cos 27q úú = 4 (1 + cos q) (1 + cos 2q)
2ê 2sin 27q cos 27q ú
ê + 11. (a) Let
ë cos 27q cos81q úû
ì 2p ü ìï 22 p üï ìï 264 p üï
é sin(3q - q) sin(9q - 3q) sin(27q - 9q) ù
+ +
P = cos í ý cos í 64 ý ...cos í 64 ý
ê
1 ê cos q cos3q cos3q cos9q cos9q cos 27 q ú
ú î 264 - 1 þ îï 2 - 1 þï îï 2 - 1 þï
= 2ê sin(81q - 27q) ú
ê +
ë cos 27q cos81q úû æ 22 p ö æ 22 p ö
1
Þ P= sin ç ÷ cos ç ÷
1 1 é sin 80 q ù
[tan 81q - tan q] = ê æ 2p ö çè 264 -1 ÷ø çè 264 -1 ÷ø
= 2sin ç
2 2 ë cos q cos 81q úû ÷
è 264 -1 ø
p+q 1 p+q æ 264 p ö
8. (b) cosec q = Þ =
p-q sin q p - q ...cos ç ÷
ç 264 - 1 ÷
Apply componendo and dividendo è ø

1 + sin q p + q + p - q 1 æ 23 p ö æ 264 p ö
= Þ P= sin ç ÷ ...cos ç ÷
1 - sin q p + q - p + q æ 2p ö çè 264 -1÷ø ç 264 -1÷
è ø
2sin ç ÷
2 2 è 264 -1ø
ì q qü ì qü
ï cos + sin 1 + tan
2 2ï = p ï 2ï = p 1 æ 265 p ö 1
Þí ý Þ í ý Þ P= sin ç ÷ = 64
q q q ï1 - tan q ï q ç ÷
æ 2 p ö 64
ï cos - sin ï 264 sin ç è 2 -1 ø 2
î 2 2þ î 2þ ÷
è 264 - 1 ø
æp qö p 2æp qö q
Þ tan 2 ç + ÷ = Þ cot ç + ÷ = 12. (c) sin x sin y + 3 cos y + 4 sin y cos x = 26
è 4 2ø q è 4 2ø p
\ 3 cos y + (sin x + 4 cos x) sin y = 26
9. (b) Given, sin 2q + sin 2f = 1/2 ...(i)
\ 3 cos y + (sin x + 4 cos x) sin y
and cos 2q + cos 2f = 3/2 ...(ii)
Square and adding, £ 9 + (sin x + 4 cos x) 2 £ 9 + 1 + 16 = 26
\ (sin2 2q + cos2 2q) + (sin2 2f + cos2 2f) \ sin x sin y + 3 cos y + 4 sin y cos x = 26
+ 2[sin 2q sin 2f + cos 2q cos 2f] = 1/4 + 9/4 cos y sin y cos x
Þ sin x sin y = =
Þ cos 2q cos 2f + sin 2q sin 2f = 1/4 3 4
Þ cos (2q – 2f) = 1/4 Þ cos2 (q – f) = 5/8 Þ 3 tan y = cosec x and tan x = 1/4
Þ 9 tan2 y = cosec2 x = (1 + cot2 x) = 17
10. (a) u = 1 + cos q + cos 2q + cos (q + 2q)
1 9
= (1 + cos 3q) + (cos q + cos 2q) Þ tan2 x + cot2 y = +
16 17
3q 3q 3q x x x x x
= 2 cos 2 + 2 cos + cos 13. (b) cos × × cos × × cos × ×.....cos × × cos ×
2 2 2 256 128 64 4 2
3q é 3q qù sin x
= 2 cos cos + cos ú
2 êë 2 2û =
æ x ö
256 sin ç
3q é 3q qù è 256 ÷ø
Similarly v = 2 sin ê cos + cos ú
2 ë 2 2û 14. (a) – 5 £ 3 sin x – 4 cos x £ 5
2
10 £ 3 sin x – 4 cos x + 15 £ 20
æ 3q qö log20 10 £ log20 (3 sin x – 4 cos x + 15) £ log20 20
\ u2 + v2 = ç cos + cos ÷
è 2 2ø
Solutions 83
15. (c) Given expression 19. (a) sin2 (sin x) – 3 sin (sin x) + 2 = 0
sin 50° + sin 140° + sin 170° {sin (sin x) – 2} {sin(sin x) – 1} = 0
= Equation has no solution.
2 sin 25° sin 70° sin 85° 20. (c) x2 + 12 + 3 sin (a + bx) + 6x = 0
4 Þ (x + 3)2 + 3 + 3 sin (a + bx) = 0
= (Using sin 2A + sin 2B + sin 2C = 4 sin A Þ (x + 3)2 + 3 = – 3 sin (a + bx)
2 L.H.S. ³ 3 but R.H.S. £ 3
sin B sin C, where A + B + C = 180°) L.H.S. = R.H.S. = 3
=2 \ x = – 3 and sin (a + bx) = – 1
16. (c) 32(1 + tan
2
x)
+ 1 - 10 × 3tan
2
x
=0 Þ sin (a – 3b) = – 1
p
Þ
2
32 (3tan x )2 + 1 - 10.3tan
2
x
=0. or a – 3b = (4n – 1) n Î Z.
2
2 1 1 1
Put 3tan x
=t; 9t2 – 10t + 1 = 0 21. (50) 5 2 + 5 2
+ log5 (sin x)
= 15 2
+ log5 cos x

1 1 1
Þ t= and t = 1
9 Þ 52 + × 5log5 (sin x) = 151/ 2 ×15 log 15 cos x
52
Þ 3tan 2 x = 3–2 and 3tan 2 x = 1 Þ 5 + 51/2 × sin x = 151/2 × cos x
1/2

Þ tan2x = –2 gives no solution and tan2 x = 1 Þ 1 + sin x = 3 cos x


gives four solutions.
17. (c) (4cos2 2x + 4 cos2x + 1) 3 sin x 1
Þ cos x - =
2 2 2 2
+ (tan x – 2 3 tan x + 3) = 0
æ pö p
Þ (2 cos 2x + 1)2 + (tan x – 3 )2 = 0 Þ cos ç x + ÷ = cos
1 è 6 ø 3
Þ cos 2x = - and tan x = 3 p p
2 Þ x + = 2np ± , n Î Z
1 6 3
Þ 2cos2 x – 1 = - and tan x = 3 p p
2 Þ x = 2np - , 2np + , n Î Z .
1 2 6
Þ cos x = ± - and tanx = 3 But we must have sin x, cos x > 0
2
\ x = 2np + p/6, n Î Z.
p 4p
Þ x= , sin x sin 3x sin 9x
3 3 22. (6) + + =0
\ Number of solutions of equation = 2. cos 3x cos 9x cos 27x
2cos2 x - 3cos x + 1 2sin x cos x 2sin 3x cos3x
18. (b) 2 sin x - 3 =1 or +
2cos3x cos x 2cos9x cos3x
2
Case I: 2 cos x – 3 cos x + 1 = 0 2sin 9x cos9x
1 p + =0
cos x = ,1 ; x = 0, 2cos 27x cos9x
2 3
sin(3x - x) sin(9x - 3x)
p or +
But at x = L.H.S. = 0° 2cos3x cos x 2cos9x cos3x
3 sin(27x - 9x)
p + =0
\ x= (rejected) \ x = 0 is a 2cos 27x cos9x
3 or (tan3x – tan x) + (tan9x – tan3x)
solution + (tan27x – tan9x) = 0
Case II: 2 sin x – 3 = 1, – 1 or tan 27x – tan x = 0 or tan x = tan 27 x
np
3 +1 3 -1 3 -1 Þ 27x = np + x, n Î I or x = , nÎI
\ sin x = , sin x = 26
2 2 2 p 2p 3p 4p 5p 6p
\ x has 2 values in [0, p]. or , , , , ,
\ Total number of solutions = 2 + 1 = 3. 26 26 26 26 26 26
Hence, there are six solutions.
84 MATHEMATICS

23. (4) Since – 2 £ sin x – 3 cos x £ 2, we have p æ x pö x


Þ x = 2mp, mÎ Zor - ç + ÷ = np + , n ÎI
4m - 6 2m - 3 2 è 2 6ø 2
-2 £ £ 2 or -1 £ £1
4-m 4-m p
\ x = 0 or x = .
2m - 3 3
If £ 1, we have p p
4-m 25. (2) Given < 3x - £ p
2 2
(2m - 3) - (4 - m) 3m - 7
£ 0 or ³0. p æ pö æ p ö -p
4-m m-4 Þ < ç 3x - ÷ £ p or - p £ ç 3x - ÷ <
2 è 2ø è 2ø 2
...(i)
3p -p
2m - 3 m +1 Þ p< 3x = or £ x < 0
Also, - 1 £ Þ £0 ...(ii) 2 6
4-m m-4
é -p ö æ p p ù
é 7ù \ xÎê ,0 ÷ È ç , ú
From Eqs. (i) and (ii), we get m Î ê -1, ú . ë 6 ø è 3 2û
ë 3û Now, 1 + cos x + cos 2x + sin x + sin 2x + sin
Therefore, the possible integers are – 1, 0, 1, 2. 3x = 0
24. (2) We have, Þ 2 cos2x + cos x + sin 2x + 2 sin 2x cos x = 0
2 Þ cos x (2 cos x + 1) + sin 2x (2 cos x + 1) = 0
æ æ pö æ p öö 2 Þ (cos x + sin 2x) (2 cos x + 1) = 0
ç cos ç x + ÷ - cos ç ÷ ÷ = sin x Þ cos x (1 + 2 sin x) (2 cos x + 1) = 0
è è 6 ø è 6 ø ø
-1
æxö æ x pö æxö æxö Þ cos x = 0 or sin x =
Þ 4sin2 ç ÷ × sin2 ç + ÷ = 4sin2 ç ÷ × cos2 ç ÷ 2
è 2ø è 2 6ø è 2ø è2ø (as for given interval, cos x > 0)
x æ x pö x p -p
\ sin = 0 or sin 2 ç + ÷ = cos 2 Þ x = or x = .
2 è2 6ø 2 2 6
Hence, there are 2 solutions.
CHAPTER
4 Principle of Mathematical Induction
1. (d) We note that P(1) = 2 and hence, k ³1 ...(i)
P(n) = n (n + 1) + 2 is not true for n = 1. Now ,
So the principle of mathematical induction is not
applicable and nothing can be said about the 2 + 2 + 2 + ...... 2 = 2 + 2 + 2 + ...... 2
validity of the statement P(n) = n (n + 1) + 2. 144 42444 3
( k +1) terms
2. (c) The product of r consecutive integers is k terms
divisible by r ! . Thus n (n + 1 ) (n + 2) (n + 3) is
divisible by 4 ! = 24. æ p ö
= 2 + 2 cosçç ÷÷ [From (i)]
è 2 k +1 ø
3. (a) Let f (n ) = 2 + 2 + ..... + 2 (number of
roots is n) é p ù 2 p p
= 2ê1+ cos k+1 ú = 2.2 cos k+2 = 2 cos k+2
Then f(1) = 2 = 2 cos p or 2 sin p ë 2 û 2 2
4 4 \ The result is true for n = k + 1. Hence, by
\ f (1) may be true for (a) as well as for (b) the principle of mathematical induction
2+ 2 p æ p ö
Again f(2) = 2 + 2 = 2´ = 2 cos 2 + 2 + ....+ 2 = 2 cosçç ÷÷ for all n Î N .
2 8 n -roots è 2n +1 ø
\ f (2) is true for (a). 4. (a) It can be proved with the help of
We check it for any integer. n
æ p ö mathematical induction that < a(n) £ n.
Let 2 + 2 + ...... 2 = 2 cosçç ÷÷ for some 2
k - roots è 2 k +1 ø
Solutions 85

200 k 1
\ < a(200) = + [From (1)]
2 k + 1 (k + 1)(k + 2)
Þ a(200) > 100 and a(100) £ 100.
1 1 1 1 k (k + 2) + 1
5. (a) Let Un = + + + .... + =
n n +1 n + 2 2n - 1 (k + 1)(k + 2)
1 1 1 1 (k + 1)2 k +1 k +1
Vn = 1 - + - + ..... + = = =
2 3 4 2n - 1 (k + 1)(k + 2) k + 2 (k + 1) + 1
Let T(n) be the statement Un = Vn
Then T(1) is true. For U1 = 1 and V1 = 1, so that Thus P(k + 1) is true whenever P(k) is true. Hence.
U1 = V1 by the principle of mathematical induction P(n)
Let T(k) be true for some positive integer K. Now, is true for all natural numbers.
é 1 1 1 ù 7. (a) Let us write the statement
U k +1 - U k = ê + + .... +
ë k + 1 k + 2 2 k + 1 úû P(n) : çæ1+ ÷ç
3öæ 5ö æ 7ö æ 2n +1ö
1+ 1+ .......ç1+ = (n +1)2
è 1øè 4÷ø çè 9÷ø è n2 ÷ø
é1 1 1 ù
– ê + + .... +
ë k k +1 2k - 1 úû We note that

1 1 1 1 1 æ 3ö
= + - =- + ...(i) P : ç1 + ÷ = 4 = (1 + 1) 2 , which is true
2k 2k + 1 k 2k 2k + 1 è 1ø
Also, Thus P (n) is true for n = 1
Suppose that P(k) is true for some natural number
é 1 1 1 ù
Vk +1 - Vk = ê1 - + - ...... + ‘k’ i.e.,
ë 2 3 2 k + 1úû
æ 3ö æ 5 ö æ 7 ö æ 2k + 1ö 2
é 1 1 1 ù çè1 + ÷ø çè1 + ÷ø çè1 + ÷ø ..... ç1 + 2 ÷ = ( k + 1)
- ê1 - + - ..... + 1 4 9 è k ø
ë 2 3 2 k - 1 úû
....(1)
1 1 Now,
=- + ...(ii)
2k 2k + 1 æ 3ö æ 5 ö æ 7 ö æ 2k + 1ö ìï 2k + 3 üï
From (i) and (ii), we find that çè1 + ÷ø çè1 + ÷ø çè1 + ÷ø ..... çè1 + 2 ÷ø í1 +
1 4 9 2ý
U k +1 - U k = Vk +1 - Vk k îï ( k + 1) þï
Since, Uk = Vk, therefore, it follows that ì
2 ï 2k + 3 ïü
Uk+1 = Vk+1 = ( k +1) í1 +
2ý [Using (1)]
\ T (n ) is true for all n Î N . îï (k + 1) þï
6. (a) Let us write the statement
1 1 1 1 n
P(n) : 1.2 + 2.3 + 3.4 + ..... + n(n + 1) = n + 1 2ì
ï (k + 1)2 + 2k + 3üï 2
= ( k +1) í ý = k + 2k + 1 + 2k + 3
îï (k +1)2 þï
1 1 1 1
we note that P(1) : = Þ = is true
= k 2 + 4 k + 4 = ( k + 2) 2 = {( k + 1) + 1}
2
1.2 1 + 1 2 2
thus P(n) is true for n = 1
Suppose that P(k) is true for some natural number Thus , P(k + 1) is true whenever P(k) is true.
‘k’ Hence by the principle of mathematical induction
P(n) is true for all natural numbers.
1 1 1 1 k
+ + + ..... + = ......(1) 8. (d) Q 21 > 12. 22 = 22, 23 < 32, 24 = 42
1.2 2.3 3.4 k ( k + 1) k + 1 But 25 > 52, 26 > 62 and so on.
Now,
4n (2n)!
1 1 1 1 1 9. (d) Let P(n) : <
+ + + ..... + + n + 1 (n !)2
1.2 2.3 3.4 k (k + 1) (k + 1)(k + 2)
For n = 2,
86 MATHEMATICS

42 4! 16 24 13. (a) n p - n is divisible by p for any natural


P(2) : < Þ <
2 + 1 (2) 2
3 4 number greater than 1.
which is true. Trick : Let n = 4 and p = 2
Let for n = m ³ 2, P(m) is true. Then (4)2 – 4 = 16 – 4 = 12, it is divisible by 2. So,
it is true for any natural number greater than 1
4m (2m)! 14. (b) For n = 1, we have
i.e. < 49n + 16n + l = 49 + 16 + l = 65 + l
m + 1 ( m !) 2
= 64 + ( l + 1), which is divisible by 64 if l = – 1
4m+1 4m 4(m + 1) (2m)! 4(m + 1) For n = 2, we have
Now, = · < · 49n + 16n + l = 492 + 16 × 2 + l = 2433 + l
m+2 m +1 m + 2 (m !) 2 (m + 2) = 64 × 38 + ( l + 1), which is divisible by 64
(2m)!(2m + 1)(2m + 2)4(m + 1)(m + 1) 2 if l = – 1
= Hence, l = – 1
(2m + 1)(2m + 2)(m !) 2 ( m + 1) 2 ( m + 2) 15. (b) Let P(n) :
[2(m + 1)]! 2(m + 1) 2 [2(m + 1)]! 1 1 1
= · < + + ..... +
[(m + 1)!] (2m + 1)(m + 2) [(m + 1)!]2
2
1× 2 × 3 2 × 3 × 4 n ( n + 1) (n + 2 )
Hence, for n ³ 2, P(n) is true. n (n + 3)
10. (b) Given that, P(n) : 3n < n! =
Now, P(7) : 37 < 7! is true 4 (n + 1) (n + 2)
Let P(k) : 3k < k! 1 1
Þ P(k + 1) : 3k+1 = 3.3k < 3.k! < (k + 1)! For n = 1, L.H.S.= = and
(Q k + 1 > 3) 1× 2 ×3 6
11. (c) Let P(n) = 2 . 42n + 1 + 33n + 1 1(1 + 3) 1
Then P(1) = 2 . 43 + 34 = 209, which is divisible R.H.S.= =
4 (1 + 1) (1 + 2) 6
by 11 but not divisible by 2, 7 or 27.
Further, let P(k) = 2 . 42k + 1 + 33k + 1 is divisible by \ P(1) is true. Let P(k) is true, then P(k) :
11, that is, 1 1 1
+ + ..... +
2.4 2 k +1
+3 3k +1
= 11q for some integer q. 1× 2 × 3 2 × 3 × 4 k ( k + 1) (k + 2 )

Now P (k + 1) = 2 . 42 k +3 + 33k + 4 k (k + 3)
= ... (i)
4 (k + 1) (k + 2)
2k +1 2 3k +1 3 2k +1 3k +1
=2 . 4 .4 + 3 .3 =16 . 2.4 + 27.3 For n = k + 1,
= 16 . 2 . 42 k +1 + (16 + 11) . 33k +1 1 1
P(k + 1) : + + .....
1× 2 × 3 2 × 3 × 4
= 16 [2. 42 k +1 + 33k +1 ] + 11 . 33k +1
1 1
= 16 . 11q + 11 . 33k +1 + +
k (k + 1) (k + 2 ) (k + 1) (k + 2 ) (k + 3)
= 11(16q + 33k +1 ) = 11m (k + 1) ( k + 4)
where m = 16q + 33k + 1 is another integer. =
4 ( k + 2 ) ( k + 3)
\ P(k +1) is divisible by 11.
1 1
L.H.S. = + + .....
\ P(n ) = 2 . 42 n +1 + 33n +1 is divisible by 11 for 1× 2 × 3 2 × 3 × 4
all n Î N . 1 1
+ +
2
12. (b) n(n - 1) = (n - 1)(n)( n + 1) k (k + 1) (k + 2 ) (k + 1) (k + 2 ) (k + 3)
It is product of three consecutive natural k ( k + 3) 1
numbers, so according to Langrange’s theorem = +
4 (k + 1) (k + 2 ) (k + 1) (k + 2) (k + 3)
it is divisible by 3! i.e., 6
[from (i)]
Solutions 87
Hence, from the principle of mathematical
(k + 1)2 (k + 4) (k +1)(k + 4) induction, the statement is true for all natural
= =
4 (k + 1)(k + 2)(k + 3) 4 (k + 2)(k + 3) numbers n.
= R.H.S. Hence, P(k + 1) is true. 18. (d) Let the statement P(n) be defined as
Hence, by principle of mathematical induction 1 1
for all n Î N, P(n) is true. P(n) : 1 + + + .....
1+ 2 1+ 2 + 3
16. (b) 3.52n + 1 + 23n + 1
Put n = 1, we get 1 2n
(3 × 53) + 24 = 391, which is divisible by 17. + =
1 + 2 + 3 + ..... + n n + 1
Put n = 2, we get
(3 × 55) + 27 = 9503, which is divisible by 17 1 1 2 2n
only. i.e. P(n) : 1+ + + ..... + =
1+ 2 1+ 2 + 3 n (n +1) n +1
17. (b) Let the statement P(n) be defined as
P(n) = 1.3 + 2.32 + 3.33 + ..... + n.3n é n (n + 1) ù
êQ sum of natural numbers = ú
(2n – 1) 3n + 1 + 3 ë 2 û
=
4 Step I : For n = 1,
Step I : For n = 1, 2 ´1 2
P(1) : 1 = = = 1, which is true.
(2.1 – 1) 31 + 1 + 3 32 + 3 1+1 2
P(1) : 1.3 = =
4 4 Step II : Let it is true for n = k,
9 + 3 12 1 1 2 2k
= = = 3 = 1.3, which is true. i.e. 1 + + + ..... + =
4 4 1+ 2 1+ 2 + 3 k (k + 1) k + 1
Step II : Let it is true for n = k, ...(i)
i.e. 1.3 + 2.32 + 3.33 + ..... + k.3k Step III : For n = k + 1,
(2k – 1) 3k + 1 + 3
= ... (i) æ 1 1 2 ö 2
4
Step III : For n = k + 1, ç1+ 1+ 2 + 1+ 2 + 3 + ..... + k (k +1) ÷ + (k +1)(k + 2)
è ø
(1.3 + 2.32 + 3.33 + ..... + k.3k) + (k + 1)3k+1
2k 2
(2k – 1) 3k + 1 + 3 k+1 = +
k + 1 (k + 1) (k + 2)
[using equation (i)]
= + (k + 1)3
4
[Using equation (i)] é 2 ù
2k (k + 2 ) + 2 2 ë k + 2k + 1û
(2k – 1) 3k + 1 + 3 + 4 (k + 1) 3k + 1 = =
=
(k + 1) (k + 2) (k + 1) (k + 2)
4 [taking 2 common in numerator part]
k +1
3 (2k – 1 + 4k + 4) + 3 2 (k + 1)
2
2 (k + 1) 2 (k + 1)
= = = =
4 (k + 1) (k + 2) k+2 ( k + 1) + 1
[taking 3k + 1 common in first and last term of
numerator part] Therefore, P(k + 1) is true, when P(k) is true.
Hence, from the principle of mathematical
3k + 1 (6k + 3) + 3 3k + 1 × 3 (2k + 1) + 3 induction, the statement is true for all natural
= = numbers n.
4 4
[taking 3 common in first term of numerator part] 19. (c) 24 º 1 (mod 5) Þ (24)75 º (1)75 (mod 5)
i.e. 2300 º 1 (mod 5) Þ 2300 × 2 º (1.2) (mod 5)
3(
k + 1) + 1
[ 2k + 2 – 1] + 3 Þ 2301 º 2 (mod 5)
= \ Least positive remainder is 2.
4
20. (d) Let P(n) be the statement given by
éë 2 ( k + 1) – 1ûù 3(
k + 1) + 1
+3 P(n) : 32n when divided by 8, the remainder is 1.
=
4 or P(n) : 32n = 8l + 1 for some l Î N
Therefore, P(k + 1) is true when P(k) is true. For n = 1,
P(1) : 32 = (8 × 1) + 1 = 8l + 1, where l = 1
88 MATHEMATICS
\ P(1) is true. 41k + 1 – 14k + 1 = 41k 41 – 14k 14
Let P(k) be true. = (27l + 14k) 41 – 14k 14 [using (i)]
Then, 32k = 8l + 1 for some l Î N ... (i) = (27l × 41) + (14k × 41) – (14k × 14)
We shall now show that P(k + 1) is true, for = (27l × 41) + 14k (41 – 14) = (27l × 41)
which we have to show that 32(k + 1) when + (14k × 27)
k
divided by 8, the remainder is 1. = 27(41l + 14 ),
Now, 32(k + 1) = 32k . 32 = (8l + 1) × 9 which is a multiple of 27.
[Using (i)] Therefore, P(k + 1) is true when P(k) is true.
= 72l + 9 = 72l + 8 + 1 = 8(9l + 1) + 1 Hence,from the principle of mathematical
= 8m + 1, where m = 9l + 1 Î N induction, the statement is true for all natural
Þ P(k + 1) is true. numbers n.
Thus, P(k + 1) is true, whenever P(k) is true. 23. (8) Let m = 2k + 1, n = 2k – 1 (k Î N)
Hence, by the principle of mathematical \ m2 – n 2
induction P(n) is true for all n Î N. = 4k2 + 1 + 4k – 4k2 + 4k – 1 = 8k
21. (133) Putting n = 1 in 11n + 2 + 122n + 1 Hence, all the numbers of the form m2 – n 2 are
We get, 111+2 + 122×1+1 = 113 + 123 = 3059, always divisible by 8.
which is divisible by 133. 24. (16) 32n+ 2 – 8n – 9, " n Î N
22. (27) Let P(n) be the statement given by Putting n = 2
P(n) : 41n – 14n is a multiple of 27 Þ 32 × 2 + 2 – 8 × 2 – 9 = 729 – 16 – 9 = 704
For n = 1, It is divisible by 16.
i.e. P(1) = 411 – 141 = 27 = 1 × 27,
which is a multiple of 27.
25. (8) 599 = (5)(52 )49 = 5(25) 49 = 5(26 - 1) 49
\ P(1) is true. = 5 ´ (26) ´ (Positive terms) –5 , So when it is
Let P(k) be true, i.e. 41k – 14k = 27l ... (i) divided
For n = k + 1, by 13 it gives the remainder –5 or (13 – 5) i.e., 8.
CHAPTER
5 Complex Numbers and Quadratic Equations
1. (a) | x1z1 – y1z2 |2 + | y1z1 + x1z2 |2 i i2 i i 2
= | x1z1 |2 + | y1z2|2 – 2Re(x1y1z1z2) 4. (b) Let z = sin ln (i ) + cos ln (i )
+ | y1z1 |2 + | x1z2 |2 + 2Re(x1y1z1z2) ì æ p ö –1 ü ì æ p ö –1 ü
ï ï ï – ï
= x1 | z1 | + y12 | z2 |2 + y12 | z1 |2 + x12 | z2 |2
2 2 = sin íln ç e 2 ÷ ý + cos íln ç e 2 ÷ ý
ç
ï è ø ï ÷ ï è ç ÷
ø ï
= x12 | z1 |2 + y12 | z2 |2 + y12 | z1 |2 + x12 | z2 |2 î þ î þ
= 2(x12 + y12) (42) = 32(x12 + y12) p p
2. (d) Q | z | = | w| and arg z = p - arg w = sin + cos = 1
2 2
Let w = reiq , then z = rei (p-q) 5. (d) Let z = (1+ i ) n1 + (1– i )n1 + (1 + i )n2

Þ z = reip . e-iq = (re -iq ) (cos p + i sin p) + (1– i ) n2


= w (-1) = -w Observe that (1+ i ) n1 + (1– i )n1
(c) Q | z - 1| + | z + 3 | £ 8 ì n1p ü
( )
3.
\ z lies inside or on the ellipse whose foci are (1, 0) = 2 í 2cos ý,
î 4 þ
and (– 3, 0) and vertices are (– 5, 0) and (3, 0). always real " n1 Î ¥
Clearly the minimum and maximum values of
Similarly (1 + i )n2 + (1– i ) n2 is always real
| z - 4 | are 1 and 9 respectively, representing
" n2 Î ¥
the distances PA and PA¢. Thus, 1 £ | z - 4 | £ 9 . Þ n1, n2 may be any positive integers
(cos a k + i sin a k )2
A' S' S A (4, 0)
6. (d) wk =
(– 5, 0) (–3, 0) (1, 0) (3, 0) P zk
Solutions 89

zk 2 zk2 1
2r 2 ( z1 – z2 ) æ 2 2 ö
= = = z We have 2 z2 – z1 )
zk zk zk Þ = z3 r ç 2 2 ÷
rk2 zk k z1 z2 è z1 z2 ø
1 1 1 [\ |z1|2 = |z2|2 = r2]
w1 + w2 + w3 = + +
z1 z2 z3 2z1 z2 A(z1)
Þ z3 =
æ1 1 1ö z1 + z2 r

= ç + + ÷ =0
è z1 z2 z3 ø O
C(z3)
\ The origin O is the centroid of DA1A2A3.
B(z2)
7. (c) If P(z1) is the reflection of Q(z2) through z1 – z2
9. (d) = cos a + i sin a
the line b z + b z = c in the argand plane. Then, z1 + z2
P(z)1
2 z1 cos a + i sin a + 1
Þ =
æ z +z ö
Rç 1 2÷
è 2 ø
– 2 z2 cos a –1 + i sin a

2 cos 2 a / 2 + 2i sin a / 2 cos a / 2


bz + bz =c

=
2i sin a / 2 cos a / 2 – 2sin 2 a / 2
Q(z)2

2cos a / 2[cos a / 2 + i sin a / 2]


æ z1 + z2 ö = 2i sin a / 2[cos a / 2 + i sin a / 2]

è 2 ÷ø
lies on the line.
z1 a z K
æ z1 + z2 ö æ z1 + z2 ö Þ = i cot Þ Given 1 =

è 2 ÷ø b çè 2 ÷ø
+ =c z2 3 z2 1

Þ b z1 + b z1 + b z2 + b z2 = 2c ... (i) 2 tan a / 2


\ tan a/2 = – 1/K Þ tan a =
1– tan 2 a / 2
Since, PQ is perpendicular to the line b z + b z = c.
Therefore –2 / K –2 K
Slope of PQ + Slope of the line = 0 Þ Þ
1–1/ K 2 K 2 –1
æ z2 – z1 ö æ – b ö
Þ ç z – z ÷ + çè ÷ =0 æ 2K ö
è 2 1ø b ø a = tan– 1 ç ÷ Þ 2 tan– 1 (K)
è 1– K 2 ø
Þ b (z2 – z1) – b ( z2 – z1 ) = 0
10. (d) z2 + az + a2 = 0
Þ b z2 – b z1 – b z2 + b z1 = 0 Þ z = aw, aw2
Adding Eqs. (i) and (ii), we get (where 'w' is non-real root of cube unity)
Þ Locus of z is a pair of straight lines
2 ( b z1 + b z2) = 2c Þ b z1 + b z2 = c and arg(z) = arg(a) + arg(w) or arg(a)
8. (b) As DOAC is a right triangle with right angle + arg(w2)
at A, |z1|2 + |z3 – z1|2 = |z3|2
2p
Þ 2|z1|2 – z3 z1 – z1 z3 = 0 Þ arg(z) = ±
3
z1 Also, |z| = |a||w| or |a||w2| Þ |z| = |a|.
Þ 2z1 – z3 – z =0 ... (1) 11. (c) Zp = r(cos q + i sin q)
z1 3
y
z
Similarly, 2z2 – z3 – 2 z3 = 0 ... (2) Q
z2
Subtracting (2) from (1) we get 2r a x

æz z ö
2 ( z2 – z1 ) = z3 ç 1 – 2 ÷ p/4 r
P(z)
è z1 z2 ø O
q x
90 MATHEMATICS

æ æ pö æ pöö æ -b ö
ZQ = 2 | z |2 ç cos ç q + ÷ + i sin ç q + ÷ ÷ a ´ ç ÷ + 2b
è è 4 ø è 4øø è a ø b
= =
é æ pö æ pö ù c æ - b ö ac
= 2r êcos ç q + ÷ + i sin ç q + ÷ ú a 2 . + ab . ç ÷ + b2
ë è 4 ø è 4ø û a è a ø
From the figure, 1 1
2 2 2 2 2 2 2 =
p 2r + r – x 3r – x 3r – x (aa + b) (ab + b) ac
cos = =
4 2 × 2r × r 2 2 \1 = \ Required quadratic equation
2r 2r2
Þ r2 = x2 Þ x = r Þ Triangle is right isosceles. x2 – (sum of roots) x + product of roots = 0
Þ acx2 – bx + 1 = 0
aw + b + cw 2 aw 2 + b + cw 16. (c) c = – 4, – b = –3
12. (d) + So, x2 + bx + c = 0 becomes x2 + 3x – 4 = 0
aw2 + bw 2 + c a + bw + cw 2 or (x + 4) (x – 1) = 0
This gives 1 and – 4 as its roots.
aw 2 + bw + cw 3 a w 3 + bw + c w 2
= +
w (aw 2 + bw + c) w (a + bw + cw 2 ) 17. (a) Let t = x + x2 + b2

1 (aw 2 + bw + c) (a + bw + cw 2 ) 1 1 x2 + b2 - x
= + Þ = =
w (aw 2 + bw + c) w (a + bw + cw 2 ) t x + x2 + b2 b2
(Q w3 = 1) 2 b 2
2 Þ t - b = 2x & t + = 2 x2 + b2
1 1 2 2w t t
= + = = = 2w 2
w w w w3 \ 2(a - x)( x + x2 + b2 )
1
13. (c) Q z + = 1 Þ z2 – z + 1 = 0 æ b2 ö
z = ç 2a - t + ÷ (t ) = 2at - t 2 + b2
è t ø
– (–1) ± (1– 4)
\ z= = – w, – w2
2 Þ a 2 + b2 - (a 2 - 2at + t 2 )
[w is cube root of unity]
and z2017 = (– w)2017 = – w, = a 2 + b 2 - (a - t )2 £ a 2 + b2
z2017 = (– w2)2017 = – w2 18. (b) a, b are the roots of
1 æ 1ö x 2 + px + q = 0 Þ a + b = - p and ab = q ...(i)
\ a = z2017 + = – çw + ÷
z 2017 è w ø
Again, a, b are roots of x 2 n + p n x n + q n = 0
= – (w + w2) = 1
n n–4 n–4 Þ a 2 n + pn a n + q n = 0 and
and 22 = 24.2 =16 2 has last digit 6.
\ b=6–1=5 b 2 n + p nb n + q n = 0
Hence, a2 + b2 = 12 + 52 = 26 Þ ( a 2 n - b 2 n ) + p n (a n - b n ) = 0
14. (c) We have ||z1| – |z2|| £ | z1 – z2|| and equality
holds only when arg z1 = arg z2 Þ a n + bn = - p n ...(ii)
Þ ||z – w| – |z – w2|| £ |w2 – w| £ 3 and equality a
Now is a root of ( x n + 1) + ( x + 1) n = 0
1 b
can hold only when |z| = 2 and not when |z| = .
2 æ an ö æ a ön
15. (a) a and b are roots of ax2 + bx + c = 0 Þ ç n + 1 ÷ + ç + 1÷ = 0
çb ÷ èb ø
b c 1 1 è ø
Þa+b= - and ab = = +
a a aa + b a b +b Þ (a n + b n ) + (a + b ) n = 0
ab + b + aa + b Þ - p n + (- p)n = 0,
=
(aa + b) (ab + b) Which holds only if n is an even integer.
Solutions 91
19. (a) Let f (x) = (x – sin b) (x – sin g) + (x – sin a) 22. (45) If 2 – i is the root then 2 + i is also the root
(x – sin g) + (x – sin a) (x – sin b) sum of roots = 4 Þ a = – 3
Now, f (sin a) = (sin a – sin b) (sin a – sin g) b
Product of roots = = (2 – i)(2 + i)
= (–) (–) = positive a
f (sin b) = (sin b – sin a) (sin b – sin g) = (+)(–) Þ b = – 15 \ ab = 45
= negative 23. (0) As the coefficients of two equations are in
f (sin g) = (sin g – sin a) (sin g – sin b) = (+)(+) reverse order, if the roots of ax2 + bx + c = 0 are
= positive a and b then the roots of second equation are
Þ Roots of f (x) = 0 are real and distinct. 1 1
20. (c) (a – 1) x2 – (a + 1)x + (a – 1) ³ 0 " x ³ 2 , . Given that one negative root is common,
a b
For a = 1, – 2x ³ 0 " x £ 2 Þ a ¹ 1 two possibilities may arise.
Þ a > 1 and f ' (x) ³ 0 " x ³ 2, f (2) ³ 0 1 1
Þ 2(a – 1)x – (a + 1) ³ 0 " x ³ 2, f (2) ³ 0 Either a = < 0 Þ a = -1 or a = < 0
Þ 2(a – 1)x ³ (a + 1) " x ³ 2, 3a – 7 ³ 0 a b
c
a +1 7 Þ ab = 1 Þ = 1 Þ c = a (not possible)
Þ x³ "x ³ 2, x ³ a
2(a –1) 3 \ a = -1 is the common root. Put a = -1 in
a +1 7 any of the equations, we get a - b + c = 0.
Þ £ 2; a ³
2(a - 1) 3 log 2 3 log 3 2
24. (7) x 2 - (3 + 2 –3 )x
7
Þ a + 1 £ 4a – 4; a ³ ; a > 1 – 2(3 log 3 2
– 2log 2 3 ) = 0
3
Þ x2 – 3(x) – 2(2 – 3) = 0 Þ x2 – 3x + 2 = 0
5 7 é7 ö Þ a = 2, b = 1 Þ a2 + ab + b2 = 4 + 2 + 1 = 7
Þ a ³ ; a ³ ; a > 1 Þ a Î ê , ¥÷
3 3 ë3 ø 25. (0) Let f(x) = (x – x1) (x – x2)(x – x3)(x – x4)
21. (1) Given that a, b, c are integers not all equal,
w is cube root of unity ¹ 1, then Þ |f(i)| = 1 + x12 1 + x22 1 + x33 1 + x44 = 1
Þ x1 = x2 = x3 = x4 = 0
| a + bw + cw2 | Þ All four roots are zero.
Þ f (x) = x4 Þ a + b + c + d = 0
æ -1 + i 3 ö æ -1 - i 3 ö 26. (2) The given relation can be rewritten as
= a + bç ÷ + cç ÷
è 2 ø è 2 ø 1 1 1 2
+ + =
æ 2a - b - c ö æ b 3 - c 3 ö a+w b+w c+w w
= ç ÷ +iç ÷ 1 1 1 2
è 2 ø è 2 ø and +
2 2 + 2
= 2
a+w b+w c+w w
1
= (2a - b - c)2 + 3(b - c) 2 2
Þ w and w are roots of
2
1 1 1 2
=
1
4a2 + b2 + c2 - 4ab + 2bc - 4ac + 3b2 + 3c2 - 6bc
+ + =
2
a+ x b+ x c+ x x
3x 2 + 2(a + b + c) x + bc + ca + ab 2
= a 2 + b2 + c 2 - ab - bc - ca Þ =
(a + x )(b + x)(c + x ) x
1
= [(a - b) 2 + (b - c ) 2 + (c - a) 2 ] Þ x3 - (bc + ca + ab) x - 2abc = 0 ...(i)
2
Two roots of the equation (i) are w and w 2 . Let
R.H.S. will be min. when a = b = c, but we cannot
take a = b = c as per question. the third root be a , then
\ The min value is obtained when any two are a + w + w2 = 0 Þ a = – w – w 2 = 1.
zero and third is a minimum magnitude integer \ a = 1 will satisfy equation (i)
i.e. 1.
1 1 1
Thus b = c = 0, a = 1 gives us the minimum value 1. Þ + + =2
a +1 b +1 c +1
92 MATHEMATICS

k2 29. (4) Q (1 + i )4 = [(1 + i )2 ]2


27. (2) Let y = x + x 2 + k 2 =
x2 + k 2 - x = (1 + i 2 + 2i )2 = (1 - 1 + 2i) 2
= 4i2 = –4 ...(i)
k2 k2
Þ = 2
x +k -x \ 2x = y -
2 ...(i)
y y 1 - pi p -i
& +
p +i 1 + pi
Let z = 2 ( k - x ) x + x 2 + k 2 ( )
(1- pi)( p - i) ( p - i)(1- pi)
æ k2 ö
= +
p +1 1+ p
= çç 2k - y + ÷y
è y ÷ø p - i - pi - p + p - pi - i - p
=
= k 2 - y 2 + 2ky = 2k 2 - ( k - y )2 £ 2k 2 p +1
z -2pi - 2i
\ £2 = = -2i ...(ii)
2 p +1
k
æ z-2ö p p æ 1- pi p -i ö
Given, z = (1 + i )4 ç
28. (3) Arg ç ÷= gives semi-circle with 4 ç p + i + 1+ pi ÷÷
è z+2ø 2 è ø
radius r = 2. p
= ( - 4)(- 2i ) = 2pi
æ w -1 ö p 4
Arg ç ÷ = gives semi circle with radius [from Eqs. (i) and (ii)]:
è w +1 ø 2
r = 1. æ | z | ö 2p
Now, ç ÷= = 4.
Consider z = 2- + 0i+ and w = -1+ + 0 + i è amp( z ) ø p / 2
Y 30. (4) If p, q, r are in A.P., p + r = 2q
p 2 + r 2 + 2 pr
w Since D ³ 0 Þ - 4 pr ³ 0
z 4
O X Þ p 2 + r 2 - 14 pr > 0
Þ| z - w |< 3 so k = 3 (as |z – w| < k and k is least p r r
Þ + > 14 Þ - 7 ³ 4 3 Þ k = 4
upper bond). r p p
CHAPTER
6 Linear Inequalities
2 1 2x - 1 2- x
1. (c) -
- 3 ³0 or 2
³ 0 , where x ¹ – 1
2
x - x +1 x + 1 x +1 x - x +1

or
(
2 ( x + 1) - x 2 - x + 1 - ( 2 x - 1) ) ³0
or 2 - x ³ 0, x ¹ -1 (as x2 – x+ 1 > 0 for " x ÎR)
Þ x £ 2, x ¹ - 1 Þ x Î (– ¥, – 1) È (–1, 2]
( x + 1) ( x2 - x + 1) 2. (c) (2x + 1) (x – 3) (x + 7) < 0
Sign scheme of (2x + 1) (x – 3) (x + 7) is as

or
(
- x2 - x - 2 ) ³0
follows :
– + – +
( x + 1) ( x 2
- x +1 ) –7 –1/2 3
- ( x - 2 )( x + 1) Hence, solution is (–¥, 7) È (–1/2, 3)
or ³0
( x + 1) ( x 2 - x + 1) 3. (c) | x - 1| £ 3 Þ -3 £ x - 1 £ 3 Þ -2 £ x £ 4
and | x - 1 | ³ 1Þ x - 1 £ -1
Solutions 93

or x - 1 ³ 1 Þ x £ 0 or x ³ 2 Þ x Îf
Taking the common values of x, we get (iii) If x ³ 1 , then
x Î [-2, 0] È [2, 4]
3 9
4. (b) The given equations are ( x + 2) - ( x - 1) < x - Þx>
2 2
ì x + 3y = 5, x ³ 1 L (i)
| x - 1| +3y = 4 Þ í 9
î - x + 3y = 3, x < 1 L (ii) Q > 1. Þ common solution is
2
and
9 æ9 ö
ì x - y = 1, y ³ 1 L (iii) x> Þ x Îç ,¥÷
x - | y - 1| = 2 Þ í 2 è2 ø
î x + y = 3, y < 1 L (iv)
Solving (i) and (iii), we get x = 2, y = 1 æ9 ö
Solving (i) and (iv), we get x = 2, y = 1 \ Solution set is x Î ç , ¥ ÷ .
è 2 ø
no solution (Q x ³ 1 and y < 1)
12x 12 | x |
Solving (ii) and (iii), we get x = 3, y = 2 7. (a) ³1Þ ³1 Q 4x 2 + 9 > 0
4x 2 + 9 4x 2 + 9
no solution (Q x < 1, y ³ 1)
5 3 Þ 4x 2 - 12 | x | +9 £ 0 Þ 4| x |2 -12| x | +9 £ 0
solving (ii) and (iv) we get, x = , y =
2 2 3
Þ (2 | x | -3) 2 = 0 Þ | x | =
no solution (Q x < 1, y < 1) 2
Hence solution is x = 2, y = 1(a unique solution) | x - 1| + a = 4 Þ | x - 1| + a = ±4
8. (b)
5. (c) 2- |1- | x || = 1 Þ 2- |1- | x ||= ±1
Þ| x - 1| = - a ± 4
Þ|1- | x ||= 1 or 3
The above equation holds if -a + 4 ³ 0
If 1- | x | = 1 Þ 1- | x |= ±1 Þ| x |= 0 or -a - 4 ³ 0
or 2 Þ x = 0 or ± 2 Þ a £ 4 or a £ -4 Þ a Î (-¥, 4] È (-¥, - 4]
If 1- | x | = 3 Þ 1- | x |= ±3 Þ| x |= -2 or 4 Þ a Î (-¥, 4]
Þ| x |= 4 Þ x = ±4 [Q | x |¹ -2] 9. (c) We know that |f(x)| = – f(x) if f ( x ) £ 0
\ Solution set is {– 4, – 2, 0, 2, 4}, hence 5 real
roots in all. x 2 - 8x + 12 x 2 - 8x + 12
\ =-
3 x 2 - 10x + 21 x 2 - 10x + 21
6. (a) The inequality is | x + 2 | - | x - 2 |< x - .
2 x 2 - 8 x + 12
Dividing the problem into three intervals : Þ £0
(i) If x < –2, then x 2 - 10 x + 21
3 3 ( x - 2)( x - 6)
- ( x + 2) + ( x - 1) < x - Þx > - Þ £ 0, x ¹ 3, 7
2 2 ( x - 3)(x - 7)
3 Þ ( x - 2)( x - 3)(x - 6)(x - 7) £ 0 , x ¹ 3, 7
But - > -2 , hence no common values
2
+ + +
Þ x Îf
2 – 3 6 – 7
(ii) If -2 £ x < 1, then
3 5 Þ 2 £ x < 3 or 6 £ x < 7
( x + 2) + ( x - 1) < x - Þx<-
2 2 Þ x Î [2, 3) È [6, 7)
5 10. (a) Case (i) : When x ³ 0
But - < -2 , hence no common values \ |x| = x
2
94 MATHEMATICS

3- x (3 - x)(4 - x) x+2
³0Þ ³0 14. (a) The inequality is log 0.2 £ 1 . The
4-x (4 - x) 2 x
Þ (x – 3) (x – 4) ³ 0 and x ¹ 4 Þ x £ 3 or x > 4 x+2
but x ³ 0 so x Î [0, 3] È (4, ¥) ...(i) L.H.S is valid if >0
Case (ii) : When x < 0 x
Þ x( x + 2) > 0 Þ x < -2 or x > 0 .
3+ x (x + 3)(x + 4)
\ |x| = –x \ ³0Þ ³0 Solving the inequality, we get (note that base < 1)
4+x (x + 4) 2
x+2 1 x+2 1 4 x + 10
Þ x < – 4 or x ³ – 3 ³ 0. 2 = Þ - ³0Þ ³0
but x < 0 so x Î (– ¥, – 4) È [–3, 0) ...(ii) x 5 x 5 5x
So union of (i) and (ii) gives 5
x (2x + 5) ³ 0 Þ x £ - or x ³ 0 .
[ -3,3] È ( -¥, - 4) È (4, ¥) 2
Taking the intersection, we get
x+y 1 æ1 1ö
11. (b) - ç + ÷ 5
2
x +y 2 2 èx yø x£- or x > 0
2
x+y x+y æ 1 1 ö æ 5ù
- = ( x + y) ç 2 -
2
x +y 2 2xy ç x + y2 2xy ÷÷ Þ x Î çç - ¥, - ú È (0, ¥)
è ø è 2û
æ 2xy - (x 2 + y 2 ) ö 15. (a) The log functions are defined if
= ( x + y ) çç 2 ÷÷
2
è (x + y ) (2 xy) ø x 2 + 6x + 9 (x + 3) 2
> 0 and x + 1 > 0 Þ >0
2(x + 1) 2( x + 1)
æ - (x - y) 2 ö
and ( x + y ) çç 2 2 ÷÷ £ 0 and x + 1 > 0 Þ x > –1
è (x + y ) (2 xy) ø Now the inequality is
y+z 1 æ1 1ö x 2 + 6x + 9
Similarly, £ ç + ÷ and log < - log 2 ( x + 1)
y 2 + z2 2 èy zø 2 -1 2(x + 1)
x+z 1 æ 1 1ö x 2 + 6x + 9
2 2
£ ç + ÷ Þ - log 2 < - log 2 (x + 1)
x +z 2 èx zø 2( x + 1)
\ Adding; we get
x 2 + 6x + 9
x+y y +z z+x 1 1 1 Þ log 2 > log 2 ( x + 1)
2 2
+ 2 2
+ 2 2
£ + + 2( x + 1)
x +y y +z z +x x y z
x 2 + 6x + 9 2
and Hence A £ B Þ > ( x + 1) Þ - x + 2x + 7 > 0
2( x + 1) 2( x + 1)
x 2 + y2 + z 2
12. (d) xy + yz + zx < 0 and m =
xy + yz + zx Þ ( x + 1)(x 2 - 2x - 7) < 0
\ (x + y + z)2 = x2 + y2 + z2 + 2 (xy + yz + zx)
\ (x + y + z)2 ³ 0 Þ x 2 - 2x - 7 < 0 [Q x + 1 > 0]
Þ x2 + y2 + z2 + 2 (xy + yz + zx) ³ 0 Þ -1 - 2 2 < x < -1 + 2 2 ,
Þ x2 + y2 + z2 ³ – 2 (xy + yz + zx)
but x > -1 Þ -1 < x < -1 + 2 2
x 2 + y2 + z 2
Þ £–2 16. (b) For the validity of inequality ax2 + 4x + a > 0,
xy + yz + zx which is possible if a > 0 and 16 – 4a2 < 0
13. (a) We have Þ a >2 ...(i)
(a1 + a2 + ... + an -1 + 2an ) (a a ...a 2a )1/n Further, the inequality can be rewritten as
³ 1 2 n–1 n
n log 5 5 + log 5 ( x 2 + 1) £ log 5 (ax 2 + 4x + a )
[Using A.M. ³ G.M.] Þ 5( x 2 + 1) £ ax 2 + 4x + a
1/n
Þ a1+ a2 + a3 + ..... + an– 1+ 2an ³ n(2c)
Solutions 95

Þ (a - 5) x 2 + 4 x + (a - 5) ³ 0 . 2sin 3x . cos 3x sin (2 . 3x ) 1


= = £
It holds if a - 5 > 0 and 16 - 4(a - 5) 2 £ 0 x
2x 2 2
3 +3 1 x –x
Þ a > 5 and a £ 3 or a ³ 7 Þ a ³ 7 ...(ii) Þ £ but (3 + 3 ) ³ 2
k 2
Combining the results of (i) and (ii) for common Þ Minimum value of k = 4
values, we get a Î [7, ¥) 22. (2) As x, y Î R, and xy > 0, so x and y will
17. (b) For x ³ -2 , x2 – x – 2 + x > 0 be of same sign.
2
Þ x > 2 Þ x Î (-¥, - 2 ) È ( 2 , ¥) 2x x 3 y 4y 2
Therefore, all the quantities , ,
Þ x Î [-2,- 2 ) È ( 2 , ¥) y3 3 9x 4
For x < – 2 are positive.
x2 + x + 2 + x > 0 or x2 + 2x + 2 > 0 Now, A.M. ³ G.M.
which is true for all x. æ æ 2x ö æ x3y ö æ 4y2 ö ö
1/3
2x x3 y 4y2
Hence x Î (-¥, - 2 ) È ( 2 , ¥) Þ 3+ + 4 ³ 3 çç çç 3 ÷÷ çç ÷÷ çç 4 ÷÷ ÷÷
18. (c) log10 x = A
y 3 9x è è y ø è 3 ø è 9x ø ø
x >0 2
=3× =2
log10 (x – 2) = B, x – 2 > 0 Þ x > 2 3
Þ A2 – 3AB + 2B2 < 0 Þ(A – 2B) (A – B) < 0
Þ (log x – 2 log (x – 2)(log x – log (x – 2)) < 0 23. (1) x + 3 - x ³ 3 - x + 3 Þ x ³ 3
Case - I : log x – 2 log (x – 2) < 0 But 3 – x ³ 0 Þ x £ 3
and log x – log (x – 2) > 0 Hence, x = 3 is the only integral solution.
Case - II :log x – 2 log (x – 2) > 0 x+3 + x
and log x – log (x – 2) < 0 24. (4) We have >1
From (1) & (2), x Î (4, ¥) x+2
19. (a) x ¹ (2n + 1) p/2, npnÎ I. The given x+3 + x x+3 -2
inequality can be written as Þ -1 > 0 Þ >0
x+2 x+2
log 2 (x 2 - 8x + 23) 3 Now two cases arise :
> Case I : When x + 3 ³ 0, i.e., x ³ – 3. Then
log 2 | sin x | log 2 |sin x |
As log2 |sin x| < 0, we get; log (x2 – 8x + 23) < 3 x+3 -2 x+3- 2
>0 Þ >0
Þ x2 – 8x + 23 < 23 = 8 Þ x2 – 8x + 15 < 0 x+2 x+2
Þ (x – 5) (x – 3) < 0 Þ 3 < x < 5 x +1
p 3p Þ > 0 Þ {(x + 1) > 0 and x + 2 > 0}
For x Î (3, 5), x ¹ p, , . Hence x+2
2 2 or {x + 1 < 0 and x + 2 < 0}
æ 3p ö æ 3p ö Þ {x > – 1 and x > – 2}
x Î (3, p) È ç p, ÷ È ç , 5 ÷ . or {x < – 1 and x < –2} Þ x > – 1 or x < – 2
è 2 ø è 2 ø
Þ x Î (-1, ¥) or x Î ( - ¥, - 2)
3 x 2 + 9 x + 17
20. (b) y = Þ x Î ( -3, -2) È ( -1, ¥) [Since x ³ -3 ]…(i)
3 x2 + 9 x + 7
Case II : When x + 3 < 0, i.e., x < – 3
3x 2 ( y - 1) + 9 x ( y - 1) + 7 y - 17 = 0 x+3 -2 -x - 3 - 2
>0 Þ >0
D ³ 0 Q x is real x+2 x+2
81( y - 1)2 - 4 ´ 3( y - 1)(7 y - 17) ³ 0 -( x + 5) x+5
Þ >0 Þ <0
Þ ( y - 1)( y - 41) £ 0 Þ 1 £ y £ 41 x+2 x+2
Þ (x + 5 < 0 and x + 2 > 0 )
\ Max value of y is 41 or (x + 5 > 0 and x + 2 < 0)
21. (4) Consider the L.H.S of the equation, Þ (x < – 5 and x > – 2)
(sin 3x) (cos 3x) or (x > – 5 and x < – 2). It is not possible.
Þ x Î (– 5, – 2) ...(ii)
96 MATHEMATICS
Combining (i) and (ii), the required solution is Þ ab ³ 0
x Î (– 5, – 2) È (– 1, ¥ ), æ 4 - x2 ö
smallest integral member = – 4 \ x çç ÷÷ ³ 0, x ¹ 0
\ Modulus of smallest integral member =|–4| = 4 è x ø
4 - x2 4 - x2 Þ x2 – 4 £ 0
25. (4) |x|+ = x+ Þ x Î [– 2, 2] – {0}
x x Number of integral values = 4.
As | a | + | b | = | a + b |
CHAPTER
7 Permutations and Combinations
1. (a) Total number of words that can be formed and 3, and a number cannot begin with 0)
using 5 letters out of 10 given different letters Similarly, the number of numbers with 5 in the
= 10 × 10 × 10 × 10 × 10 (as letters can repeat) middle
= 1, 00, 000 = 5P4 – 4P3, etc.
Number of words that can be formed using 5 \ The required number of numbers
different letters out of 10 different letters = (4P4 – 3P3) + (5P4 – 4P3) + (6P4 – 5P3) +
= 10P5 (none can repeat) 9

10! ..................... + (9P4 – 8P3) = å ( n P4 - n -1 P3 )


= = 30, 240 n=4
5! 5. (b) C ...... = 4! = 24
\ Number of words in which at least one letter D ...... = 4! = 24
is repeated M ...... = 4! = 24
= total words-words with none of the letters
S C ...... = 3! = 6
repeated
= 1,00,000 – 30,240 = 69760 S D ...... = 3! = 6
2. (c) X – X – X – X – X. The four digits 3, 3, 5, 5
S M C DW=1
can be arranged at (–) places in 4! = 6 ways. S M C WD=1
2!2! 6. (b) m = Number of permutations of letters of
The five digits 2, 2, 8, 8, 8 can be arranged at
the word BHARAT, when B and H are never
5! together
(X) places in = 10 ways.
2!3! 6! 5!
Total no. of arrangements = 6 × 10 = 60 ways. = – × 2! (keeping (BH) together and
2! 2!
3. (c) The letter of word COCHIN in alphabetic permuting with A, R, A, T)
order are C, C, H, I, N, O. = 360 – 120 = 240 and n = number of permutations
Fixing first letter C and keeping C at second place, of the letters of the word BHARAT when each
rest 4 can be arranged in 4! ways. word begins with B an d ends with
Similarly the words starting with CH, CI, CN are
4! in each case. 4! m 240
T= = 12 \ = = 20
Then fixing first two letters as CO next four places 2! n 12
when filled in alphabetic order give the word 7. (c) Let the square has dimensions n × n.
COCHIN. If n is even, total no. of blue (diagonal) tiles
\ Numbers of words coming before COCHIN = n + n = 2n
are 4 × 4! = 4 × 24 = 96 If n is odd, then total no. of blue tiles
4. (d) The smallest number, which can occur in = n + n – 1 = 2n – 1
the middle is 4. (the middle tile is be counted twice).
Hence, 2n – 1 = 121
The number of numbers with 4 in the middle
Þ n = 61
= 4 P4 – 3 P3 So, number of red tiles = 61 × 61 – 121 = 3600
(Q The other four places are to be filled by 0, 1, 2
Solutions 97
8. (c) 1 9 8
= . 2 = 2 = 256.
2
Numbers Numbers 12. (a) Arrangement does not matter because of
Number of
ending ending Total descending order there should be only one
digits
with 0 with 5 arrangement
1 0 1 1 \ required number of ways = 15C3 × 24C2
2 8 9 17 1 ................ 24 25 26 ................ 40
3 9.8 = 72 8.8 = 64 136
4 9.8.7 = 504 8.8.7 = 448 952 Selecting any 2 out of Selecting any 3 from
24 15
Total 1106 1 to 24 = C2 26 to 40 = C3
9. (d) 10 identical boxes can contain 12 balls such 13. (b) Let the number of children in the class = n.
that each box contains at least one ball in the Number of groups of 3 children = nC3
following cases: Each child would go to zoo n– 1C2 times.
Case (i) : 8 boxes contains single ball each + 2 \ According the question: nC3 = 84 + n – 1C2
boxes contains two balls each n(n - 1)(n - 2) (n - 1)(n - 2)
Single ball boxes Boxes with doubleball
Þ = 84 +
64748 6447448 6 2
2W,6B; (WW)(WW) Þ n(n – 1) (n – 2) = 504 + 3(n – 1) (n – 2)
Þ (n – 1) (n – 2) (n – 3) = 504
3W,5B; (WW)(WB) Þ (n – 1) (n – 2) (n – 3) = 7 × 8 × 9 Þ x = 10
4W, 4B; (WW)(BB); (WB)(WB) 14. (d) We will consider the following cases:
5W,3B; (WB)(BB) Case Flags No.of signals
6W, 2B; (BB)(BB) 4 alike and 2 4 white and 2 red
6!
=15
Case (ii): 9 boxes single ball each + 1 box others alike 4!2!
containing 3 balls 4 alike and 2 6!
4 white,1red and1blue = 30
others different 4!
Singleballboxes Triple ball boxes
6474 8 6474
8 3alike and 3 6!
3 white,3red = 20
3W + 6 B; (WWW ) others alike 3!3!
3alike and 2 3 white,1blue,
4W + 5B; (WWB ) 2 6!
others alike and1 2 red or 3 red, 1 C1 × =120
5W + 4 B; (WBB ) different 3!2!
blue, 2 white
6W + 3B; ( BBB ) Total 185
\ Total 10 ways are there
10. (c) Required number of ways = (Total number 15. (c) Let the number of men = n
of ways of arranging 7 persons) – (Number of Number of women = 2
ways where two American are together + Number \ Total participants = (n + 2)
of ways when two Britishian are together – Now number of games played by men among
Number of ways when both American and both the m selves = 2(nC2)
Britishian are together) Number of games men played with women = 4n
= 6! – [2 × 5! + 2 × 5! – 4! × 2! × 2!)] According to question 2(nC2) = 4n + 66
= 720 – [480 – 96]= 240 + 96 = 336 Þ n(n – 1) = 4n + 66 Þ n2 – 5n – 66 = 0
11. (b) The candidate is unsuccessful if he fails in Þ (n – 11)(n + 6) = 0 Þ n = 11 or n = – 6
9 or 8 or 7 or 6 or 5 papers. \ Total number of participants = 11 + 2 = 13
\ The number of ways to be unsuccessful 16. (b) Suppose there n players in the beginning.
The total number of games to be played was
= 9 C9 + 9 C8 + 9 C7 + 9 C6 + 9 C5 equal to nC2 and each player would have played
= 9 C0 + 9 C1 + 9 C2 + 9
C3 + 9 C 4 n – 1 games.
Let us assume that A and B fell ill. Now the total
1 9 number of games of A and B is (n – 1) + (n – 1) – 1
= ( C0 + 9 C1 + ......... + 9 C9 ) = 2n – 3. But they have played 3 games each.
2
Then their remaining number of games is
98 MATHEMATICS
2n – 3 – 6 = 2n – 9. Given, \ Number of ways
nC – (2n – 9) = 84
2 é 1 1 1 1ù
Þ n2 – 5n – 150 = 0 or n = 15 = 10C6 × 4! ê1– + – + ú
ë 1! 2! 3! 4!û
17. (b) Required ways = 8C5 – (Number of ways
when R2 or R3 are unused) 10! é1 1 1 ù
= × 4! – +
R1 6!4! êë 2 6 24 úû
R2 é12 – 4 + 1ù
= (10 × 9 × 8 × 7) ê = 1890
R3
ë 24 úû
23. (6) Number of numbers beginning with 1 = 120
= 8C5 – (6C5 + 6C5) = 8C5 – (12) = 56 – 12 = 44 1
18. (c) The number of triangles with vertices on Number of numbers beginning with 2 = 120
p p
different lines = C1 × p C1 = p3.
C1 × 2
The number of triangles with 2 vertices on one Starting with 31 ................................. = 24
line and the third vertex on any one of the other 3 1
two lines Starting with 3214 ................................. = 2
p( p - 1) 3 2 1 4
3
= C1 { p
C2 ´ 2p
C1 } = 6p
2 Finally = 1 3 2 1 5 4 6
\ The required number of triangles Hence, unit place digit of 267th number is 6.
= p3 + 3p2(p – 1). 24. (7) If x denotes the number of times he can
take unit step and y denotes the number of times
19. (c) Let A1, A2, ..., A10 be vertices of a regular he can take 2 steps, then x + 2y = 7.
polygon of 10 sides. We must have x = 1, 3, 5
The number of ways of selecting 3 vertices If x = 1, the steps will be 1 2 2 2
is 10C3.
4!
The number of ways of selecting 3 consecutive Þ Number of ways = =4
vertices is (i.e., A1 A2 A3, A2 A3 A4, ..., A10 A1 A2) 3!
= 10. If x = 3, the steps will be 1 1 1 2 2
The number of ways of selecting three vertices 5!
such that two vertices are consecutive = (First Þ Number of ways = 2!.3! = 10
select 2 consecutive vertices, leave their
neighbouring two vertices and select one more If x = 5, the steps will be 1 1 1 1 1 2
from the remaining 6 vertices) is Þ Number of ways = 6C1 = 6
If x = 7, the steps will be 1 1 1 1 1 1 1
10 × 6C1 = 60 The total number of required
selections is Þ 7C0 = 1
10C – 10 – 60 = 120 – 70 = 50 Hence total number of ways = N = 21
3 Þ N/3 = 7
20. (b) Division of objects can be (0, 1, 4) or (0, 2, 3) 25. (9) In this case number of participants from
So total number of distribution ways 10 different countries are 1, 2, 3… 10. number
5! 5! of ways that they can be arranged in a row
= ´ 3!+ ´ 3! = 30 + 60 = 90 ways such that all the participants from the same
0!.1!.4! 0!.2!.3! country were together is (10!)(1!)(2!)(3!)(4!)….
21. (8) a + b + c = 21 Þ 3b = 21 Þ b =7 (10!)
[Q a + c = 2b] Hence K = (10!)(1!)(2!)(3!)(4!)…. (10!) highest
Þ a + b + c = 21 Þ a + c = 14 power of 10 or that of 5 can be given by only
Þ l = 14 – 1C2 – 1 = 13C1 = 13 (5!)(6!)(7!)(8!) (9!)(10!)(10!) which is
Hence l – 5 = 13 – 5 = 8 1+1+1+1+1+2+2=9
22. (9) If exactly 6 of his matching are correct, then
the remaining 4 statements from column I must
be incorrectly matched with 4 statements from
column II
Solutions 99
CHAPTER
8 Binomial Theorem
1. (a) 2 60 = (1 + 7) 20 6. (c) n
C0 + n C1 ( ax )1 + n C2. (ax)2 + ........
= 20
C0 . + 20
C1. 7 + 20 2
C2 . 7 + = 1 + 8x + 24x2 + ... (i)
20 Compairing coefficients of x 0 , x1 , x 2 in
............ + C20 . 7 20
equation (i),
20
\ the remainder = C0 . = 1 n
C0 = 1 Þ 1 = 1 ...(ii)
2. (c) 32 = 25 Þ (32) 32 = (25 ) 32 n
C1 a = 8 Þ na = 8 ...(iii)
= 2160 = (3 – 1)160 = 3m + 1, m Î N n(n – 1) 2
3232 and n C2 . a 2 = 24 Þ a = 24 ... (iv)
= 25(3m + 1)
3m+1
\ (32) = (32) 2
8
= 23(5 m + 1) 2 2 = 4. 85m + 1 Put n = in equation (iv), from equation (iii)
a
4(7 + 1)5 m + 1 = 4(7n + 1), n Î N = 28n + 4
éæ 8 ö 2 8 ù
3232 2
\ When 7 divides (32) , remainder = 4 Þ ê çè ÷ø – ú a = 48 Þ 64 – 8a = 48
ëê a a úû
n æ r –1 ö
3. (d) å ç å n Cr .r C p .2 p ÷ Þ a=2 \ n=4
r =1 çè p = 0 ÷
ø a–n 2–4 1
\ = =–
n æ r – 1 ö a+n 2+ 4 3
= å n C r ç å r C p .2 p ÷ 7. (a) 17 2009 + 112009 – 7 2009 =
r =1 ç p=0 ÷
è ø
n
(17 2009 – 72009 ) + 112009 = (17 – 7)
n
= å nCr { (1 + 2)r – 2r }= å n
Cr 3r – n Cr 2r (17 2008 + 17 2007.71 + ..... + 7 2008 ) + 11
2009
r=1 r =1 2009 2009
n n n Cleary last digit in 17 –7 is 0.
= n(4 –1) – (3 –1) = 4 – 3
4.
2 6
(d) Given expression is (1 + x - 2 x ) Also, last digit in 112009 is 1.

= 1 + a1 x + a2 x 2 + ..... + a12 x12 Last digit of 17 2009 + 112009 – 72009 is


0 + 1 = 1.
Put x = 1 both side;
153
0 = 1 + a1 + a2 + .... + a12 … (i)
Put x = – 1 both sides; 8. (c) Let x = 2 \ a = x2 + 2 x + 1
2

(-2)6 = 1 - a1 + a2 – .... + a12 …(ii) Let y = 2204 \ b = y2 – y + 1


Adding (i) and (ii), Þ N = x16 – 1 Þ N = (x4 – 1) (x4 + 1)(x8 + 1)
2 + 2a2 + 2a4 + .... + 2a12 = 64 Þ N = (x4 – 1) (x2 + 2 x + 1 ) (x2 – 2 x + 1 )(x8 + 1)
64 - 2 Þ N = y6 – 1 = (y3 – 1) (y3 + 1)
Þ a2 + a4 + .... + a12 = = 31
2 Þ (y3 – 1)(y + 1)(y2 – y + 1)
5. (a) The last term = nCn. (–1 / 2 )n 9. (c) N = 20C7 – 20C8 + 20C9 – 20C10 + ..... – 20C20
= (1/3. 3 9 )log3 8. = (20C7 + 20C9 + 20C11 + ..... + 20C19) –
log3 8.
n n/2
æ 1 ö (20C8 – 20C10 + ..... + 20C20)
\ (– 1) .(1/2) = ç 5/3 ÷
è3 ø = (20C0 + 20C2 + 20C4 + 20C6) –
5 (20C1 + 20C3 + 20C5)
- . 3log3 2
= 3 3 = 3-5 log 3 2 = 2 -5 = (1 + 190 + 4845 + 38760) – (20 + 1140 + 15504)
So, t5 = C4 (2 ) (–1/ 2 )4 = 10C4. =
10 1/3 6 10
C10–4. = 43796 – 16664 = 27132 = 3 × 4 × 7 × 19 × 17
100 MATHEMATICS

10. (a) x = T7 = nC6(31/3)n – 6. (4–1/3)6 é n n + 1ù


y = Tn – 5 = nCn – 6(31/3)6. (4–1/3)n – 6 So, from (iii) and (iv), x Î ê ,
ën +1 n úû
Þ y = 12x
(8 5) (6 2)
n 100 – r r
Cn – 6(31/3)6 (4–1/3)n – 6 13. (a) t r +1 = 100
Cr . . As 2 and
= 12. nC 6(31/3)n – 6.(4–1/3)6
Þ 12 = (121/3)12 – n Þ n = 9 5 are coprime, t r +1 will be rational if 100 – r is a
11. (d) Given expression multiple of 8 and r is a multiple of 6.
Also 0 £ r £ 100.
( x1/ 3 )3 + (1)3 ( x - 1) \ r = 0, 6, 12, ............. , 96
= –
x 2/3
–x 1/ 3
+1 x1/ 2
( x1/ 2 - 1) Þ 100 – r = 4, 10, 16, ............, 100
But 100 – r is to be a multiple of 8.
( x1/ 3 + 1) ( x 2 / 3 - x1/ 3 + 1) So, 100 – r = 0, 8, 16, 24, ... 96.
= The common terms in (1) and (2) are 16, 40, 64
( x 2 / 3 - x1/ 3 + 1)
and 88.
( x1/ 2 + 1) ( x1/ 2 - 1) \ r = 84, 60, 36, 12 give rational terms.
– \ The number of irrational terms
x1/ 2 ( x1/ 2 - 1) = 101 – 4 = 97.
r
= ( x + 1) – (1 + x -1/ 2 ) = x
1/ 3 1/ 3
– x
–1/ 2 æ cos a ö
14. (c) t r +1 = 10 Cr (x sin a )10 – r . çè x ÷ø .
10
æ x +1 x -1 ö
Þ çè 2 / 3 1/ 3 - ÷ It is independent of x if r = 5
x - x + 1 x - x1/ 2 ø 10
1/ 3 –1/ 2 10
\ The term independent of x = C5 . sin 5 a .
= (x -x )
1 1
Tr +1 in ( x1/ 3 - x –1/ 2 )10 is cos 5 a = 10
C5 . 5 (sin 2a )5 £ 10
C5 5
2 2
10
Cr ( x1/ 3 )10 - r . ( -1)r . ( x -1/ 2 )r (10)!1
= . 5
2
æ 10 - r r ö (5!) 2
çè - ÷
3 2ø
= ( -1)r 10
Cr x which is T
æ 10 - r r ö
15. (c) If Tr + 1 ³ Tr , then r +1 ³ 1
independent of x if çè - ÷ =0 Þ r=4 Tr
3 2ø
25 – r + 1 . 3x
Hence required coefficient =
10 4
C4 (-1) = 210. Þ ³1
r 2
2n n
12. (c) Middle term = Cn . x = t n+1 26 – r .
Þ 3 ³1 (Q x = 2)
If tn + 1 is the greatest term also, then r
1
t n+1 ³ tn ....(i) Þ 78 – 3r ³ r \ r £ 78 Þ r £ 19
4 2
t n+1 ³ t n+ 2 .....(ii)
Þ r = 19
From (i),
2n
Cn . x n ³ 2 n C . x n –1 Largest term = (r + 1)th term = 20th term
n –1
16. (c) Middle term in the expansion is
(2n)! (2n)! th
Þ ³ Þ (n + 1) x ³ n æ 10 ö
n! n! (n – 1)!(n + 1)! çè + 1÷ø i.e., 6th term.
2
n
Þ x³ ...(iii) 7 1 63
n +1 Thus T6 = 7 Þ 10C5 5 . x5 sin5 x =
From (ii), 8 x 8
2n
Cn . x n ³ 2 n Cn +1 . x n +1 63 1
Þ 252.sin 5 x = Þ sin 5 x =
(2n)! (2n)! 8 32
Þ ³ x
1
n!n ! (n –1)!(n + 1)! π
Þ sin x = \ x = nπ + (-1)n
n +1 2 6
Þ x£ ...(iv)
n
Solutions 101
17. (d) The cofficients of the integral powers of x
are = 2[ n C0 2n + n C2 2n - 2.3 + n C4 .2n - 4.32 +.............]
40 40 40 = 2 × Integer = Integer
C0 , C2 . 22 , C4 . 24 , ..... , 40 C40 . 2 40 Q I + f + F is integer Þ f + F must be integer..
Þ (1 + 2) 40 = 40
C0 + 40 C1 2+ 40 C2 . 22 +...... \ 0 £ f < 1 and 0 < F < 1 Þ 0 < f + F < 2
40 Þ f + F = 1 Þ F = 1 – f \ ( I + f ) (1 – f )
+ C40 . 2 40
= (I + f ) F = (2 + 3)n (2 – 3) n = 1
Þ (1 – 2) 40 = 40
C0 – 40 C1 . 2 4 æ 34 - k ö æ xk ö
+ 40 C2 . 22 –......+ 40 C40 . 2 40 21. (1) å çç (4 - k )! ÷÷ çç ÷
÷
Adding, 3 40 + 1 = 2 × (required sum) k =0 è øè k! ø
18. (b) 4 æ 34- k ö æ x k ö 4!
p( n ) n n n
C 0 . C1. C 2 ..... C n n = å çç (4 - k )! ÷÷ çç k ! ÷÷ 4!
= k =0 è øè ø
+
p(n + 1) n 1
C 0 .n +1 C1.n +1 C 2 ......n +1 C n +1 4 4 4- k k
Ck .3 x (3 + x )4
1 æç n C 0 ö÷ æç n C1 ö÷ æ n Cn ö = å 4!
=
4!
= n +1 .......ç ÷ k =0
C 0 çè n +1 C1 ÷ø çè n +1 C 2 ÷ø ç n +1 C
è
÷
n +1 ø
According to the question.
(3 + x)4 32
1æ 1 öæ 2 ö æ n +1ö =
= ç ÷ç ÷........ç ÷ 4! 3
1 è n + 1ø è n +1 ø è n -1ø 4
or (3 + x) = 256 or x + 3 = 4 or x=1
é n
C r +1 ù
êQ n +1 r = ú 22. (4) Expression = (1 – x)5 .(1 + x)4 (1 + x2)4
êë C r +1 n + 1 úû = (1 – x) (1 – x2)4 (1 + x2)4 = (1 – x) (1 – x4)4
\ so the coefficient of x13 = – 4C3 (–1)3 = 4
(n +1)! n! p (2002) (2002) 2001
= = \ = 23. (0) We have 3C0 – 5C1 + 7C2 + ...
n+1
(n + 1) (n + 1) n p (2001) (2001)! + (–1)n (2n+3)Cn
19. (d) = 3C0 – 3C1 + 3C2 + ...+ (–1)n 3Cn – 2C1 + 4C2+...
C1 C 3 C 5 n n (n - 1)(n - 2) + (–1)n 2n Cn
+ + + ..... = + n
= 3 [C0 – C1 + C2 + ....+ (–1) Cn]
2 4 6 2 4!
–2[C1 – 2C2 + .... (–1)nnCn]
n (n - 1)(n - 2)(n - 3)(n - 4) =3×0–2×0=0
+ + ......
6! 24. (5) We have
1 é (n + 1)n (n + 1)n (n - 1)(n - 2) é 1 1 1 1 ù
11! ê + + + ... +
= ê
n + 1 ë 2!
+
4!
+ ë1!10! 3!8! 5!6! 11!1!úû
11 11 11 11
(n + 1)n (n - 1)(n - 2)(n - 3)(n - 4) ù = + + + ... +
+ .....ú 1!10! 3!8! 5!6! 11!
6! û = 11C1 + 11C3 + 11C5 + 11C7 + 11C9 + 11C11
1 n +1 = sum of the even coefficient in the expansion of
= [ C 2 + n +1C 4 + n +1C6 + .....] (1 + x)11 = 210
n +1
Therefore K = 10
1 2n - 1 25. (6) (1 – 2x + 5x2 – 10x3) [C0 + C1x + C2x2 + ...]
= éë 2n +1-1 -n +1 C0 ùû =
n +1 n +1 = 1 + a1x + a2x2 + ...
20. (b) Given (2 + 3)n = I + f, where I is integer n(n - 1)
Þ a1 = n – 2 and a2 = – 2n + 5
2
and 0 £ f < 1. We note that (2 + 3 ) (2 – 3 )
n a21 = 2a2. Therefore, we have
= 1. So let us assume that F = (2 – 3) .
Clearly 0 < F < 1. Now, (n – 2)2 = n (n – 1) – 4n + 10
Þ n2 – 4n + 4 = n2 – 5n + 10
I + f + F = (2 + 3) n + (2 – 3)n \ n=6
102 MATHEMATICS

CHAPTER
9 Sequences and Series
1. (d) We have
Sn = 13 + 3. 23 + 33 + 3. 43 + 53 + ............
= c2 é 4
ë å n2 - 4 å n + nùû
Let n = 2 m é 4n(n + 1)(2n + 1) 4n(n + 1) ù
= c2 ê - + nú
3 3 3
Then S2m = ( 1 + 3 + 5 + ...... to m terms) ë 6 2 û

+ 3(23 + 43 + 63 + ...to m terms) é 2(2n2 + 3n + 1) - 6(n + 1) + 3 ù


= c2 n ê ú
= {13 + 23 + 33 + 43 + ...... + (2m – 1)3 + (2m)3} ëê 3 ûú
– {23 + 43 + ....+(2m)3} + 3{23 + 43 + 63 + ..... +
(2m)3} é 4n2 - 1ù n(4n 2 - 1)c 2
= c2 n ê ú=
2 êë 3 úû 3
é 2m(2m + 1) ù 3
= ê ú + 8 × 2{ 13 + 23 + 3 4. (c) d = a2 – a1 = a3 – a2 = ... = an – an – 1
ë 2 û
\ sin d [sec a1 sec a2 + sec a2 sec a3 + ...
3 + sec an–1 sec an]
+ ... + m }
sin(a 2 - a1 ) sin(a 3 - a 2 )
2 2 m2 (m + 1)2 = + + ...
= m (2m + 1) + 16. cos a1 cos a 2 cos a 2 cos a 3
4
sin(a n - a n -1 )
n2 (2n 2 + 6 n + 5) n +
= [Put m = ] cos a n cos a n -1
4 2
2. (b) 1, log9 (31 – x + 2), log3 (4.3x – 1) are in A.P. sin a 2 cos a1 - cosa 2 sin a1
=
Þ 2 log9 (31– x+2) = 1 + log3 (4.3x – 1) cosa1 cos a 2
Þ log3 (31 – x + 2) = log33 + log3 (4.3x – 1)
sin a 3 cos a 2 - cos a 3 sin a 2
Þ log3 (31– x + 2) = log3 [3(4 × 3x – 1)] + + ...
Þ 31– x + 2 = 3 (4.3x – 1) cos a 2 cos a 3
3 = tan a2 – tan a1 + tan a3 – tan a2 + ... +
Put 3x = t Þ + 2 = 12t - 3 or 12t2 – 5t – 3 = 0; tan an – tan an– 1 = tan an – tan a1
t
1 3 æ 1 1 1 ö
Hence t = - , 5. (b) y – x = 3 çè 2 + 2 + 2 + ...÷ø
3 4 2 4 6
3 æ 1 1 1 ö
Þ 3x = (as 3 x ¹ - ve )
x – z = çè 2 + 2 + 2 + ...÷ø
4
2 4 6
æ 3ö \ (y – x) = 3(x – z) Þ 4x = y + 3z
Þ x = log3 ç ÷ or x = log3 3 – log3 4
è 4ø x y z
Þ x = 1 – log3 4 Þ2 = +
3 6 2
3. (c) Given that for an A.P, Sn = cn2
a +b b+c
Then, Tn = S n - S n -1 = cn 2 - c (n - 1)2 6. (b) Given , b, are in A.P..
1– ab 1– bc
= (2n - 1)c
a+b b+ c
\ Sum of squares of n terms of this A.P Þb– = –b
1– ab 1– bc
= åTn2 = å (2n - 1)2.c2
Solutions 103

Þ (a – c)2 = 1 – 1 = 0 Þ a = c but
– a(b 2 + 1) c(b 2 + 1)
Þ = a ¹ c as given that a < b < c
1– ab 1– bc \ We consider a + c = 1 and ac = – 1/4
Þ a + c = 2abc
Now, given quadratic equation is Þ (a – c)2 = 1 + 1= 2 Þ a – c = ± 2
2ac x2 + 2abc x + 2abc = 0 but a < c Þ a - c = - 2 ....(ii)
(Substituing a + c
= 2abc and then cancelling 2ac) 1 1
Solving (i) and (ii) we get a = -
Þ x2 + bx + b = 0 2 2
7. (d) Given mid terms tn = 1 and tn + 1 = 7 x x
\ tn + tn + 1= 8 = t1 + t2n 10. (c) = 5 Þ r = 1-
1- r 5
and tn + 1 – tn = 6 = d (common difference of Since G.P. contains infinite terms
A.P.) \ – 1< r<1
tn + tn + 1 = 8
x x
\ a + (n – 1)d + a + nd = 8 \ a + 6n = 7 Þ -1 < 1 - < 1 Þ - 2 < - < 0
Now 4t1t2n = [(t1 + t2n)2 – (t2n – t1)2] 5 5
= 64 – 36(2n – 1)2 [as t2n – t1 = (2n – 1) × 6]
Þ – 10 < x < 0. Þ 0 < x < 2 Þ 0 < x < 10.
\ t1t2n = 16 – 9(2n – 1)2 5
\ 16 – 9(2n – 1)2 + 713 ³ 0 11. (c) In the quadratic equation ax2 + bx + c = 0
\–4£n£5 \ n=5 b c
Hence, from a + 6n = 7, a = – 23 D = b 2 - 4 ac and a + b = - , ab =
a a
a+b b+c a 2 + b2 = (a + b)2 - 2ab
8. (d) Given, b 2 = ac, x = ,y=
2 2
b 2 2c b2 - 2ac
a =
= 2- =
c 2a 2c a a a2
Now, + = +
x y a+b b+c
3 3 b3 3c æ b ö
2( ab + ac + ac + bc) and a + b =- - ç- ÷
= =2 [Q b 2 = ac] a 3 a è aø
ab + ac + b 2 + bc æ b - 3abc ö
3
= -ç ÷
b b é 1
+ = 2b ê +
1 ù è a3 ø
Again, ú
x y ëa +b b + cû ATQ a + b, a2 + b2, a3 + b3 are in G.P.
2 3
=
2b(b + c + a + b)
=2 Þ - b , - b - 2ac , - (b - 3 abc) are in G.P.
ab + ac + b2 + bc a a2 a3
2
æ a cö æ b bö æ b2 - 2 ac ö b æ b3 - 3 abc ö
\ ç + ÷ ç + ÷ = 4. Þ ç ÷ = ç ÷
è x yø è x yø è a2 ø aè a3 ø
9. (d) Given that a, b, c are in A.P. Þ b + 4a c – 4ab c = b – 3ab2c
4 2 2 2 4

Þ 2b = a + c Þ 4a2c2 – ab2c = 0 Þ ac D = 0
but given a + b + c = 3/2 Þ 3b = 3/2 Þ c D = 0 ( Q In quadratic a ¹ 0)
Þ b = 1/2 and then a + c = 1 12. (c) Let first term = a, common ratio = r, where
–1 < r < 1
Again a2, b2, c2, are in G.P. Þ b4 = a2 c2
a a3
2
Þ b = ± ac Þ ac = or -
1 1 Then, = 2 and = 24
4 4 1- r 1 - r3
and a + c = 1 ....(i)
Considering a + c = 1 and ac = 1/4
104 MATHEMATICS
(Q r > 1)
1- r3 1
\ =
(1 - r ) 3 3 1+ 5
\ [r] = 1. Also – <–r<–1
2
i.e., 1 – 2r + r2 = 3 (1 + r + r2 ) or 2r 2 + 5r + 2 = 0 \ [– r] = – 2
-1 [r] + [– r] = 1 – 2 = – 1.
\ r = -2 or 16. (c) We have 2b = a + c and a, p, b, q, c are in
2
A.P.
1
As – 1 < r < 1 \ we have r = - a+b b+c
2 Þ p= ,q=
2 2
3 3 3
\ The series is 3 - + - + ... Again, p ' = ab and q ' = bc
2 4 8
13. (b) Given b2 = ac (Q a, b, c are in G. P.) and (a + b)2 - (b + c )2
2(log 2b – log 3c) = log a – log 2b \ p2 - q2 =
4
+ log 3c – log a
[Q given terms are in A. P.] (a - c )(a + c + 2b)
= = (a - c)b = p '2 - q '2
2 4
æ 2b ö æ 3c ö 3c
Þ log ç ÷ = log ç ÷ Þ b = 17. (d) Let S = 1 + 2.2 + 3. 22 + 4 .23 + 5.24 + ....
è 3c ø è 2b ø 2
+ 100 . 299
b2 3b 9c \ 2S = 1.2 + 2.22 + 3.23 + ... + 99.299 + 100 . 2100
Now, a = = =
c 2 4 Subtracting, we get
\ a is the largest side. –S = 1 + 1.2 + 1.22 + .... + 1.299 – 100. 2100
b2 + c2 – a 2 = (1 + 2 + 22 + ... + 299) – 100 . 2100
Now, cos A =
2bc
1(2100 - 1)
= - 100.2100 = 2100 - 1 - 100.2100
9c 2 81 2 -1
+ c2 – c2
4 16
=
3c
= negative \ S = 100.2100 - 2100 + 1 = 99.2100 + 1
2´ ´c 18. (a) Consider two real numbers (a + b) and
2
\ A > 90° \ triangle is obtuse. (c + d). Using G. M. £ A. M.

14. (c) y = 2 x , being in the first quadrant. The ( a + b) + ( c + d )


We get ( a + b)(c + d ) £
sequence of x-coordinate 1, 2, 4, 8, ................ 2
\ the sequence of y-coordinate 2
Þ M £ 2 Þ M£ 1
2, 2 2 , 2 4 , 2 8 , ................ is a G.P. where
the common ratio is 2. Also a, b, c, d are positive. Therefore
M = (a + b) (c + d) > 0. \ 0 < M £ 1
yn = 2( 2)n –1 = ( 2)n + 1
n
n+2 æ 1ö
15. (c) Let sides of triangle be a, ar, ar 2 19. (a) tn = .ç ÷
n(n + 1) è 2ø
Since r > 1 \ ar 2 is greatest side
n
2
\ a + ar > ar Þ r2 – r – 1 < 0 2(n + 1) - n æ 1 ö
= .ç ÷
n(n + 1) è 2 ø
1- 5 1+ 5 1+ 5
<r< Þ 1<r<
2 2 2
Solutions 105

n-1 n From Eqs. (i) and (ii) , we get


1 æ1ö 1 æ1ö r2 – 2r + 1 = 16r – 64 Þ r2 – 18r + 65 = 0
= ç ÷ – . .
n è 2ø n + 1 çè 2 ÷ø (r – 5) (r – 13) = 0
r = 5 or 13
n ìï1 æ 1 ö° 1 æ 1 ö1 üï If r = 5, then a(80 – 64) = 64 and hence a = 4. In
Sn = å t n = í ç ÷ - ç ÷ ý this case the numbers are 4,20,100 and their sum
n=1 ïî1 è 2 ø 2 è 2 ø ïþ is 124.
23. (189) log6 (abc) = 6 Þ (abc) = 66
ìï 1 æ 1 ö1 1 æ 1 ö 2 üï
+ í ç ÷ - ç ÷ ý + .......... b 36
Let a = and c = br Þ b = 36 and a =
ïî 2 è 2 ø 3 è 2 ø ïþ r r
Þ r = 2, 3, 4, 6, 9, 12, 18
ìï 1 æ 1 ö n -1 1 æ 1 ö üï
n
1
- æ 1ö
+ í ç ÷ ç ÷ ý =1– Also 36 ç1– ÷ is a perfect cube. Þ r = 4
n è 2ø n + 1 è 2ø ï ( n + 1)2 n è rø
îï þ
20. (a) As A.M. ³ G.M., we get Þ a + b + c = 9 + 36 + 144 = 189.
1/ 3 24. (3) We have
1æ 2 b b ö æ 2 b b ö
ç ax + + ÷ø ³ çè ax × × ÷ø 2 6 10 14
3è 2x 2 x 2 x 2x S = 1 + + 2 + 3 + 4 + .......¥ ....(i)
3 3 3 3
1/ 3
1 æ 2 b ö æ ab ö 1
Þ ç ax + ÷ø ³ çè ÷ø
3è x 4 Multiplying both sides by , we get
3
b 1 1 2 6 10
But the least value of ax2 + is c, therefore S = + 2 + 3 + 4 + ........¥ ....(ii)
x 3 3 3 3 3
1/ 3 Subtracting eqn. (ii) from eqn. (i), we get
æ ab2 ö
3ç ÷ ³ c Þ 27ab2 ³ 4c3
è 4 ø 2 1 4 4 4
S = 1 + + 2 + 3 + 4 + ........¥
3 3 3 3 3
21. (0.60) Let a = first term of G.P. and r = common
ratio of G.P.; Then G.P. is a, ar, ar2 2 4 4 4 4
Þ S = + 2 + 3 + 4 + ........¥
a 3 3 3 3 3
Given S ¥ = 20 Þ = 20
1- r 4
Þ a = 20(1 – r) ... (i) 2 3 = 4´3 Þ S =3
Also a 2 + a 2 r 2 + a2r4 + ... to ¥ =100 Þ S=
3 1 3 2
1-
a2 3
Þ =100 25. (990) Let S1= 2 + 3 + 5 +9 + 16 +............+ xn
1 - r2
Þ a2 = 100(1 – r)(1 + r) ... (ii) S1 = 2 + 3 + 5 + 9 + ..........x n -1 + x n
2 2
From (i), a = 400(1 – r) ;
0 = 2 + [1 + 2 + 4 + 7 + ......
From (ii), we get 100(1 – r)(1 + r) = 400(1 – r)2
+ to (n - 1) term] - xn
Þ 1 + r = 4 – 4r Þ 5r = 3 Þ r = 3/5 = 0.60
22. (124) Let b = ar and c = ar2. Given that a, ar, ar2 \ x n = 2 + [1 + 2 + 4 + 7 + .....to (n - 1) terms]
– 4 are in AP. Therefore Again let
a + (ar2 – 64) = 2(ar) ...(i) S2 = 1 + 2 + 4 + 7 + ........+ tn–1
2
Þ a(r – 2r + 1) = 64
S2 = 1 + 2 + 4 + 7..... + t n - 2 + t n -1
Again a, ar – 8, ar2 – 64 are in GP. Therefore
a(ar2 – 64) = (ar – 8)2 ...(ii)
0 = 1 + [1 + 2 + 3 + ....... + (n - 2) term] - t n -1
Þ a(16r – 64) = 54
106 MATHEMATICS
(c – a)3 = 2(a – d) + (b – d)2 + (c – d)3
(n - 2)(n - 1) n 2 - 3n + 4 becomes – 2D + kD2 + (2D)3 = – 6D + 4D2 – D3
t n -1 = 1 + =
2 2 Þ 9D2 + (k – 4)D + 4 = 0
n -1 Since D is real, we have (k – 4)2 – 4(4) (9) ³ 0
1 3
\ S2 = å t n -1 = 2 Sn 2 - 2 Sn + 2S1 or k2 – 8k – 128 ³ 0 or (k – 16) (k + 8) ³ 0
\ k Î(– ¥, – 8] È [16, ¥)
n =1
Hence, the smallest positive value of k = 16.
1 ( n - 1) n ( 2n - 1) 3 n ( n - 1)
= - + 2( n - 1) 28. (1) x Î (0, p), implies |cos x| < 1. Thus,
2 6 2 2 S¥ = 1 + |cos x| + cos2 x + |cos3 x| + ... up to ¥
= 1 + |cos x| + |cos x|2 + |cos x|3 + ... up to ¥
é 2n 2 - n 3n ù
= (n - 1) ê - + 2ú 1
ëê 12 4 úû =
1– | cos x |
n -1 é 2
= 2n - n - 9n + 24 ù 3
12 ë û \ 33/(1 – |cos x|) = 36 Þ =6
1– | cos x |
n -1 é 2 n 3 - 6n 2 + 17n - 12
= n - 5n + 12ù = 1 1
6 ë û 6 Þ |cos x| =
2
Þ cos x = ±
2
n 3 - 6n 2 + 17n - 12 p 2p 1
\ x n = 2 + S2 = 2 + Þ x= , Þ ( x1 + x2 ) = 1.
6 3 3 p
29. (6) For the given A.P., we have
n 3 - 6n 2 + 17n 2(2a + b) = (5a – b) + (a + 2b)
=
6 Þ b = 2a ...(1)
So, x20 = 990 Also for the given G.P., we have (ab + 1)2 =
26. (5) Let k and k + 1 be removed. Then, (a – 1)2 (b + 1)2 ...(2)
Putting b = 2a from (1) in (2), we get a = 0, – 2, or
n
(n + 1) - 2k - 1 1
2 105
= 4
n-2 4
1 1
Þ 2n 2 - 103n - 8k + 206 = 0 But a > 0, so a = and b = 2a =
Since n and k are integers, so n must be even, 4 2
Hence, (a– 1 + b– 1) = 2 + 4 = 6.
4m 2 + 103(1 - m) 30. (2) Let the two quantities be a and b
say n = 2m, then k = . Clearly
4 b–a 2(b – a )
(1 – m) must be divisible by 4. G = ab ; p = a + ;q=a+
3 3
Let m = 1 + 4t, then we get k = 16t2 – 95t + 1 and
k (b – a )
1£ k < n (kp – q) = ka + –a
3
Þ 1 £ 16t 2 - 95t + 1 < 8t + 2
(b – a ) b–a
Þ t = 6 and so, n = 50 –2 = a( k –1) + ( k – 2)
3 3
27. (8) Since a, b, c, d are in A.P., we have 2k (b – a) (b – a )
b – a = c – b = d – c = D (let common difference) kq – p = ak + –a–
3 3
or d = a + 3D
Þ a – d = – 3d and d = b + 2D b–a
Þ a(k – 1) + (2k – 1)
or b – d = – 2D 3
Also c = a + 2D or c – a = 2D \ (kp – q) (kq – p) = ab \k=2
\ Given equation 2(a – b) + k(b – c)2 +
Solutions 107
CHAPTER
10 Straight Lines
1. (a) Given P º (a, b) 13 - 3
Now, AB is y – 3 = (x – 4)
x y 7-4
Given line is + =1 ...(i)
a b Þ 3y – 10x + 31 = 0.
If line (i) cuts x and y axes at A and B respectively,
then A = (a, 0) and B = (0,b). Y (7,13)
1 Ry = x
Also the area of D OAB = S i.e. ab = S
2
A (3,4)
Þ ab = 2S P
Since line (i) passes through P( a , b )
a b a ab (4,3)
+ =1 Þ + =1 A’
a b a 2 S
X
2
Þ a b – (2S)a + 2 a S = 0
Since a is real, 4S 2 – 8abS ³ 0 Þ S ³ 2ab æ 31 31 ö
It intersects y = x at ç , ÷ , which is the
Hence the least value of S = 2ab . è 7 7ø
2. (b) a, b, c are the roots of equation required point P.
4. (c) Equation of AO is 2x + 3y – 1 + l (x + 2y – 1) = 0
x 3 - 3 x 2 + 6x + 1 = 0. Where l = – 1, since the line passes through
So, a + b + c = 3, ab + bc + ca = 6 and the origin. So, x + y = 0
abc = – 1
Now, the centroid of the triangle is æ aö
Since AO isperpendicular to BC, (– 1) ç - ÷ = – 1
è ø b
æ 1 1 1ö
+ + \ a =–b
ç ab + bc + ca ab bc ca ÷
ç , ÷
3 3 A
ç ÷
è ø
x+

0
1=
2y

-
æ ab + bc + ca a + b + c ö 3y O
-1

i.e. ç , +
è 3abc ÷ø 2x
=0

3
B ax + by - 1 = 0 C
æ6 3 ö
=ç , ÷ or (2, – 1)
è 3 -3ø Similarly, (2x + 3y – 1) + m (ax – ay – 1) = 0
3. (c) Consider a point A’ , the image of A in y = x will be the equation of BO for m = – 1.
\ Coordinates of A’ = (4, 3) BO is perpendicular to AC
[Notice that A and B lie to the same side with
respect to y = x]. ì (2 - a) ü æ 1 ö
Þ í- ý ç - ÷ = – 1. \ a = – 8, b = 8.
Then PA = PA’. Thus, PA + PB is minimum, î 3+ a þ è 2ø
if PA’+ PB is minimum, i.e. if P, A’, B are collinear.
108 MATHEMATICS
5. (d) The coordinates of A are (a, 0) and of B are
(0, b). If the coordinates of C are (x1, x1) then æ6 ö
y = 6 intersects y = mx at Q ç , 6 ÷
area of the èm ø
1 2
D AOC = OA × x1 and the area of the
Thus PQ = æç - ö÷ + ( 6 - 2 )2 < 5
6 2
2
èm mø
1
D BOC = OB × x1 . 2
2 æ4ö
According to the given condition, Þ ç ÷ + 16 < 25
èmø
1 1 2
OA × x1 = 2 × OB × x1 Þ OA = 2OB æ 4ö 4 3 1 3
2 2 Þ ç ÷ < 9 Þ -3 < < 3 Þ - < <
è mø m 4 m 4
Þ a = 2b.

\ æ 4ö æ 4 ö
x m Îç - ¥, - ÷ È ç , ¥÷
B ( 0, b) Ry
=
è 3ø è 3 ø
8. (a) Let P(1, 0) and Q(–1, 0), A(x, y)
C
AP BP CP 1
Given: = = =
O A (a, 0) AQ BQ CQ 2
x y Þ 2AP = AQ Þ 4(AP)2 = AQ2
Equation of AB is therefore + =1 Þ 4[(x – 1)2 + y2] = (x + 1)2 + y2
2b b
or x + 2y = 2b Þ 4(x2 + 1 – 2x) + 4y2 = x2 + 1 + 2x + y2
Þ 3x2 + 3y2 – 8x – 2x + 4 – 1 = 0
Since C lies on it, x1 + 2 x1 = 2b
Þ 3x2 + 3y2 – 10x + 3 = 0
2b a
Þ x1 = = 10
3 3 Þ x2 + y2 – x +1=0 ...(1)
Hence the coordinates of C are 3
\ A lies on the circle given by (1). As B and C
æ 2b 2b ö æa aö also follow the same condition.
çè 3 , 3 ÷ø or çè 3 , 3÷ø
.
\ Centre of circumcircle of DABC = centre of
6. (c) ry
circle given by (1)
ry+3x+2=0
æ5 ö
7/2
4y+x-14=0 = ç ,0÷ .
è3 ø
9. (b) Let C = (x1, y1)
5/3 3y-2x-5=0
A(– 3, 2)
1

X
–5/2 –2 –1 –2/3 O 1 2 3 4 2
–1
–2 E
5 7 1
From diagram it is clear that £ b £ . B C
3 2 D
(– 2, 1) (x1, y1)
æ2 ö æ x1 - 2 y1 + 1 ö
7. (c) y = 2 intersects y = mx at P ç , 2 ÷ ç , ÷
èm ø è 2 2 ø
æ x - 5 y1 + 3 ö
Centroid, E = ç 1 , ÷
è 3 3 ø
Solutions 109
Since centroid lies on the line b
12. (d) Slope of the line in the new position is ,
3x + 4y + 2 = 0 a
since it is ^ to the line ax + by + c = 0 and it cuts
æ x -5ö æ y1 + 3 ö
\ 3ç 1 ÷ + 4ç ÷+2 = 0 the x-axis at (2,0). Hence, the required line passes
è 3 ø è 3 ø
b
Þ 3x1 + 4y1 + 3 = 0 through (2, 0) and its slope is . Required eq. is
Hence vertex (x1, y1) lies on the line 3x + 4y + 3 = 0 a
10. (d) Circumcentre = (0, 0) b
y-0 = ( x - 2)
æ (a + 1) (a - 1)
2 2ö a
Centroid = ç 2 , 2 ÷ Þ ay = bx - 2b Þ ay - bx + 2b = 0
è ø
We know the circumcentre (O), 13. (d) Let L1 (x, y) = x – y – 1 and L 2 (x, y)
Centroid (G) and orthocentre (H) of a triangle = 2 (x – y ) + 5
lie on the line joining the O and G.
Then, from figure P(a, 2) lies below L 2 ,
HG 2
Also, = Þ Coordinate of orthocentre 2a - 4 + 5 1
GO 1 So <0 Þ a > -
-2 2
3(a + 1) 2 3(a - 1)2
= ,
2 2 L 2 : 2(x – y) + 5 = 0
Now, these coordinates satisfies eqn given in P (a, 2)
option (d)
Hence, required eqn of line is L1 : x - y - 1 = 0
(a – 1)2 x – (a + 1)2 y = 0
11. (a) The line passing through the intersection
of lines ax + 2by + 3b = 0 and
bx - 2ay - 3a = 0 is
Also P(a, 2) lies above L1 ,
ax + 2by + 3b + l (bx – 2ay – 3a) = 0
Þ (a + b l ) x + (2b – 2a l )y + 3b – 3 l a = 0 a - 2 -1
So >0 Þ a<3
As this line is parallel to x-axis. -1
\ a + b l = 0 Þ l = – a/b 1
Taking intersection, we get - <a<3
a 2
Þ ax + 2by + 3b – (bx – 2ay – 3a) = 0
b
æ 1 ö
2a 2 3a 2 Þ a Î ç - , 3÷ .
Þ ax + 2by + 3b – ax + y+ =0 è 2 ø
b b
14. (b)
æ 2a 2 ö 3a 2 A(5, – 1)
y ç 2b + ÷ + 3b + =0
ç b ÷ø b
è
æ 2 2 ö æ 3b2 + 3a 2 ö
y ç 2b + 2a ÷ = -ç ÷
ç b ÷ ç b ÷
è ø è ø H
2 2 (0, 0)
-3(a + b ) -3
y= =
2(b2 + a 2 ) 2
B (– 2, 3)
C (a, b)
So it is 3/2 units below x-axis.
110 MATHEMATICS

Let the third vertex of DABC be (a, b). Now, In DPCB, (PB)2 = (PC)2 + (CB)2
Orthocentre = H(0, 0) (By Pythagoras theoresm)
Let A (5, – 1) and B (– 2, 3) be other two vertices 2
of DABC. æ 2ö a2
Þ a2 = ç ÷ +
Now, (Slope of AH) × (Slope of BC) = – 1 è 5ø 4

æ -1 - 0 ö æ b - 3 ö a4 4 3a 2 4
Þ çè ÷ç ÷ = -1 Þ a2 - = Þ =
5 - 0 ø è a + 2ø 4 25 4 25
Þ b – 3 = 5 (a + 2) ...(1)
Similarly, 16 16 4 3 4 3
a2 = Þa= = ´ =
(Slope of BH) × (Slope of AC) = – 1 75 75 5 3 3 15

æ 3ö æ b + 1ö 4 3
Þ -ç ÷ ´ç = -1
è 2 ø è a - 5 ÷ø \ Length of Equilateral triangle ( a) =
15
Þ 3b + 3 = 2a – 10 16. (a) Let, L1 (x, y) = x + y + 1
Þ 3b – 2a + 13 = 0 ...(2)
On solving equations (1) and (2) we get and L 2 (x, y) = 2x – 3y – 5
a = – 4, b = – 7 \ L1 (10, – 20) = 10 – 20 + 1 = – 9 Þ ‘–’ve
Hence, third vertex is (– 4, – 7).
15. (b) and L 2 ( 10, – 20) = 20 + 60 – 5 = 75 Þ ‘+’ve
\ Equation of the bisector will be
P (1, 2)
x + y +1 æ 2 x - 3 y - 5ö
= -ç ÷
2 è 13 ø

Þ x ( 13 + 2 2 ) + y ( 13 – 3 2 ) + ( 13
A C B 3x + 4y = 9
–5 2 )=0
Shortest distance of a point (x1, y1) from line 17. (c) If a point is equidistant from the two
intersecting lines, then the locus of this point is
ax1 + by1 - c the angle bisector of those lines.
ax + by = c is d =
a2 + b2 Now, let (h, k) be the point which is equidistant
from the lines 4x – 3y + 7 = 0 and 3x – 4y + 14 = 0
Now shortest distance of P (1, 2) from 3x + 4y
= 9 is Then 4h - 3k + 7 = ± 3h - 4k + 14
4 2 + ( -3) 2 32 + ( -4) 2
3(1) + 4(2) - 9 2
PC = d = =
2
3 +4 2 5 Þ 4h - 3k + 7 = ± (3h - 4k + 14)
Given that DAPB is an equilateral triangle Þ h + k – 7 = 0 and 7h – 7k + 21 = 0
Let 'a' be its side Hence locus of (h, k) is x + y – 7 = 0

a and x – y + 3 = 0
then PB = a, CB =
2
Solutions 111
18. (7) The given lines are
2 - 2 -1 1
2x + y = 9/2 … (i) = =
and 2x + y = – 6 … (ii) 5 5
Signs of constants on R.H.S. show that two
lines lie on opp. sides of origin. Let any line æQ Perpendicular distance of ö
through origin meets these lines in P and Q ç ÷
respectively then req. ratio is OP : OQ ç ax + by = c from (x1 , y1 )is ÷
ç ÷
Y ç ax + by1 + c ÷
p= 1
(0, 9/2) ç ÷
è a 2 + b2 ø
B
P
Let AB = BC = AC = x
C (–3, 0) A X
x' O (9/4, 0) So, in DABD , AD 2 + BD 2 = AB 2
2
Q D (0, –6) æ 1 ö x2 æ BC ö
Þ çç ÷÷ + = x2 çQ BD = ÷
è 5ø 4 è 2 ø
y'
Now in DOPA and DOQC, 4 2 2
Þ x2 = Þx= =
Ð POA = Ð QOC (ver. opp. Ð' s) 15 15 K
Ð PAO = Ð OCQ (alt. int. Ð' s) Þ K = 15
\ DOPA ~ DOQC (by AA similarity) 20. (0) Since 3 .1 – 4 + 1 < 0 ,
OP OA 9 / 4 3
\ = = = so 3 sin q – cos q + 1 £ 0
OQ OC 3 4
\ Req. ratio is 3 : 4.
3 1 1
Þ sin q – cos q £ –
9 -6 m 2 2 2
= OP : OQ = : = 3: 4 =
20 5 n
æ pö 1
Þ m + n = 7. Þ sin ç q - ÷ £ –
è 6ø 2
19. (15) Let ABC be an equilateral triangle with base
BC.
7p p 11p 4p
So, AD ^ BC Þ £ q– £ Þ £ q £ 2p
6 6 6 3
A (2,–1)
Þ maximum value of sin q is 0.
21. (4) Family of lines can be written as
(3x + 7 y + 11)sec q + cosec q (5 x - 3 y - 11) = 0

B D C Þ (5 x - 3 y - 11) + tan q (3 x + 7 y + 11) = 0


x+2y =1 So, for any value of q this line passes through
2 + 2(-1) - 1 the point of intersection of lines
Now, length AD = 5 x - 3 y - 11 = 0and 3x + 7 y + 11 = 0 i.e from
(1)2 + (2)2
P(1, – 2).
112 MATHEMATICS

Let A( x, y ) be a point on the line k + h 5+ 4


Þ = =9
x- y+3=0 k –h 5–4
Applying cosines rule in DPAB
PB2 = PA2 + AB2 - 2PA. AB cos q ³ PA2 + AB2 24. (5) A(1, – 2)

-2PA. AB = ( PA - AB )2

=0

x–
+y

y=
Þ ( PA - AB) 2 £ PB 2

3
2x
So, maximum value of
B x + py = q C
| PA - PB | is PB = 4 + 36 = 2 10.
æ q 2q ö æ 3 p + q q 3ö
Þ k 2 = 40 çè 2 p 1 , 2 p 1÷ø çè p + 1 , p + 1÷ø
22. (1) Let the equation of L2 be L1 + lL = 0
Þ (1 + l)x + (2 + l)y + 3 + l = 0 P is the orthocenter. Therefore, AP ^ BC

1+ l æ 1 ö æ 3 + 2ö 5
Slopes of L2, L and L1 are – , – 1, – 1/2 or ç – ÷ ç = – 1 or = 1 or p = 5
2+l ÷
è p ø è 2 –1 ø p
Since L is the bisector of the angle between L1 27 – 2q
and L2 Since BP ^ AC, we have =–1
18 + q
1+ l or q = 27 + 18 or
– +1 q = 45
2+ l –1 + 1/ 2
\ = Þl=–3 \ p + q = 5 + 45 = 50
1+ l 1 + 1/ 2
1+ 25. (4) Since y = x is the perpendicular bisector of
2+l
the side AB and A = (1, 2), we have B = (2, 1).
So the equation of L2 is y + 2x = 0 Since the image A'(x, y) of A in the angular bisec-
tor x – 2y + 1 = 0 lies on the line BC, we have
Þ m = – 2 and 812m2 + 3 = 812 × 4 + 3 = 3251
23. (9) tana, tanb, tanl are the roots of equation x –1 y – 2 – 2(1– 2(2) + 1) 4
= = =
t3 – 12t2 + 15t – 1 = 0 (given). 1 –2 12 + 2 2 5
So, tana, tanb, tanl = 12 and tana, tanb,
æ 9 2ö
tanl = 1. Also Stana tanb = 15 Therefore, A' = ç , ÷
è 5 5ø
When A(tana, cota), B(tanb, cotb) and C(tanl,
Since equation of BC is the equation of BA', we
cotl)
have the equation of BC as
The centroid G(h, k)
1– (2 / 5)
y–1= ( x – 2)
æ tan a + tan b + tan l cot a + cot b+ cot l ö 2 – (9 / 5)
=ç , ÷
è 3 3 ø Þ y – 1 = 3(x – 2)
Þ 3x – y – 5 = 0
æ 12 D tan a.tan b ö So that a = 3, b = – 1. Hence, a – b = 4.
=ç , ÷ = (4, 5)
è 3 3(tan a tan b tan l) ø
Solutions 113
CHAPTER
11 Conic Sections
1. (a) Since 4x + 3y – 4 = 0 is dividing the
circumference in the ratio 1 : 2, angle subtended Þ y = x + (5 2 – 4)
at the centre = 2 p / 3. For no solution c > 5 2 – 4
Also the perpendicular distance from the centre
\ c Î (5 2 – 4, ¥ ).
of the given line is 5 5. (b) Equation of the given circle is
Þ Radius = 10 Þ Equation of the circle is
x 2 + y 2 + 2gx + 2fy + c = 0 ......(i)
x 2 + y 2 - 10 x - 6 y - 66 = 0.
2. (b) Y
y
P given circle
C(4 cos q , 4 sin q )
R

A B
x Q
(– 4, 0) O

O circumcircle X
of DOPQ
1
A = . 8 . 4 sinq = |16 sin q| Equation of the chord of contact PQ, drawn from
2
1 2 15 the origin (0, 0) to the given circle will be
Now, sinq can be equal to , ,... gx + fy + c = 0 .....(ii)
16 16 16 Eq. of any circle passing through the intersection
i.e. there are 15 points in each quadrant. points of the given circle and the chord PQ can
3. (c) If (a, 0) is the centre C and P is (2, – 2), be written as
then Ð COP = 45°. 2 2
( x + y + 2gx + 2fy + c ) + l (gx + fy + c) = 0
Since the equation of OP is x + y = 0.
....(iii)
\ OP = 2 2 = CP. Hence OC = 4 If this circle passes through the origin, then we
Y have,
c + l c = 0 gives l = – 1
Putting the above value of l in equation (iii)
C
X
gives the equation of the required circle as
O A B
x 2 + y 2 + gx + fy = 0
P
(2

6. (b)
,

2 2
–2

B x + y =1
)

The point on the circle with the greatest x


coordinate is B.
a = OB = OC + CB = 4 + 2 2 .
P (–2, –1) (0, –1)
2
4. (d) Since y = | x | + c and x + y 2 – 8 | x | – 9 = 0
both are symmetrical about y-axis we consider Any line through ( – 2, – 1) is y + 1 = m ( x + 2 )
the case x > 0, when the equations become
2 2m - 1
y = x + c and x + y 2 – 8x – 9 = 0 . Equation of It touches the circle if =1
1 + m2
tangent to circle x 2 + y 2 – 8x – 9 = 0 parallel to
4
y = x + c is y = (x – 4) + 5 1 + 1 Þ m = 0,
3
114 MATHEMATICS

4 9. (b) The equations of the circles are


\ Equation of PB is y + 1 = ( x + 2) x 2 + y 2 - 10 x - 10 y + l = 0 ...(i)
3
Þ 4 x– 3y+ 5=0 and x 2 + y 2 - 4 x - 4 y + 6 = 0 ...(ii)
A point on PB is (– 5, – 5), (we can choose some C1 = centre of (i) = (5, 5)
other point as well) C2 = centre of (ii) = (2, 2)
Its image by the line y = – 1 is P¢ ( – 5 , 3). d = distance between centres = C1C2
Hence equation of incident ray PP¢ is
= 9 + 9 = 18
3 +1
y -3 = ( x + 5) Þ 4 x + 3 y + 11 = 0 r1 = 50 - l , r2 = 2
-5 + 2 For exactly two common tangents we have
7. (b) Circle : x2 + y2 + 3x = 0,
r1 - r2 < C1C2 < r1 + r2
æ 3 ö Þ
Centre, B = ç – , 0÷ 50 - l - 2 < 3 2 < 50 - l + 2
è 2 ø
Þ 50 - l - 2 < 3 2 or 3 2 < 50 - l + 2
3
Radius = units. Þ 50 - l < 4 2 or 2 2 < 50 - l
2
Y Þ 50 - l < 32 or 8 < 50 - l
Þ l > 18 or l < 42
Required interval is (18, 42)
A(0, 1) 10. (b) Equation of required circle :
S : (x – 1)2 + (y – 1)2 + l (x – y) = 0
S' : x2 + y2 – 2y – 3 = 0
Common chord of S = 0 and S¢ = 0 is S – S ' = 0
X¢ B O X (l – 2) x – (l + 4) y + 5 = 0
æ 3 ö Centre of S ' : (0, – 1) lies on common chord
çè – , 0÷ø
2 Þ l =–9
9
S : (x – 1)2 + (y – 1)2 – 9 (x – y) = 0 Þ r =
2
Y¢ 11. (c) 2
y = 8x
Line : y = mx + 1 A
y-intercept of the line = 1 2 2
x +y =3 2
\ A = (0, 1)
OA
Slope of line, m = tan q =
OB
B
1 2
Þ m= = Þ 3m – 2 = 0 We 2
have : x + (8x) = 9
3 3
Þ x2 + 9x – x – 9 = 0
2
8. (b) Let the variable circle is Þ x (x + 9) – 1 (x + 9) = 0
Þ (x + 9) (x – 1) = 0 Þ x = –9, 1
x 2 + y 2 + 2 gx + 2 fy + c = 0 ...(i)
It passes through (a, b) for x = 1, y = ± 2 2 x = ± 2 2
L1 = Length of AB
\ a 2 + b2 + 2 ga + 2 fb + c = 0 ...(ii)
2 2
(i) cuts x + y = 4 orthogonally
= (2 2 + 2 2) 2 + (1 - 1) 2 = 4 2
\ 2( g ´ 0 + f ´ 0) = c - 4 Þ c = 4 L2 = Length of latus rectum = 4a = 4 × 2 = 8
2 2
L1 < L2
\ from (ii) a + b + 2 ga + 2 fb + 4 = 0 12. (c) Equation of the tangent at P (x1, y1) to
\ Locus of centre (–g,–f ) is y2 = 4ax is yy1 – 2ax – 2ax1 = 0 ...(i)
a 2 + b2 - 2ax - 2by + 4 = 0 Equation of the chord of y2 = 4a(x + b) whose
mid-point is (x', y') is
or 2ax + 2by = a 2 + b 2 + 4
Solutions 115
yy' – 2ax – 2ax' – 4ab 15. (d) As per the definition, the locus must be an
= y'2 – 4a x' – 4ab or yy' – 2ax – (y'2 – 2a x') = 0 ellipse, with given points as foci and 10 as its
...(ii) major axis. Since the line segment joining (2, –3)
Equation (i) and (ii) represent the same line. and (2, 5) is parallel to y-axis, therefore, ellipse is
y1 2a 2 ax vertical.
\ = =–
y' 2a 2
y ' - 2ax ' 4
\ 2 be = 8 and 2b = 10 Þ b = 5 and e =
This gives y' = y1 and then 2ax1 = y'2 – 2ax' 5
= y12 – 2ax' = 4ax1 – 2ax' \ x' = x1 2
\ a = b 2 (1 - e 2 ) = 9 and centre of the
\ mid-point (x', y') = (x1, y1). ellipse is (2, 1)
13. (d) The given curve is y = x2 + 6
\ Equation of the required ellipse is
Equation of tangent at (1, 7) is
1 ( x - 2) 2 ( y - 1)2
( y + 7) = x .1 + 6 Þ 2x - y + 5 = 0 ...(i) + =1
2 9 25
As given this tangent (1) touches the circle x2 y2
x2 + y2 +16x + 12y + c = 0 at Q 16. (a) For ellipse 2
+ =1
4 32
Centre of circle = (– 8, – 6). a = 4, b = 3
Q æ3ö
2
7
=0 Þ e = 1–ç ÷ =
–y+5 è4ø 4
2x
P(1,7) C
\ Foci are ( )
7, 0 and – 7,0 ( )
(–8,–6) Centre of circle is at (0, 3) and it passes through
(± )
7, 0 , therefore radius of circle

( 7)
2
+ ( 3) = 4
Then equation of CQ which is perpendicular to 2
=
(1) and passes through (– 8, – 6) is
17. (d) x2 = 8y ...(i)
1
y + 6 = – ( x + 8) Þ x + 2y + 20 = 0 ...(ii) x2
2 + y2 = 1 ...(ii)
Now Q is pt. of intersection of (i) and (ii) 3
\ Solving eqs (i) & (ii) we get ; x = – 6, y = – 7 8y 1
\ Req. pt. is (– 6, – 7). From (i) and (ii), + y 2 = 1 Þ y = – 3,
14. (a) Since S = (a, 0) = (1, 0), the circle is of the 3 3
form (x – 1)2 + y2 = r2 When y = – 3, then x2 = – 24, which is not possible.
Suppose AB is a common chord. Since this is 1 2 6
equidistant from the focus and the vertex. When y = , then x = ±
3 3
M(1/2,0) lies on AB and AB is double ordinate of Point of intersection are
the parabola, let A = (1/2, y) so that
æ 2 6 1ö æ 2 6 1ö
æ1ö ç , ÷ and ç - , ÷
y2 = 4 ç ÷ Þ y = ± 2 è 3 3ø è 3 3ø
è2ø
1
æ1 ö æ1 ö Required equation of the line, y - =0
Þ A = ç , 2 ÷ and B = ç , - 2 ÷ 3
è 2 ø è 2 ø Þ 3y – 1 = 0
Since DAMS is right-angled triangle, we have
x2 y2
1 9 18. (c) Given ellipse is + =1 ... (i)
SA2 = SM2 + MA2 = + 2 = = (Radius)2 16 4
4 4
Hence, the equation of the circle is Equation of a circle centered at (1, 0) can be
written as (x –1)2 + y2 = r2 ... (ii)
9 The abscissae of the intersection points of the
(x – 1)2 + y2 =
4 circle and the ellipse is given by the equation
116 MATHEMATICS
See figure B (4, 2) is one end of the minor axis of
16 - x 2 2
( x - 1)2 + =r ( x - 4)2 y 2
4 the ellipse + = 1 and (– 1, 2) is one
25 4
end of the major axis of the second ellipse.
Therefore,
(1, 0) (4, 0)
AB = 5, OB = 16 + 4 = 20 , OA = 1 + 4 = 5
2 2 We have (OA)2 + (OB)2 = 25 = (AB)2
i.e. 4 ( x – 2x + 1) + 16 – x = 4 r 2
p
2
i.e. 3 x – 8x + 20 – 4 r = 0
2 Therefore Ð AOB =
2
If the circle lies inside the ellipse, then the roots 21. (1) C
of the above equation must be imaginary or equal
Q
i.e. D £ 0 i.e. 64 + 12 (4 r 2 – 20) £ 0 S
11
Þ r= £ R B
3
11
Hence, greatest value of r = and the A P
3
equation of required circle is Point A ( 33 + 3, 0) lies on the given circle,
11 x2 + y2 – 6x – 8y – 24 = 0
( x - 1)2 + y 2 =
3 PQ and AB intersect inside the circle.
2 Let PR = a, RS = b, QS = c
i.e. 3 ( x + y 2 ) – 6x – 8 = 0. Since PR × RQ = AR × RB Þ a(b + c) = 3 × 7
x2 y 2 Also, QS × SP = 3 × 7 Þ c(a + b) = 3 × 7
19. (a) Ellipse is + =1 Þ a = c \ PR/QS = 1
16 3
Now, equation of normal at (2, 3/2) is 22. (11) The centre C of the circle = ( 5,7 ) and the
radius
16 x 3 y
- = 16 - 3 = 52 + 7 2 + 51 = 5 5
2 3/ 2
13 PC = 122 + 52 = 13 Þ q = PA = 13 - 5 5
Þ 8x – 2y = 13 Þ y = 4 x -
2
and p = PB = 13 + 5 3
13
Let y = 4 x - touches a parabola \ G.M. of p and q
2
y2 = 4ax. = pq = (13 - 5 5(13 + 5 5))
We know, a straight line y = mx + c touches a
parabola y2 = 4ax if a – mc = 0 = 169 - 125 = 2 11 = 2 k Þ k = 11.
æ 13ö
\ a - ( 4) ç - ÷ = 0 Þ a = – 26 A C (5, 7)
è 2ø
Hence, required equation of parabola is P (–7, 2) B
y2 = 4 (– 26)x = – 104 x
20. (a) y
(0, –1)
A B 2
23. (4) Tangent to y = 8 (x + 2) is
2
O
y = m (x + 2) +
x m
2 c æ 1ö
c = 2m + Þ = çè m + ÷ø
m 2 m
Solutions 117
25 25
1 c = c – c 2 Þ 4c 2 – 25 c + 25 = 0
Q m+ ³2 Þ ³ 2 Þ c³4 4 4
m 2 5
Þ The minimum value of c = 4. Þ c= ,5
24. (3) The locus of the point of intersection of 4
tangents to the parabola y2 = 4 ax inclined at 5 5 1 1
For c = , 1 + t 2 = Þ t2 = Þt =±
an angle a to each other is tan2a (x + a)2 4 4 4 2
= y2 – 4ax For c = 5, 1 + t 2 = 5 Þ t = ± 2
Given equation of Parabola y2 = 4x {a = 1}
Point of intersection (–2, –1) æ1 ö
A º ç , 1÷ , B º (4 , 4), C º (4, – 4) and
tan2a (–2 + 1)2 = (–1)2 – 4 × 1 × (–2) è4 ø
Þ tan2a = 9 Þ tan a = ± 3 Þ |tan a| = 3 æ1 ö
D º ç , -1÷
x2 y2 5 4 è4 ø
25. (27) + = 1 Þ e2 = 1 - = AD = 2 and BC = 8, distance between AD and BC
9 5 9 9
2 15
Þ e= =
3 4
\ Area of trapezium ABCD
æ 5ö
One end of latusrectum is ç 2, ÷ 1 15 75
è 3ø = ( 2 + 8) × = sq. units.
2 4 4
æ 5ö 2x y 27. (2) Due to symmetry the desired area
Equation of tangent at ç 2, ÷ is + =1
è 3ø 9 3 1
Q = 4 ´ area of DS1OS3 = 4 ´ ae ´ be1
æ 2ö æ 2ö
ç ae, b ÷ S 2
ç - ae, b ÷ 3
ç a ÷ø ç a ÷ø
è è

R F¢ C F P
O
S2 S1
æ 2ö æ 2ö
ç- ae, - b ÷ ç ae, - b ÷
ç a ÷ø ç a ÷
è è ø
S S4
1 9 27 Where e1 is eccentricity of conjugate hyperbola
Area of D CPQ = × ×3= = 2 ´ 2e ´ 3e1 = 12ee1
2 2 4
\
27 Now b 2 = a 2 (e 2 - 1) Þ e 2 = 13/ 4
Area of quadrilateral PQRS = 4 × = 27.
4 1 1 13
and 2 + 2 = 1 Þ e12 =
26. (4) Focus of the parabola y 2 = 4x is (1, 0) e e1 9
So diagonals are focal chord.
13 13
\ Required area = 12 ´ ´ = 26
AS = 1 + t 2 = c (say) 2 3
1 1 é 1 1 1ù 28. (9)
Q + 25 =1 êëQ AS + CS = a úû C
c -c y
4 L
B P
A 2 A Ö12
30° 30° Q(4 + 4Ö3, 0)
x
O N(4, 0)
S
B M(1, 0)

D D
C
x=1
118 MATHEMATICS
Common chord of both the circles is x = 1. For maximum length of the common chord, it
Now, we have to find the ratio of areas of must pass through the centre of the smaller circle
equilateral triangles ANB and CQD. ( r2 , r2 ) , so
Now in triangle OPN,
ON = OP cosec 30° = 2 × 2 = 4. r1
4r2 = r1 + r2 Þ =3
Area of triangle NAB r2
1 a + b + 2ab
MN . AB = MN . AM = MN. MN tan 30° Þ = 3 Þ 2 ( a + b ) = 4 2 ab
2 a + b - 2 ab
= (ON – OM)2 tan 30°
1 9 Þ ( a + b ) 2 = 8ab Þ a 2 - 6 ab + b 2 = 0
= (4 - 1) 2 = sq. units.
6b ± 36b2 - 4b2
= (3 ± 2 2 ) b
3 3
Þ a=
Now in triangle NLQ, 2
NQ = NL cosec 30° = 4 3. a
1 Þ = 3± 2 2
Since area of tr iangle CQD = QM .CD b
2 30. (5) The tangent at any point A(2sec q, tan q)
= QM .CM x sec q y tan q
is given by - = 1.
QM. QM tan 30° = (MN + NQ) 2 tan 30° 2 1
1 57 + 24 3 It meets the line x - 2 y = 0
= (3 + 4 3) 2 = sq. units. x sec q x tan q 2
3 3 Þ - =1 Þ x =
57 + 24 3 2 2 sec q - tan q
So, ratio of area of trianlges = . æ 2 1 ö
9 Þ Q ºç , ÷ ...(1)
29. (5) Let r be the radius of the circle. Its equation è sec q - tan q sec q - tan q ø
is x 2 + y 2 - 2r ( x + y ) + r 2 = 0. Since it passes Also, the tangent meets the line x + 2 y = 0 at
through P(a , b) R, so
x x
a 2 + b 2 - 2r ( a + b ) + r 2 = 0 Þ sec q + tan q = 1
2 2
Solving r1 = a + b + 2ab ...(1) 2
Þ x=
r2 = a + b - 2ab sec q + tan q
Now, the equations of two circles are æ 2 -1 ö
Þ Rºç , ÷ ...(2)
x 2 + y 2 - 2r1 ( x + y ) + r12 = 0 and è sec q + tan q sec q + tan q ø
Now,
x 2 + y 2 - 2r2 ( x + y ) + r2 2 = 0
2 2 + 12 2 2 + 12
The common chord is S1 - S2 = 0 CQ.CR =
(sec q - tan q) 2 (sec q + tan q) 2
Þ 2 ( r2 - r1 ) ( x + y ) + r12 - r2 2 = 0
= 22 + 12
Þ 2 ( x + y ) = r1 + r2 Þ CQ.CR=5
CHAPTER
12 Limits & Derivatives
1. (d) lim f ( x) = lim {- h}cot{-h} \ lim f ( x ) does not exist.
x ®0 - h ®0 x ®0
= lim (1 - h) cot(1 - h) = cot 1 x x
æ x 2 + 5x + 3 ö æ 4x +1 ö
h ®0 2. (a) lim ç 2 ÷ = xlim çè1+ 2 ÷
tan 2 {h} tan 2 h x ®¥ çè x + x + 2 ÷ø ®¥ x + x + 2ø
lim f ( x ) = lim = lim =1
x ®0+ h ®0 h 2 - [h]2 h®0 h2
Solutions 119
( 4 x +1) x
é x2 + x + 2 ù x2 + x + 2 1 e x - sin x
êæ 4 x + 1 ö 4 x+1 ú = 2 lim [using L¢ Hospital's rule]
x® 0 ( x - 2) x n - 3
= lim êç1 + 2 ÷ ú
x ®¥ ê è x + x + 2ø ú For this limit to be finite, n – 3 = 0
êë úû Þ n=3
1
4+ 6. (c) lim [(sin x)1/x + (1/x)sin x]
lim x x® 0
2
4x + x x ®¥ 1+ 1 + 2 sin x
lim
2
= e4 x x2 1/ x æ 1ö
= e x®¥ x + x + 2 = e = lim (sin x ) + lim ç ÷
3. (b) According to the question x®0 x®0 è x ø
æ x2 ö æ x3 ö æ 1ö
lim sin x log ç ÷
lim ç 3 – ÷ £ lim f ( x) £ lim ç 3 + ÷ è xø
x ®0 ç 12 ÷ø x®0 x ®0 ç 9 ÷ø =0+ e x®0
è è [Q | sin x | < 1 when x ® 0]
Þ (3 – 0) £ lim f ( x ) £ (3 + 0)
x ®0 - log x -1/ x
lim lim
x ® 0 cosec x x ® 0 - cosec x cot x
Hence lim f ( x) = 3 (from Sandwitch Theorem) =e =e
x®0 [Using L' Hospital rule]
4. (c) Let y sin x
lim . tan x
1 k k k 1/ n = e x ® 0 x = e0 = 1
= lim k ((n + 1) (n + 2) ...(n + n) )
n®¥ n
7. (c) Limit is of the form 1¥ , so
1
Þ ln y = lim px
tan
px
n®¥ n tan
æ aö 2a ì æ aö ü 2a
lim ç 2 - ÷ = lim í1 + ç1 - ÷ ý
æ æ n + 1 ök æn+2ö
k
æn+nö ö
k
x®a è xø x®a î è xø þ
ç ln ç ÷ + ln ç ÷ + ... + ln ç ÷ ÷
ç è n ø è n ø è n ø ÷ø æ
aö px æ x - aö px
è = lim ç1 - ÷ tan = lim ç tan
x®a è ø x 2a ex®a è x ø ÷
k æ æ n + 1ö æ n + 2ö æ n + nö ö e 2a
= lim ç ln ç ÷ + ln çè ÷ + ... + ln çè ÷
n ®¥ n è è n ø n ø n ø ÷ø Let x – a = h we get the limit
æ h ö p
n
æ ln (n + r ) - ln n ö = lim ç ÷ tan ( a + h)
= lim k .
n®¥
å çè
n
÷ø e h® 0 è a + h ø 2a
r =1 h æ p ph ö
= lim tan ç + ÷
æ
n
ln (n + r ) - ln n ö h®0 a + h è 2 2a ø
å
e
= k . lim çè ÷ø
n®¥ n h æ æ ph ö ö
r =1 = lim ç - cot ç ÷ ÷
kk h®0
e a+hè è 2a ø ø
æ æ 1ö ö æ 4ö æ 4ö
= k . ç 2 ç ln 2 - ÷ ÷ = ln ç ÷ . Þ y = ç ÷ . -h p 1
è è 2ø ø è eø è eø = lim . .
5. (c) Given that, h® 0
e æ ph ö 2 a p / 2 a
(a + h) tan ç ÷
x è 2a ø
(cos x - 1) (cos x - e )
lim = finite non-zero 2a 2
x®0 xn -2a ph / 2a - -
number = lim . = e pa = e p
eh®0 p(a + h) tan(ph / 2a)
(1 - cos x ) (1 + cos x )(e x - cos x ) 8. (a) Q ax 2 + bx + c = 0 has roots a and b
= lim n
x®0 x (1 + cos x ) a b
æ sin x ö æ e x - cos x ö æ
2 then 2 + + c = 0
1 ö x x
= lim ç 2 ÷ . ç ÷ . çè ÷
x®0 è x ø è x n - 2
ø 1 + cos x ø 1 1
i.e., cx 2 + bx + a = 0 has roots and .
a b
2 e x - cos x 1
= lim 1 . .
x ®0 xn - 2 2
120 MATHEMATICS

æ 1ö æ 1ö cos2 (sin2(sin2).........(sin2(x)).....) ( x + 4 + 2)
Þ cx 2 + bx + a = c ç x - ÷ x- ÷
è aø èç bø = lim .
p
x®0 ìï æ (x + 4) - 2ö üï
æ 1 - cos(cx 2 + bx + a ) ö sin ípç ÷ ý
Now lim îï è x ø ïþ
ç ÷

1 è 2(1 - ax ) 2 ø æ (x + 4) - 2ö
a
1 çp x ÷
ì 2 æ cx 2 + bx + a ö ü 2
è ø
ï sin ç ÷ï cos 2 0 (2 + 2) 4
ï è 2 øï = . =
= lim í 2 ý 1 p p
x ®1/ a ï (1 - ax ) ï cos2 x
ï ï 10. (b) lim éê11/ cos x + 12 / cos x + ..... + n1/ cos x ùú
2 2 2
î þ x ®p / 2 ë û
æ cx + bx + a ö
2
sin ç ÷ = tlim (1t + 2t + .......nt )1/ t
è 2 ø ®¥
= lim é 1 ù
x®1/ a (1 - ax) = t ³ 1ú
ê Putting 2
ë cos x û
1/ t
ìc æ 1ö æ 1ö ü t 1/ t
éæ 1 öt æ 2 öt ænö ù
t
sin í ç x - ÷ ç x - ÷ ý = lim ( n ) êç ÷ + ç ÷ + ...... + ç ÷ ú
î 2 è a ø è bø þï t ®¥ êëè n ø è n ø è n ø úû
= lim
x®1/ a æ 1ö 1/ t
-a ç x - ÷ éæ 1 öt æ 2 öt ænö ù
t
è aø
= n lim êç ÷ + ç ÷ + ...... + ç ÷ ú
æ c æ 1 ö æ 1ö ö t ®¥ êè n ø ènø è n ø úû
c æ 1ö ë
sin ç ç x - ÷ ç x - ÷ ÷ x-
è 2 è aø è bø ø 2 çè bø÷ = n (0 + 0 + ...... + 1)0 = n
= lim . lim ì log e n log e ( n + 1)
x®1/ a c æ 1 ö æ 1ö x®1/ a -a 11. (b) lim í ´
çx- ÷ x- n®¥ î log e (n - 1) log e n
2 è aø èç bø÷
log e (n + 2) log e n k ïü
c æ 1 1ö ´ ´ .... ´ ý
- log e ( n + 1) log e (n k - 1) ïþ
2 çè a bø÷ c æ 1 1ö
= 1. = -
2a èç a bø÷
ìï log e n k üï log e n
-a lim í ý = k lim
n®¥ î log
ï e ( n - 1) þï n ®¥ log e ( n - 1)

9. (b) Let P 1
Let n = , then
h
cos2 (1 - cos 2 (1 - cos 2 (......cos 2 ( x))......
= lim æ 1ö
x ®0 ìï æ ( x + 4) - 2 ö üï log e ç ÷
è hø - log e ( h)
sin í p ç ÷ø ý = k lim = k lim
è x h® 0 æ 1ö h® 0 log e (1 - h) - log e ( h )
ïî ïþ log e ç 1 - ÷
è hø
2 2 2 2
cos sin (1 - cos (......cos ( x))...... 1 1
= lim = k lim =k× =k
x ®0 ïì æ ( x + 4) - 2 ö ïü h®0 ì log e (1 - h) ü (1 - 0)
sin í p ç ÷ý í1 - ý
ïî è x ø þï î loge ( h) þ
cos (sin (sin (......(sin 2 ( x))......)
2 2 2 12. (b) Q lim 1 {[12 x] + [22 x] + [32 x] + ... + [ n2 x]}
= lim n®¥ n3
x®0 ïì æ x + 4 - 2 ö ïü
sin íp ç ÷ýì
x ø þï ï æ x + 4 - 2 ö üï ì n 2 ü æ n 2 ö
îï è ï å [r x ] ï ç å r x - {r x}÷
2
íp ÷ý
ìï æ x + 4 - 2 ö üï îï çè x ø þï í r =1
= lim ï
ý ç r =1 ÷
íp ç ý ï = lim ç ÷ø
x ÷ n ®¥ î n 3 þ n®¥ è n3
ïî è ø þï
Solutions 121
æ n (n + 1)(2 n + 1) ö 2
ç
x. n
{r 2 x}÷ æxö 2 x
= lim ç 6 -å 3 ÷ a ç ÷ tan
n3 2 x è 2ø 2
n®¥
çè r =1 n ÷ = lim ´ tan = a lim ´
ø x®0 g ( x ) 2 x ®0 g ( x )
æxö
2

(1)(1)(2) x ç ÷
= x. -0= è2ø
6 3
13. (c) lim f ( x) = lim f ( x ) x2 x2 4b
= a lim \ lim =
x ®3- x ®3+ x ®0 4 g ( x ) x ®0 g ( x ) a
( x +3) x
(27) 27 -9 1 - cos( x - 3) g (1 - cos 2 x) g (2sin 2 x)
Þ lim = lim l Now, lim 4
= lim
x ®3- 3x - 27 x ®3+ ( x - 3) 2
x ®0 x x®0 x4
æ x + 3 x -18 ö
2 g (2sin 2 x) (2sin 2 x) 2 a a
= lim ´ = ´4 = .
ç ÷ x ® 0 (2sin 2 x ) 2 x4
32 ç 3 9 - 1÷ 4b b
ç ÷ l
è ø= 16. (a) Let f ( x ) = lx 2 + mx + n
Þ lim
x ®3- 33 (3x -3 - 1) 2 Þ f '( x) = 2lx + m
2
1 x + 3x -18 l 1 l 2 Now , f (1) = f ( -1) Þ l + m + n = l - m + n
Þ lim = Þ ×9 = Þ l = .
x®3 3 9( x - 3) 2 27 2 3 Þ m=0
æ p ö \ f '(x) = 2lx
sec 2 ç ÷
æ æ p öö è 2-bx ø
14. (a) Here, lim ç sin 2 ç ÷÷ \ f '(a1) = 2la, f '(a2 ) = 2la2 , f '(a3 ) = 2la3
x ®0 è è 2 - ax ø ø
As a1, a2 , a3 are in A.P..
æ p ö
sec 2
ì æ p ö ü çè 2 -bx ÷ø \ f '(a1 ), f '(a2 ), f '(a3 ) are in A. P..
= lim í1 - cos 2 ç ÷ý
x ®0 î è 2 - ax ø þ 17. (b) Since, f(x) is a polynomial function
ì ü
satisfying
ïï 2 æ p ö 1 ïï æ1ö æ1ö
lim - ícos ç ÷× ý f ( x) × f ç ÷ = f ( x ) + f ç ÷,
x®0 ï è 2 - ax ø cos 2 æ p ö ï
ç ÷ è xø è xø
=e îï è 2 -bx ø þï
ì æ p ö æ p ö pa ü
\ f ( x) = x n + 1 or f ( x ) = - x n + 1
ï 2sin ç 2 - ax ÷ cos ç 2- ax ÷´ ï
ï è ø è ø (2- ax )2 ï If f ( x) = - x n + 1, then f (4) = -4n + 1 ¹ 65
lim - í ý
x ®0 ï æ p ö æ p ö pb ï
2sin ç ÷ cosç ÷´
ï
î è 2 -bx ø è 2 -bx ø (2 -bx ) ïþ 2 So, f ( x ) = x n + 1 Since, f(4) = 65
=e
[using L'Hospital's rule] \ 4 n + 1 = 65
æ 2p ö
sin ç ÷
è 2 - ax ø × a × (2 - bx )
2 Þ n = 3 \ f ( x) = x3 + 1 Þ f '( x) = 3 x 2
- lim
x ®0 æ 2 p ö b (2 - ax )2
sin ç ÷
2 - bx ø
\ f '(l1 ) = 3l12 , f '(l2 ) = 3l22 , f '(l3 ) = 3l32
=e è
Since, l1, l2, l3 are in GP.
a (2 -bx )3 a \ f '(l1 ), f '(l2 ), f '(l3 ) are also in GP..
- lim × -
x ®0 b (2 - ax )3
=e =e b.
g ( x) f (a) - g (a) f ( x)
f (1 - cos x) 18. (5) lim
15. (c) lim x®a x-a
x ®0 g ( x ) sin 2 x g (a + h) f (a) - g ( a) f (a + h)
= lim
æ xö æ 2 xö h®0 h
f ç 2 sin 2 ÷ ç 2sin ÷ [For x = a + h]
è 2ø è 2ø
= lim ´
x®0 æ x ö æ x öæ xö g (a + h) f (a) - g (a) f (a) + g (a) f (a) - g (a) f (a + h)
g ( x ) ç 2 sin 2 ÷ 4 ç sin 2 ÷ç cos 2 ÷ = lim
h®0 h
è 2ø è 2 øè 2ø
122 MATHEMATICS

é g (a + h ) - g ( a ) ù é f (a + h) - f (a) ù 1- x
ú - hlim ïì - ax + sin ( x - 1) + a ïü1- x 1
lim f (a ) ê g (a ) ê ú
= h ®0 ë h û ®0 ë h û lim í =
22. (2) ý
= f (a) g' (a) – g (a) f ' (a) = 2 × 2 – (– 1) × 1 = 5 î x + sin ( x - 1) - 1 ïþ
x ®1 ï 4
lim ( tan x (sin x -1)) 1+ x
x® p ìï a (1 - x ) + sin ( x - 1) üï
tan x Þ lim í ý
19. (1) lim (sin x) =e 2

p x ®1 ïî ( x - 1) + sin ( x - 1) ïþ
2 1+ x
ì sin( x - 1) ü
sin 2 x - sin x æ0 ö ïï - a + ï
= lim çè form÷ø Þ lim í x -1 ï
x®p cos x 0 ý
e 2 x ®1 ï sin ( x - 1) ï
1+
lim
2sin x cos x -cos x ïî x - 1 ïþ
x® p - sin x 2
= e 2 = e0 =1 æ -a + 1 ö 1
20. (0.50) Required limit, Þç ÷ = Þ a = 0 or 2
è 2 ø 4

l = lim
(x + x+ x -x ) \ Largest value of a is 2.
23. (1) Let
x ®¥ x x
x+ x+ x + x y= =
3
x 1 x
x+ x + 2/3
Dividing numerator & denominator by x , we 3 x x + 3
x ....¥
get x + x ....¥
x 5/ 3 x
x = =
1+ x 5/ 3
+y y
x x+
l = lim x2 /3
x ®¥
x+ x or y 2 + ( x 5/ 3 ) y - x 5/ 3 =0
1+ 2
+1
x - x5 / 3 ± x10 / 3 + 4 x5 / 3
\y=
1 2
1+
x 1 1 - x 5 / 3 + x10 / 3 + 4 x5 / 3
= lim = = = 0.50 = (Q y > 0)
x ®¥ 1 1 1+1 2 2
1+ + 3/2 +1
x x 4 x5 /3
=
x (1 + a cos x ) - b sin x 2( ( x10/ 3 + 4 x 5/ 3 ) + x5 / 3 )
21. (6) lim =1
x®0 { f ( x )}3 =
2
ìï x 2 x 4 üï ìï x3 x5 üï æ 4 ö
x + ax í1 - + - ......ý - b í x - + - ......ý ç1 + 5/ 3 ÷ + 1
ïî 2! 4! þï îï 3! 5! þï = 1 è x ø
Þ lim
x ®0 { f ( x)} 3 2 2
\ lim y = = = 1.
1+ a - b æ a b ö æ a bö x ®¥ 1+ 0 + 1 2
+ ç - + ÷ + x 2 ç - ÷ + .......
x2 è 2! 3!ø è 4! 5!ø 24. (1) a( x3 - 1) + ( x - 1) = 0
Þ lim =1
3
x ®0 ì f ( x) ü or ( x - 1)(ax 2 + ax + a + 1) = 0
í ý
î x þ a, b ¹ 1 so, a, b are roots of
a b ax 2 + ax + a + 1 = 0
Þ 1 + a - b = 0 and - + =1
2! 3! a +1
a + b = -1, ab =
5 3 a
Þa=- and b = - . Thus b – 3a = 6
2 2 (1 + a) x3 - x2 - a ( x3 - x2 ) + a( x3 -1)
lim = lim

1 (e1-ax -1)( x -1) x®
1 (e1-ax -1)( x -1)
a a
Solutions 123

[x2 + a(x2 + x +1)] (1 + a)x2 + ax + a 25. (36) Let 3x = t2


= lim = lim

1 (e1-ax -1) x® æ e
1 1-ax
-1 ö t2 +
27
- 12
a aç
ç 1 - ax ÷÷ (1 - ax) t2
è ø lim t 4 - 12t 2 + 27
t ®3 1 3 = lim
éæ1+ a ö 2 ù - t ®3 t -3
êç a ÷ x + (1)x +1ú 2 t t2
= lim a ë
è ø û = lim a (abx -(a+b)x +1)

1 (1- ax) x®
1 (1-ax) (t 2 - 3)(t + 3)(t - 3)
a a = lim
t ®3 t -3
(1 - (a ) x )(1 - (b) x) a (a - b) = (3 – 3) (3 + 3) = 36.
2
= lim a = .

1 (1 - ax ) a
a

CHAPTER
13 Mathematical Reasoning
1. (a) Inclusive “or”. 17 is a real number or a
positive integer or both. 9. (a) p q p Ù q (p Ù q)Þ p
2. (c) p ® (~ p Ú q) has truth value F. T T T T
It means p ® (~ p Ú q) is false. T F F T
It means p is true and ~ p Ú q is false.
Þ p is true and both ~ p and q are false. F T F T
Þ p is true and q is false. F F F T
3. (b) ~p : Ashok does not work hard
\ ( p Ù q) Þ p is a tautology..
Use '®' symbol for then
(~p ® q) mean = If Ashok does not work hard 10. (b)
then he gets good grade. pÞq ~q Þ ~p p Þ q Û ~q Þ ~p
4. (d) When p is false and q is true, then p Ù q is
T T T
false, pÚ ~ q is false.
F F T
(Q both p and ~q are false)
T T T
and q Þ p is also false,
only p Þ q is true. T T T

5. (d) ~ p Ù q = ~ (q ® p) 11. (b) The truth value of ~(~p) « p as follow


6. (a)
p ~p ~(~p ) ~(~p) ®p p ®~(~p ) ~(~p )«p
p q p Ù q p Ú q ~(p Ú q) (p Ù q) Ù ~ (p Ú q) T F T T T T
T T T T F F F T F T T T
T F F T F F
Since last column of above truth table contains
F T F T F F only T.
F F F F T F Hence ~ (~p) ® p is a tautology.
12. (c)
\ (p Ù q) Ù (~ (p Ú q)) is a contradiction.
p q p®q ~p ~pÚq (p®q)«~(pÚq)
7. (b) (p Ù ~ q) Ù (~ p Ù q) = (p Ù ~ q) Ù (~ q Ù q)
T T T F T T
= f Ùf = f
T F F F F T
(By using associative laws and commutative laws)
F T T T T T
\ (pÙ ~ q) Ù (~ p Ù q) is a contradiction.
F F T T T T
8. (b) p Þ q is logically equivalent to ~ q Þ ~ p
\ (p Þ q) Û (~ q Þ ~ p) is a tautology but
not a contradiction.
124 MATHEMATICS

13. (d) p Þ (~ p Ú q) is false means p is true and 18. (c) The inverse of the proposition
~ p Ú q is false. (p Ù ~ q) ® r is ~ (p Ù ~ q) ® ~ r
Þ p is true and both ~p and q are false. Þ p is true º ~ p Ú ~ (~q) ® ~ r º ~ p Ú q ® ~ r
and q is false 19. (a) ~ ((~ p) Ù q) º ~ (~ p) Ú ~ q º p Ú (~ q)
14. (c) Let p : 2 + 3 = 5, q : 8 < 10 20. (c) ~ ( p Þ q) º p Ù ~ q
Given proposition is : p Ù q . \ ~ (~ p Þ ~ q) º ~ p Ù ~ (~ q) º ~ p Ù q .
Its negation is ~ ( p Ù q) = ~ p Ú ~ q Thus ~ (~ p Þ ~ q) º ~ p Ù q
\ we have 2 + 3 ¹ 5 or 8 </ 10. 21. (c) ~ [ (p Ú q) Ù (q Ú ~ r)]
º ~ ( p Ú q) Ú ~ (q Ú ~ r)
15. (b) We know that p ® q is false only when p is º (~ p Ù ~ q) Ú (~ q Ù r)
true and q is false. 22. (c) Statement given in option (c) is correct.
So p ® (~ p Ú q) is false only when p is true and ~ [p Ú (~ q) ] = (~ p) Ù ~ (~ q) = (~ p) Ù q.
(~ p Ú q) is false. 23. (d) Since ~ (p Ú q) º ~ pÙ ~ q
But (~ p Ú q) is false if q is false because ~ p is
false. (By De-Morgans’ law)
Hence p ® (~ p Ú q) is false when truth value of \ ~ (p Ú q) ¹ ~ p Ú ~ q
p and q are T and F respectively. \ (d) is the false statement.
16. (a) We know that the contrapositive of p ® q 24. (d) We know that ~ (p ® q) º p Ù ~q
is ~ q ® ~ p. \ ~((p Ù r) ® (r Ú q)) º (p Ù r) Ù [~(r Ú q)]
So contrapositive of p ® (~q ® ~r) is º (p Ù r) Ù (~r Ù ~q)
~ (~q ® ~r) ® ~p º ~ q Ù [~ (~r)] ~p 25. (d) Let P = A Í B, Q = B Í D, R = A Í C
Q ~ (p ® q) º p Ù ~q º ~ q Ù r ® ~p
Contrapositive of (P Ù Q) ® R is (® ~ R ® ~
17. (a) ~ [ p Ú (~ p Ú q)] º ~ p Ù ~ (~ p Ú q)
(P Ù Q)).
º ~ p Ù (~ (~ p)Ù ~ q) º ~ p Ù ( p Ù ~ q ) . ~R®~ PÚ~Q

CHAPTER
14 Statistics
n1 (400) + n 2 (480)
\ 430 = Þ 30n1= 50n2
1. n C1 + 2. n C2 + 3. n C3 + .... + n. nCn n1 + n 2
1. (a) X =
n
C1 + nC2 + .... + n Cn n1 5
Þ =
n2 3
n n
å r. nCr n å n -1 Cr -1
n.2n -1
4. (c) Sum of 6 numbers = 30 × 6 = 180
Sum of remaining 5 numbers = 29 × 5 = 145
X= r =1 = r =1 = n .
n n \ Excluded number = 180 – 145 = 35.
2 -1
å n
Cr å n
Cr 5. (c) Let the items be a1, a2, ........, an.
r =1 r =1 a + a + ........ + a n
then X = 1 2 .
2. (a) Let the mean of the remaining 4 n
observations be x1 . Now, according to the given condition:
(a + 1) + (a 2 + 2) + ........ + (a n + n)
a + 4 x1 nM - a X= 1
Then, M = Þ x1 = . n
(n - 4) + 4 4 1 + 2 + 3 + ........ + n n(n + 1)
=X+ =X+
n1 x1 + n 2 x 2 n 2n
3. (a) x= (using sum of n natural nos.)
n1 + n 2
n +1
Q x1 = 400, x 2 = 480, x = 430 =X+ .
2
Solutions 125
6. (a) Median is given as
N 10. (a) We know that Q.D
-F
M = l+ 2 ´C 5 5
f = ´ M .D. = ´12 = 10
where 6 6
l = lower limit of the median - class 3 3
f = frequency of the median class \ S.D = ´ Q.D. = ´ 10 Þ S .D. = 15.
N = total frequency 2 2
F = cumulative frequency of the class just 11. (d) Since S.D. £ Range = b – a
before the median class \ Var ( x ) £ (b – a )2 or (b - a)2 ³ Var ( x) .
C = length of median class.
Now, given, M = 25, N = 100, F = 45, 12. (d) Since 0 < y < x < 2y
C = 20 – 30 = 10, l = 20. x x
\ By using formula, we have \ y> Þ x- y <
2 2
50 – 45 \ x – y < y < x < 2x + y.
25 = 20 + ´ 10
f y+x
Hence median = = 10
50 50 2
25 – 20 = Þ5= Þ f = 10. Þ x + y = 20. ...(i)
f f And range = (2x + y) – (x – y) = x + 2y.
101 + d (1 + 2 + 3 + ......+100) But range = 28 \ x + 2y = 28 ...(ii)
7. (b) Mean = From equations (i) and (ii), x = 12, y = 8
101
d × 100 × 101 ( x - y ) + y + x + (2 x + y ) 4 x + y
=1+ = 1 + 50 d \ Mean = =
101 × 2 4 4
Q Mean deviation from the mean = 255 y 8
= x + = 12 + = 14.
1 4 4
Þ [|1 - (1 + 50d ) | + | (1 + d ) - (1 + 50d ) | + 13. (b) Let xi be n observations, i = 1, 2, ...n
101
| (1 + 2d )| Let X be the mean and M.D be the mean
-(1 + 50d ) | +....+ | (1 + 100d ) - (1 + 50 d ) |] = 255 deviation about X .
If each observation is increased by 5 then new
Þ 2d [1 + 2 + 3 + ... + 50] = 101´ 255
mean will be X + 5 and new M.D. about new
50 ´ 51
Þ 2d ´ = 101´ 255 æ n
xi ö
2 mean will be M.D. çQ Mean = å nø
÷
101´ 255 25755 è i =1
Þ d= = = 10.1
50 ´ 51 2550 14. (a) The first n natural numbers are 1, 2, 3,
................n
8. (c) Clearly mean A = 0.
1 + 2 + 3 + 4 + ... + n
Their mean, x =
Standard deviation s = å ( x - A)2 n
2n n(n + 1) n + 1
= = .
2n 2
( a - 0) + ( a - 0) + ...(0 - a) 2 + ...
2 2
2= [Q The sum of Ist n natural numbers is n (n + 1) ]
2n 2
a 2 .2n Now, Variance = s 2 =
å i ( x - x ) 2
= = | a | . Hence | a | = 2.
2n n

9. (c) Using, = å ( xi - 2 xx i + x )
2 2 ù
në û
n1 (s12 + d12 ) + n 2 (s22 + d 22 ) 22
s=
n1 + n 2
=
3
.
=
å xi2 - 2 x å xi + x 2 .n
n n n
126 MATHEMATICS

2 1
=
å xi2 - 2 x 2 + x 2 = å xi2 - æ å xi ö =
20
´ 400 = 20 = 22 ´ 5 .
ç ÷
n n è n ø 16. (d) If initially all marks were xi then
[Since frequency of each variate is one]
å ( xi - x )2
n(n + 1)(2n + 1) si2 = i .
Q å xi2 = .
6 N
2 Now each is increased by 10
n(n + 1)(2n + 1) æ (n + 1) ö
\ Variance = -ç
è 2 ÷ø
2
6n å ëé( xi +10)-( x +10)ûù å ( xi - x )2
i i
si2 = = = si2
(n + 1)(2n + 1) (n + 1)2 N N
= -
6 4 Hence, variance will not change even after the
æ 2n + 1 n + 1ö (n + 1)(n - 1) grace marks were given.
= (n + 1) ç - ÷ = 17. (a) C.V. (1st distribution) = 60, s1 = 21
è 6 4 ø 12 C.V. (2nd distribution) = 70, s2 = 16
n2 - 1 Let x1 and x2 be the means of 1st and 2nd
= .
12 distribution, respectively, Then
15. (b) Let the observations be x1, x2, ...., x20 and x s
C.V. (1st distribution) = 1 ´ 100
be their mean. Given that, variance = 5 and n = 20. x1
We know that, 21 21
\ 60 = ´100 or x1 = ´ 100 = 35
1 20
( ) x1 60
å ( xi - x )
2
Variance s 2 = s2
n i=1 and C.V. (2nd distribution) = ´100
x2
1 20
i.e. 5 = å
20 i =1
( x i - x )2 16
´ 100 or, x =
16
i.e., 70 = 2 ´ 100 = 22.85
20 x2 70
å ( xi - x )
2
or = 100 ...(i) 18. (93.32) When each observation is multiplied
i =1 by 2, then variance is also multiplied by 2.
If each observation is multiplied by 2 and the We are given, 2, 4, 5, 6, 8, 17.
new resulting observations are yi, then When each observation multiplied by 2, we get
1 4, 8, 10, 12, 16, 34.
yi = 2xi i.e., xi = yi . \ Variance of new series = 22 × Variance of given
2
data = 4 × 23.33 = 93.32.
1 20 1 20 19. (0) We know that,
Therefore, y = å
n i =1
yi = å 2x i
20 i=1
s
Coefficient of variation = ´ 100
x
1 20 s1
= 2. å xi
20 i =1 \ CV of 1st distribution = ´ 100
30
s1
1 Þ 50 = ´ 100 [CV of 1st distribution = 50
i.e., y = 2x or x = y. 30
2 (given)]
On substituting the values of xi and x in eq. (i), Þ s1 = 15
we get
s2
20 2 Also, CV of 2nd distribution = ´ 100
æ1 1 ö 25
å çè 2 yi - 2 y ÷ø = 100 s2
i =1
20 Þ 60 = ´100
25
å ( yi - y )
2
i.e., = 400 .
60 ´ 25
i =1 Þ s2 = Þ s2 = 15
Thus, the variance of new observations 100
Thus, s1 – s2 = 15 – 15 = 0.
Solutions 127

20. (78) å x = 170, å x 2 = 2830 increase in 1


i.e., 8.24 = [(3.4)2 + (2.4)2 + (1.6)2 + x2 + y2
5
å x = 10 , then å x¢ = 170 + 10 = 180 – 2 × 4.4 (x + y) + 2 × (4.4)2 ]
Increase in å x 2 = 900 - 400 = 500 , then or 41.20 = 11.56 + 5.76 + 2.56 + x2 + y2
– 8.8 ×13 + 38.72.
å x¢2 = 2830 + 500 = 3330 Therefore, x2 + y2 = 97 … (ii)
2 But from eq. (i), we have
1 æ1 ö
Variance =
n
å x '2 - ç å x ' ÷
èn ø
x2 + y2 + 2xy = 169
From eqs. (ii) and (iii), we have
… (iii)
2 2xy = 7 2 … (iv)
1 æ1 ö
= ´ 3330 - ç ´ 180 ÷ = 222 - 144 = 78. On subtracting eq, (iv) from eq. (ii), we get
15 è 15 ø
x2 + y2 – 2xy = 97 – 72
21. (8.25) i.e. (x – y)2 = 25 or x – y = ± 5 … (v)
2
11 ×12.23 æ 11.12 ö So, from eqs. (i) and (v), we get
-1 ç -1 ÷ x = 9, y = 4 when x – y = 5
2
s = 6 -ç 2 ÷ = 8.25 or x = 4, y = 9 when x – y = –5.
10 è 10 ø Thus, the remaining observations are 4 and 9.
22. (5) We have, Required difference = 5.
CV of 1st distribution (CV1) = 50. 24. (18) Var (1, 2, ....., n) = 10
CV of 2nd distribution (CV2) = 60
2
s1 = 10 and s2 = 15. 12 + 2 2 + ..... + n2 æ 1 + 2 + ..... + n ö
Þ -ç ÷ = 10
s n è n ø
We know that, CV = ´ 100
x 2
(n + 1)(2n + 1) æ n + 1 ö
s 10 Þ -ç ÷ = 10
\ CV1 = 1 ´ 100 Þ 50 = ´ 100 6 è 2 ø
x1 x1
Þ n2 – 1 = 120
10 ´ 100 Þ n = 11
Þ x1 = = 20 .
50 Var (2, 4, 6, ....., 2m) = 16 Þ Var (1, 2, ....., m) = 4
s Þ m2 – 1 = 48 Þ m = 7
Also, CV2 = 2 ´ 100
x2 Þ m + n =18
15 ´100 25. (52) Mean
Þ 60 =
x2 3 + 7 + 9 + 12 + 13 + 20 + x + y
=x = = 10
15 ´100 8
Þ x2 = Þ x 2 = 25. Þ x + y = 16 ...(i)
60
Thus, x 2 - x1 = 25 - 20 = 5. S ( xi )2
23. (5) Let the other two observations be x and y. Variance = s2 = - ( x )2 = 25
8
Therefore, the series is 1, 2, 6, x, y.
9 + 49 + 81+144 +169 + 400 + x 2 + y 2
1+ 2 + 6 + x + y s2 = - 100 = 25
Now, mean ( x ) = 4.4 = 8
5
or 22 = 9 + x + y. Þ x2 + y2 = 148 ...(ii)
Therefore, x + y = 13 … (i) From eqn. (i), (x + y)2 = (16)2
5 Þ x2 + y2 + 2xy = 256
1
2
Also, variance (s ) = 8.24 =
n å ( xi - x )2 Using eqn. (ii), 148 + 2xy = 256
i =1 Þ xy = 52
128 MATHEMATICS

CHAPTER
15 Probability-1
1. (d) Total number of outcomes So, for each ai Î A , there are four possibilities.
S = {(1, 1), (1, 2), (1, 3), (1, 4), (2, 1), (2, 2), (2, 3),
\ Total no. of cases = 4 × 4 × ............ × 4 (n times) = 4n
(2, 4), (3, 1), (3, 2), (3, 3), (3, 4), (4, 1), (4, 2), (4, 3), (4, 4)}
Further out of above four possibilities, first three
n(S) = 16
satisfy ai Î P È Q
Number of favourable outcomes
So, the number of cases, when exactly r element
E = {(2, 1), (3, 1), (3, 2), (4, 1), (4, 2), (4, 3), (4, 4)}
of A belong to P È Q = n Cr (3)r
n(E) = 7
n
n(E) 7 Cr (3)r
Required probability = = \ Required probability =
n(S) 16 4n
2. (b) Given equation 5. (a) Since the chairs are numbered, so for
counting of total number of cases it is equivalent
100
x+ > 50 Þ x 2 - 50x + 100 > 0 to linear permutation. Hence, total number of
x cases = 10!
Þ ( x - 25) 2 > 525 If two particular persons A and B sit together
then the total number of linear arrangements =2!
Þ x - 25 < - (525) or x - 25 > (525) 9!. Consider one of such arrangements in which
the arrangement started at chair 1 (C1) and ends
Þ x < 25 - (525) or x > 25 + (525)
at chair 10 (C10).
As x is positive integer and (525) = 22.91 , we C1 – C2 –C3 – ......... –C9 –C10
If two persons sit at C1 and C10 then it will lead
must have x £ 2 or x ³ 48 to 2! 8! new arrangements. So the favourable
Let E be the event for favourable cases and S be number of cases = 2! 9! + 2! 8! = 2!8! (10)
the sample space.
2!8!(10) 2
\ E = {1, 2, 48, 49, ......100} \ Probability = =
10! 9
\ n(E ) = 55 and n(S) = 100 6. (b) Total ways of distribution = 45 Þ n(S) = 45
Hence the required probability Total ways of distribution so that each child get
n( E) 55 11 atleast one game = 45 - 4 C1 35 + 4C2 25 - 4 C3
P(E) = = = .
n( S ) 100 20 = 1024 - 4 ´ 243 + 6 ´ 32 - 4 = 240
3. (b) Total number of combinations of numbers n(E) = 240
on the cube and the tetrahedron = 6 × 4 = 24 n( E ) 240 15
Favourable number of ways of getting a sum required probablity = = = .
not less than 5 n(S ) 45 64
= sum of coefficients of x6, x7, .... x10 in the product 7. (d) A chess board is a square divided into 64
2 3 4 5 6 equal squares.
= (x + x + x + x + x + x ) In 1st diagonal we have only 1 square
( x + x 2 + x3 + x 4 ) In 2nd diagonal we have only 2 squares
= ( x 2 + 2 x 3 + 3 x 4 + 4 x5 + 4 x 6 In 3rd diagonal we have 3 squares so selection
can be done in 3C3 ways
+ 4 x7 + 3x8 + 2 x9 + x10 ) In 4 diagonal we have 4 squares so selection
= 4 + 4 + 4 + 3 + 2 + 1 = 18 can be done in C34 ways
18 3 And so on
\ Required probability = = Hence the total number of ways in which 3
6´ 4 4
4. (b) Let A = {a1, a2, a3, ........... an}. For any squares can be chosen
ai Î A , 1 £ i £ n , we have following choices 2(3C3 + 4C3 + 5C3 + 6C3 + 6C3 + 7C3) + 8C3
(1) ai Î P and ai Î Q (2) ai Î P and ai Ï Q [Note that we do not have 2 × 8C3]
(3) ai Ï P and ai Î Q (4) ai Ï P and ai Ï Q Hence the total number of favourable ways m
Solutions 129

= 4(3C3 + 4C3 + 5C3 + 6C3 + 7C3) + 2 × 8C3 = 392. 170


64 × 63 × 62 64 \ Required probability = 1 - 22
.
And total number of ways = C3 C5
1× 2 × 3
log b
11. (b) log a b =
= 32 × 21 × 64 log a
Hence the required probability Let b = 2m and a = 2n where m and n denote the
m 392 7 exponents on the base 2 in the given set then
= = =
n 32 × 21 × 62 744 m
8. (c) The total numbers of arrangements is log a b =
n
11! 11! Therefore, logab is an integer only if n divides m.
=
2!2!2! 8 Now, total no. of ways m and n can be chosen
The number of arrangements in which C, E, H, = 25 × 24 = 600.
6! 11! For favourable cases
I, S appear in that order 11 C5 = Let n = 1, So m can take values 1, 2, 3, 4,
2!2!2! 8 × 5!
11! 5, 6, ..., 24 = 24
1 if n = 2 m = 4, 6, 8, 10, 12, 14, 16, 18,
\ Required Probability = 8 × 5! = 20, 22, 24 = 11
11! 120
n = 3m = 6, 9, 12, 15, 18, 21, 24 =7
8 n = 4m = 8, 12, 16, 20, 24 =5
9. (c) Player A can win if A throws (1, 6) or (6, 1) n = 5m = 10, 15, 20, 25 =4
and B throws ((1, 1), (2, 2), (3, 3), (4, 4), (5, 5) or n = 6m = 12, 18, 24 =3
(6, 6)). Thus the number of ways is 12. n = 7m = 14, 21 =2
Similarly the number of ways in which B can win n = 8m = 16, 24 =2
is 12. n = 9, 10, 11, 12 m = 1 for each =4
Total number of ways in which either A wins or B 62
wins = 24.
Thus the number of ways in which none of the Hence, required probability = 62 = 31 .
two wins = 64 – 24. 600 300

6 4 - 24 53 12. (d)
\ The required probability = 4
= .
6 54
10. (c) Value 50p. 25p. 10p. n(S) = Number of total ways
No. 2 5 15
14!
5 coins out of 22 can be chosen in 22C2 ways. = 14 P12 = = 7 ´ 13!
Let x be the no. of 50 P coins selected. y be the 2!
no. of 25p. coins selected and z be the no. of 10 The girls can be seated together in the back seats
p. coins selected. We desire that 50x + 25y + 10z leaving a corner seat in 4 × 3! = 24 ways and the
< 150 or 10x + 5y + 2z < 30 where 0 £ x £ 2; or boys can be seated in the remaining 11 seats in
11 11! 1
0 £ y £ 5, 0 £ z £ 14 and x + y + z = 5. Thus P9 = = ´ 11! ways
following cases are impossible. 2! 2
\ n(E) = Number of favourable ways
Case (i) : x = 2, y = 2, z = 1
No. of ways of such selections 1
= 24 ´ ´11! = 12!
= 2C2 ´ 5C2 ´ 15C1 = 1´10 ´15 = 150. 2
n( E ) 12! 1
Case (ii) : x = 1, y = 4, z = 0 The required probability = = =
n(S ) 7 ´13! 91
No. of ways of such selections = 2C1 ´ 5C4 = 10.
13. (a) We know that P (exactly one of A or B occurs)
Case (iii) : x = 2, y = 3, z = 0. No. of ways of such = P (A) + P (B) – 2P (A Ç B).
selection = 2C2 ´ 5C3 ´ 15C0 = 1´10 = 10. Therefore, P (A) + P (B) – 2P (A Ç B) = p … (i)
Similarly, P (B) + P (C) – 2P (B Ç C) = p … (ii)
\ Total = 150 + 10 + 10 = 170 ways are to be rejected.
and P (C) + P (A) – 2P (C Ç A) = p …(iii)
130 MATHEMATICS
Adding (i), (ii) and (iii) we get For maximum value of P(B) both P(A È B) and
2 [P (A) + P (B) + P (C) – P (A Ç B) P(A Ç B) should be maximum, maximum value of
– P (B Ç C) – P (C Ç A)] = 3p P(A È B) is 1 and maximum value of P(A Ç B)
Þ P (A) + P (B) + P (C) – P (A Ç B) = is 0.3.
– P (B Ç C) – P (C Ç A) = 3p/2 … (iv) Hence P(B) = 1 + 0.3 – 0.5 = 0.8.
We are also given that, Similarly For minimum value of P(B) both P(A È B)
P (A Ç B Ç C) = p2 … (v) and P(A ÇB) should be minimum, minimum value
Now, P (at least one of A, B and C) of P(AÈB) = P(A) = 0.5 and minimum value of
= P (A) + P (B) + P (C) – P (A Ç B) – P (B Ç C) P(A Ç B) = is 0.1.
– P (C Ç A) + P (A Ç B Ç C) Hence P(B) = 0.5 + 0.1 – 0.5 = 0.1.
So required range is (0.1, 0.8).
3p 3 p + 2 p2
= + p2 = [using (iv) and (v)] 17. (a) Total number of outcomes = 52C4 = 270725
2 2 E = Event of exactly two spade cards and exists
14. (d) Since, one and only one of the three events two aces.
E1, E2 and E3 can happen, therefore P (E1) + P A = Event of 1 spade ace, 1 non-spade ace and 1
(E2) + P (E3) = 1 .....(i) spade card and 1 non-spade card
Q Odds against E1 are 7 : 4 B = Event of 2 non-spade aces and 2 spade cards.
4 4 1( 3C1 )( 12 C1 )( 36C1 ) 3 ´ 12 ´ 36 1296
Þ P(E1 ) = = .....(ii) P ( A) = =
4 + 7 11 52
C4 270725 270725
Q Odds against E2 are 5 : 3 3
C2 ´ 12 C2 3 ´ 66 198
3 3 P( B) = = =
Þ P (E 2 ) = = ......(iii) 52
C4 270725 270725
3+5 8
From (i), (ii) and (iii), we have, Now, E = A È B and A Ç B = f. This implies
4 3 1494
+ + P(E 3 ) = 1. P ( E ) = P ( A È B) = P ( A) + P ( B ) = .
11 8 270725
4 3 88 - 32 - 33 18. (0.80) Total no. of arrangements of the letters
Þ P(E3 ) = 1 - - =
11 8 88 10!
of the word UNIVERSITY is .
23 23 2!
= = No. of arrangements when both I's are together
88 23 + 65
Hence odds against E3 are 65 : 23. = 9!
So. the no. of ways in which 2 I’s do not together
15. (a) Let p1, p2 be the chances of happening of
the first and second event, respectively; then, 10!
= - 9!
according to the given conditions, we have 2!
3 10!
1 - p1 æ 1 - p2 ö - 9!
p1 = p22 and =ç ÷ 2! 10!- 9! 2!
p1 è p2 ø \ Required probability = =
10! 10!
3 2!
1 - p22 æ 1 - p2 ö 2
Þ =ç ÷ Þ p2 (1 + p2 ) = (1 - p2 ) 10 ´ 9!- 9!.2! 9![10 - 2] 8 4
p22 p
è 2 ø = = = = = 0.80
10! 10 ´ 9! 10 5
1 1 19. (0.50) Total number of numbers = 4! = 24
Þ 3 p2 = 1 Þ p2 = and so p1 = .
3 9 For odd nos. 1 or 3 has to be at unit's place
16. (c) Let P(A) = probability that Ashmit will pass, If 1 is at unit place, then total number of numbers
and P(B) = probability that Bishmit will pass, = 3! = 6
then from the given information And if 3 is at unit place, then total number of
P(A) = 0.5, 0.1 £ P(A Ç B) £ 0.3. numbers = 3! = 6
We have to find the value of P(B). \ Total number of odd number = 6 + 6 = 12
Since P(A È B) = P(A) + P(B) – P(A Ç B) 12 1
Or P(B) = P(A È B) + P(A Ç B) – P(A) \ Required probability = = = 0.50
24 2
Solutions 131

20. (0.33) Total number of cases = 3n


C2 Þ ab(ab - 1) = 0 Þ ab = 1 or ab = 0

3n(3n - 1) Þ a = 1, b = 1; a = w, b = w2
= [Cube roots and unity]
2
a = 1, b = 0, a = 0, b = 0
Now x3 + y3 = ( x + y ) ( x 2 - xy + y 2 )
\ n( S ) = Number of quadratic equations which
x3 + y3 is divisible by 3 if 3 divdes x + y or divides
are unchanged by squaring their roots = 4
x2 – xy + y2 we arrange the 3n numbers in 3
and n(E) = Number of quadratic equations have
sequences.
equal roots = 2.
A : {1, 4, 7, ............., 3n – 2}
B : {2, 5, 8, .............., 3n – 1} n( E ) 2 1 p
\ Required probability = = = = .
C : {3, 6, 9, ............., 3n} n(S ) 4 2 q
Clearly we must choose either one number from Þ p + q = 3.
the first sequence and other number from the 23. (4) The number of ways to answer a question
second sequence or both numbers from the third = 25 – 1 = 31.
sequence only. i.e. In 31 ways only one correct.
\ Number of favourable cases Let number of choices = n
n (n - 1) n n 1
= n × n + n C2 = n 2 + = (3n - 1) Now, according to the question >
2 2 31 8
n 31
(3n - 1) Þn> Þ n > 3.8 \ Least value of n = 4.
2 1 8
\ Required probability = = = 0.33
3n(3n - 1) 3 24. (5) Let S be the sample space and E be the event
2 of getting a large number than the previous number.
21. (3) Total no. of arrangements = 15! \ n( S ) = 6 ´ 6 ´ 6 = 216
Extreme chairs are occupied by girls, thus there Now, we count the number of favourable ways.
are four gaps among 5 girls where boys can be Obviously, the second number has to be greater
seated. Let the number of boys in these four gaps than 1. If the second number is i(i > 1), then the
be 2x + 1, 2y + 1, 2z + 1 and 2t + 1, then number of favourable ways = (i – 1) × (6 – i)
2x + 1 + 2y + 1 + 2z + 1 + 2t + 1 = 10 \ n(E) = Total number of favourable ways
Þ x+y+z+t=3 6
Where x, y, z, t are integers and = å (i - 1) ´ (6 - i )
i =1
0 £ x £ 3, 0 £ y £ 3, 0 £ z £ 3, 0 £ t £ 3 =0+1× 4+2×3+3×2+4×1+5×0
\ The number of ways of selecting positions = 4 + 6 + 6 + 4 = 20.
for boys Therefore, the required probability,
= coefficient of x3 in (1 + x + x2 + x3)4 n( E ) 20 5
4 P( E ) = = = =p (given)
æ 1 - x4 ö n(S ) 216 54
= coefficient of x3 in ç ÷
è 1- x ø \ 54 p = 5
= coefficent of x3 in (1 – x4)4 (1 – x)–4= 6C3 = 20 25. (5) Let A be the event of a boy and B the event
\ Number of arrangements of boys and girls of having cat eyes. So
with given condition = 20 × 10 ! × 5 ! 20 1 20 1
P ( A) = = and P ( B ) = =
40 2 40 2
\ Required probability = 20 ´ 10!´ 5! = 20 10 1
15! 3003 Now, P ( A Ç B) = =
40 4
n 3003 \ P ( A È B ) = P( A) + P ( B ) - P ( A Ç B )
Þ = =3
1001 1001 1 1 1 3 a
22. (3) Let a and b be the roots of the quadratic = + - = =
equation. According to question, 2 2 4 4 b
a + b = a 2 + b2 and ab = a 2b2 = a 2 + b 2 = 32 + 4 2 = 5
132 MATHEMATICS
CHAPTER
16 Relations & Functions-2

1. (d) P = {( a, b) : sec 2 a - tan 2 b = 1} Þ x = w1 y and y = w2z for some w1, w2 ÎQ


For reflexive : Þ x = w1 ( w2 z ) = ( w1 × w2 ) z
sec 2 a - tan 2 a = 1 (true " a) Þ xRz as w1 × w2 ÎQ
For symmetric :
sec2 b – tan2 a = 1 m p p r
Now let n S q and q S t ; where n, q, t ¹ 0 .
L.H.S. 1 + tan 2 b - (sec 2 a - 1)
3. (a) x Î R Þ x - x + 5 = 5 is an irrational
= 1 + tan 2 b - sec 2 a + 1 number.
= – (sec2 a – tan2b) + 2 = – 1+2 = 1
So, Relation is symmetric. \ ( x, x) Î R
For transitive : So, R is reflexive.
if sec2 a – tan2 b = 1 and sec2 b – tan2 c = 1 ( 5, 1) Î R because 5 - 1 + 5 = 2 5 - 1
sec2 a – tan2 c = (1 + tan2 b) – (sec2 b – 1)
= –sec2b + tan2b + 2 = – 1 + 2 = 1 which is an irrational number.
So, relation is transitive. But (1, 5) Ï R
Hence, relation P is an equivalence relation. \ R is not symmetric.
2. (b) Reflexivity : xRx as x = 1 . x and 1 is a
rational number. We have, ( 5, 1), (1, 2 5) Î R because
m m 5 -1+ 5 = 2 5 -1
S iff m . n = m . n, which is true.
n n
If 1 - 2 5 + 5 = 1 - 5 are irrational numbers.
Thus, R and S are reflexive relations.
Symmetricity : Let xRy Also, ( 5, 2 5) Î R and 5 - 2 5 + 5 = 0
Þ x = wy for some rational number w which is not an irrational numbers.
1 \ ( 5, 2 5) Ï R
Þy= x
w So, R is not transitive.
Þ yRx provided w ¹ 0, but if x = 0 and y is any 4. (a) f ( x) is onto
non-zero integer, then x = 0y and there exist no \ S = range of f (x).
rational number w for which y = wx. Now f (x) = sin x - 3 cos x + 1
Þ y R x thus R is not symmetric. æ pö
= 2sin ç x - ÷ + 1
è 3ø
m p
Now, let S
n q æ pö
Q -1 £ sin ç x - ÷ £ 1
è 3ø
p m
Þ mq = pn Þ pn = mq Þ S
q n æ pö
Þ S is a symmetric relation. -1 £ 2sin ç x - ÷ + 1 £ 3
è 3ø
Transitivity : Let xRy and yRz.
\ f ( x) Î[ -1, 3] = S .
Solutions 133
5. (b)
6. (c) Let f (x) ¹ 2 be true and f (y) = 2, f (z) ¹ 1 are 10. (d) We have, f ( x) = x + x2 = x+ | x |
false Clearly, f is not onto as
Þ f (x) ¹ 2, f (y) ¹ 2, f (z) = 1 f (-1) = f (-2) = 0 but -1 ¹ -2
Þ f (x) = 3, f (y) = 3, f (z) = 1
but then function is many one, similarly two other Also, f is not onto as f ( x) ³ 0, "x Î R.
cases. \ Range of f = (0, ¥) Ì R.
7. (c) Total number of functions from A ® B
= 34 = 81 and number of onto mappings 11. (b) f (x) = sin x + cos x, g (x) = x2 – 1
= coefficient of x4 in 4! (ex – 1)3. Þ g (f (x)) = (sin x + cos x)2 – 1 = sin 2x
= coefficient of x4 in 4! (e3x – 3e2x + 3e2x + 3ex – 1) p p
£ 2x £
Clearly g (f (x)) is invertible in –
2 2
æ 34 3.24 3.14 ö
+ [Q sin q is invertible when – p/2 £ q £ p/2]
= 4! çç – – 0 ÷÷
è 4! 4! 4! ø p p
= 81 – 48 + 3 = 81 – 45 = 36 Þ £x£ .

4 4
\ Number of functions from A ® B, which are 12. (b) Here
not onto is 81 – 36 = 45.
2 g 2 ( x ) = gog ( x) = g{g ( x)} = g (3 + 4 x)
8. (b) Let f ( x ) = ax + bx + c as it touches
X-axis at x = 3 = 15 + 42 x = (42 - 1) + 42 x
-b
Þ =3 g3 ( x) = gogog(x) = g(15 + 42 x) = 3 + 4(15 + 42 x)
2a
Þ b = -6 a ...(1) = 63 + 43 x = (43 - 1) + 43 x
Also, 9a + 3b + c = 0 ...(2) Generalizing, we get
4a – 2b + c = 3 ...(3)
g n ( x) = (4n - 1) + 4n x = y (say)
3 18 27
Þa= ,b=- ,c= then x = ( y + 1 - 4n )4 - n
25 25 25
3 2 Þ g - n ( y ) = ( y + 1)4- n - 1
Þ f ( x) = ( x - 6 x + 9)
25
\ g - n ( x) = ( x + 1)4- n - 1
9. (b ) f ( x ) = sgn( x - x 4 + x 7 - x8 - 1)
13. (d) f ( x) = 3 | x | - x - 2 and g (x) = sin x
7
For x Î (0, 1); x - 1 < 0, x - x < 0 4
for fog (x) = 3 | sin x | - sin x - 2 which is
4 7 8 defined if
\ x - x + x - x -1 < 0
3 |sinx | – sin x – 2 ³ 0
For x Î (1, ¥ ); x < x 4 , x 7 < x8
If sin x > 0 then 2 sin x – 2 ³ 0 Þ sin x ³ 1
\ x - x 4 + x 7 - x8 - 1 < 0 p
Þ sin x = 1 Þ x = 2np +
4 7 8
Also for x = 1; x - x + x - x - 1 = -1 2
If sin x < 0 then – 4 sin x – 2 ³ 0
Thus x - x 4 + x7 - x8 - 1 < 0 for all x Î R +
1
Þ – 1 £ sin x £ –
Hence sgn( x - x 4 + x 7 - x8 - 1) = -1"x Î R + . 2
Therefore f (x) is many-one and into.
7p 11p ù
Þ x Î é2np + , 2np +
êë 6 6 úû
134 MATHEMATICS

é
x Î ê 2np +
7p
, 2np +
11p ù ì pü
È í 2 m p + ý , n, m Î I
and gof ( x) = x, " xÎ[0,1]
6 ûú î
.
ë 6 2þ Only condition that g(x) should satisfy for
2
gof ( x) = x, " xÎ[0,1] is that g(x) should attain
æ 3 1 ö all values in [0, 1] when rangte of f (x) a subset of
14. (b) f ( x) = sin 2 x + ç cos x + sin x ÷
ç 2 2 ÷ (–1, 1) is used as image for g(x). Thus, there can
è ø
be infinite such functions g(x) with domain
æ1 3 ö [–1, 1] and range [0, 1]
+ cos x ç cos x - sin x ÷ 18. (14) If set A has m elements and set B has n
ç2 2 ÷
è ø elements then number of onto functions from A
to B is
3 1 3 n
= sin 2 x + cos 2 x + sin 2 x +
4 4 2
cos x sin x
å (-1)n-r n
Cr r m where 1 £ n £ m
r =1
1 3
+ cos 2 x - cos x sin x Here E = {1, 2,3, 4}, F = {1, 2}
2 2
5 5 m = 4, n = 2
= (sin 2 x + cos 2 x ) = \ No. of onto functions from E to F
4 4
2
æ5ö
Now, y = g (f (x)) = g ç ÷ = 1.
è4ø
= å (-1)2-r 2Cr (r )4
r =1
Clearly y = 1 is a straight line. = (-1) 2C1 + 2C2 (2) 4 = – 2 +16 = 14.
2
15. (a) Given that f (x) = x and g(x) = sin x, " x Î R.
Then (gof) (x) = sin x2 19. (0) f ( x) = (a - xn )1/n , x > 0
Þ (gogof) (x) = sin (sin x2) Þ f ( f (x)) = [a -{(a - xn )1/ n}n ]1/ n = x
Þ (fogogof) (x) = sin 2 (sin x2)
2
As given that (fogogof) (x) = (gogof) (x) Also g (x) = x + px + q
Þ sin2 (sin x2) = sin (sin x2) \ g (x) – x = 0 is quadratic equation
Þ sin (sin x2) = 0, 1
x 2 + ( p - 1) x + q = 0
2 p
Þ sin x = np or ( (4n + 1) where n Î Z Given that this equation has imaginary roots
2
\ x 2 + ( p - 1) x + q > 0 for all real x
2 2
Þ sin x = 0 Q sin x Î[ -1,1] [\ coefficient of x2 = 1 > 0]
Þ x2 = np Þ x = ± np where n Î W \ g ( x) - x > 0 for all real x
16. (b) Let h(x) = g( f (x)) where, g(x) is injective Þ g (g(x)) – g (x) > 0 " x Î R
and h(x) is surjective.
\ Codomain of h(x) = Range of h(x). Now g (g (x)) – f (f (x)) = g (g (x)) – x
Range of h(x) = [0, 2] = [g (g(x))– g (x)] + [g(x) – x] > 0 " x Î R
Þ codomain of g is [0, 2]. So, g(x) must be \ The equation g (g (x)) – f (f (x)) = 0
surjective. has no real root.
Now, domain of g = [–1, 1] which must be range 20. (1) g (x) = 1 + x – [x];
of f.
But codomain of f = [–1, 1] ì –1, x < 0
Þ f must be surjective. ï
f (x) = í 0, x = 0
17. (d) We have, –1< f (0) < f (1) <1 ï 1, x > 0
î
g = [–1,1] ®[0,1]
Solutions 135
For integral values of x; g (x) = 1.
æp ö é æp ö ù
For x < 0; (but not integral value) x – [x] > 0 Þ g (x) > 1. Þ f (1) = cos2 1 + cos ç + 1÷ × êcos ç + 1÷ - cos1ú
For x > 0;(but not integral value) x – [x] > 0 è3 ø ë è3 ø û
Þ g (x) >1 æp ö é æ p ö pù
\ g (x) ³ 1, " x \ f (g (x)) = 1, " x. Þ f (1) = cos2 1+ cos ç +1÷ × ê-2sin ç +1÷ sin ú
è3 ø ë è 6 ø 6û
e x - e- x
21. (6) Let y = g ( x) = æp ö æp ö
2 Þ f (1) = cos 2 1 - cos ç + 1÷ sin ç + 1÷
è3 ø è6 ø
Þ e2 x - 1 = 2 ye x
Substituting ex = t, we have t2 – 2yt – 1 = 0 1é æp ö æ p öù
Þ f (1) = cos 2 1 - sin ç + 2 ÷ + sin ç - ÷ ú
2 êë è 2 ø è 6 øû
2 y ± 4 y2 + 4
Þt= = y ± y2 +1 1é 1ù
2 Þ f (1) = cos 2 1 - êcos 2 - ú
2ë 2û
Þ e x = y + y2 + 1 (Q e x > 0)
1 1 3
2
= cos 2 1 - (2 cos 2 1 - 1) + =
Þ x = ln( y + y + 1) 2 4 4
\ gof (1) = f ( f (1)) = f (3/ 4) = 2.
Þ g -1 ( y ) = ln( y + y 2 + 1)
24. (2) Since, f (x) is symmetric about y = x.
-1
Þ f -1 ( x) = f ( x )
2
\ g ( x ) = f ( x) = ln( x + x + 1)

(Q g ( f ( x)) = x Þ g -1 ( x) = f ( x)) \ f 2 ( x) = ( f -1(x))2 - px × f (x) f -1(x) + 2x2 f (x).

æ 22 2 ö Þ f 2 ( x) = f 2 ( x ) - px × f ( x ) × f ( x) + 2 x 2 f ( x )
æ e 22 - 1 ö ç e -1 + æ e22 - 1 ö ÷
\fç ÷ = ln çç ÷ + 1
ç 2e11 ÷ ç 2e11 2 e11 ÷ ÷ Þ f ( x ) × {2 x 2 - px × f ( x )} = 0
è ø ç è ø ÷
è ø
2x2
æ 22 2 ö Þ f ( x) = [as f ( x ) ¹ 0 ]
e -1 æ e 22 + 1 ö px
= ln ç + çç 11 ÷
÷ = ln[e11 ] = 11.
ç 2e11 ÷ ÷
ç è 2e ø ÷ 2x
è ø Þ f ( x) = Þ p = 2.
p
ìï - x + 1, x ³1
22. (4) f -1 ( x ) = í the solution of 25. (7) Here, f(1) = 1
ïî1 + - x , x£0
f(2) = f [f (1)] + f [2 – f (1)], using f(1) = 1
f ( x) - f -1 ( x) = 0 are x = –1, 1, 0, 2. f(2) = f(1) + f (1) = 2.
æp ö f(3) = f [f(3)] + f[3 – f(2)] = f (2) + f(1) = 2 + 1 = 3.
23. (2) f ( x ) = cos 2 x + cos 2 ç + x ÷
è3 ø Thus, f (n) = n
æp ö æ3ö 1 20 1
- cos x × cos ç + x ÷ ; g ç ÷ = 2;
è3 ø è4ø
\ å f (r ) = 30 [1 + 2 + 3 + .... + 20]
30 r =1
gof(x) = ?
1 20(20 + 1)
æp ö æp ö = ´ = 7.
f (1) = cos 2 1 + cos 2 ç + 1÷ - cos1cos ç + 1÷ 30 2
è 3 ø è3 ø
136 MATHEMATICS

CHAPTER
17 Inverse Trigonometric Functions
1. (c) Let tan–1 x = a and tan –1y = b 3. (a) Given
Þ tan a = x, tan b = y p
The given system of equations is : cos -1 b1 + cos-1 b2 + .... + cos -1 bk = k
a tan a + b sec a = c and a tan b + b sec b = c 2
\ a and b are roots of a tan q + b sec q = c -1 p
2 2
We know that cos x £ "x ³ 0
Þ (b sec q ) = (c - a tan q) 2
-1 p
Þ (a 2 - b 2 ) tan 2 q - 2ac tan q + c 2 - b 2 = 0 \ cos br £ "r = 1, 2,3,...., k
2
2ac k
\ tan a + tan b = 2 and kp
a - b2 Þ å cos-1b r £ 2
r =1
c 2 - b2 So the given equality holds only if
tan a tan b =
a 2 - b2 p
2 2 cos -1 b1 = cos -1 b2 = ..... = cos -1 b k =
2ac c -b 2
\ x+ y = and xy =
2
a -b 2
a2 - b2 Þ b1 = b2 = .... = b k = 0 .
k
a 2 - c2
Þ 1 - xy = 2
a -b 2 Thus A= å ( br ) r = 0 .
r =1
x+ y 2ac (1 + x 2 )1/ 3 - (1 - 2 x)1/ 4
\ =
1 - xy a - c 2
2 Required limit = lim
x ®0 x + x2
é 1ù é 1ù
(c) f ( x) = sin-1 ê x2 + ú + cos-1 ê x2 - ú
1 1
2. (1 + x 2 ) -2 / 3 (2 x ) - (1 - 2 x) -3 / 4 ( -2)
ë 2û ë 2û = lim 3 4
x®0 1+ 2x
é 1ù é 1 ù
= sin -1 ê x 2 + ú + cos-1 ê x2 + - 1ú (L’ Hospital Rule)
ë 2 û ë 2 û 1
=
é 1ù æé 1ù ö 2
= sin -1 ê x2 + ú + cos-1 ç ê x2 + ú - 1÷ 4. (c) sin–1 (log[x]) is defined if -1 £ log[ x ] £ 1
ë 2 û èë 2û ø
and [ x] > 0
1 1 é 1ù
Since x 2 + ³ , ê x2 + ú = 0 or 1 as 1
2 2 ë 2û Þ £ [ x ] £ e Þ [x] = 1, 2 Þ x Î [1, 3)
e
é 1ù
sin -1 ê x 2 + ú is defined only for these two Again, log(sin -1[ x ]) is defined if
ë 2û
values. sin -1[ x ] > 0 and -1 £[ x ] £ 1
é 1ù Þ [ x ] > 0 and - 1 £ [ x] £ 1 Þ 0 < [ x] £ 1
Hence ê x2 + ú = 0
ë 2û Þ x Î[1, 2)
é 2 1ù \ Domain of f ( x) = [1, 2)
Þ f ( x) = sin -1 0 + cos -1 (-1) = p ê x + ú = 1
ë 2û For 1 £ x < 2, [x] = 1
-1 ( ) -1
Þ f ( x) = sin 1 + cos 0 = p . p p
\ f ( x) = sin -1 0 + log = log , "x Î [1, 2)
Therefore range of f ( x) = {p} 2 2
Solutions 137

pü 1 1
\ Range of f ( x) = ì
-1
ílog ý . + tan + tan -1 + ...... +
î 2þ 1 + (n - 1)n 1 + n(n + 1)
5. (c) We have 1 n + 19
S1 = S x1 = sin 2b tan -1 = tan -1
1 + (n + 19) (n + 20) n + 21
S2 = S x1x2 = cos 2b
S3 = S x1x2x3 = cos b -1 n - 1 1
Þ tan + tan -1
S4 = x1x2x3x4 = – sin b n +1 1 + n ( n + 1)
4 S - S3 1
So that å tan -1 xi = tan -1 1 - 1S + tan -1
i =1 2 + S4 1 + (n + 1) (n + 2)
sin 2b - cos b 1 n + 19
= tan -1 + ...... + = tan -1
1 - cos 2b - sin b 1 + (n + 19) (n + 20) n + 21
cos b ( 2sin b - 1) Þ tan-1
1
+ tan-1
1
+ ...... +
= tan -1 1+ n(n +1) 1+ (n +1)(n + 2)
sin b ( 2sin b - 1)
= tan cot b = tan -1 ( tan ( p / 2 - b ) ) = p / 2 - b
1 -1 n + 19 n -1
= tan - tan -1
1 + ( n + 19) ( n + 20) n + 21 n +1
f ( x ) = tan
æ
-1 ç (
12 - 2 x 2 ö
÷ ) æ 1 ö -1 æ 1 ö
6. (d)
ç x4 + 2 x 2 + 3 ÷
Þ tan-1 ç 2 ÷ + tan ç 2 ÷ +...... +
è ø è n + n +1 ø è n + 3n + 3 ø
-1 1
æ ö tan
= tan ç-1
ç 2 3 -1 ÷( ÷
) 1 + ( n + 19) ( n + 20)
æ n + 19 n - 1 ö
ç x2 + 3 + 2 ÷ ç - ÷
ç ÷
è x2 ø = tan -1 ç n + 21 n + 1 ÷
2 3 çç 1 + n + 19 ´ n - 1 ÷÷
As x + ³ 2 3 [using AM > GM] è n + 21 n + 1 ø
x2
3 -1 20
Þ x2 + 2 + 2 ³ 2 + 2 3 = tan 2
=S
x n + 20n + 1
æ 2 3 -1 ö
÷= p ( ) \ tan S = 2
20
\ ( f ( x ) ) max = tan çç
-1 n + 20n + 1
è
2 3 +1 ÷
ø
12 ( ) -1
9. (c) sin x - -1 = cos x -
1 -1 1
cos -1 x
( x + 1) ( x - 1)
2 2 2 sin x
7. (c) - 4 x2 = 2
= | x2 - 1 | (sin x . cos-1 x + 1)
-1
Þ (sin-1 x - cos-1 x) =0
Þ |tan–1 | x || = | x2 – 1 | sin -1 x . cos-1 x
Draw the graphs of y = |tan –1 |x|| and y = |x2 – 1|
y Þ sin-1 x = cos-1 x or sin -1 x cos-1 x +1 = 0
2
y = |x – 1| 1 æp ö
p/2 Þ x= or sin -1 x ç - sin -1 x ÷ + 1 = 0
2 è 2 ø
–1
y = |tan – |x|| p æ p 2 ö
± ç + 4÷
2 ç 4 ÷
1 è ø
–3 –2 –1 0 1 2 3 Þ x= or sin -1 x =
2 2
From the graph, it is clear that equation has four 1 p æ p2 ö
roots. Þ x= or sin -1 x =
- ç1 + ÷
2 4 ç 16 ÷
8. (c) We know that, è ø
10. (b) (cot–1 a)x2 – (tan –1 a)3/2 x + 2(cot–1 a)2 = 0
1 1 1
tan-1 + tan -1 + tan -1 + ...... Equation has real roots
1+ 2 1+ 2 ´ 3 1+ 3´ 4
138 MATHEMATICS
13. (b) The given relation is possible when
( ) - 8 ( cot -1 a )
3 3
\ D ³ 0 Þ tan -1 a ³0
a 2 a3
a- + + ... = 1 + b + b 2 + ...
\ tan -1 a ³ 2cot -1 a = p - 2 tan -1 a 3 9
Also
p
Þ tan -1 a ³ Þa³ 3 a2 a3
3 -1 £ a - + + ... £ 1& -1 £ 1+ b + b2 + ... £ 1
3 9
( tan-1 a )
3/ 2
a 1
Sum of roots > 0 Þ >0 Þ | b |< 1 Þ| a |< 3 and =
2 cot -1 a a 1- b
1+
tan–1 a > 0 Þ a > 0 3
Product of roots > 0 Þ 2 cot–1 a > 0 3a 1
Þ = , there are infinitely many
Þ aÎR Þ a ³ 3 a + 3 1- b
( n2 + 1)( n2 - 2n + 2)
solution. But in given options it is satisfied only
-1 1
11. (b) Tn = sec when a = 1 and b = - .
( n2 - n + 1)
2
3
æ 2
–1 4 r + 3
ö –1 1
(n 2 + 1)2 ( n 2 - 2n + 2) 14. (d) Tr = cot ç ÷ = tan 1
Þ sec 2 Tn = è 4 ø 1+ r2 –
(n 2 - n + 1) 2 4
(n + 1) 2 + ( n 2 + 1) - 2 n( n 2 + 1)
2
æ 1ö æ 1ö
Þ sec 2 Tn = çè r + ÷ø – çè r – ÷ø
(n 2 - n + 1) 2 = tan –1
2 2
æ 1ö æ 1ö
1 + (n2 + 1 - n)2 1+ ç r + ÷ ç r – ÷
Þ sec2 Tn = è 2ø è 2ø
(n 2 - n + 1) 2
æ 1ö æ 1ö
Þ tan Tn = 2
1 = tan –1 ç r + ÷ – tan –1 ç r – ÷
è 2ø è 2ø
n - n +1
æ 1ö 1
n - ( n - 1) Sn = å Tr = tan -1 ç n + ÷ – tan –1
Þ tan Tn = è 2 ø 2
1 + n ( n - 1)
æ 4n ö
Þ tan–1 n – tan–1 (n – 1) = tan –1 ç ÷
\ S = T1 + T2 + T3 +....+ Tn è 2n + 5 ø
\ S = tan –1 n 1
12. (c) (sin–1 x)3 – (cos–1 x)3 + (sin –1 x) (cos–1 x) S ¥ = lim S n = tan –1 2 = cot –1 .
n®¥ 2
p3
(sin–1 x – cos–1 x) = æ 1 ö æ (n + 1) - n ö
16 15. (c) Tn = tan -1 ç 2 = tan-1 ç
è n + n + 1÷ø è 1 + (n + 1) n ÷ø
(sin–1 x – cos–1 x) {(sin –1 x)2 + (cos–1 x)2 = tan (n + 1) – tan –1 n
–1
Þ T1 = tan (1/3) = tan –1 2 – tan–1 1
–1
p3 T2 = tan–1 (1/7) = tan –1 3 – tan–1 2
+ (2 cos–1 x sin –1 x)} = ..... ..........................................
16
Tn = tan–1 (n + 1) – tan–1 n
p3 On adding
(sin–1 x – cos–1 x) (sin –1 x + cos–1 x)2 =
16 T1 + T2 + T3 +.....+Tn = tan–1 (n + 1) – tan–1 1
p2 p3 æ n ö
(sin–1 x – cos–1 x) = = tan -1 ç
4 16 è n + 2 ÷ø
p p 3p
2sin -1 x - = ; 2 sin –1 x = \ lim (T1 + T2 + T3 + ..... + Tn )
n®¥
2 4 4
3p 3 x x æ 1 ö
Þ sin–1 x = Þ x = sin or cos = lim tan -1 ç ÷
8 8 8 n®¥ è 1+ 2 / n ø
Solutions 139

p æ 1 ö ì -1 æ 1 ö -1 æ 1 ö ü
= tan–1 1 = = sin -1 ç ÷ + ísin ç ÷ - sin ç ÷ý
4 è 2ø î è 2ø è 3 øþ
16. (b) x3 + bx2 + cx + 1 = 0 ; f(–1) = b – c < 0
f(0) = 1 > 0 Þ –1 < a < 0 ì æ 1 ö -1 æ 1 ö ü
a = –B + ísin -1 ç ÷ - sin ç ÷ý
B Î (0, 1) î è 3ø è 4 øþ
æ 2 sin B ö ì æ 1 ö -1 æ 1 ö ü
y = -2 tan -1 ( cosec B ) - tan -1 ç + ísin -1 ç ÷ - sin ç ÷ ý + ...
÷ î è 4ø è 5 øþ
è cos 2 B ø
æ 2cosec B ö 2sin B æ 1 ö p p
= - ç p + tan -1 - tan -1 = -p = 2 sin -1 ç ÷ =2. =
2 ÷ è 2ø 4 2
è 1 - cosec B ø cos 2 B
p
17. (a) cos -1 x 3 + cos -1 x = is given equation a3 æ1 a ö b3 æ1 bö
2 19. (c) cosec 2 ç tan -1 ÷ + sec 2 ç tan -1 ÷
...(i) 2 è2 bø 2 è2 aø
p 1 1
Now cos -1 x 3, cos-1 x ³ 0 and their sum is = a3 + b3
2 æ -1 æ a ö ö æ -1 b ö
1 - cos ç tan ç ÷ ÷ 1 + cos ç tan ÷
Þ cos-1 x 3, cos-1 x both must belong to è è b øø è aø
1
é pù = a3
ê 0, 2 úû
æ æ
b
öö
ë
1 - cos ç cos -1 ç ÷÷
Þ x 3, x Î [0,1] çè çè a 2 + b2 ÷ø ÷
ø
p
Now from (i) cos -1 x 3 + cos -1 x = +b3
1
2 æ ö
p a
Þ cos -1 x 3 = - cos-1 x 1 + cos ç cos -1 ÷
2 çè a 2 + b2 ÷ø
Þ cos -1 x 3 = sin -1 x 1 1
= a3 + b3
b a
1- 1+
Þ cos -1 x 3 = cos -1 1 - x 2 2 2
a +b a + b2
2
Þ x 3 = 1- x 2 as cos –1 x is one-one
æ ö
function a3 b3
= a 2 + b2 ç - + ÷
1 çè
3x 2 = 1 - x 2 Þ 4 x2 = 1 Þ x = ± a 2 + b2 - b a 2 + b2 + a ÷ø
2
1
But x Î[ 0, 1] Þ x = is the only possible
2 2
= a +b ç a
æ
2ç 3
( a2 +b2 +b ) ( 3
+b
a2 +b2 -a ) ö
÷
÷
solution ç a2 b2 ÷
Þ (a) is the correct option. è ø
é n - n -1 ù é ù
-1 = a 2 + b2 êa æç a 2 + b2 + b ö÷ ú
18. (a) Let Tn = sin ê ú ë è øû
ê n ( n + 1) ú
ë û
+b æç a 2 + b2 - a ö÷
é 1 1 1 1ù è ø
= sin -1 ê 1- - 1- ú
ë n n + 1 n +1 nû = a 2 + b2 ( a + b ) a 2 + b2
æ 1 ö -1 æ 1 ö = (a + b)(a 2 + b 2 )
Tn = sin -1 ç ÷ - sin ç ÷
è n ø è n +1 ø 20. (a) cot–1(2.12) + cot–1(2.22) + cot–1(2.32) +...
¥ ¥
\ S = sin -1 æ
ç
1 ö
÷ + T2 + T3 + T4 + ...
=
r =1
( )
å cot -1 2.r 2 =
r =1
æ 1 ö
å tan -1 çè 2r 2 ÷ø
è 2ø
140 MATHEMATICS

¥ é (1 + 2r ) + (1 - 2r ) ù sin A cos A
= å tan -1 êê1 - (1 + 2r )(1 - 2r ) úú =
(sin A + cos 2 A)(sin 2 A - cos 2 A)
2
r=1 ë û
¥ sin 2A 1
=- = - tan 2A
= å éë tan -1 (1 + 2r ) + tan -1 (1 - 2r ) ùû 2 cos 2 A 2
r =1 Hence,
= tan–1 3 – tan–1 1 + tan–1 5 – tan–1 3 + tan–1 7 æ1 ö
– tan–1 5 +....+ tan –1 ¥ tan -1 ç tan 2 A÷ + tan -1 (cot A) + tan -1 (cot 3 A)
p p p è2 ø
=- + =
4 2 4 ì p
1/ x ïï p if 0 < A <
4 [Q tan -1 (- x) = - tan -1 x]
æp -1 ö =í
21. (1) Let y = lim ç - tan x ÷ ï0 p p
x®¥ è 2 ø if < A <
–1 1/x ïî 4 2
= lim (cot x)
x®¥ é p pù
23. (2) (sin–1x) Î ê - , ú
log cot -1 x æ ¥ ö ë 2 2û
\ log y = lim çè form÷ø
x ¥ p2
( )
x®¥ 2
\ sin -1 x £
Þ (sec–1 y)2, (tan–1 z)2 > 0
–1 4
log y = lim ( 0. ¥ form)
x®¥(1 + x ) cot -1 x
2 p2 p2
\ RHS ³ \ (sin–1 x)2 =
(1 + x 2 )-1 æ0 ö 4 4
log y = – lim çè form÷ø \ (sec–1 y)2 + (tan–1 z)2 = 0
x®¥ cot -1 x 0
or sec–1 y = tan –1 z = 0
-2 x p
-1
(1 + x 2 )2 \ sin x = ± , y = 1, z = 0
lim 2
= – x®¥ -1 \ x = ±1, y = 1, z = 0
24. (3) cos (2 sin–1 (cot (tan –1 (sec (6 cosec–1x)))))
1 + x2 = –1
lim x 1
= – 2 x®¥ = 2 lim =0 p
1 + x2 x® ¥ 2 x sin–1 (cot (tan–1 (sec (6 cosec–1 x)))) = ±
\ y = e0 =1 2
cot (tan–1 (sec (6 cosec–1x))) = ± 1
p p
22. (0) We know that cot A > 1 if 0 < A < tan–1 (sec (6 cosec–1x)) = ±
4 4
p p sec (6 cosec–1 x) = ±1
and cot A < 1 if <A<
4 2 6 cosec x = ± 3p, ± 2p, ± p
–1

tan -1 (cot A) + tan -1 (cot 3 A) p p p 2


cosec–1x = ± , ± , ± Þ x = 1, ,2
2 3 6 3
cot A + cot 3 A
= p + tan -1 ,
(Q x > 0)
1 - cot 4 A
p 25. (7) f ( x ) = 3cos -1 ( 4 x ) - p is defined
If 0 < A <
4
and tan–1(cotA) + tan–1 (cot3A) -1 p 1 1
If cos 4 x ³ Þ 4 x £ or x £ ...(i)
p p 3 2 8
cot A + cot 3 A
= tan -1 if < A < -1 1
4
1 - cot A 4 2 Also, -1 £ 4 x £ 1 or £x£ ...(ii)
4 4
cot A + cot 3 A cot A cosec2 A.sin 4 A Therefore, from eqs. (i) and (ii), we have domain:
Also,
4
=
1 - cot A sin 4 A - cos 4 A é -1 1 ù
x Î ê , ú Þ 4a + 64b = 7
ë 4 8û
Solutions 141
CHAPTER
18 Matrices

é cos 2 q cos q sin q ù é 2 - 1ù é3 2ù é1 1ù


1. (a) AB = ê ú \ (B -1 )(C -1 ) = ê úê ú=ê ú
êëcos q sin q sin 2 q úû ë- 3 2 û ë5 3û ë1 0û

é1 1 ù
é cos2 f cos f sin f ù \A = ê ú
ê ú ë1 0û
êë cos f sin f sin 2 f úû
é1 0 ù é 0 - tan (q / 2) ù
3. (c) I– P = ê ú –ê ú
écos q cos f + cos q cos f sin q sin f
2 2
ë0 1 û ë tan (q / 2) 0 û
=ê 2 2
êë cos f cos q sin q + sin q sin f sin f é 1 tan (q / 2) ù
= ê - tan (q / 2) 1 ú
ë û
cos 2 q cos f sin f + sin 2 f sin q cos qù
ú
cos q cos f sin q sin f + sin 2 q sin 2 f úû écos q - sin q ù
\ (I – P) ê sin q cos q ú
ë û
écos q cos f(cos q cos f + sin q sin f)

é 1 tan (q / 2) ù écos q - sin q ù
ë sin q cos f(cos q cos f + sin f sin f = ê - tan q / 2 1 ú. ê sin q cos q ú
ë û ë û
cos q sin f(cos q cos f + sin q sin f ù
sin q cos f(cos q cos f + sin q sin f)úû é cos q + tan( q / 2) sin q - sin q + tan(q / 2) cos q ù
= ê - tan(q / 2) cos q + sin q tan(q / 2) sin q + cos q ú
écos q cos f cos( q - f) cos q sin f cos(q - f)ù ë û
=ê ú
ë sin q cos f cos( q - f) sin q sin f cos(q - f) û é 1- 2sin2 (q / 2) -2sin(q / 2)cos(q / 2) ù
ê 2 + tan(q / 2)(2cos2 (q / 2) -1) ú
Clearly AB is the zero matrix if cos( q - f) = 0 + 2sin ( q / 2)
=ê ú
ê- tan (q / 2)(2cos2 q / 2 -1) tan(q / 2)(2sin(q / 2) cos(q / 2))ú
p ê ú
ë+2sin (q / 2) cos(q / 2) +(1 - 2sin2 (q / 2)) û
i.e. q - f is an odd multiple of .
2
é 1 - tan (q / 2) ù
é2 1ù é- 3 2 ù = ê ú = I+P
2. (a) Let B = ê ú and C = ê ú ë tan q / 2 1 û
ë 3 2û ë 5 - 3û
-1 -1
Given BAC = I Þ B (BAC) = B I é 1 – xù é 1 – y ù
4. (c) (1 – x)–1 (1 – y)–1 ê úê ú
ë – x 1 û ë– y 1 û
Þ I(AC) = B -1 Þ AC = B-1
é 1 + xy –( x + y ) ù
Þ ACC -1 = B -1C -1 Þ AI = B -1C -1 = (1 + xy – (x + y))–1 ê –( x + y ) 1 + xy ú
ë û
\ A = (B -1 )(C -1 ) é x+ yù
–1 ê 1 –
-1 1 é 2 - 1ù é 2 - 1ù æ x+ y ö 1 + xy ú
Now B = ê ú= ê- 3 2 ú = ç1 – ÷ ê ú = A( z)
4 - 3 ë- 3 2 û ë û è 1 + xy ø ê x+ y ú
ê – 1 + xy 1 ú
1 é - 3 - 2 ù é3 2ù ë û
C -1 = ê ú=ê
9 - 10 ë- 5 - 3 û ë5 3 úû
Þ A( x) A( y) = A( z)
142 MATHEMATICS

5. (d) We have
é0 2b cù
é1 (c) Let A = ê a -c ú .
0 ù é1 0ù é1 0 ù é1 0ù
8. b
A2 = ê
1/ 2 1 ú ê1/ 2 1ú = ê 2(1/ 2) 1ú = ê1 1úû ê ú
ë ûë û ë û ë
ëê a -b c ûú
é1 0 ù é1 0 ù é1 0 ù
A4 = ê úê ú=ê ú; é0 a aù
ë1 1û ë1 1û ë 2 1û
Now A = ê 2b
T
b -b ú
ê ú
é1 0 ù é1 0 ù é1 0 ù êë c -c c úû
A8 = ê 2 1ú ê 2 1ú = ê4 1ú
ë ûë û ë û
Q A is orthogonal.
é1 0ù \ AAT = I
In general, by induction An = ê ú
ë n / 2 1û
é 0 2b c ù é 0 a a ù é1 0 0 ù
é1 0ù Þ a b - c 2b b -b ú = ê 0 1 0 ú
ê ú ê
Þ A50 = ê ú ê úê ú ê ú
ë 25 1û ëê a -b c ûú êë c -c c ûú ëê 0 0 1 ûú
6. (d) As aij = (i2 + j2 – ij) (j – i)
aji = (j2 + i2 – ji) (i – j) = – aij é 4b2 + c 2 2b 2 - c 2 -2b 2 + c 2 ù
Þ A is skew symmetric Þ Tr(A) = 0. ê ú
Also |A| = 0. Þ ê 2b 2 - c 2 a 2 + b2 + c 2 a 2 - b2 - c 2 ú
ê ú
ê -2b2 + c 2 a 2 - b2 - c 2 a 2 + b2 + c 2 ú
é 3 1 ù ë û
ê ú
2 2 ú é1 0 0ù
7. (a) Given that, P = ê
ê 1 3ú = ê 0 1 0ú
ê– ú ê ú
ë 2 2 û ëê 0 0 1ûú
é1 1ù T T 2005
A=ê ú and Q = P A P and X = P Q P Equating the corresponding elements, we get
ë0 1û 4b2 + c2 = 1 ....(i)
We observe that Q = P A PT 2b2 – c2 = 0 ....(ii)
Þ Q2 = (P A PT) (P A PT) = P A (PT P) A PT a2 + b2 + c 2 = 1 ....(iii)
= PA (I A) PT a2 – b2 – c 2 = 0 ....(iv)
\ P A2 PT From solving (i), (ii) and (iii) we get,
Proceeding in the same way, we get 1 1 1
a=± ;b=± ; c=± ;
Q 2005 = P A2005 PT 2 6 3
é1 1ù é1 2 ù
Also A = ê ú Þ A2 = ê ú é 0 1 0 ù é 0 1 0ù
ë0 1û ë0 1 û 9. (a) A2 = AA = ê 0 0 1 ú ´ ê 0 0 1 ú
ê ú ê ú
and proceeding in the same way êë p q r úû êë p q r úû
é1 2005 ù
A2005 = ê
ë0 1 úû é0 0 1 ù
ê ú
Now, X = PTQ2005 P =ê p q r ú
= PT(PA2005 PT)P = (PT P) A2005 (PT P) ê ú
ë pr p + qr q + r 2 û
é1 2005 ù
= IA2005 I = A2005 = ê
ë0 1 úû
Solutions 143
Again
1
\ y= (13 - 5 + 5m)
é0 0 1 ù é 0 1 0ù 9
ê ú Þ – 27 = 8 + 5m (Given y = – 3)
A3 = A2 A = ê p q r ú ´ êê 0 0 1 úú
ê \m=–7
2 ú ê p q r úû
ë pr p + qr q + r û ë 12. (b) We have, AB = A and BA=B.
é p q r ù Now, AB = A Þ (AB) A = A . A Þ A (BA) = A2
ê 2 ú Þ AB = A2 (Q BA = B) Þ A = A2 (Q AB = A)
= ê pr p + qr q+r ú Again, BA = B Þ (BA)B = B2 Þ B(AB) = B2
ê 2 2 2 3ú
ë pq + r p pr + q + qr p + 2qr + r û Þ BA=B2 (Q AB = A) Þ B = B2 ( Q BA = B)
\ A and B are idempotent matrices.
ép 0 0ù é 0 q 0ù 13. (b) Given that
ê ú
= êê 0 p ú
0ú + ê 0 0 qú X = A1 + 3 A 33 + ... ( 2 n – 1 ) ( A 2n -1 )2n -1
êë 0 0 ú
pû ê 2 ú
ë pq q qr û We know that if A is a skew-symmetric matrix
then AT = –A
é 0 0 r ù 2n -1
ê 2 ú XT = - [A1 + 3 A33 + ... ( 2 n – 1) ( A2n-1 ) ]
+ ê pr qr r ú
ê 2 3ú = –X, so skew-symmetric
ë pr pr + qr 2 pr + r û
éa b c ù
é1 0 0 ù é 0 1 0ù ê ú
14. (c) A = ê d e f ú A2 = A × A = A Aq
= p êê 0 1 0 úú + q êê 0 0 1úú
ëê g h i úû
êë 0 0 1 úû êë p q r úû
[Q Given that A = Aq]
é0 0 1 ù éa b c ù éa d g ù
ê ú
+r ê p q r ú 2 ê ú ê ú
Þ A = êd e f ú êd e h ú
ê ú
ë pr p + qr q + r 2 û êë g h i úû ê c f i ú
ë û

= pI + qA + rA2 \ A3 - rA2 - qA = pI . é| a |2 + | b |2 + | c |2 ad + be + cf ag + bh + ci ù
2 2
10. (a) Given A = 2A – I Þ A A = (2A – I) A ê ú
2 2 2
Þ ê da + eb + fc | d | + | e | + | f | dg + eh + fi ú
Þ A3 = 2A2 – A = 2 (2A – I) – A ê
2 2 2
ú
ê ga + hb + ic gd + he + if | g | +| h | +| i | ú
= 4A – 2I – A = 3A – 2I ë û
Again A3 A = (3A –2 I) A = 3 A2 – 2A
Þ A4 = 3 (2A – I) – 2A = 4A – 3I é0 0 0ù
........ ê ú
........ = ê 0 0 0 ú (Q Given A2 = O)
In general An = nA – (n –1) I êë 0 0 0 úû
11. (d) PQ = I Þ P–1 = Q
Þ |a|2 + |b|2 + |c|2 = 0
Now the system in matrix notation is PX = B
Þ Re(a) = Re(b) = Re(c) = Im(a) = Im(b)
\ X = P–1 B = QB
= Im(c) = 0
éxù é2 2 1 ù é1 ù Similarly
ê ú 1ê úê ú Re(d) = Re(e) = Re(f) = Im(d) = Im(e) = Im(f) = 0
\ ê yú = ê 13 - 5 m ú ê1 ú and Re(g) = Re(h) = Re(i) = Im(g) = Im(h)
9
êëz úû êë- 8 1 5 úû êë5úû = Im(i) = 0
144 MATHEMATICS

= BN+1 + ( N + 1) BNC
é 0 + 0i 0 + 0i 0 + 0i ù
ê ú = BN[ B + (N + 1) C]
\ A = ê 0 + 0i 0 + 0i 0 + 0i ú
Thus K = N Þ K/N = 1.
êë 0 + 0i 0 + 0i 0 + 0i úû
19. (1023) (AB)×(AB) = A(BA)B = A3B2
= Null matrix of order 3 × 3
(AB)(AB)(AB) = A7B3
15. (d) (XY)T = YTXT n
Y T = (ABC – CBA)T = CTBTAT – ATBTCT s o (AB)n = A2 - 1 Bn
= – CBA + ABC = Y s o k = 210 – 1 = 1023
XT = (ABC + CBA)T = CTBTAT + ATBTCT 20. (27) Observe A1A2
= – CBA – ABC = – X é1 1 ù é1 2 ù é1 3 ù é1 (2 + 1) ù
=ê úê ú=ê ú=ê 1 úû
é cos q sin q ù ë0 1û ë0 1û ë0 1û ë0
16. (d) A = ê ú
ësin q – cos q û
é1 3 ù é1 3 ù é1 (3 + 2 + 1) ù
\ AAT = I ...(i) and A1A2A3 = ê úê ú=ê ú
Now, C = ABAT ë 0 1û ë 0 1û ë0 1 û
Þ ATC = BAT ...(ii) So, in general A1A2A3 ... An
Now ATCnA = ATCCn – 1 A = BATCn – 1 A é1 n + ( n –1) + ( n – 2) + ... + 3 + 2 + 1ù
(from (ii)) =ê ú
= BATCCn – 2 A = B2ATCn – 2 A = ... ë0 1 û
é 1 0ù n(n + 1)
= Bn – 1ATCA = Bn – 1BATA = Bn = ê ú So = 378 Þ n = 27
ë – n 1û 2
21. (4) Given that AT A = I
é x1 ù
ê ú éa b cù
17. (c) Let X = ê x2 ú , (x1 x2 x3)
êë x3 úû where A = êê b c a úú
êë c a b úû
æ a11 a12 a13 ö æ x1 ö
ça a a ÷ çx ÷ =0 éa b c ù
ç 21 22 23 ÷ ç 2 ÷
çè a ÷ç ÷ \A = êb c a ú
T
31 a32 a33 ø è x3 ø ê ú
êë c a b úû
a11 x12 + a22 x22 + a33 x32 + (a 12 + a 21 )x 1 x 2 +
\ AT A = I
(a13 + a31)x1x3 + (a23 + a32)x2x3 = 0
It is true for every x1, x2, x3, éa b c ù éa b c ù é1 0 0ù
then a11 = a22 = a33 = 0, a12 + a21 = 0, a13 + a31 Þ êb c a ú êb c a ú = ê0 1 0ú
ê úê ú ê ú
= 0, a23 + a32 = 0 êë c b úû êë c b úû êë 0
a a 0 1 úû
\ A is a skew symmetric matrix
a21 = – 2008; a31 = – 2010
a32 = 2012 é a2 + b2 + c2 ab + bc + ca ab + bc + ca ù
ê ú
18. (1) We have, BC = CB, and Þ ê ab + bc + ca a2 + b2 + c2 ab + bc + ca ú
AN + 1 = (B + C) N + 1 ê ú
ê ab + bc + ca ab + bc + ca a2 + b2 + c 2 ú
= N+1C0 B N+1 + N+1C1 BNC + N+1C2BN–1C2 + ë û
N+1
... + CrB N+1–rC r + . . . .
é1 0 0 ù
But given that C2 = 0 Þ C3 = C4 = .... = Cr = 0
= êê0 1 0 úú
Hence, AN+1 = N+1CNBN+1 + N+1C1BNC
êë0 0 1 úû
Solutions 145
Þ a2 + b2 + c 2 = 1 ...(i)
and ab + bc + ca = 0 ...(ii) é 0 0 0ù é0 0 0ù
abc = 1 (given) ...(iii) A = ê 0 0 0ú and A = ê 0 0 0 úú
ê
2 ú 3 ê

Now a3 + b3 + c3 – 3abc ëê16 0 0úû ëê 0 0 0 ûú


= (a + b + c) (a2 + b2 + c2 – ab – bc – ca) \ An = O, n ³ 3
Þ a3 + b3 + c3 = (a + b + c) [1 – 0] + 3 × 1
Now P50 = (I + A)50 = 50C0 I50 + 50C1 I49 A
[Using (i), (ii) and (iii)]
3 3 3 + 50C2 + I48 A2 + O
Þa +b +c =a+b+c+3
= I + 50A + 25 × 49 A2.
Now (a + b + c)2 = a2 + b2 + c2 + 2 (ab + bc + ca)
Þ (a + b + c)2 = 1 \ Q = P – I = 50A + 25 × 49 A2.
50

Þ (a + b + c) = 1 Þ q21 = 50 × 4 = 200
[Q (a + b + c) ¹ – 1 as a, b, c all are + ve Þ q31 = 50 × 16 + 25 × 49 × 16 = 20400
numbers.]
Þ q32 = 50 × 4 = 200
\ We get
a3 + b3 + c 3 = 4 q31 + q32 20600 q + q 32
\ = = 103 Þ 31 =1
q 21 200 103 q 21
-1 + i 3
22. (1) z = Þ z3 = 1 and 1 + z + z2 = 0
2 é2 –1 ù é 2 –1 ù é1 0 ù
24. (2) A2 = ê ú ê ú= ê ú = I;
é (- z)r ë 3 – 2 û ë 3 – 2 û ë0 1 û
z 2s ù é( - z) r z 2s ù
P = ê 2s
2 úê ú x = 2, 4, 6, 8....
êë z z r úû êë z 2s z r úû Þ S(cosx q + sinx q) = (cos2 q + sin2 q)
+ (cos4 q + sin4 q) + (cos6 q + sin6 q) + ... + ...
é z 2r + z 4s
ê

(
z 2s (- z)5 + z r ù
ú ) cos 2 q sin 2 q
= + = cot2 q + tan2 q
2s
( r
êë z (-z) + z
r
) 4s
z +z 2r ú
úû 1 – cos 2 q 1– sin 2 q
Using AM ³ GM,
For P2 = – I we should have
z2r + z4s = 1 and z2s ((–z)r + zr) = 0 1
(cot 2 q + tan 2 q) ³ cot 2 q tan 2 q
Þ z2r + zs + 1 = 0 2
Þ r is odd and s = r but not a multiple of 3. Þ (cot2 q + tan2 q) ³ 2
Which is possible when s = r = 1 Þ So, minimum value = 2
\ only one pair is there.
25. (2) As A2 = O, Ak = O k 2.
é 1 0 0ù é 0 0 0ù
Thus, (A + I)50 = I + 50A
23. (1) ê ú
P = ê 4 1 0ú = I + êê 4 0 0úú = I + A
Þ (A + I)50 – 50A = I
êë16 4 1úû êë16 4 0úû
\ a = 1, b = 0, c = 0, d = 1
146 MATHEMATICS

CHAPTER
19 Determinants
1. (a) Given determinant
a2 b2 c2
2
1 a a Þ a b c =0 (using R 3 ® R3 + 2 R2 )
cos(n - 1) x cos nx cos(n + 1) x 1 1 1
=0
sin (n - 1) x sin nx sin (n + 1) x
a2 b2 - a 2 c 2 - a 2
æ C ® C2 - C1 ö
1 + a 2 - 2a cos x a a2 Þ 1 b-a c-a = 0 ç 2 ÷
è C3 ® C3 - C1 ø
Þ 0 cos nx cos(n + 1) x = 0 1 0 0
0 sin nx sin (n + 1) x
Þ (b2 - a 2 )(c - a) - (b - a)(c 2 - a 2 ) = 0
By applying C1 ® C1 + C3 – 2 cos x C2 Þ (b - a)(c - a )(b - c ) = 0
By expanding Þ a = b or b = c or c = a
(1 + a2 – 2a cos x) [cos nx sin (n + 1) x Þ D ABC is an isosceles.
– sin nx cos (n + 1) x]= 0
Now, (1 + a2 – 2a cos x) sin (n + 1 – n) x = 0 a2 b sin A c sin A
2
Þ (1 + a - 2a cos x)sin x = 0 3. (d) D = b sin A 1 cos A
c sin A cos A 1
1 + a2
sin x = 0 or cos x =
2a sin A sin B sin C
Using, = = =k
æ 1 + a2 ö a b c
As a ¹ 1 \ç ÷ >1 Þ sin A = ak , sin B = bk , sin C = ck So,
è 2a ø
Þ cos x > 1 It is not possible. \ sin x = 0 a2 abk ack
2 2 2 D = abk 1 cos A .
a b c
2. (d) ( a + 1) 2
(b + 1) 2
( c + 1) 2 = 0 ack cos A 1

( a - 1) 2 ( b - 1) 2 ( c - 1) 2 Take a common from C1 and R1 both, we get


1 bk ck
a2 b2 c2 2
D = a bk 1 cos A
Þ4 a b c =0
ck cos A 1
(a - 1)2 (b - 1)2 (c - 1)2
1 sin B sin C
(taking R2 ® R2 - R3 )
= a 2 sin B 1 cos A
a2 b2 c2 sin C cos A 1
Þ a b c =0
Operate
1 - 2a 1 - 2b 1 - 2c
C2 ® C2 - C1 sin B and C3 ® C3 - C1 sin C
(u sin g R3 ® R3 - R1 )
Solutions 147

1 0 0 p
Where 0 < a , b, q <
2 2 4
D = a sin B 1 - sin B cos A - sin B sin C
sin C cos A - sin B sin C 1 - sin 2 C 1 1 1
\ D = - sin(b - a ) cos(b - a ) cos(b - a + q)
= a 2 [(1 - sin 2 C - sin 2 B + sin 2 B sin 2 C )
- cos b sin b sin(b - q)
- (cos A - sin B sin C ) 2 ]
Operating C3 - C1 sin q - C2 cos q , we get
2 2 2 2
= a [sin A - sin B - sin C + 2sin B sin C cos A] 1 1 1 - sin q - cos q
The above expression does not represent area D = - sin(b - a ) cos(b - a ) 0
or perimeter of the triangle.
- cos b sin b 0
1 1 + sin P sin P (1 + sin P) = (1 - sin q - cos q)[cos b cos(b - a) - sin b sin(b - a)]
4. (d) D = 1 1 + sin Q sin Q (1 + sin Q) Þ D = [1 - (sin q + cos q)] cos(2b - a)
1 1 + sin R sin R (1 + sin R)
p
Q 0 < a , b, q < \ sin q + cos q ¹ 1
1 sin P sin P + sin 2 P 4
p
= 1 sin Q sin Q + sin 2 Q (C2 ® C2 – C1) Also 2b - a < Þ cos(2b - a ) ¹ 0
4
1 sin R sin R + sin 2 R \ D ¹ 0 Þ the three points are non-collinear..
n -1
1 sin P sin 2 P 6. (d) å r = 1 + 2 + 3 + ... + (n – 1) = n(n - 1)
r =1 2
= 1 sin Q sin 2 Q (C3 ® C3 – C2)
n -1
1 sin R sin 2 R å (2r - 1) = 1 + 3 + 5 + ... + [2 (n – 1) – 2]
r =1
= (n – 1)2
1 sin P sin 2 P
n -1
0 sin Q - sin P sin 2 Q - sin 2 P å (3r - 2) = 1 + 4 + 7 + .. + (3n – 3 – 2)
=
2 2 r =1
0 sin R - sin P sin R - sin P
(n - 1)(3n - 4)
(R2 ® R2– R1, R3 ® R3 – R1) =
= (sinQ – sinP) (sinR – sinP) 2
n -1
1 sin P sin 2 P \ å Dr
r =1
0 1 sin Q + sin P
0 1 sin R + sin P
Sr S (2r - 1) S (3r - 2)
= (sinQ – sinP) (sinR – sinP) (sinR – sinQ)
n
D > 0 if P < Q < R = n -1 a
Now 2
D < 0 if P > Q > R n(n - 1) (n - 1)(3n - 4)
(n - 1)2
Hence the sign of D cannot be determined. 2 2
5. (d) The given points are
n -1
P ( - sin(b - a), – cos b ) , Q (cos(b - a ),sin b) å D r consists of (n – 1) determinants in
r =1
R (cos(b - a + q),sin(b - q)) L.H.S. and in R.H.S every constituent of first
148 MATHEMATICS
row consists of (n – 1) elements and hence it f "( x ) g "( x ) h "( x )
can be splitted into sum of (n – 1) determinants.
a b c = 12mx2 + 6nx + 2r ...(2)
n -1
\ å Dr p q r
r =1 Again differentiating w.r.t.x, we get
n(n - 1) (n - 1)(3n - 4) f "'( x ) g "'( x ) h "'( x )
(n - 1)2
2 2 a b c
= 24mx + 6n ... (3)
n p q r
= n -1 a =0
2 putting x = 0 in (2), we get
n(n - 1) (n - 1)(3n - 4) f "(0) g "(0) h "(0)
(n - 1)2
2 2 2r = a b c ...(4)
(Q R1 and R3 are identical) p q r
n -1 Putting x = 0 in (3), we get
Hence, value of å Dr is independent of
f "'(0) g "'(0) h "'(0)
r= 1
both 'a' and 'n'. 6n = a b c ...(5)
7. (a) We have a2 + b2 + c2 + ab + bc + ca £ 0 p q r
\ (a + b)2 + (b + c)2 + (c + a)2 £ 0 From (5) and (4), we get
\ a + b = 0, b + c = 0 and c + a = 0
\ a=b=c=0 f "'(0) – f "(0) g "'(0) – g "(0) h "'(0) – h "(0)
a b c
2 2 2
(a + b + 2) a +b 1 p q r
2 2 2 = 2(3n – r)
Þ 1 (b + c + 2) b +c
10. (b) We have
c2 + a 2 1 (c + a + 2)2
1 1 1
4 0 1
a b c = (a – b)(b – c)(c – a)(a + b + c)
= 1 4 0 = 65 3 3
a b c3
0 1 4 ...(1)
1 1 1
1 1 1 1 1
8. (b) |A | = –1
= a b c
|A| 5
Also, a b c = abc 1 1 1
| (AB)T | = |AB| = |A×(adj A)| = |A|×|adj (A)|
= 5 × 52 =53 a 3 b 3 c3 a 2 b2 c2

1 1 (taking a, b, c common from R1, R2, R3)


\ ||A– 1|(AB)T| = | (AB)T| = 3 |AB| = 1
5 5
bc ac ab
9. (a) Differentiating given equation w.r.t.x, we
get = 1 1 1 (Multiplying R1 by abc)
2 2 2
a b c
f '( x) g '( x ) h '( x)
a b c 1 1 1
= 4mx3 + 3nx2 + 2rx + 5
p q r = a 2 b2 c 2 then,
...(1) bc ac ab
Again differentiating w.r.t.x, we get
Solutions 149

1 1 1 Operate R2 ® R2 - R1 ; R3 ® R3 - R1 , then

D= ( x – a)2 ( x – b) 2 ( x – c) 2 a+a a a
( x – b)( x – c) ( x – c)( x – a) ( x – a)( x – b) -a b 0 =0
-a 0 c
= (a – b)(b – c)(c – a)(3x – a – b – c)
Now given that a, b, c are all different, then Þ aab + c(ab + ab + aa ) = 0 since a, b, c ¹ 0
1 Þ a (bc + ca + ab) + abc = 0
D = 0. Therefore, x = (a + b + c)
3
1 æ 1 1 1ö
t 2 - 3t + 4 \ = -ç + + ÷
11. (a) l= a è a b cø
t 2 + 3t + 4
a 1 1
Þ (l - 1)t 2 + 3(l + 1)t + 4(l - 1) = 0
14. (c) 1 b 1 =0
Since, t is real
1 1 c
Þ 9(l + 1) 2 - 16(l - 1)2 ³ 0
Þ (3l + 3 - 4l + 4)(3l + 3 + 4l - 4) ³ 0 a –1 0 1–c
Þ 0 b – 1 1 – c = 0,
1
Þ ( 7 - l )(7l - 1) ³ 0 Þ £l£7 1 1 c
7
R1 ® R1 – R3, R2 ® R2 – R3
3 -1 4
Þ (1 – a) (1 – b) (1 – c)
Now, D = 1 2 -3
6 5 l
–1 0 +1
[Determinant of coefficients of equations]
0 –1 1 = 0,
é 1 ù
= 7 (l + 5) ¹ 0 êQ 7 £ l £ 7 ú 1 1 c
ë û
1– a 1– b 1– c
Hence the given system of equations has a
unique solution.
æ 1 ö æ 1 ö
12. (c) Determinant of coefficients C1 ® ç ÷ C1 , C 2 ® ç ÷ C2 ,
è1 – a ø è1 – b ø
t t +1 t -1 t 1 -1
= t +1 t t + 2 = t + 1 -1 1 æ 1 ö
C3 ® ç ÷ C3
t -1 t + 2 t t -1 3 1 è1 – c ø
Þ (1 – a) (1 – b) (1 – c)
t 1 -1
= 2t + 1 0 0 ì 1 1 c ü
= -4(2t + 1) í (0 + 1) – (–1 + 0) + (1– 0)ý = 0
2t - 1 4 0 î (1– a) (1– b) 1– c þ
1 1 1 c
For non-trivial solution t = -
. Þ + + = 0,
2 1– a 1– b 1– c
13. (a) The given system of equations will have as a ¹ 1, b ¹ 1, c ¹ 1 (given)
a non-trivial solution if,
1 1 c – 1+ 1
a+a a a Þ + + =0
1– a 1– b 1– c
a a+b a =0 1 1 1
Þ + + =1
a a a+c 1– a 1– b 1– c
150 MATHEMATICS
15. (a) The given system of equations will have a
non-trivial solution if a1 + pb1 b1 + qc1 c1 + ra1
a+a a a 18. (d) D¢ = a2 + pb2 b2 + qc2 c2 + ra2
a a+b a =0 a3 + pb3 b3 + qc3 c3 + ra3
a a a+c
a1 b1 + qc1 c1 + ra1
Operating R2 ® R2 – R1 and R3 ® R3 – R1, we get
= a2 b2 + qc2 c2 + ra2
a+a a a
a3 b3 + qc3 c3 + ra3
–a 0 =0b
–a 0 c
pb1 b1 + qc1 c1 + ra1
or aab + c(ab + ab + aa) = 0
or a(bc + ca + ab) + abc = 0 + pb2 b2 + qc2 c2 + ra2
1 æ 1 1 1ö pb3 b3 + qc3 c3 + ra3
or = –ç + + ÷ (Q a, b, c ¹ 0)
a è a b cø In the first determinant, apply C3 ® C3 – rC1 and
16. (c) The given system is consistent. then C2 ® C2 – qC3.
Therefore, In second determinant take p common from C1
1 1 –1 and then apply
C2 ® C2 – C1. Then take q common from C2 and
D= 2 –1 –c =0
apply
– b 3b – c C3 ® C3 – C2. Finally taking r common from C3,
or c + bc – 6b + b + 2c + 3bc = 0 we have ultimately D¢ = (1 + pqr)D.
5b 19. (d) det{Adj(Adj A)} = (14)4 = |A|(n – 1)2
or 3c + 4bc – 5b = 0 or c= Þ |A| = 14 = 3x + 11 Þ x = 1
4b + 3
Now, c < 1 20. (c) A(Adj A) = |A|In;
5b 5b |A| = xyz – (8x + 4y + 3z) + 28
Þ < 1 or –1 <0
4b + 3 4b + 3 é 68 0 0 ù
b–3 æ 3 ö ê ú
or < 0 Þ b Î ç – ,3÷ = 60 – 20 + 28 = 68 Þ A(Adj A) = ê 0 68 0 ú
4b + 3 è 4 ø
êë 0 0 68úû
17. (c) We have,
æ np ö (-1)n n !
0 cos ç x + ÷ 21. (0) Apply C3 ® C3 – C1
è 2 ø ( x + 3)n +1
dn np (-1)n n ! æ 3pö æ 5p ö
[ f ( x ) ] = 0 cos sin2 ç x + ÷ sin2 ç x + ÷
è 2ø è 2ø
sin(2x + 5p)sin(2p)
dx n 2 3n +1
a a 3
a5 æ 3p ö æ 5pö æ 5pö
Þ sin çè x + ÷ø sin ç x + ÷
è
2cos ç x + ÷ sin(p) = 0
2 2ø è 2ø
æ 3pö æ 5pö æ 5pö
sin ç x - ÷ sin ç x - ÷ 2cos ç x - ÷ sin(-p)
è 2ø è 2ø è 2ø
np ( -1)n n !
0 cos Q All elements of C3 are zero.
2 3n +1 22. (20) The matrices in the form
dn np ( -1)n n !
\ [ f ( x) ]x = 0 = 0 cos =0 é a11 a12 ù
dx n 2 3n +1 êë a21 a22 úû , aij Î {0, 1, 2}, a11 = a12 are
a a3 a5
é 0 0/1/ 2ù é 1 0/1/ 2ù é 2 0/1/ 2ù
(Q R1 and R2 are identical) êë0/1/ 2 0 úû , êë0/1/ 2 1 úû , êë0/1/ 2 2 úû
Solutions 151
At any place, 0/1/2 means 0, 1 or 2 will be the
element at that place. x sin x cos x x sin x cos x
2 2 2
Hence there are total 27 = 3 × 3 + 3 × 3 + 3 × 3) + 2 x - sec x -3x + x - tan x - x3
matrices of the above form. Out of which the
matrices which are singular are 2 x sin 2 x 5x 2 2cos 2 x 5

é 0 0 / 1/ 2 ù é 0 0 ù é1 1ù é 2 2ù Operate
êë 0 , , ,
0 úû êë1/ 2 0 úû êë1 1úû êë 2 2úû 1 1 1
R2 ® R2 , R3 ® R3 , R2 ® R2 ,
Hence there are total 7(= 3 + 2 + 1 + 1) singular x x x
matrices. respectively on three determinants
Therefore number of all non-singular matrices in
the given form = 27 – 7 = 20 1 cos x - sin x
f '( x) tan x
é3 0 0ù = x - - x2
ê ú x x
23. (0.20) A = ê0 3 0ú
2 x sin 2 x 5x
êë0 0 3úû
det (adj(adj(A))) = |A|4 = 274 sin x cos x x
x sin x cos x
ìï 27 4 üï 1 tan x
í ý = = 0.20 + 2 x - sec 2 x -3x 2 + x - - x2
îï 5 þï 5 sin 2 x
x
24. (0) 2 5 2 2cos 2 x 5
x
log a + (n –1)log r log a + (n +1)log r log a + (n + 3)log r
1 1 0 0 0 1
log a + (n + 5)log r log a + (n + 7)log log a + (n + 9)log r f '( x )
Lt = 0 -1 0 + 0 -1 0
log a + (n +11)log r log a + (n +13)log r log a + (n +15log r x ®0 x
0 0 0 2 2 5
D = C3 ® C3 – C2 C2 ® C2 – C1
log a + (n –1) log r 2 log r 2log r 0 0 1
Þ log a + (n + 5) log r 2log r 2 log r = 0 + 0 -1 0
log a + (n + 11) log r 2 log r 2log r 2 2 5
= 0 + 2(0 + 1) + 2(0 + 1) = 4
(1 + ap)2 (1 + bp)2 (1+ cp)2
27. (0) The two roots of the equation are 1 ± i 3 ,
25. (16) (1 + aq)2 (1 + bq)2 (1+ cq)2
so that we can take
(1 + ar )2 (1 + br )2 (1 + cr )2 æ1 3ö
a = 1 + i 3 = 2 çç 2 + i 2 ÷÷ = 2eip/3 and
1 2a a 2 1 p p2 è ø
= 1 2b b2 ´ 1 q q 2 = 2 × 2D1 × 2D2 æ1 3ö
b = 1 – i 3 = 2 çç 2 - i 2 ÷÷ = 2e–ip/3
1 2c c 2 1 r r2 è ø

= 8D1D2 = 8 ×
1
× 4 = 16 a + b = 2, a2 + b2 = 22 e
i2 p / 3
(
+ e-i2p / 3 )
2
2p
= 22 . 2 cos = – 22 ,
1 cos x - sin x 3
26. (4) 2
f '( x) = x - tan x - x3 (ip
a3 + b3 = 23 e + e
-ip
= – 24 , )
2 x sin 2 x 5x
a4 + b4 = 24 ( e )
i4 p / 3
+ e-i4 p / 3 = – 24
152 MATHEMATICS
29. (1) We have,
and a5 + b5 = 25 e (
i5 p / 3
+ e-i5p / 3 = 25 ) éa b ù
A= ê ú
2 -22 -2 4 ëc d û
éa b ù éa b ù
Determinant = -22 -24 -2 4 Þ A2 = ê úê ú
ëc d û ëc d û
-2 4 -24 25
é a 2 + bc ab + bd ù
1 -2 -2 2 =ê ú
1 -2 -22 êë ac + cd bc + d 2 úû
= – 28 1 2
2
2 = – 28 0 6 6 = 0. Given, A2 = A and ad – bc = 0
-1 -1 1 0 -3 -3 é a 2 + bc ab + bd ù é a b ù
\ ê ú=ê ú
êë ac + cd bc + d 2 úû ë c d û
(1 + x) 2 (1 + x )4 (1 + x)6 Þ ab + bd = b Þ b(a + d) = b
3 Þ a+d=1
28. (0) Let f (x) = (1 + x ) (1+ x)6 (1 + x)9
1 x x2
(1 + x) 4 (1 + x )8 (1 + x)12
2
x4
Coefficient of 'x' is f ' (0) 30. (2) –3x4 + 1 x =0
1 x3 x6
2(1 + x )2 4(1 + x )3 6(1 + x)5
3 1 1 1
f ' (x) = (1+ x ) (1 + x)6 (1 + x )9
1 x x2
(1 + x )4 (1 + x)8 (1 + x )12 Þ –3x4 + x3 =0
1 x2 x4
(1 + x )2 (1 + x )4 (1 + x )6 Now, apply R1 ® R1 – R2, R2 ® R2 – R3
+ 3(1 + x)3 6(1 + x )5 9(1 + x)8
0 1- x 1 - x2
(1 + x )4 (1 + x )8 (1 + x )12
2
x 2 - x4
Þ –3x4 + x3 0 x - x =0
(1 + x )2 (1 + x)2 (1+ x)6 1 x2 x4
+ (1 + x )3 (1 + x)6 (1 + x )9
Þ –3x4 + x3[x2(1 – x2)(1 – x)
4(1+ x)3 8(1 + x )7 12(1 + x )11
– x(1 – x)(1 – x2)] = 0
Put x = 0, f ¢(0) = 0 Þ x4[– 3 + (1 – x) (1 – x2) (x – 1)] = 0
x = 0, x = 2
Solutions 153
CHAPTER
20 Continuity and Differentiability

1. (b) f is continuous at x = p / 4 , 3. (d) If f (a) = a, then obviously x = a is the


solution.
if lim f ( x ) = f (p / 4) . Let f (a) > a and g(x) = f (x) – x then g (a) > 0 and
x ®p / 4
g( f (a)) = f ( f (a)) – f (a) = a – f (a) < 0
2
Now, L = lim (sin 2x ) tan 2x Since g(x) is continuous, so at least for one
x ®p / 4 c Î(a, f (a)) , g(c) = 0.
Þ log L = lim tan 2 2x log sin 2x Similarly we can argue for f (a) < a.
x ®p / 4
lim sin x ln x
sin x +
log sin 2 x æ ¥ ö 4. (b) f (0+ ) = lim x = e x®0
= lim ç ÷ x ®0 +
x ®p /4 cot 2 2 x è ¥ ø
ln x 1x
2 cot 2x 1 lim lim
= lim =- =e x ® 0+ cosec x =e x ® 0+ - cosec x cot x
2
x ®p / 4 - 2 cot 2x cos ec 2x.2 2
æ sin x ö
or L = e -1 / 2 - lim ç
+ è x ÷ø
tan x
=e x®0 = e -1´ 0 = 1
\ f (p / 4) = e -1 / 2 = 1 / e
\ f (0- ) = g (0) = 1
ì1 - [ x] Let g (x) = ax + b Þ b = 1 Þ g ( x ) = ax + 1
ï , x ¹ -1
2. (d) f (x) = í 1 + x and For
ïî1 , x = -1
é sin x ù
x > 0, f '( x ) = esin x ln(| x|) êcos x ln(| x |) +
ì1
ï
,x < 0 ë x úû
f (x) = í1 - x
,x ³ 0 f '(1) = 1(0 + sin1) = sin1
îï1 + x
f (-1) = - a + 1 Þ a = 1 - sin1
ì1 ,x < 0
ï g ( x) = (1 - sin1) x + 1.
f (2x) = í1 - [ 2 x ] , x > 0
ï1 + [ 2 x ] 5. (b) We apply check for continuity at x = 0
î
LHL = lim f ( x) = lim f (0 - h)
x ® 0- h® 0
ì1 ,x< 0
ï 1 ¥
= lim (cos h + sin h) -cosec h ( 1 from )
ï1 ,0 £ x < h ®0
ï 2
Þ f ( 2x) = í 1 = exp{ lim (cos h + sin h - 1) ´ - cosec h}
ï0 , £ x £1 h® 0
2
ï 1 3
ï- ,1 £ x < ì æ 2h h hö
î 3 2 = exp í lim ç -2sin 2 + 2sin 2 cos 2 ÷
1 î h ® 0 è ø
Þ f (x), for all values of x where x <
2 1 ü
1 ´- ï
continuous function and for x = and x = 1 h hý
2 2sin cos ï
2 2þ
f (x) be a discontinuous function.
154 MATHEMATICS
7. (a) f (x) = min {x + 1, | x | + 1}
ì æ h höü Þ f (x) = x + 1 " x Î R
sin - cos ï
ïï ç 2 2 ÷ ï = e-1
= exp í lim ç ÷ý Y
ïh®0 ç h
cos ֕
îï è 2 ø ïþ
y=–x+1 y=x+1
RHL = lim+ f ( x ) = lim f (0 + h)
x ®0 h ®0
(0, 1)
1/ h
e + e 2/ h + e3/ h
= lim
h ®0 ae 2/ h + be3/ h X' X
(– 1, 0)
3/ h -2 / h -1/ h
e (e +e + 1) 1
= lim =
h® 0 e3/ h
{ae -1/ h
+ b} b Y'
Hence, f (x) is differentiable everywhere for all
é ù x Î R.
êëQhlim e-1/ h = 0ú 8. (c) As per question,
®0 û
p = left hand derivative of |x –1| at x = 1
\ For continuity at x = 0 Þ p = –1
1 Also lim g ( x ) = p
e -1 = a = b -1 Þ a = , b = e x ®1+
e
ìï (1 + sin px )t - 1 üï ( x - 1) n
6. (b) f (x) = lim í ý
Where g (x) = , 0 < x < 2,
t ®¥ ï (1 + sin px )t - 1 ï log cos m ( x - 1)
î þ m, n are integers, m ¹ 0, n > 0
Q sin p x > 0 (in I and II quadrants) \ we get,
\ 2n p < p x < (2n + 1) p
Þ 2n < x < 2n + 1, n Î I ( x - 1) n
lim = -1
and sin px < 0 (in III and IV quadrrants) x ®1+ log cos m ( x - 1)
\ (2n + 1) < p x < (2n + 2) p
Þ 2n + 1 < x < 2n + 2, n Î I and hn
Þ lim = -1
sin p x = 0 if x = 0, 1, 2,......... h® 0 log cos m h

ì
1-
1 hn
ï t Þ lim = -1
ï lim (1- sin px) , 2n < x < 2n +1 h®0 m(log cos h)
ït®¥ 1 [Using L' Hospital's rule]
ï 1+
f (x) = í (1+ sin px)t n h n-1 cos h
ï Þ lim = -1
(1+ sin px)t + 1 2n +1 < x < 2n + 2 h®0 m( - sin h)
ï lim ,
ï t®¥ (1+ sin px)t + 1 [Using L' Hospital's rule]
ï x = 0,1,2,.....
î 0, n hn - 2 cos h
Þ lim = 1 Þ n = 2 and m = 2
m æç
h® 0 sin h ö
ì 1, 2n < x < 2n + 1 è h ø ÷
ï
= í -1 2n + 1 < x < 2n + 2 9. (b) Let | f (x) | £ x2, "x Î R
ï 0, x = 0,1, 2..........
î Now, at x = 0, | f (0) | £ 0
Þ f (0) = 0
If k Î I, f (k) = 0, but lim f ( x) = 1 or – 1
x® k f (h ) - f (0) f (h)
\ f ¢ (0) = lim = lim ...(1)
according as h® 0 h-0 h® 0 h
k Î (2n, 2n + 1) or k Î (2n + 1, 2n + 2)
Solutions 155

f (h) 14. (d) Let x3 = cos p and y3 = cos q


£|h| 2
Now, (Q | f ( x ) | £ x ) Given 3 3
h (1 - x 6 ) + (1 - y 6 ) = a (x – y )

f (h) f (h) Þ (1 - cos 2 p ) + (1 - cos 2 q )


Þ - | h |£ £ | h | Þ lim ® 0 ...(2)
h h® 0 h = a(cos p – cos q)
(using sandwich Theorem) Þ sin p + sin q = a(cos p–cos q)
\ from (1) and (2), we get f ¢ (0) = 0,
i.e. - f (x) is differentiable, at x = 0 æ p + qö æ p - qö
Þ 2sin ç cos ç
Since, differentiability Þ Continutity è 2 ÷ø è 2 ÷ø
\ | f (x) | £ x2, for all x Î R is continuous as p - qö æ p + qö
well as differentiable at x = 0. = –2a sin æç ÷ sin ç ÷
è 2 ø è 2 ø
æ xx – x– x ö æ ( x) 2 x – 1 ö
10. (a) f ( x) = cot –1 ç ÷ = cot –1 ç ÷ æ p - qö 1 æ 1ö
Þ tan ç =- Þ p – q = tan–1 çè - ÷ø
ç
è 2 ÷
ø
ç 2xx ÷
è ø è 2 ÷ø a a

æ 2x x ö æ 1ö
= tan –1 çç –1 x
÷÷ = –2 tan x Þ cos–1x3 – cos–1 y3 = tan–1 çè - a ÷ø
x 2
è –1 + ( x ) ø Differentiate w.r.t. x, we have
2
\ f ¢( x ) = – ´ x x (1 + log e x ) 3x2 3 y2 dy dy x 2 1 - y 6
1 + ( x x )2 - + =0 Þ = .
1 - x6 1 - y 6 dx dx y 2 1 - x 6
–2
\ f ¢(1) = ´ 1 = –1 Hence, f (x, y) = x2 / y2
2
15. (b) f ¢ (x) = n (1 + x)n –1, f ¢¢ (x) = n (n – 1) (1 + x)n – 2
11. (c) We have, g = inverse of f = f –1
\ f n (x) = n!, f n (0) = n!
Þ g(x) = f –1 (x) Þ f [g(x)] = x
Differentiating w.r.t. x, we get f ' [g (x)]. g ' (x) = 1 n (n –1) n!
Þ 1+ n + + ... +
1 2! n!
\ g ' (x) = = 1 + [g (x)]3 = nC0 + nC1 + nC2 + ... + nCn = 2n
f '[ g (x)]
16. (c) Given ea cos a = 1 ....(i)
é 1 1 ù
êQ f ¢ ( x) = \ f ¢[ g ( x)] = b

3
and e cos b = 1 .....(ii)
êë 1+ x 1 + [ g ( x)] úû –x
Let f(x) = e – cos x, then f(x) is continuous and
12. (b) Since, 2 < x < 3, \ [x] = 2 differentiable.
æ 2p ö Also, f(a) = f(b) = 0 (from (i) and (ii))
\ f ( x) = sin ç - x 2 ÷ Therefore by Rolle's MVT, f '(x) = 0 has at least
è 3 ø
one root in (a, b).
\ f ¢( x) = cos çæ 2p - x 2 ÷ö (-2 x) Þ - e - x + sin x = 0 for at least one x Î ( a, b)
è 3 ø
17. (b) We have, f(x) = x|sin x|, x Î R
æ 5p ö 5p ì x sin x, x Î(2 np,(2 n + 1) p)
\ f ¢ç =2
ç 3 ÷÷ 3 f ( x) = í
è ø î – x sin x, x Î(2n + 1) p , 2pn)
13. (c) Put x n = cos a , y n = cos b f ( x ) – f ( np )
f '(np ) = lim
a -b x ®nx x – np
Þ tan = -a
2 x | sin x |
Þ cos–1(xn) – cos–1( yn) = 2 tan–1(– a) = lim
x – np
x ® nx
y n -1 dy x n -1 x n -1 Clearly, f(x) is differentiable for all x except
Þ = = n -1
1 - y 2n dx 1 - x2n y x = np, n = ±1, ±2, ±3,....
156 MATHEMATICS

18. (c) We have,


sin x2
ì 20. (d) lim f ( x ) = lim
æ pö for x ¹ 0 x ®0 x2
f ( x ) = í x 2 cos ç ÷ x®0
î è xø
æ sin x 2 ö
0 for x = 0 = xlim ç
®0 è x 2 ø
÷ x = (1)(0) = 0 = f (0)
For x = 0, f (0) = 0
2 p Þ f (x) is continuous
and lim x cos =0.
x®0 n Now, to check the differentiability of f (x)
Þ f (x) is continuous at x = 0 R.H.D.
f (0 – h) – 0 sin h 2
f (0 + h) - 0 -0
Now Lf ¢ (0) = lim = lim = lim h =1
x ®0 –h h® 0 h h®0 h
æ pö
h 2 cos ç ÷ – 0
è hø p sin h2
= lim = lim – h cos = 0 . f (0 - h ) - 0 -0
x ®0 –h x®0 h L.H.D. = lim = lim -h =1
h® 0 -h h® 0 -h
Simillarly, R f ¢ (0) = 0
Þ differentiable at x = 0. R.H.D. = L.H.D.
p Þ f (x) is differentiable
Now, f (x) is not differentiable at cos =0
x Now, differentiate f (x) w.r.t. x, we get
p p p x cos( x 2 )2 x - sin( x 2 )
cos = 0 Þ = (2 n +1)
x x 2 f ¢(x) =
x2
2
Þ x= 2 sin x 2
2 x +1 f ¢(x) = 2cos x -
x2
Q f (x) is not differentiable at
ì 2 sin x
2
2 ï 2cos x - 2 , x¹ 0
x Î[–1,1]: x = , n ÎZ f ¢(x) = í x
2n + 1
ï 1 , x =0
19. (d) Let f : R ® R be a continuous function î
such that f (x2) = f (x3) ...(i) for all x Î R Now, to check the continuity of f ¢(x)
Put x = –x, in eqn. (i), then f (x2) = f (–x3) 2
from (i) f (x3) = f (–x3) lim f ¢ ( x ) = lim 2cos x 2 - sin x = 2 – 1 = 1
x®0
Let x3 = t, then f (t) = f(–t) x ®0 x2
Þ f (x) is an even function Þ f ¢(x) is continuous
Now take x3 = t, then from (i)
f (t2/3) = f (t)
Put t = t2/3 2 f ( x + h ) - f ( x)
2/3 21. (0) f '(x) = lim
Þ f (t (2/3) ) = f (t ) h®0 h
(2/3) 2
Þ f (t) = f (t 2/3 ) = f (t )
3 n f ( x + h) - f ( x) (h)2
= f (t (2/3) ) ... = f (t (2/3) ) | f '( x) | = lim £ lim
This is true for all t Î R and any n Î I. h®0 h h®0 h
n
æ 2ö Þ | f '( x) | £ 0
When n ® ¥, ç ÷ ® 0
è 3ø
Þ f '( x) = 0
0
Then f (t) = f (t ) = 1
Hence, f (x) is a constant function and Þ f (x) = constant
therefore it is differentiable everywhere As f (0) = 0 Þ f (1) = 0.
Solutions 157

22. (4) Q x = cosec q - sin q é æ xö æ xö æ x ö æ x öù


24. (5) F ¢( x) = ê f ç ÷ . f ¢ ç ÷ + g ç ÷ g ¢ ç ÷ ú
Þ 2 2 2
ë è ø è ø 2 è 2 ø è 2 øû
x + 4 = (cosec q - sin q) + 4
Here, g (x) = f ' (x)
= (cosec q + sin q)2 ....... (i) and g' (x) = f '' (x) = – f (x)
and y 2 + 4 = (cosec n q - sin n q) 2 + 4 So, F ¢( x) = f çæ ÷ö g çæ ÷ö - æ xö æ xö
x x
f ç ÷ gç ÷=0
2
è ø è ø 2 è2ø è2ø
= (cosec n q + sin n q)2 ........(ii)
Þ F (x) is constant function
æ dy ö So, F (10) = 5
ç ÷
dy è dq ø 25. (2) Given, | f (x) – f (y) | = | x – y | for all in [0, 1]
Now , =
dx æ dx ö | f '( x) – f (y) |
ç ÷ Þ =1
è dq ø | x– y |
| f (x) – f (y) |
n(cosecn-1q)(-cosec q cot q) - n sin n-1 q cos q Þ lim = 1Þ | f '(x) | =1
= x® y x– y
-cosec q cot q - cos q
Þ f '( x) = ±1Þ f ( x ) = ± x xÎ[0,1]
n n -1
n(cosec q cot q + sin q cos q) \ f (x) has exactly 2 function.
=
(cosec q cot q + cos q) 26. (3) Taking logarithm of both sides, we get
n cot q (cosecn q + sin n q) é æ 1 öù
= log y = x êlog ç1 + ÷ ú
cot q (cosec q + sin q) ë è x øû
2
n(cosecn q + sin n q) n y + 4 1 x2 æ 1 ö æ 1ö
= = Þ y1 ( x ) = ç- ÷ + log ç 1 + ÷
(cosec q + sin q) y x + 1 è x2 ø è xø
x2 + 4
[From (i) and (ii)] 1 æ 1ö
Squaring both side, we get =- + log ç 1 + ÷ ......... (1)
x +1 è xø
2
n 2 ( y 2 + 4) Since y (2) = (1 + 1/2)2 = 9/4
æ dy ö
çè ÷ø =
dx ( x 2 + 4) æ 1 3ö
So, y1 (2) = (9/4) çè - + log ÷ø
3 2
2
æ dy ö Multiplying (1) by y and then differentiating, we
or ( x 2 + 4) ç ÷ - n 2 y 2 = 4n 2 = kn 2
è dx ø get
Þ k = 4. æ 1 æ 1 öö
y2 ( x ) = y1 ( x ) ç - + log ç 1 + ÷ ÷
23. (1) We have, è x +1 è x øø
f (x) = cos x cos 2x cos22 x cos 23 x .....
cos 2n–1x æ 1 x æ 1 öö
n
sin 2 x
+ y ( x ). ç + ç- ÷÷
Þ f ( x) = è ( x + 1)2 x + 1 è x 2 ø ø
2n sin x
So,
2n cos 2n x sin x - sin 2n x cos x
Þ f '( x) = æ 1 3ö æ1 1ö
2n sin 2 x y2 (2) = y1 (2) ç - + log ÷ + y(2) ç - ÷
n n-1 è 3 2ø è9 6ø
æ p ö 2 cos 2 p
Þ f 'ç ÷ =
è 2ø
= cos 2n–1 p 2
2n æ 9ö æ 1 3ö 1
= ç ÷ ç - + log ÷ -
= (–1) 2 n – 1 = 1 è 4ø è 3 2ø 8
158 MATHEMATICS
27. (3) Here,
ì 1 1
ï 3 - cos x - , | sin x |<
y= ( x - sin x ) + ( x - sin x) + ...¥ ) ï
29. (4) f`(x) = í
2 2
ï 2 + cos x + 1 1
, | sin x |³
So, y = ( x - sin x) + y \ y = x – sin x + y
2 ïî 2 2
dy dy ì 1 1
Differentiating, we get : 2 y
dx
= 1 - cos x +
dx ï 3 - cos x - , | cos x |>
ï 2 2

p 2 p 1 p 3 ï 2 + cos x + 1 1
At x = , y - y = -1 Þ y2 - y + = - , | cos x |£
2 2 4 2 4 ïî 2 2

æ 1ö 2p - 3
2 ì 1 1
Þ çy- ÷ = Þ (2 y - 1) = ± 2p - 3 ïï 3 - cos x + 2 , | cos x |> 2
è 2 ø 4 =í
dy ï 2 + cos x + 1 , | cos x |£ 1
From Eq. (i), we get (2 y - 1) = 1 - cos x ïî 2 2
dx
Thus, f (x) is discontinuous at
dy 2 1 p 3p 5 p 7 p
\ = dx | cos x |= or x = , , ,
dx p 2p - 3 Þ - 2p = 3 2 4 4 4 4
x=
2
dy x=
p
2 2 3 3
30. (3) Put cos f = ;sin f = ; tan f =
1 13 13 2
lim f ( x) = x × a cot x + b 1
28. (2) Þ lim = -1 -1
x ®0 3 x®0 x2 3 y = cos {cos( x + f)} + sin {cos( x - f)}
-1 p
xa + b tan x 1 = cos {cos( x + f)} + - cos -1{cos(f - x)}
Þ lim = 2
x ® 0 x 2 tan x 3 p
= x+f+ -f+ x
æ x3 ö 2
ax + b ç x + + ...¥ ÷
è 3 ø 1 p
Þ lim = y = 2 x + ; z = 1 + x2
x®0 3 æ tan x ö 3 2
x ç
è x ÷ø dy
æb ö dy dx 2 1 + x 2
(a + b) x + ç ÷ x3 + ...¥ Now, = =
è3ø 1 dz dz x
Þ lim 3
= dx
x®0 x 3
So a + b = 0 æ dy ö 10
\ç ÷ =
Also, b = 1 Þ a = –1 Þ a2 + b2 = 2 è dz ø x= 3 3
4
Solutions 159
CHAPTER
21 Application of Derivatives

1. (d) Given f (x) = tan –1 (sin x + cos x)


i.e. 3( x 2 - 6 x + 8)e < 1 i.e. x2 – 6x + (8 – 1/3e) < 0
1
f '(x) = .(cos x - sin x )
1 + (sin x + cos x) 2 i.e. ( x - (3 + 1 + 1/ 3e ))( x - (3 - 1 + 1/ 3e ) < 0
æ 1 1 ö Û 3 - 1 + 1/ 3e < x < (3 + 1 + 1/ 3e )
2. ç cos x - sin x÷
è 2 2 ø Hence x Î (4, 3 + 1 + 1/ 3e )
=
2 Similarly if x < 2, then f ' (x) < 0,
1 + (sin x + cos x )
p p If log 3(x2 – 6x + 8) e > 0
æ ö
çè cos .cos x - sin .sin x÷ø i.e., x < 3 - 1 + 1/ 3e or x > 3 + 1 + 1/ 3e
4 4
=
1 + (sin x + cos x) 2 Hence x Î (3 - 1 + 1/ 3e , 2)
æ pö 4. (a) Here f ¢ ( x) > 0"x Î R
2 cos ç x + ÷
è 4ø Þ 2cos 2 x - 8(a + 1)cos x
\ f '(x) =
1 + (sin x + cos x )2 -(4a2 + 8a - 14) > 0"x Î R
if f ' (x) > 0 then f (x) is increasing function.
p p p Þ 4cos2 x - 8(a + 1)cos x - (4a2 + 8a -12) > 0
Hence f (x) is increasing, if - < x + <
2 4 2 Þ cos 2 x - 2(a + 1) cos x - ( a 2 + 2a - 3) > 0
3p p
Þ - <x< Þ (cos x - a )(cos x - b) > 0
4 4
æ p pö
where a = (a + 1) + 2a 2 + 4a - 2
Hence, f (x) is increasing when n Î ç - , ÷
è 2 4ø and b = (a + 1) - 2a 2 + 4a - 2
2. (b) If f ( x ) = x1/ x , then Þ cos x - b < 0 since cos x - a < 0 "a > 0
1
f ' ( x ) = 2 é x1/ x (1 - ln x ) ù Þ (a + 1) - 2a 2 + 4a - 2 > cos x "x Î R
x ë û
f is decreasing if x > e and f is increasing if x < e. Þ (a + 1) - 2a 2 + 4a - 2 > 1
As e < 3 <4 < 5 < 6 < 7 2
\ Max {31/ 3 , 41/ 4 ,51/ 5 , 61/ 6 , 71/ 7 } = 31/ 3
Þ 2 a 2 + 4a - 2 < a Þ a + 4 a - 2 < 0
Þ a Î (-2 - 6, 6 - 2)
Also 1 < 2 < e \ Max {1, 21 / 2 } = 21 / 2
Hence a Î (0, 6 - 2) since a > 0.
But 21 / 2 = 41 / 4 < 31 / 3
x
\ the greatest number is 31 / 3 5. (c) We have f (x) = ,0 < x £ 1
sin x
3. (a) f '( x) = 2 x log 27 - 6 log 27 sin x - x cos x
Þ f ¢ ( x) =
+ (6 x - 18) log( x 2 - 6 x + 8) sin 2 x
where sin2 x is always +ve, when 0 < x £ 1 . But
(3 x 2 - 18 x + 24)(2 x - 6) to check Nr., we again let h (x) = sin x – x cos x
+
x2 - 6x + 8 Þ h ¢( x ) = x sin x > 0 for 0 < x £ 1
ë ( )
= 6 ( x - 3 ) éê log 3 + log x 2 - 6 x + 8 + 1ùú
û
Þ h (x) is increasing Þ h (0) < h (x),
when 0 < x £ 1
(
= 6 ( x - 3) log 3e x - 6 x + 8
2
) Þ 0 < sin x – x cos x, when 0 < x £ 1
Þ sin x – x cos x > 0, when 0 < x £ 1
For f (x) to be defined x 2 - 6 x + 8 > 0
Þ f ' (x) > 0, x Î (0,1] Þ f (x) is increasing on (0, 1]
Þ x < 2 or x > 4
x
If x > 4 then f '( x) < 0 if log 3( x 2 - 6 x + 8)e < 0 Again g ( x) =
tan x
160 MATHEMATICS

tan x - x sec 2 x ì
Þ g ¢( x) = , when 0 < x £ 1 ï2 x, for all x Î[1, 2)
tan 2 x ï
Here tan2 x > 0 But to check Nr. we consider Þ f ¢ ( x ) = í x, for all x Î[2,3)
p (x) = tan x – x sec2 x ï 2x
ï , for all x Î[3, 4)
p ¢ ( x ) = sec 2 x - sec2 x - x.2 sec x.sec x tan x î3
2
Þ p ¢( x ) = -2 x sec x tan x < 0 for 0 < x £ 1 Clearly, f ¢ (x) > 0 in each of the intervals
(1, 2), (2, 3) and (3, 4). So, f (x) is increasing in each
Þ p (x) is decreasing, when 0 < x £ 1 of these intervals. It is to note here that f (x) is
Þ p (0) > p (x) Þ 0 > tan x – x sec2 x not increasing on [1, 4] because values of f (x) in
\ g ¢( x) < 0 (1.5, 2) are greater than the values of f (x) in (2, 3).
Hence g (x) is decreasing when 0 < x £ 1 . p
9. (b) We have, A + B =
-1 u p 3
6. (c) Given that g(u) = 2 tan (e ) -
2 p 3 - tan A
p \B = - A Þ tan B =
\ g(–u) = 2 tan ( e - u ) -
-1 3 1 + 3 tan A
2 Let Z = tan A. tan B. Then,
-1 æ 1 ö p
= 2 tan ç u ÷ - 3 - tan A 3 tan A - tan 2 A
èe ø 2 Z = tan A. =
p ép ù p 1 + 3 tan A 1 + 3 tan A
= 2cot ( e ) - = 2 ê - tan -1 ( eu ) ú -
-1 u
2
2 ë2 û 2 3x - x
p p Þ Z= , where x = tan A
-1 u -1 u 1 + 3x
= p - 2 tan (e ) - = - 2 tan (e ) = – g(u)
2 2
\ g is an odd function. dZ ( x + 3 ) ( 3x - 1)
Þ =-
2eu dx (1 + 3x ) 2
Also g '(u) = > 0 , " u Î (– ¥, ¥)
1 + e 2u dZ 1
For max Z, =0Þ x = , - 3.
\ g is strictly increasing on (–¥, ¥) . dx 3
x2
7. (b) Consider the function f (x) = 3 in the x ¹ - 3 because A+B = p/3 which implies that
x + 200 x = tan A > 0. It can be easily checked that
interval [1, ¥).
d 2Z 1
x(400 – x3 ) < 0 for x = . Hence, Z is maximum
Since the derivative f ¢(x) = 3 2
is dx 2
3
( x + 200)
positive at 0 < x 3 400 and negative at x > 3 400 , the 1 1
for x = i.e. tan A = or A = p / 6.
function f(x) increases at 0 < x < 3 400 < 8 it follows 3 3
that the largest term in the sequence can be either a7 1
or a8. Since a7 = 49/534 > a8 = 8/89, the largest For this value of x, Z = .
3
49
term in the given sequence is a7 = . 10. (a) f '( x) = (4a - 3) + ( a - 7) cos x
543
8. (c) We have, For non-existence of critical point f '( x) ¹ 0 for
x2 + 1 any x Î R .
f ( x) = , for all x Î [1, 4)
[ x] 3 - 4 a for any x Þ 3 - 4a > 1
ì Þ cos x ¹
ï x 2 + 1, for all x Î[1, 2) a-7 a-7
ï 2 Þ | 4a - 3 | - | a - 7 |> 0
ï x +1
Þ f ( x) = í , for all x Î[2, 3) 4
ï 2 (i) a ³ 7 Þ 4a - 3 - a + 7 > 0 Þ a > - .
3
ï x2 +1
ï , for all x Î[3, 4) Hence a ³ 7
î 3
Solutions 161

3 Y
(ii) If £ a < 7 Þ 4a - 3 + a - 7 > 0
4
Þ 5a > 10 Þ a > 2 Þ 2 < a < 7 2
3
(iii) If a < Þ 3 - 4a + a - 7 > 0
4
X
4 –3 –2 O
Þ a<-
3
æ 4ö 13. (b) f '( x ) = sin x cos x(3sin x + 2l)
\ a Î ç -¥, - ÷ È (2, ¥) For maximum or minimum
è 3ø
11. (a) Let H be the height of the cone and a be 2l
f '(x) = 0 Þ sin x = 0 or cos x = 0 or sin x = -
its semi vertical angle. Suppose that x is the 3
radius of the inscribed cylinder and h be height
p p æ 2l ö
then \ Critical pointsin æç - , ö÷ arex = 0, sin -1 ç - ÷
h = QL = OL – OQ = H – x cot a è 2 2 ø è 3 ø
V= volume of cylinder = px 2 ( H - x cot a) One of these is point of minima and other is point
1 2l
Also p = p(H tan a) 2 H ...(i) of maxima, provided - 1 < - <1
3 3
dV 3 3
= p(2 Hx - 3 x 2 cot a ) Þ – <l<
dx 2 2
O But if l = 0 , then sinx = 0, which gives only one
critical points
a æ 3 3ö
x \ l Î ç - , ÷ - {0}
Q è 2 2ø
p
14. (a) For 0 < x £ ; [cos x] = 0
2
æ pù
Hence, f (x) = 1 for all çè 0, ú
L P 2û
M
æ pö
dV 2 Trivially f (x) is continuous on çè 0, ÷ø
So, = 0 Þ x = 0 , x = H tan a; 2
dx 3 This function is neither strictly increasing nor
strictly decreasing and its global maximum is 1.
d 2V 15. (c) Let R and S be the positions of men P and
= -2pH < 0 Q at any time t. Since velocities are same
dx 2 x = 2 H tan a
3
2 \ OR = OS = x (say) and given dx = v and let SR = y
So, V is maximum when x = H tan a and dt
3
4 1 S
q = Vmax = p H 2 tan 2 a H
9 3
4 p3 p tan 2 a 4 x y
= = p [ from (i)]
27 p tan 2 a 9
Hence p : q = 9 : 4 Q 45°
2 O P x
12. (a) The graph of y = 2 - x + 5 x + 6 is drawn in R
Now in triangle ORS, applying cosine rule, we get
the adjacent figure. Clearly f (x) will have maxima at
y2 = x2 + x2 – 2x . x cos 45º = 2 x 2 - x 2 2
x = –2 only if a 2 + 1 ³ 2 Þ | a | ³ 1
\ y = x (2 - 2)
162 MATHEMATICS

dy dx Dividing (1) by (2)


\ = { (2 - 2)} = v (2 - 2)
dt dt dy 3cos 2q 3(cos 2 q - sin 2 q)
Hence the required rate at which they are = q = q
dx e (sin q + cos q) e (sin q + cos q)
being separated is v 2 - 2. 3 ( cos q + sin q ) (cos q - sin q)
dy
2 2 =
c dy c dx eq ( sin q + cos q )
16. (a) Here y = Þ =-
x+a dx ( x a)2
+ dy 3(cos q - sin q)
=
( x + a) 2 dx eq
Slope of normal is Þ > 0 (for all x) dy
c2 Given tangent is parallel to x-axis then =0
dx
\ x cos a + y sin a = p is normal if, 3(cos q - sin q)
0=
cos a eq
- > 0 or cot a < 0 or cos q – sin q = 0 Þ cos q = sin q
sin a
i.e., a lies in II or IV quadrant. tan p p
So, Þ tan q = 1 Þ tan q = Þ q=
4 4
æ p ö æ 3p ö 19. (c) We have,
a Î ç 2 np + , (2n + 1)p ÷ È ç 2np + , (2n + 2)p ÷
è 2 ø è 2 ø y3 – 3xy + 2 = 0
17. (c) Any point on x2 – y2 = a2 is dy æ dy ö
(a sec q, a tan q) Þ 3y2 - 3 ç x + y÷ = 0
dx è dx ø
This point is nearest to y = 2x if the tangent at
this point is parallel to y = 2x dy y
Þ = 2
dy x dx y - x
Now, x 2 - y 2 = a 2 Þ = If the tangent is parallel to x-axis, then
dx y
dy y
æ dy ö =0Þ 2 = 0 Þ y = 0.
Þ ç ÷ = cosec q dx y -x
è dx ø(a sec q ,a tan q ) But, y = 0 does not satisfy equation (i). So, there
p is no point on the curve where tangent is parallel
cosecq = 2 Þ q = to x-axis. Therefore, H = f.
6 20. (a) We have,
æ p pö 1
Hence the point is ç a sec , a tan ÷ i.e. p(x) > x2, p(0) = 0, p”(0) =
è 6 6ø 2
æ 2a a ö Let g(x) = p(x) –x2
ç , ÷ , Clearly they lie on the line 2y = x g(x) > 0, " x ¹ 0 and g(0) = p(0) – 0 = 0
è 3 3ø Þ x = 0 should be minima.
18. (c) Given, y = 3 sin q.cos q \ g¢¢(x) should be ³ 0 at x = 0
dy Now, g¢(x) = p¢(x) – 2x
= 3[sin q(- sin q) + cos q(cos q)] g¢¢(x) = p¢¢(x) – 2
dq
dy 1 3
g¢¢(0) = p¢¢(0) – 2 = – 2 = –
= 3[cos 2 q - sin 2 q] = 3 cos 2q ...(1) 2 2
dq But g¢¢(0) ³ 0 Þ No polynimial exists.
and x = eq sin q
21. (40) Let the speed of the train be v and distance
dx to be covered be s so that total time taken is s/v
= eq cos q + sin q eq hours. Cost of fuel per hour = kv2 (k is constant)
dq
dx 3
= eq (sin q + cos q) ...(2) Also 48 = k. 162 by given condition \ k =
dq 16
Solutions 163

3 2 24. (9.22) Equation of parabola


\ Cost to fuel per hour v . y = x2 – 4 and 2y = 4 – x2
16
Other charges per hour are 300.
Total running cost,
3s 300 s y = x2 – 4
æ 3 ös D C
C = ç v 2 + 300 ÷ = v +
è 16 øv 16 v
dC 3s 300 s
= - 2 = 0 Þ v = 40
dv 16 v A B
2 4 - x2
d C 600 s y=
= > 0 \ v = 40 results in minimum 2
dv 2 v3
running cost
22. (1) Given x + y – ln (x + y) = 2 x +5 æ 4 – h2 ö
Let point B = ç h, ÷
dy 1 æ dy ö è 2 ø
Þ 1+ - ç1 + ÷ = 2
dx x + y è dx ø æ 4 – h2 ö
A = ç – h, ÷ C = (h, h2 – 4)
dy x + y + 1 è 2 ø
Þ = D = (–h, h2 – 4)
dx x + y - 1
\ Area of rectangle ABCD
dt a+b
= = ¥ when a + b = 1. æ 4 – h2 ö
dx (a,b ) a + b - 1 = AB × BC = 2h × ç – h 2 + 4÷
è 2 ø
23. (260)
dA
A Þ A = 12h – 3h3 Þ = 12 – 9h2
dh
For maxima of minima
dA
= 0 Þ 12 – 9h2 = 0
dh
120° 2 2
h= ± \ maximum at h=
O B 3 3
Let OA = x km, OB = y km, AB = R 3
(AB)2 = (OA)2 + (OB)2 – 2 (OA) (OB) cos 120° æ 2 ö æ 2 ö 24 8
\ A = 12 ç ÷ – 3 ç ÷ = –
è 3ø è 3ø 3 3
æ 1ö
R2 = x2 + y2 – 2 xy ç - ÷ = x 2 + y 2 + xy
è 2ø 16 16 3 16 ´173
= = ÞA= = 9.22
...(1) 3 3 3
R at x = 6 km, and y = 8 km 25. (2) Q P(x) is non -zero polynomial and
R = 62 + 82 + 6 ´ 8 = 2 37 P(1 + x) = P(1 – x) for all x
Now, differentiate w.r.t. x, we get
Differentiating equation (1) with respect to t P¢(1 + x) = –P¢(1 – x)
dR dx dy æ dy dx ö Put x = 0, P¢(1) = –P¢(1) Þ P¢(1) = 0
2R = 2 + 2y + ç x + y ÷ and P(1) = 0 Þ P(x) touch x-axis at x = 1
dt dt dt è dt dt ø \ P(x) = (x – 1)2 Q(x)
1 Since, m is the lergest integer such that
= [2 ´ 8 ´ 20 + 2 ´ 6 ´ 30 + (8 ´ 30 + 6 ´ 20)] (x – 1)m divides polynomial P(x).
2R Þ m = 2 such that (x – 1)m divides P(x) for all
dR 1 260 k such P(x)
= [1040] = =
dt 2 ´ 2 37 37 37
164 MATHEMATICS

26. (2) f '( x) = (b 2 - 3b + 2)(-2 sin 2 x) + b - 1 ¹ 0 i.e. x23 - x13 = 3x12 ( x2 - x12 )
for any x Î R
Þ x22 + x12 + x1 x2 = 3 x12
Þ (b - 1){1 - (b - 2)(2sin 2 x)} ¹ 0
Þ b ¹ 1 and Þ 2 x12 - x1 x2 - x22 = 0 Þ x2 = -2 x1
Similarly x3 = -2 x2 = 4 x1
1 æ 3 ö æ 5ö
> 1 Þ b Î ç , 2 ÷ È ç 2, ÷ x4 = -2 x3 = -8 x1
2(b - 2) è 2 ø è 2ø
....................................
When b = 2, f (x) = x + sin 3 Þ f '( x) = 1 ¹ 0
x2n = -22n -1 x1
æ 3 5ö
\ b Î ç , ÷ Þ integral value of b = 2 3( y1 + y2 + y 3 ........ + y2n )
è 2 2ø
27. (1) Equation of given curves are = ( x13 + x23 + x33 + ......... + x2 n3 ) + 2n
2 2
ax + by = 1 ...(i)
(( -8) 2 n - 1) x3
2 2 = x13 + 2 = - 1 (82 n - 1) + 2n
and a1 x + b1 y = 1 ...(ii) -8 - 1 9
dy dy a x i.e. y1 + y2 + y3 + .... + y2n
From (i) 2ax + 2by =0\ =- ...(iii)
dx dx b y 3
æx ö æ 2n ö
dy = - ç 1 ÷ (26 n - 1) + ç ÷ ,
From (ii) 2a1 x + 2b1 y = 0, è 3ø è 3 ø
dx
dy a x Now put n = 30 and x1 = 2 then
\ =- 1 ...(iv) 3
dx b1 y
y1 + y2 + y3 + ...... + y60 = - æç 2 ö÷ ( 2180 - 1) + 20
Curve (i) and (ii) will cut each other at right è3ø
dy æ 2183 - 8 ö
angles if the product of the values of for 2183 - 8
dx Þ S =-+ 20 or S + ç ÷ = 20
the two curves is –1 27 è 27 ø
Þ 5k = 20 Þ k = 4
æ a x ö æ a1 x ö aa1 x 2 29. (9) Sectorial area AOB is removed and the
i.e., ç - ÷ç - ÷ = -1 or = -1
è b y øè b1 y ø bb1 y 2 remaining part be folded into a cone of height
...(v) h and radius r.
from (i) and (ii), ax 2 + by 2 = a1x 2 + b1 y 2 ArcAB ArcAB
Q q = Angle = = = Arc AB
2 2 radius 1
or (a - a1 ) x = (b1 - b) y A A

x2 b -b I I
\ = 1 ...(vi) O q q O q I
y 2 a - a1 h
I I r
x2
Putting the value of in (v), we get B B
y2 circular sheet after removed part conical vessel
2 2
aa1 æ b1 - b ö 2pr = 2p - q and r + h = 1
ç ÷ = -1 \ volume of cone
bb1 è a - a1 ø
a - a1 b - b b - b1 1 1 1 1 1 1
or =- 1 = or - = - V = pr 2 h = pr 2 1 - r 2
aa1 bb1 bb1 a1 a b1 b 3 3
p2 r 4 (1 - r 2 )
dy Þ V2 =
28. (4) = x 2 Þ Equation of tangent at 9
dx 2 4
p (r - r 6 )
P1 ( x1 , y1 ) is y - y1 = x12 ( x2 - x1 ) Let, y = V 2 =
9
Solutions 165

dy p2 The point P (-(a - 2), 6) lies on it.


\ = (4r 3 - 6r 5 ) and
dr 9
( a - 2) 2 36
d2y p2 \ + =1
= (12r 2 - 30r 4 ) a2 b2
dr 2 9
2
dy
\ r= ç ÷
æ2ö or 36 = 1 - ( a - 2) = 4(a - 1)
For max or min of y, = 0 2
b 2 2
a a
dr è3ø
Then, 9 ( a - 1)
2 2
or
2
=
d y p æ .2 4ö 16 2 b a2
= çè 12 - 30. ÷ø = - p Y
dr 2 r = 2 / 3 9 3 9 27
\ y is maximum and hence V is also maximum C (0,b)
(– (a–2),6) P
æ 2ö
at r = ç ÷ ....(1) 6
è 3ø
2
1 2 X
A D O B
\ Removed sectorial area = .(1) .q
2 (–a, 0) (a, 0)
1 1 9a2
A1 = .q = (2p - 2pr ) ì 1 ü
2 2 or b2 = = 9 ía + 1 + ý
(a - 1) î a - 1þ
æ 2ö
= p(1 - r ) = p çç1 - ÷ [from (1)] d (b) 2 ïì 1 ïü d 2 (b 2 ) 18
è 3 ÷ø \ = 9 í1 - ý and =
2 2
Also, A = p da îï ( a - 1) þï da (a - 1)3
2
1+ db 2
A 1 3 = 3+ 6 = m + n For extremum value of b, =0
\ = = da
A1 2 2
1- 1-
3 3 9(2)2
Þ we get a = 2 \ b2 = Þ b = 6m
Þ m + n = 3 + 6 = 9. (2 - 1)
x2 y2 Hence greatest height of the arch is 6 m.
30. (6) Equation of the ellipse is + =1
a2 b2
CHAPTER
22 Indefinite Integration

sin x sin( x - a + a)
1. (b) ò sin( x - a) dx = ò dx 2 x (nx 2 n + 2 - (n + 1) x 2 n + 1)
sin( x - a) = hn ( x ) = g 'n ( x ) =
sin( x - a ) cos a + cos( x - a)sin a ( x 2 - 1)2
= ò dx 2x
sin( x - a ) Now f ( x) = lim hn ( x ) = as 0 < x <1
= ò {cos a + sin a cot( x - a )}dx n®¥ ( x - 1) 2
2

= (cos a) x + (sin a) logsin( x - a) + C 2x


\ A = cos a, B = sin a Thus ò f ( x)dx = ò ( x2 - 1)2 dx
2. (d) Let gn ( x )
1 1
2 4 2n x2n+ 2 - 1 = - 2 = +c
= 1 + x + x + .... + x = x - 1 1 - x2
x2 - 1
æ f ( x) g '( x) - f '( x) g ( x ) ö æ g ( x ) ö
So, 2 x + 4 x 3 + ..... + 2nx 2 n-1
3. (c) ò çè f ( x ) g ( x)
÷ ln ç
ø è f ( x) ø
÷ dx
166 MATHEMATICS
g ( x) ln a2 x b3 x
Let =t =ò dx
f ( x) 6a 2 x b 3 x
f ( x ) g '( x) - g ( x) f '( x ) Let a 2 x b3 x = t .
Þ dx = dt ;
( f ( x )) 2 Then t ln a 2b3dx = dt .
ln t (ln t )2 Therefore,
ò t dt =
2
+C
I =ò
1 ln t
dt
1 + nx n -1 - x 2 n
2 3 2
6 ln a b t
4. (c) We have ò e x dx 1 æ - ln t -1 ö
(1 - x n ) 1 - x 2 n = 2 3ç
- ò 2 dt ÷
6ln a b è t t ø
é 1 + xn nx n -1 ù 1 æ ln et ö
x
= òe ê + údx =- ç ÷+k
ê 1 - x n (1 – x n ) 1 - x 2 n ú 6 ln a 2 b3 è t ø
ë û
1 æ ln a 2 x b3 x e ö
1 + xn =- 2 3ç ÷+k
=e
x
+C 6ln a b çè a 2 x b3 x ÷ø
1 - xn
(c) I n = (sin x + cos x)n dx
1 + xn
8.
ò
Here, f ( x) = , then
(sin x + cos x) n -1 2( n - 1)
1 - xn = (sin x - cos x) + I n -2
nx n -1 n n
f '( x ) = t4 8 t4
(1 - x n ) 1 - x 2 n Þ I5 = (sin x - cos x) + I3 = (sin x - cos x )
5 5 5
òe
x
and ( f ( x) + f '( x ))dx = e x f ( x) + C æ
8 t 2
4 ö
+ ç (sin x - cos x) + I1 ÷
5. (d) lim n 2 ( x1/ n - x1/(n +1) ) 5 çè 3 3 ÷ø
n®¥

= lim
h® 0 h 2
1 ( x h
h
- x +1
h ) t4
5 15
8
= (sin x - cos x ) + t 2 (sin x - cos x )
32
æ æ h ö ö h + (sin x - cos x) + C
1 è èç h - h +1ø÷ ø h +1 15
= lim x -1 x
h® 0 h 2 (sin x - cos x ) 4
= (3t + 8t 2 + 32) + C ;
æ h 2 ö 15
ç x h +1 - 1÷ æ 1 ö h where t = (sin x + cos x).
= lim ç ÷ .ç ÷ x h +1 = ln x.1.1 9. (b) Let I be the given integral. We can see that
h®0 ç h2 ÷ è h + 1ø
çè ÷ cos q + sin q ö
h +1 ø the derivative of loge æç ÷ is 2 sec 2q,
\ f ( x) = ln x è cos q - sin q ø
so that using integration by parts we have
So, I = ò xf ( x)dx = ò x ln xdx
sin 2q æ cos q + sin q ö
x2 x2 1 x2 x2 I= loge ç ÷
= ln x - ò . dx = ln x - +C 2 è cos q - sin q ø
2 2 x 2 4
sin 2q
x 2n - 1 -ò × (2 sec 2q)d q
6. (a) f ( x ) = lim 2n = -1 (Q 0 < x < 1) , 2
n® ¥ x +1
sin 2q æ cos q + sin q ö
so ò sin -1 x( f ( x))dx = - ò sin -1 x dx = log e ç ÷ - ò tan 2q d q
2 è cos q - sin q ø
= - [x sin -1 x + 1 - x 2 ] + C =
sin 2q æ cos q + sin q ö 1
log e ç ÷ - log e (sec 2q) + c
æ ln aa x /2 x
ö 2 è cos q - sin q ø 2
ln bb
7. (b) I = ò x ç 5 x /2 3 x + 2 x 4 x ÷dx Therefore
ç 3a b 2a b ÷
è ø f ( x) = log e (sec 2q).
Solutions 167
10. (d) We have, Put tan x = t
1 (t - 1)
ò x [log ex e × loge2 x × loge3 x e] dx I =- ò (t + 1) t3 + t2 + t
dt

1 t 2 -1

x loge ex log ee 2 x.loge e 3 x
dx =- ò (t 2 + 2t + 1) t3 + t 2 + t
dt

1 1
1-
= ò x(log dx t2 1 2
e e + loge x)(log e e2 + loge x) =- òæ 1ö 1
dt , put 1 + t + = u
t
çt + 2 + ÷ t +1+
(loge e3 + loge x) è t ø t
d (loge x) 2du

(1 + loge x)(2 + log e x)(3 + log e x)
\ I=-
ò 1+ u2 = -2 tan -1 u + c,

1 1
=ò dt , where t = log x where u = 1 + tan x +
(1 + t )(2 + t )(3 + t ) e tan x
æ1 1 1 1 1 ö b
= òç × - + × ÷ dt 13. (c) Let ax 2 = b sin 2 q Þ x = sin q
è 2 1+ t 2 + t 2 3 + t ø a
1 2ax dx = 2b sin q cos qd q
= log | 1 + log e x | - log | 2 + log e x | 2b sin q cos q b
2 Þ dx = dq = cos qd q
1 2ax a
+ log | 3 + log e x | +C b cos qd q
1
2 I= ò
a æ b 2 ö
= log{log e ex} - log{log e e 2 x} çè a + b. sin q÷ø . b cos q
a
2
dq
1
+ log{log e e 3 x} + C
2
= a ò (a 2 + b 2 sin 2 q)
dx sec 2 qd q
11. (a) I = ò ( x - b) ( x - a)(b - x )
= a
ò (a 2 sec2 q + b2 tan 2 q)
Put x = a sin 2 q + b cos 2 q Putting tan q= z Þ sec 2 qd q = dz then
dz
dx = 2(a - b) sin q cos q dq
Also, ( x - a) = (b - a ) cos 2 q
I= a ò
a (1 + z 2 ) + b 2 z 2
2
dz
( x - b) = ( a - b) sin 2 q = a ò (a 2 + b 2 ) z 2 + a 2
2(a - b) sin q cos q dq
\I = ò (a - b) sin 2 q (b - a) sin q cos q a dz

=
2 2
2 dq 2 (a + b a2
z2 +
b - a sin q b - a ò
ò
= = cos ec 2 q dq
2 (a + b2 )
2

2 2 a a 2 + b2 æ z a2 + b2 ö
= (- cot q) + c = cot q + C = . tan -1 ç ÷+c
b-a a -b a 2 + b2 a
è
ç a ÷
ø
Now x = a sin 2 q + b cos 2 q 1 æ z a 2 + b2 ö
\I= tan -1 ç ÷ + c,
Þ x cos ec 2 q = a + b cot 2 q 2 2
a(a + b ) çè a ÷ø
Þ x(1 + cot 2 q) = a + b cot 2 q where z = tan q .
x-a 2 x-a 14. (b) Given ò f ( x ) sin x cos xdx
\cot q = ; \I= +C
b-x a -b b- x 1
= ln f ( x) + c
12. (a) I = - ò (tan x - 1) sec 2 xdx
dx 2( b - a 2 )
2

(tan x + 1) tan 3 x + tan 2 x + tan x


168 MATHEMATICS
Differentiating both sides w.r.t.x then +(n - 1) ò (sin x + cos x)n- 2 [2 - (sin x + cos x) 2 ] dx
1 f '( x )
f ( x) sin x cos x = .
2 2
2(b - a ) f ( x ) = (sin x + cos x )n -1 (sin x - cos x )
f '( x) +2(n - 1) I n -2 - (n - 1) I n
Þ 2(b 2 - a 2 ) sin x cos x = Therefore,
{ f ( x )}2
I n + (n - 1) I n - 2(n - 1) I n- 2
f '( x)
Þ 2b 2 sin x cos x - 2 a 2 sin x cos x = = (sin x + cos x )n -1 (sin x - cos x )
{ f ( x )}2
Integrating both side w.r.t. x we get Þ nI n - 2( n - 1) I n- 2

1 = (sin x + cos x )n -1 (sin x - cos x ).


2 2 2 2
-b cos x - a sin x = - 17. (d) We have,
f ( x) f ' (x) = ln(x2 + 1)
1
or f ( x) = 2 2 f (x) = ò ln ( x 2 –1) dx
( a sin x + b 2 cos2 x )
sec6α sec18α sec54α 2x2
15. (a) We have +
cosec2α cosec6α cosec18α
+ f (x) = xln(x2 – 1) – ò x2 - 1dx
sin 2a sin 6a sin18a æ x 2 –11 ö
= + + f (x) = xln(x2 – 1) – 2ò ç +
÷ dx
cos 6a cos18a cos 54a 2
è x –1 x –1ø 2
sin 2a 1 sin 4a
Now, = æ x –1ö
cos 6a 2 cos 2a cos 6a f (x) = xln(x2 – 1) – 2x – ln ç +C
è x + 1ø÷
1 é sin 6a cos 2a - cos 6a sin 2a ù æ 1ö
= ê ú f (2) = 2ln(3) – 4 –ln çè ÷ø + C = 0 [Q f (2) = 0]
2ë cos 2a cos 6a û 3
1 Þ C = 4 – 3ln3
= (tan 6a - tan 2a )
2 \ f (x) = xln(x2– 1) – 2x – ln
( x –1) + 4 – 3ln 3
sin 6a 1 ( x +1)
Similarly, = (tan18a - tan 6a ) \ the above function is defined for S
cos18a 2
infinite C values possible in set S
and sin18a = 1 (tan 54a - tan18a ) such that f ¢ (x) = ln(x2 – 1)
cos 54a 2 1/ x
1 æ f ( x) ö
Thus integral = ò (tan 54a - tan 2a ) dx 18. (1) We have lim ç1 + x + ÷ = e3
x ®0 è x ø
2
1/ x
1 é l n | sec54a | l n | sec 2a | ù æ æ f ( x) ö ö
= ê - úû + c Þ lim ç1 + x ç1 + ÷÷ = e3
2ë 54 2 x ®0 è è x2 ø ø
16. (b) Let x2 + f ( x) 1
I n = ò (sin x + cos x ) n -1 (sin x + cos x ) dx é x ù x x
ê æ x 2 + f ( x) ö x 2 + f ( x ) ú
Take, u = (sin x + cos x )n -1 Þ lim êç 1 + ÷ ú = e3
x ®0 ê ç x ÷
ú
è ø
and dv = (sin x + cos x )dx êë úû
2
so that, v = sin x - cos x x + f ( x)
Þ = 3 Þ f ( x) = 2 x2
Therefore, using integration by parts we have x2
I n = (sin x + cos x)n -1 (sin x - cos x) Therefore
2
-ò (n - 1)(sin x + cos x) n- 2 ò f ( x)log e x dx = 2ò x loge x dx
´ (cos x - sin x )(sin x - cos x ) dx é x3 x3 1 ù
= 2 ê loge x - ò × dx ú (By Parts)
= (sin x + cos x )n -1 (sin x - cos x ) êë 3 3 x úû
+(n - 1) ò (sin x + cos x)n -2 (cos x - sin x) 2 dx 2 2 2 æ 1ö
= x3 log e x - x3 + c = x 3 ç log e x - ÷ + c
3 9 3 è 3ø
= (sin x + cos x )n -1 (sin x - cos x ) Þ a+b=1
Solutions 169
q
(cos 2 x + sin 2 x ) sin 3
19. (0.60) Let I = ò dx 21. (3) I =ò 2 dq
(2 cos x - sin x) 2 q
cos cos3 q + cos2 q + cos q
(cos x + 2sin x ) cos x 2
= ò dx æ q qö 2q
(2 cos x - sin x )2 çè 2sin .cos ÷ø .sin
2 2 2
Integrating by part , taking cos x as the first =ò dq
2q 3 2
(cos x + 2 sin x ) 2 cos cos q + cos q + cos q
and as the second function, 2
q
(2 cos x - sin x ) 2 2 sin 2 sin q d q
we have =ò 2
2q
ì 1 ü - sin xdx 2 cos cos q + cos2 q + cos q
3
= cos x í ý-ò 2
î 2 cos x - sin x þ (2cos x - sin x)
- sin x dx Put cos q = t Þ - sin q d q = dt
ì 1 ü
= cos x í ý+ò q
Also cos q = 2cos 2 - 1 = 1 - 2sin 2 = t
q
î 2 cos x - sin x þ (2cos x - sin x)
2 2
cos x 1- t
= ( - dt )
(2 cos x - sin x) \ I= ò 2
1 2 (1 + t ) t 3 + t 2 + t
- (2 cos x - sin x ) - ( -2 sin x - cos x)
+ò 5 5 dx 1 (t 2 - 1)dt
(2 cos x - sin x ) = ò
2 (t + 1)2 t 3 + t 2 + t
é g d ù
êë N = l Dr + m dx Dr úû æ 1ö
çè1 - 2 ÷ø (dt )
cos x 1 1 t
= - = ò
(2 cos x - sin x) 5 2 æ 1 ö 1
çè t + + 2÷ø t + + 1
2 ( -2 sin x - cos x) t t
ò dx - 5 ò 2 cos x - sin x dx 1
Put t + + 1 = u 2 Þ æç1 - 1 ö÷ dt = 2u du
cos x 1 t è t2 ø
= - x
(2cos x - sin x ) 5 1 2udu
I= ò
2
- ln | 2 cos x - sin x | +c
5
2 1+ u2 u( )
Þ |a + b| = 0.60. -1 1
= tan u = tan -1 1 + + 1 + c
1 - (cot x)2010 t
20. (2012) Let I = ò dx -1 1/ 2
tan x + (cot x) 2011 = tan (cos q + sec q + 1) + c
(sin x )2010 - (cos x )2010 (sin x ) 2011 cos x So, f (q) = cosq + secq + 1 > 2 + 1 =3
=ò 2010
× dx f ( x + h) - f ( x )
(sin x ) (sin x ) 2012 + (cos x )2012 22. (2) f '( x ) = lim
[(sin x)2010 - (cos x) 2010 ]sin x cos x h® 0 h
=ò dx æ æ h öö
(sin x )2012 + (cos x ) 2012 f ç x ç 1 + ÷ ÷ - f ( x)
è x øø
Put t = (sin x) 2012 + (cos x )2012 . Then = lim è
h ®0 h
dt = [2012(sin x )2011 cos x æ hö
f ( x) × f ç 1 + ÷ - f ( x)
+2012(cos x )2011 (- sin x )]dx = lim è xø
h ®0 h
= 2012[(sin x)2010 - (cos x )2010 ](sin x cos x) dx
Therefore (Q f ( xy ) = f ( x ) × f ( y ) " x, y Î R )
1æ 1 ö 1 æ hö
I =ò ç ÷ dt = log e | t | + c f ç1 + ÷ - 1

= f ( x ) lim è
t è 2012 ø 2012
1 h ®0 h
= log e | (sin x) 2012 + (cos x) 2012 | + c
2012
170 MATHEMATICS
ì hæ æ h öö ü 2 é -1 1 æ 2 + t öù
ï 1 + ç1 + g ç ÷ ÷ - 1 ï =
ï x è x øø ï ê tan (t ) + ln çç ÷ú + C
= f ( x ) í lim è
ý 3 ëê 2 2 è 2 - t ÷ø ûú
ï h ®0 h ï 2 1
ïî ïþ \A= , B =
3 3 2
ì 1æ æ h ö öü 2 1
= f ( x ) × í lim ç1 + g ç ÷ ÷ ý \12 A + 9 2 B - 3 = 12 × + 9 2 - 3 = 8.
îh ®0 x è è x ø øþ 3 3 2
h sin8 x sin8 x × cos x
As h ® 0; ® 0 25. (4) I = ò dx = ò dx
x cos x cos2 x
æhö sin8 x
Þ g ç ÷ ® 0 as lim g ( x ) = 0 (given)
èxø x®0 =ò × cos x dx
(1 - sin 2 x )
1 f ( x)
\ f '( x ) = f ( x) × Þ =x Put sin x = t , cos x dx = dt
x f '( x)
t8 t8 -1 + 1
f ( x) x2 \I = ò dt = ò dt
Þò dx = + C Þ k = 2. 1- t2 1- t2
f '( x) 2 (t 2 - 1)(t 2 + 1)(t 4 + 1) 1
=ò dt + ò dt
23. (1) I = ò {sin(100 x + x) × (sin x)99 } dx 1- t 2
1 - t2
= ò {sin(100 x ) cos x + cos100 x sin x}(sin x )99 dx 1
= ò (t 2 + 1)(-1 - t 4 ) dt + ò dt
= ò sin(100 x ) cos x × (sin x )99 dx 1- t2
1424 3 1442443 1
I II = ò (-t 6 - t 4 - t 2 - 1) dt + ò dt
+ ò cos(100 x ) × (sin x)100 dx 1- t2
-t 7 t 5 t 3 1 1+ t
sin(100 x)(sin x )100 = - - - t + ln +C
= 7 5 3 2 1- t
100 - sin 7 x sin 5 x sin 3 x
100 = - -
cos(100 x)(sin x )100 dx
100 ò
- 7 5 3
1 1 + sin x ...(1)
+ ò cos(100 x )(sin x )100 dx - sin x + ln +C
2 1 - sin x
2
sin(100 x)(sin x )100 æ x xö
= +C ç sin + cos ÷
100
Now, ln è
2 2ø
l 100 2
Þ l = 100, m = 100 Þ = = 1. æ x xö
m 100 ç cos - sin ÷
è 2 2ø
dx
24. (8) ò 1 + tan x / 2 æp xö
(cos x - sin x )(1 + sin x cos x) = 2 ln = 2 ln tan ç + ÷ ...(2)
(cos x - sin x )dx 1 - tan x / 2 è4 2ø
= 2ò \ From (1) and (2)
(cos x - sin x) 2 (2 + (sin x + cos x) 2 - 1) - sin 7 x sin5 x sin 3 x
(cos x - sin x )dx \I = - - - sin x
= 2ò 7 5 3
((sin x + cos2 x ) - 2sin x cos x )
2
æp xö
+ ln tan ç + ÷ + C
(1 + (sin x + cos x ) 2 ) è 4 2ø
(cos x - sin x)dx 7 5
- sin x sin x sin x sin x 3
= 2ò = + - +
(2 - (sin x + cos x ) 2 )(1 + (sin x + cos x) 2 ) a b c d
dt æp xö
= 2ò where t = sin x + cos x + ln tan ç + ÷ + C (given)
(2 - t )(1 + t 2 )
2
è 4 2ø
dt 2é 1 dt ù Þ a = 7, b = -5, c = 3, d = -1
= 2ò = êò dt + ò ú
2 2 2
(2 - t )(1 + t ) 3 ë 1+ t 2 - t2 û Þ (a + b + c + d ) = (7 - 5 + 3 - 1) = 4.
Solutions 171
CHAPTER
23 Definite Integration

1. (a) Since the required function is a polynomial, p p


é sin 2 px ù é sin 2qx ù
the absciassae of the points of inflection can = êx + ú + ê x - 2q ú = 2p
only be among the roots of the second derivative. ë 2 p û0 ë û0
Consequently 2p

òe
cos q
p ''( x ) = ax( x - 1)( x + 1) = a ( x3 - x ) 3. (c) I= cos(sin q)d q = Real part of
Since at the point x = 0, p '(0) = tan 60° = 3, 0
2p
x
æ x 4 x2 ö
òe
cos q
{cos(sin q) + i sin(sin q) d q
ò
P '( x ) = P ''( x )dx + 3 = a ç
è 4
- ÷+ 3
2ø 0
0 2p
hen, since P(1) =1, we get cos q i sin q
P (x) =
x
= Real part of òe e d q = Real part of
æ x 5 x3 7 ö 0
ò P '( x )dx + 1 =a ç - + ÷ + 3( x - 1) + 1 2p 2p
ei q
1
è 20 6 60 ø
òe
cos q+i sin q
d q = Real part of òe dq
60( 3 - 1) 0 0
Since, P (-1) = -1 so , a = = Real part of
7
Thus 2p é 2iq
iq e e3iq ù
1 1
é 3 -1 5 3 ù òêê1 + e + + + ............ ú d q
ò P( x)dx = ò êë 7 (3x - 10 x ) + x 3 úûdx 0 ë
2! 3! ûú
0 0 = Real part of
1 2p
3 - 1 æ x6 5 4 ö x2 é 1 ù
= ç - x ÷+ 3 ò êë1 + (cos q + i sin q) + 2! (cos 2q + i sin 2q) + ...úû d q
7 è 2 2 ø 2 0
0 2p
é 1 ù
=
3 -1 æ 1 5 ö
ç - ÷+
7 è 2 2ø 2
3 3 3 2
= +
14 7
= ò êë1 + cos q + 2! cos 2q + .......úû dq
0
2p
p é sin 2 q ù
(cos2 px + sin2 qx – 2 cos px sin qx) dx = ê q + sin q + + .......ú = 2p
2. (d) I= ò ë 2.2! û0
-p
4. (a) Let q = p + d , r = p + 2d , s = p + 3d
Q sin 2 qx, cos2 px
are even functions of x and
p + sin x p + d + sin x -2d + sin x
cos px . sin qx is an odd function.
p p \ f (x) = p + d + sin x p + 2d + sin x -1+ sin x
\ ò cos 2 px dx = 2 ò cos 2 px dx p + 2d + sin x p + 3d + sin x 2d + sin x
-p 0
p p Applying R ® R1 + R3 - 2 R2 , we get
ò sin 2 qx dx = 2 ò sin 2 qx dx 0 0 2
-p 0
p f ( x) = p + d + sin x p + 2d + sin x -1 + sin x
and ò cos px sin qx dx = 0 p + 2d + sin x p + 3d + sin x 2d + sin x
-p
p p = 2[( p + d + sin x)( p + 3d + sin x)
2 2
\ I = 2 ò cos px dx + 2 ò sin qx dx = 0 -( p + 2d + sin x )2 = -2d 2
0 0 2
p p
æ 1 + cos 2 px ö
= 2ò ç
æ 1 - cos 2qx ö
÷ dx + 2 ò ç ÷ dx
Given ò f ( x)dx = -4
0
è 2 ø 0
è 2 ø 0
2
p p
Þ 2
= ò (1 + cos 2 px) dx + ò (1 - cos 2qx) dx ò ( -2d )dx = -4 d 2 = 1 Þ d = ±1
0
0 0
172 MATHEMATICS
p /2 1 b
dx Þ tan -1 f 2 ( x ) ³ b-a
5. (a) ò a2 cos2 x + b2 sin2 x 2 a
0 1é -1 2 -1 2 ù
p /2
p/2 2 Þ b-a £ ê lim tan f ( x) - lim tan f ( x) ú
1 sec x dx 2 ëê x ® b - +
sec2 x dx
ò ûú
x®a
=
= ò b2 tan 2 x + a 2 b 2
0 æ aö
tan 2 x + ç ÷
2
=
p
0 24
è bø
¥ 1
ò 0 (1 + cos
8
1 dt 8. (b) Given x )( ax 2 + bx + c ) dx
=
b2 ò æ aö
2
(Putting tan x = t )
2
0 t2 +ç ÷
è bø
= ò 0 (1 + cos8 x )(ax 2 + bx + c) dx
¥ 1
1
b bt 1 æ pö p = ò (1 + cos8 x )(ax 2 + bx + c ) dx
= ´ tan -1 = ç ÷= 0
b 2 a a 0 ab è 2 ø 2 ab 2

Thus
p
=
p
Þ ab = 8
+ ò1 (1 + cos8 x)(ax 2 + bx + c) dx
2ab 16 2
ò 1 (1 + cos
8
Now minimum value of a cos x + b sin x is Þ x )( ax 2 + bx + c ) dx = 0
- a 2 + b2 , which is least if a = b = 2 2 b
Þ - a 2 + b 2 ³ - 8 + 8 = -4 Now we know that if
òa f ( x)dx = 0 then it
sin x means that f (x) is + ve on some part of (a, b) and
6. (a) Let f ( x) = – ve on other part of (a, b).
x
x cos x - sin x But here 1 + cos8 x is always + ve,
\ f '( x ) = \ ax2 + bx + c is + ve on some part of [1 , 2]
x2 and – ve on other part [1, 2]
( x - tan x) cos x ép p ù \ ax 2 + bx + c = 0 has at least one root in (1, 2).
= < 0" x Îê , ú
2 Þ ax2 + bx + c = 0 has at least one root in (0,2).
x ë4 3 û
sin x x2
\ f ( x) = is decreases on the interval
ép p ù
x 9. (c) f ( x) = ò (t - 1)dt
x
ê4, 3 ú
ë û Þ f '(x) = (x -1)2x - (x -1) = (x -1)(2x2 + 2x -1)
2
Þ The least value of the function
\ f '(w ) = (w - 1)(2w 2 + 2w - 1)
æ p ö sin( p / 3) 3 3
m= f ç ÷ = = = (w - 1){2( -1) - 1} = 3(1 - w )
è 3ø ( p / 3) 2p
and the greastest value of the function \ | f '(w) |= 3 | 1 - w |
æ p ö sin(p / 4) 2 2 Now,
M = fç ÷= =
è 4ø ( p / 4) p | 1 - w |2 = (1 - w )(1 - w) = (1 - w )(1 - w 2 )
therefore
p/3 = 1 - w - w 2 + w3 = 3
æ p pö 3 3 sin x æ p pö 2 2
-
èç 3 4 ø÷ 2p
<
xò dx < ç - ÷
è 3 4ø p 10.
\ | 1 - w |= 3 Þ | f '(w) |= 3 3
(a) We have
p/4
[Mean Value Theorem of Integral Calculus] d é h ( x) ù
3
p/3
sin x 2 ò
dx êë g ( x )
f (t ) dt ú = f ( h( x)) h '( x) - f ( g ( x)).g '( x)
Hence,
8
< ò x
dx <
6
û
f (2) ´ 2 ´ 2 ´ 1 8
= f (2)
p/4 =
1 f ( x ) f '( x ) p p
3 2´
7. (d) f '( x ) ³ f ( x ) + Þ ³1 4
f ( x) 1 + f 4 ( x) 2
sec x
Integrating on the interval (a, b), we get
b b ò2 f (t )dt
é0 ù
f ( x ) f '( x) Let L = lim êë 0 form úû
ò 1 + f 4 ( x) dx ³ ò dx x®
p
4 x2 -
p 2

a a 16
Solutions 173
On applying L' Hospital's rule, we get If is possible when f (x) = 0.
d é sec x
2 ù Hence, " p ÎR. There is no non-zero continous
ê
dx êë 2 ò f (t )dt ú
úû
function.
Hence, S ÎR.
L = lim 13. (a) We have,

p d æ 2 p ö 2
x - ÷ 1 1
dx çè
4
16 ø L = lim nò dx
x ®¥ 0 (1+ x 2 )n
f (sec 2 x ).2sec 2 x tan x Q (1 + x2)n > 1 + nx2
L = lim 1 1

p 2x Þ <
4 (1+ x 2 )n 1+ nx 2
æ pö p p
f ç sec 2 ÷ .2.sec 2 . tan 1 dx 1 dx
è 4 ø 4 4 8 Þ ò <ò
= = . f (2) 2
0 (1 + x ) n 0 1 + nx 2
p p
2. 1 dx 1
4 Þ ò < [tan –1 nx ]10
p 0 (1 + x 2 ) n n
11. (c) Let I = ò0 (1– | sin 8 x) |) dx 1 dx 1
p p Þ ò < tan –1 n
Þ I= ò0 dx – ò0 | sin 8 x ) | dx 0 (1 + x 2 ) n n
p 8´ 8 Þ L
Þ I= p– ò | sin 8 x | dx
= lim n ò
1 1
dx < lim
n
(tan –1 n )
0 0 2 n x®¥ n
p x ®¥ (1+ x )
8

Þ I = p –8ò sin8x dx p
Þ L < tan –1 ¥ =
0
2
p 1 é p ù
[Q sin 8x is periodic with period ] <L<2
8 \
2 êëQ 2 < 2 úû
p /8
é – cos8 x ù x
Þ I = p –8 ê
ë 8 úû 0 14. (a) f (x) = 2ò t f (t )dt + 1 ...(i)
Þ I = p + (cos p – cos0) = p – 2 0
12. (d) Given, \ f (0) = 1
x Now, differentiate (i) w.r.t. x
ò f ( x)dt = pf ( x) ...(1) f ¢(x) = 2x f (x)
0 f ¢ ( x)
Differentiating, we get, = 2x
f ( x)
Þ f (x) = p f ¢(x) ...(2)
Integrate both sides, we get
f '( x ) 1 ln | f (x) | = x2 + c ...(ii)
Þ =
f ( x) p Q f (0) = 1 Þ c = 0
Integrating, we get, Then from eqn. (ii), ln | f (x) | = x2
x x 2
Þ log f (x) = p + C Þ f (x) = e Þ f (1) = e
1
Þ f (x) = Aex/p x
Putting x = 0 in eqn. (i) we get,
...(3) 15. (a) J = ò 1 + x8 dx
0
0 = p f (x) pf (0) = 0
Q 0 < x8 < 1
Case I f (0) = 0, p ¹ 0 1 1
Þ A=0 (from 2) x 1 é x2 ù 1
Þ f (x) = 0 pf (0) = 0 Þ J > ò 2 dx Þ J > 2 ê 2 ú Þ J >
Þ f ¢ (x) = 0 (from (2)) 0 ëê úû0 4
1 1
Þ there is no non-zero continuous to f (x). x é x2 ù 1
Case II p = 0 J < ò 1 + 0 dx Þ J < ê 2 ú Þ J <
x 0 ê
ë û0 ú 2
\ ò0 f (t )dt = 0, " x Î R Hence, (I) is true and (II) is false.
174 MATHEMATICS
16. (a) It is given that f (x) = | sin x |, ( f ( x ))2 - 42
x = lim
x -1
g(x) = ò f (t ). dt x ®1
2 f ( x). f ¢( x )
0
= lim (L-Hospital rule)
2 x ®1 1
and p(x) = g(x) – x = 2f (1) . f ' (1) = 2 × 4 × 2 = 16
p
2
Now, p(x + p) = g(x + p) – (x + p) - sin x sec x x2 -1
p
p+ x 19. (0) f (- x ) = -cosec x x sin x cos x
2
= ò f (t )dt - x - 2
p - tan x x tan x x2 + 1
0
p p+ x = - f ( x ) Þ f ( x) is an odd function
2
=ò ò
f (t )dt +
p
x-2 f (t )dt - p /3
0 p
Q f (x) is periodic function with period p
Þ ò f ( x) dx = 0.
p+ x x -p /3
x
\ ò f (t )dt = ò f (t )dt 2
3ò e-t dt - 3x + x3
p 0
p 0
2 20. (0.30) We have lim
Þ p(x + p) = ò sin x dx + g ( x ) -
p
x-2 x®0 x5
0
æ 0ö
2 ç form ÷
= 2 + g(x) – x – 2 è 0ø
p Using L’ Hospital’s rule we get
Þ p(x + p) = p(x) for all x 2 2
17. (a) If is given that, x2 + (f (x))2 £ 1 3e - x - 3 + 3x 2
3 e- x - 1 + x 2
lim lim =
Þ f (x) £ 1 - x 2 x® 0 5x4 5 x® 0 x4
1 1 æ 0ö
f ( x )dx £ ò 1 - x 2 dx ç form ÷
Þ ò è 0ø
0 0
Again using L' Hospital’s rule, we get
1 1
éx 2 1 -1 ù 2
Þ ò f ( x)dx £ êë 2 1 - x + 2 sin ( x) úû0 3
lim
e - x .( -2 x ) - 0 + 2 x
0
1
5 x® 0 4 x3
p
Þ ò f ( x)dx £ 4 Þ f (x) = 1 - x2 3 2 - e- x + 1
2
æ 0ö
ç form ÷
0 = . lim
1 1 5 4 x®0 x 2 è 0ø
2 2 Again using L' Hospital’s rule we get
f ( x) 1 - x2
ò 1- x ò
2
Þ 2
dx = 2
dx 3 -e - x .(-2 x ) 3 3
1 1 (1 - x ) lim = 1 = = 0.30
10 x® 0 (2 x) 10. 10
2 2
1 p/2
2
ò
dx 21. ( 2) I (n) = ò q.sin n q d q .
= 0
1 1 - x2 p/2

ò q.sin n - 2 q(1 - cos 2 q) d q


2
Þ I ( n) =
-1 p p
1/ 2 p
= ésin xù = - = 0
ë û1/2 4 6 12 p/2

ò (q.cos q).cos q.sin n - 2 qd q


f ( x)
= I (n - 2) –
ò 2t dt
(t 2 ) |4f ( x ) 0
4 = lim
18. (16) lim sin n -1 q p / 2
x ®1 x -1 x ®1 x -1 = I (n - 2) - q.cos q.
n -1 0
Solutions 175
p/2 23. (0) Using integration by parts
n -1
+ ò [q.(- sin q) + cos q]. sin q d q
æ cos pt ö
1 1
cos pt
0 n -1
p/2
I = ç -2 sin qt
è p ø0 ÷ ò
+ 2q cos qt.
p
dt
1 0
= I (n - 2) -
(n - 1) ò n
q.sin q d q
2 2q
1

p/2
0 =-
p
sin q cos p +
p ò
cos qt.cos ptdt
1 n -1
ò
0
+ cos q. sin q dq
2q éæ
1
n -1 2 sin pt ö
0 = - sin q cos p + êç cos qt
1 1 p/2 p p êè p ÷ø 0
= I (n - 2) - .I (n) + .sin n q ë
(n - 1) ( n - 1)( n) 0 1 ù
q
Þ
n
n -1
I (n) = I (n - 2) +
1
(n - 1)(n)
, p
+ ò
sin pt sin qtdt ú
ú
0 û
n -1 1 2 2 q æ sin p cos q q I ö
Þ I (n) - I (n - 2). = 2. = - sin q cos p + +
n n p p çè p p 2 ÷ø
1
Þ n I ( n ) - ( n - 1) I ( n - 2 ) = é q2 ù 2 2q
n or, I ê1 - ú = - sin q cos p + 2 sin p cos q
Put n = 2010 2 p
ëê p ûú p
1
Þ 2010 I ( 2010 ) - 2009 I ( 2008) = é 2q tan p 2 ù
2010 = cos p cos q ê 2
- tan q ú
-1 êë p p úû
Þ [2010 I (2010) - 2009 I (2008)] = 2010
x x As p and q are different roots of equation,
tanx = x
ò
22. (0) We have x (1 - t ) f (t )dt = t f (t )dt ò Þ tan p = p and tan q = q
0 0 \ I = 0.
Differentating both sides with respect to x 3 1 3
ò [ x ] dx = ò [ x ] dx + ò [ x ] dx
we get, 24. (2)
x

ò
x(1 - x ) f ( x) + (1 - t ) f (t )dt = xf ( x ) 0
é
0
ì0, if
1
0 £ x < 1ù
0 êQ éë x ùû = í ú
x
ë î1, if 1 £ x < 3û
Þ
ò
x 2 f ( x ) = (1 - t ) f (t )dt 1 3
= ò 0 dx + ò 1dx = [ x ]1 = 2
3
0
Differentiating again with respect to x on both 0 1
sides, we get x
x 2 f ¢( x ) + 2 xf ( x ) = (1 - x ) f ( x) lim
F ( x) 1
=
ò f (t )dt
Þ lim x –1
25. (7) x ®1 G ( x) 14 x®1
Þ
f ¢ ( x) 1 - 3 x
= 2 ò t f ( f (t )) dt
–1
f ( x) x 1 1
Integrating both the sides, we get Q ò-1 f (t )dt = 0 and ò-1t f ( f (t )) dt = 0
1 f(t) being odd function
ln f ( x ) = - + 3lnx + l
x \ Using L Hospital’s rule, we get
1 f ( x) 1
Þ ln éë x f ( x) ùû + = l and f (1) = 1
3 lim =
x x ®1 x f ( f ( x )) 14
Þ l =1 f (1) 1 1/ 2 1
æ 1ö Þ = Þ =
1 çè 1- ÷ø f ( f (1)) 14 æ 1 ö 14
Þ f ( x) =
e x
. fç ÷
è 2ø
x3
Thus lim f ( x) = 0 æ 1ö æ 1ö
x ®¥ Þ f ç ÷ =7 Þ fç ÷ =7
è 2ø è 2ø
176 MATHEMATICS

CHAPTER
24 Application of Integrals
b
1. (d) Given ò f (x)dx = b2 +1 - 2 = 2[ln | sec x ||0p / 4 +ln | sin x ||pp // 24 ]
1
Differentiate with respect to b é 1 ù
= êln 2 + ln 1 - ln ú = 2[2ln 2] = ln 4.
b x ë y

f (b) = Þ f ( x) =
2
b +1 x2 +1
2. (b) If 1 £ x < 2 Þ [x] = 1 Þ [ y] = ±1
Þ y Î[-1, 0) È [1, 2)

x
O p/4 p/2
5. (d) Eliminating y from two equations,
We get 2mx + 4 = x 2 Þ x 2 - 2m - 4 = 0
x1 and x2 are roots of this quadratic equation

If 2 £ x < 3 Þ [x] = 2 Þ y = ±2 y = mx + 2
Þ y Î[-2, -1) È [2, 3) 2
If 3 £ x < 4 Þ [x] = 3 Þ y = ±3
Þ y Î[ -3, -2) È [3, 4) x1 O x2
If 4 £ x < 5 Þ [x] = 4 Þ y = ±4 \ x1 + x2 = 2m and x1x2 = -4
Þ y Î[-4, -3) È [4, 5) x2
æ x2 ö
Clearly, the required area consists of eight Now, ò ç ç mx + 2 - ÷ dx
2 ÷ø
squares, each of area unity (see figure) x1 è
\ Required area = 8 sq. units m 2 2 1 3 3
3. (a) As point lies on the line. Locus of the point = ( x2 - x1 ) + 2( x2 - x1 ) - ( x2 - x1 )
is straight line perpendicular to given line passing 2 6
ém 1 2 2 ù
x = ( x2 - x1 ) ê ( x2 + x1 ) + 2 - ( x1 + x1x2 + x2 )ú
through (2 3, -1) i.e. y + =1 ë2 6 û
3
= ( x2 + x1 ) 2 - 4 x1 x2
3 ´1
Þ required area = .
2 ém 1 2 ù
ê 2 ( x2 + x1 ) + 2 - 6 {( x2 + x1 ) - x2 x1}ú
ë û
y = 3x - 7
2
é m2 + 4 ù
= 4m + 16 ê ú
O êë 3 úû
P
Clearly above is minumum if m = 0
6. (d) Required area = 4A , where
p p
A = ò ( x + sin x)dx - ò xdx
4. (c) Here f ( x ) = min{| tan x |,|cot x |}
0 0
p/4 p/2
Required area = 2 ò tan xdx + 2 ò cot xdx
0 p/4
Solutions 177
Both the curves pass through origin.
1
y=x
\ Required area = ò ( xe x - xe- x )dx
0
1
A p y = sin x + y = f(x)
–1
= ò x(e x - e- x )dx
f (x) 0
1

ò
x -x x1 -x
p 2p = x(e + e ) |0 - (e + e )dx
0
p2 p2 æ 1ö 1 2 1
A = - cos p + cos0 - =2 sq. units. = çe + ÷ - e
x
+ e- x =
2 2 è eø 0 e 0
7. (b) Let the equation of curve
y2(2a – x) = x3 ...(i) 9. (a) If both x + y - 1 and 2 x + y - 1 are
and equation of line x = 2a ...(ii) positive, then x + y – 1 + 2x + y – 1 = 1
The given curve is symmetrical about x-axis and Þ 3x + 2y = 3
passes through origin. x y
x3 Þ + =1
From (i) we have, y2 = 1 3/ 2
2a - x If x + y - 1 is positive and 2 x + y - 1 is
x3
But < 0 for x > 2a and x < 0 negative, then (x + y – 1) – (2x + y – 1) = 1
2a - x Þ – x – 1 = 0 Þ x = –1
So, curve does not lie in the portion x > 2a and If first negative and second positive, then
x < 0, therefore curve lies in 0 £ x £ 2a . –( x + y – 1) + (2x + y – 1) = 1
\ Area bounded by the curve and line Þ x–1=0
2a 2a Þ x=1
x3 / 2 If both negative, then
= ò ydx =
ò 2a - x
dx
– (x + y – 1) – (2x + y – 1) = 1 Þ– 3x – 2y + 1 = 0
0 0
Put x = 2a sin2q and dx = 4a sin q cosq dq x y
Þ – 3x – 2y + 1 = 0 Þ + =1
p/2 1/ 3 1/ 2
é3 1 pù
ò 8a
2
\ I= sin 4 q dq = 8a 2 ê . . ú Y
0
ë4 2 2û 3
m +1 n +1ö
æ p/2 G G
2 2 ÷
ç using
çè ò
sin m x cos n x dx =
2G
m+n+2 ÷
÷
2
3/2
0
2 ø
3x

3x
+2

3pa 2
+
y=

2y

= sq. unit
1

2 1/2
3

8. (a) Y y = x ex y = xe–x X' O 1/3 1 X


–1

–1

) Y'
1,e \ Required area
A(
B(1, 1
e) æ1 ö 1æ4 ö 1 2
X = ç ´ 2 ´ 3÷ - ç ´ 2÷ + ´ 1 ´
O x=1 è2 ø 2è3 ø 2 3
= 2 sq units
Given curves are y = xe x and y = xe - x
10. (c) y = 4 - x 2 is the parabola EMA
æ 1ö y = |x – 2| is the pair of straight lines DCA and ABF,
Line x = 1 meets the curves at A(1, e) and B ç 1, ÷ . y = (x – 2)1/3 is the curve IAHBG
è eø
178 MATHEMATICS
Y p
y=2–x M (0, 4)
y = x –2
Also when x ® 0, t ® 0 and when x ® ,
G 4
C(–1, 3)
D (–2, 4) y = (x–2)
1/3
p
H B (3, 1) t ® tan
8
p
tan
X 8 2 -1
E (–2, 0) O A (2, 0) 4t 4t
\ A= ò 2 2
dt = ò dt
0 (1 + t ) 1 - t 0 (1 + t ) 1 - t 2
2

I (0, –2) 12. (d) Given curves are y = x2 and y = x3


Also, x = 0 and x = p, p > 1
Thus f (x) = max. {4 – x2, |x – 2|, (x – 2)1/3} is Now, intersecting point is (1, 1)
y = x3
ì 2 - x - 2 £ x < -1
ï 2
ï4 - x , - 1 £ x < 2 y = x2
f ( x) = í
1/ 3
ï( x - 2) , 2 £ x < 3
ï x - 2, 3 £ x £ 4
î (p ,0)
Now, area bounded by the curve and x-axis. (0.0)(1,0) x = p
-1 2 3
= ò (2 - x )dx + ò ò
(4 - x) dx + ( x - 2)1/3 dx
2 -1 0
4

( ) ( )
1 p
ò
+ ( x - 2)dx 2 3 3 2
Required area = ò x - x dx + ò x - x dx
0 1
3
1 p
7 3 3 59 1 x3 x4 x 4 x3
= +9+ + = sq. units. = - + -
2 4 2 4 6 3 4 4 3
11. (b) The given curves are 0 1
x 1 æ 1 1 ö æ p4 p3 1 1ö
1 + sin x 1 + tan Þ = ç - ÷ +ç - - + ÷
y= = 2 ...(1) 6 è 3 4ø è 4 3 4 3ø
cos x x
1 - tan 1 1 1 1 1 3 p 4 – 4 p3
2 Þ – + + – =
6 3 4 4 3 12
x
1 - sin x 1 - tan p ( 3 p - 4)
3
and y = = 2 ...(2) Þ = 0 Þ p3 (3p – 4) = 0
cos x x 12
1 + tan 4
2 Þ p = 0 or
\ The area bounded by the above curves, by 3
Since, it is given that p > 1 \ p can not be zero.
p
the lines x = 0 and x = is given by 4
æ
4
x xö
Hence, p =
3
p/4
ç 1 + tan 2 1 - tan ÷
2 13. (d) Let I be the smaller portion and II be the
A= ò ç - ÷ dx greater portion of the given figure then,
0 ç 1 - tan x x
1 + tan ÷
è 2 2ø
x
2
x+

p/4 2 tan Y
y=

2
= ò x
dx
(0, 2)
0 1 - tan 2
2 I
x 1 x
Let tan = t Þ sec2 dx = dt X¢ II X
2 2 2 (– 2, 0)
2
Þ dx = dt
1+ t2


Solutions 179
0 The graph of given region is as follows-
Area of I = ò éê 4 - x 2 - ( x + 2 ) ùú dx
-2 ë û
0 0
éx 4 æ xö ù é x2 ù (1, 2)
=ê 4 - x 2 + sin -1 ç ÷ ú - ê + 2 x ú (– 4, 1)
ë2 2 è 2ø û Q
-2 ëê 2 ûú -2
P

- é 4 ù p S T R
= ëé 2sin ( -1) ûù - ê - + 4ú = 2 ´ - 2 = p - 2
1
(– 3, 0)
ë 2 û 2
Now, area of II = Area of circle – area of I.
= 4p – (p – 2) = 3p + 2 Required area
= Area (trap PQRS) – Area (PST + TQR)
area of I p-2
Hence, required ratio = = 1 é -3 1 ù
area of II 3p + 2 = ´ (1 + 2 ) ´ 5 - ê ò-4 -x - 3 dx + ò-3 x + 3 dx ú
2 ë û
14. (a) The given curve is y = tanx ...(1) éæ 3/2 ö-3 æ 3/2 ö1 ù
15 ê 2 ( -x - 3) 2 ( x + 3)
p = - êç ÷ +ç ÷ ú
when x = , y = 1
4 2 ç -3 ÷ ç 3 ÷ ú
êëè ø-4 è ø-3 úû
Equation of tangent at P is
15 é 2 16 ù 15 3
æ 2 pö æ pö = - + = - 6 = sq. units
y – 1 = ç sec ÷ ç x - ÷ 2 êë 3 3 úû 2 2
è 4ø è 4ø 16. (b) Since,
Y
y = 2x + 1 –
p y = x 2 + x +10
2
1 1
æp ö
P ç ,1÷
Þ x2 + x + = y –10 +
è4 ø 4 4
2
O æ 1 ö æ 39 ö
Þ ç x + ÷ =ç y - ÷
X¢ M X
L
è 2ø è 4ø
y = tan x Þ Latusrectum of the above parabola is 1.
\ length of chord is also 1.

Area is maximum when chord is latusrectum.
Area of shaded reion =
p
or y = 2x + 1 – ...(2) 0
2 Area of rectangle OABC – 2 ò parabola
–1/2
Area of shaded region
= area of OPMO – ar (DPLM) -1
,10
p 2
1 B C(10, 10)
= ò 0
4 tan x dx - (OM - OL)PM
2
p
1 ì p p - 2ü
= [ log sec x ] - í - 4
0 ý ´1
2î4 4 þ
1é 1ù
= êlog 2 - ú sq. unit
2ë 2û A –1/2 O
0
15. (c) y³ x + 3 Þ y2 = x + 3
= 10 – 2 ò ( x 2 + x +10)dx
ï ( x + 3 ) if x < -3
ì- –1
Þ y2 = í ....(i)
ïî ( x + 3 ) if x ³ -3 2
0
é x3 x2 ù
x+9 = 10 – 2 ê + +10 ú
Also y £ and x £ 6 ....(ii)
5 ëê 3 2 ûú –1
2
Solving (i) and (ii) we get intersection points as
(1, 2), (6, 3), (– 4, 1), (–39, –6) é –1 1 10 ù 1
= 10 – 2 ê + – ú =
ë 24 8 2 û 6
180 MATHEMATICS
Y
sin x x sin x x 2
17. (b) y = + e and y = +
x x 2
Required area of the region

ï sin x x
æ sin x x 2 ö üï
= ò1 í + e - ç + ÷ ý dx O 1 2 3
X

îï x è x 2 øï
þ
2 21. (4) The desired region consists of four disjoint
2æ x x ö 2 é x x ù 3
squares. So the area = 1 × 4 = 4 square unit.
= ò1 ç e - 2 ÷ dx = ê e - 6 ú
è ø êë úû1
6
2 8 æ 1ö 2 7 5
= e - -çe- ÷ = e -e-
6 è 6ø 6 4
3
18. (1) The graph of f (x) = min{x – [x], – x – [–x]}
is shown as in figure. Therefore -2 -1
2 l
1 2 3 45
ò min {x - [ x ], - x - [- x ]}dx -2
-3
-2
æ1 1ö 22. (1) The curves y = [a] x2 and
= Area of shaded region = 4ç ´ 1 ´ ÷ = 1
è 2 2ø 1
Y y = [ a ] x 2 represent parabolas which are
2
1 symmetric about y-axis. The equation
y2 – 3y + 2 = 0 gives a pair of straight lines
2 y = 1, y = 2 which are parallel to x-axis.
X Thus the area bounded is shown as;
–2 –3/2 –1 –1/2 0 1/2 1 3/2 2
19. (2) The area of the region is Y
2 2
log e n y = [a]x y =1/2 [a]x
æ -x 1ö æ n - 1 - loge n ö ...(1)
A=4 ò çè e - ÷ø dx = 4 çè
n n ÷ø
y=2
0
y=1
1
1–
n
O X
From above figure the required area
– logen logen y =2 2 æ ö
2y y
–1+
1 =2 ò ( x2 - x1 ) dy = 2 ò ç [ a ] - [ a ] ÷ dy
n y =1 1 è ø
Now, n - 2 ³ 2loge n Þ 2n - 2 - 2log e n ³ n 2( 2 - 1)
2
2( 2 - 1) 2 3/2 2
n - 1 - log e n 1
=
[a] ò y dy =
[a]
- .( y )1
3
Þ ³ 1
n 2
1 4 ( 2 - 1) 3/2
= . (2 - 1)
\ From (1), A ³ 4 ´ =2 3 [a]
2
20. (3.33) Here both functions f (x) and g(x) 4 (5 - 2 - 2 2 ) 4 (5 - 3 2 )
are periodic. Thus required area = =
3 [a ] 3 [a]
æ1 ö 1
é x 3/2 x 3 ù \ Area is greatest, when [a] is least, i.e., 1.
ç
ò ÷
2
= 10 ( x - x )dx = 10 ê - ú \ Area is greatest, when [a] = 1
ç ÷ êë 3 / 2 3 úû 0
è0 ø Þ a Î éë1, 2 )
10
= = 3.33.
3
Solutions 181

f ( x + h) - f ( x) 2x 2
23. (2) f ¢(x)= lim Thus f ( x) = ( x - 4) . f ( x) meet the x-axis
h® 0 h 3
at (0, 0) (–2, 0) and (2, 0).
f ( x(1 + h / x )) - f ( x )
= lim Since f ( - x) = - f ( x ) , the curve y = f ( x ) is
h® 0 h
symmetrical about the origin.
f ( x) f (1 + h / x ) - f (1)
= lim Also, as a = 2, f '( x) > 0 for x < -2 / 3,
x h® 0 h/x
f ( x) x > 2 / 3 and f '( x) < 0 for
= · f ' (1)
x -2 / 3 < x < 2 / 3 .
2 f ( x) f '( x) 2 Thus, shape of y = f ( x ) is as shown in figure
\ f ¢ (x) = or =
x f ( x) x Y
Integrating both sides, we get, f (x) = cx2 C(2, 0)
Since f (1) = 1, \ c = 1
Y
f (x) = x2
E B(2, 0)
A O X
2 D
y= x2 + y 2 = 4
1+ x2
F
(–1,0) O (1,0) X Area of the region ACBDOEA
So, f (x) = x 2 0 2
= 2p - ò f ( x ) dx + ò f ( x )dx
2 -2 0
Now, 2
= x2 Þ x 4 + x2 - 2 = 0 0 0
1+ x
Þ x2 = 1 Þ x = ± 1
But ò-2 | f ( x) | dx = - ò-2 f ( x)dx
0 2
é1 æ 2 ù = - ò f ( -t )( -1) dt = ò f (t )dt

Required area = 2 ê ò ç - x ÷ dx ú 2 0
êë 0 è 1 + x 2 ø ú
û \ Area (ACBDOEA) = 2p
Thus, the area of the other region
é 3 ù1 ép 1 ù AEODBFA = 2p.
- x
= 2 ê 2 tan x - ú = 2 ê - ú
1 25. (7)
ë 3 û0 ë2 3 û
Y
æ 2ö 4
= çè p - ÷ø sq. units.
3
24. (0) Since y = f ( x ) has relative extremes at
3
x = ± 2 / 3, these points are critical points, and
hence they must be roots of f '( x) = 0 (clearly
2
is differentiable everywhere). Thus
f '( x) = a ( x - 2 / 3)( x + 2 / 3) 1
2
= a ( x - 4 / 3)
Þ f ( x ) = a ( x 3 / 3 - 4 x / 3) + b . This passes O
–2 –Ö 2 –1 1 Ö2 2 X
through (0, 0) and (1, –2). So b = 0 and
a (1/ 3 - 4 / 3) = -2 Þ a = 2 . –Ö3 Ö3
182 MATHEMATICS
As we know that fractional part of any thing must lie \ Area of region bounded by y = {x} 2
between 0 and 1 thus between x-axis for the x Î [0, 2] is
Y 1 2 3
= ò x 2 dx + ò ( x 2 - 1) dx + ò ( x 2 - 2) dx
0 1 2
{x}2 2
x 2 –1 x 2 – 2 x 2 – 3 + ò ( x 2 - 3) dx
3
x2 7
A0 = 2+ 3-
3
æ 7ö
\ Required area = 2A0 = 2 ç 2 + 3 - ÷
O 1 2 3 2 X è 3ø

CHAPTER
25 Differential Equation and its Applications

1. (c) Let P (x, y) be the point on the curve passing


through the origin O (0, 0) and let PN and PM be f ( x ). f '( x)
or ³1
the lines parallel to the x-and y-axis, respectively. If 1 + f 4 ( x)
the equation of the curve is y = y (x), the area POM Integrating with respect to x, from x = a to x = b
1
( )
x x b
tan –1 ( f 2 ( x)) ³ b – a
equals ò ydx and the area PON equals xy - ò ydx . 2 a
0 0 1ì –1 2 ü
Assuming that 2(POM) =PON, we therefore have or (b – a) £ í lim (tan ( f ( x ))ý
x x x
2 î x ®b - þ
2ò y dx = xy - ò y dx Þ 3ò y dx = xy (
– lim tan –1 ( f 2 ( x))
x ®a +
)
0 0 0
y p
Þ b–a £
P(x, y) 24
N 3. (c) Differentiating w.r.t. x the given equation,
we have 1 - ( f '( x)) 2 = f ( x)
If y = f(x), then we have,
x
O M 2
æ dy ö dy
1 - ç ÷ = y2 = ± 1- y2
è dx ø dx
dy
Þ = ±dx
Differentiating both sides of this gives 1 - y2
dy dy dy dx
3y = x + y Þ 2 y = x Þ =2 Þ sin -1 y = C ± x Þ y = sin(C ± x)
dx dx y x
Since f(0) = 0 so 0 = sin C Þ C = 0
Þ log y = 2log x + C Þ y = Cx 2 Thus f ( x ) = ± sin x
With C being a constant. This solution
represents a parabola. We will get a similar result As f ( x ) ³ 0 for x Î [0, 1] so we have
if we have started instead with 2 (PON) = POM f ( x ) = sin x.
3 1 Since sin x < x " x > 0, we get f ( x) < x for
2. (c) f '( x ) ³ f ( x ) +
f ( x) 0 < x £ 1. Thus f (1/ 2) < 1/ 2 and
4
or f '( x ). f ( x) ³ 1 + f ( x ) f (1/ 3) < 1/ 3.
Solutions 183

4. (a) Put y = x sec q


Þ 2v cos v = x dv (v sin v - cos v)
dy dq dx
\ = x sec q tan q + sec q
dx dx Separating the variables, we get
From the given equation, 2dx æ v sin v - cos v ö
Þ =ç dv
æ dq
x 3 ç x sec q tan q + sec q÷
ö x è v cos v ÷ø
è dx ø Integrating 2 ln x + ln c = ln sec v - ln v
= x3 sec3 q+ x3 sec2 q tan q sec v
Þ cx 2 = Þ sec v = cx 2 v
Dividing both sides by x 3 sec q , we get v
dq y
x tan q + 1 = sec 2 q+ sec q tan q Þ sec = cxy
dx x
dq where ‘c’ is an arbitrary constant.
x = (tan q+ sec q) 7. (c) Differentiating xy = C , we get y + xp = 0
dx
dx dy
or (sec q- tan q)d q = where p = .
x dx
Integrating we get
Replacing p by p + tan p / 4 = p + 1 ,
ln(sec q+ tan q) - ln sec q = ln x + ln c 1 - p tan p / 4 1 - p
y 2 - x2 p +1
or 1 + = cx Þ y + y 2 - x 2 = cxy we have y + x=0
y 1- p
where c is an arbitrary constant. dy y / x + 1
5. (a) Put x2y2 = z Þ = Putting y / x = v , the last
dy
dx y / x - 1
Given, x 2 × 2 y + y 2 × 2 x = tan( x 2 y 2 ) equation reduces to
dx
dv v + 1 v 2 - 2v - 1
d 2 2 x = -v= -
( x y ) = tan( x 2 y 2 ) put x2y2 = z dx v - 1 v -1
dx dx 2(v - 1)
Now, given expression transform to Þ 2 =- dv
x v 2 - 2v - 1
dz
= tan z \ ò dx = ò cot z dz Þ 2ln x = - ln (v 2 - 2v - 1) + ln C
dx
x = ln (sin z) + C Þ ln x 2 ( y 2 / x 2 - 2 y / x - 1) = ln C
p p Þ y 2 - 2 xy - x 2 = C , which does not
when x = 1, y = Þ z= Þ C=1
2 2 represent a pair of straight lines if C ¹ 0 .
\ x = ln sin (x2y2) + 1 \ ln sin (x2y2) = x – 1 8. (a) The given equation can be written as
sin (x2y2) = ex – 1 æ dx dy ö ( x2 dy – y 2 dx)
æ yö ç - ÷+ =0
è x y ø ( x – y) 2
6. (b) x ç cos ÷ ( y dx + xdy)
è xø
æ dy dx ö
æ yö ç – ÷
= y sin ç ÷ ( x dy - y dx ) æ dx dy ö è y 2 x 2 ø
èxø Þ ç – ÷+ =0
è x y ø æ 1 1 ö2
2
Dividing both sides by x dx we get ç – ÷
è y xø
æ y ö æ y dy ö y æ y ö æ dy yö dy dx
Þ ç cos ÷ ç + ÷ = sin ç ÷ ç - ÷ –
è ø è
x x dx ø x è x ø è dx xø æ dx dy ö y 2 x2
Which is homogeneous equation Þ ç - ÷+ =0
è x y ø æ 1 1 ö2
dy dv ç – ÷
Putting y = vx we get = v+x èx yø 1
dx dx Integrating, we get: ln | x | – ln | y | – =c
æ dv ö æ dv ö æ1 1ö
or cos v ç v + v + x ÷ = v sin v ç x ÷ ç x – y÷
è dx ø è dx ø è ø
184 MATHEMATICS
10. (b) The given equation can be written as
x xy x xy
Þ ln – = c Þ ln + =c ( ydx - xdy) + x xy (x + y)dx + y xy ( x + y)dy = 0
y y-x y x- y
a 2 dx x y Þ ( y dx - x dy ) + ( x + y ) xy ( x dx + y dy ) = 0
9. (b) Here, × = + -2 y dx - x dy æ x ö x æ x 2 + y 2 ö
xy dy y x Þ + ç + 1÷ × dç ÷÷ = 0
y2 ç
dy 2 èy ø y è 2 ø
Þ a2 = ( x + y 2 - 2 xy )
dx æxö æ x2 + y 2 ö æ x ö x
Þ dç ÷+d ç +1 =0
dy ç 2 ÷÷ ç y ÷ y
Þ ( x - y )2 × = a2 , è yø è øè ø
dx æxö
dç ÷
dy dv æ x2 + y2 ö è yø
put x - y = v \1 - = Þ d çç ÷÷ + =0
dx dx è 2 ø æ x + 1ö × x
æ dv ö ç ÷
Þ v 2 ç1 - ÷ = a 2 èy ø y
è dx ø æ x2 + y 2 ö æ -1 æ x ö ö
dv v 2 dv or d çç ÷÷ + 2d çç tan çç ÷÷ ÷÷ = 0
Þ v 2 - a 2 = v2 Þ 2 = dx è 2 ø è è y øø
dx v - a2 Integrating both the sides, we get
æ a2 ö
Þ ç1 + 2 dv = dx x2 + y2
ç v - a 2 ÷÷ Þ + 2 tan -1
x
=C
è ø 2 y
Integrating both the sides, we get
Hence, (b) is the correct answer.
a v-a dy y3
v + log = x+C 11. (a) = 2x
2 v+a dx e + y 2
a æ x- y-aö dy y 3e -2 x
Þ (x - y) + log ç ÷ = x+C Dividing by e2x : \ =
2 è x- y+aø dx 1 + y 2 e-2 x

Þ y+C =
a æ x- y-aö
log ç \ dy + y 2 e -2 x dy = y 3e -2 x dx
÷
2 è x- y+aø dy
\ ò 2 + ò 2( ye -2 x dy - y 2 e -2 x dx ) = 0
It passes through (3a, a) y
a æ1ö dy
Þ a+C = log ç ÷ \ ò 2 + ò d (e -2 x y 2 ) + c = 0
2 è 3ø y
a æ1ö æ 2 + log 3 ö \ 2ln | y | + e-2 x y 2 = c
Þ C = -a + log ç ÷ Þ C = - a ç ÷
2 è3ø è 2 ø 12. (b) Given curve is a n -1 y = x n ...(i)
a æ x- y-aö Differentiating equation (i) w.r.t. x, we get
\ y = (2 + log 3) + log ç ÷ dy
2 è x- y+aø a n -1 = nx n -1 ...(ii)
dx
x- y-a
Þ = e y - k , where k = a (2 + log 3) Eliminating a from equations using (i) & (ii), we get
x- y+a 2 x n dy
y -k = nx n-1 ...(iii)
x - y 1+ e y dx
\ =
a 1 - e y-k dx dy
Putting - in equation (iii) at the place of ,
æ 1 + e y -k ö dy dx
Þ x = y + aç ,
ç 1 - e y -k ÷÷ we get
è ø
x n dx - xdx
a - . = nx n -1 , Þ = n,
where k = (2 + log 3) y dy ydy
2 Þ – xdy = ny dy,
Which is required equation of curve.
Hence, (b) is the correct answer. x2 ny 2
Þ - +c = , Þ ny2 + x2 = constant.
2 2
Solutions 185

13. (c) Given, equation of normal at P (1,1) is (ii) put sin x - x = t Þ (cos x - 1)dx = dt
ay + x = a + 1 \ y 2 - x = log e 2 ò 2t dt
æ dy ö \ y 2- x = 2t + c
\ Slope of tangent at P = a Þ ç ÷ =a
è dx ø(1,1) \ y = 2 x + t + c2 x
Given
\ y = 2sin x + c2 x
dy dy æ dy ö 16. (c) Rearranging the terms of equation, we get
µy Þ = ky Þ ç ÷ =k=a
dx dx è dx ø (1,1) dt g '( x ) t2
Y -t =-
dx g ( x) g ( x)
Q 1 dt 1 g '( x) 1
Þ- 2 + = ...(i)
t dx t g ( x ) g ( x)
P(1, 1) 1 1 dt dz
curve Normal Let z = Þ - 2 =
X t t dx dx
O P
Thus, from (i) we obtain dz + g '( x ) z = 1
dy dy dx g ( x ) g ( x)
= ay Þ = adx
dx y dz
(variable being separated) which is clearly linear in z and with
dx
Þ ln y = ax + c g '( x )
ò dx
It is passing through (1, 1) then c = – a I.F. = e g ( x ) = elog[ g ( x )] = g ( x)
a ( x -1) Þ Thus, complete solution is
Þ equation of the curve is y = e
1
æ dy ö z × g ( x) = ò g ( x ) × dx + C
14. (a) Let P be (x, y). C is ç x + y , 0 ÷ and B is g ( x)
è dx ø
1 g ( x)
æ dy ö Þ g ( x) = x + c Þ =t
çè 0, y - x ÷ø . Centre of the circle through O, t x+c
dx 2 2
C, P and B has its centre at the mid-point of BC. 17. (a) 1 = lim t f ( x) - x f (t ) æç 0 form ö÷
Let it be (a, b) then t®x t-x è0 ø
2
dy dy 2tf ( x ) - x f '(t )
2a = x + y and 2b = y - x = lim
dx dx
t ®x 1
Þ 2 xf ( x) - x2 f '( x) = 1
dy
B Þ x2 - 2 xy = -1, y = f ( x )
P dx
dy 2 1
Þ - y=- 2
O dx x x
C A which is linear equation whose I.F.
1
D
= e -2log x = 2 .
Now, (a, b) lies on y = x x
Multiplying both sides of the last equation by
so, y - x dy = x + y dy Þ dy = y - x I.F., we get
dx dx dx x + y
d æ y ö 1 y 1
dy
- y log e 2 = 2sin x (cos x - 1) log e 2 ç ÷ = - 4 Þ 2 = 3 + K.
15. (a) dx è x 2 ø x x 3x
dx
Since y = f(1) = 1.
This is linear differential equation
2 1 2 2
I.F. = e - e ò dx = e - x loge 2 = 2- x
log 2 so K = . Thus y = + x
3 3x 3
(i) Solution is dy
dy x
y 2 - x = ò 2 - x 2sin x (cos x - 1) log e 2 dx 18. (2) The equation is y = e dx Þ x = ln y
dx
186 MATHEMATICS
19. (0) (2ny + xy logex) dx = x logex dy Again differentiating both sides w.r.t. x
dy æ 2n ö y ( x) = x 2 y '( x) + y ( x )2 x + xy ( x )
Þ =ç + 1÷ dx
y è x log e x ø or (1 - 3x ) y ( x ) = x 2 y '( x)
Þ log( y ) = 2n log | log x | + x + c and c = 0 y '( x ) æ 1 3 ö
or = -
(Q curve passes through (e, ee)) y ( x ) çè x 2 x ÷ø
2n
\ y = e x + log(log x) = e x (log x )2n Integrating, we get
1
Þ f ( x) = e x (log x)2 n l n y ( x ) = - - 3l n x + l n c
x
Now, æ x3 y ( x ) ö
ì 1 1 x3 y ( x)
ï® ¥ if x<
e
or l n ç ÷ = - or = e -1/ x
ï è c ø x c
ï 1
g ( x) = lim f ( x) = í 0 if <x<e ce -1/ x
n ®¥ e or y ( x) =
ï x3
ï ® ¥ if x>e
ï æ 1ö
î So, y(1) = e Þ c = e2 \ y çè ÷ø = 8
e
2
22. (5) Put x = r sec q and y = r tan q So,
\ ò g ( x ) dx = 0
2 2 2
1/ e x -y = r ... (1)
20. (2) The given differential equation can be y
and sin q = ...(2)
dx x
written as = xy[ x 2 sin y 2 + 1]
dy then differentiating (1) we get,
2 xdx - 2 ydy = 2rdr
1 dx 1
Þ - y = y sin y 2 or xdx - ydy = rdr ... (3)
3 dy 2
x x
This equation is reducible to linear equation, so and differentiaing (2) we get,
1 xdy - ydx
putting - = u, the last equation can be = cos qd q
x 2 x2
written as 2
du or xdy - ydx = x cos qd q
+ 2uy = 2 y sin y 2
dy = r 2 sec 2 q cos qd q = r 2 sec qd q ...(4)
2 Substituting values from (3) and (4) in the given
The integrating factor of this equation is e y .
differetial equation, we get
So required solution is
2 2
ue y = ò 2 y sin y 2 .e y dy + C rdr æ 1+ r2 ö 1+ r2
= çç 2 ÷÷ =
r 2 sec 2 q d q è r ø r
= ò (sin t )et dt + C (t = y 2 )
dr
2 1 y2 2 2
or = sec q d q
Þ ue y = e (sin y - cos y ) + C (1 + r 2 )
2
2 Intergrating both sides,
Þ 2u = (sin y 2 - cos y 2 ) + Ce - y
2 ln(r + (1 + r 2 )) = ln(sec q + tan q) + ln c
Þ 2 = x 2 [cos y 2 - sin y 2 - 2Ce - y ] Where c is an arbitrary constant.
21. (8) Differentiating both sides w.r.t., x of the
given equation or (r + (1 + r 2 )) = c(sec q+ tan q)
x x
æ ö
x. y ( x ) + ò y (t )dt .1 = ( x + 1) x. y ( x ) + ò ty (t )dt x+ y
or ( ( x2 - y2 ) + (1+ x2 - y2 ) = c ç ÷
0 0 çè (x2 - y2 ) ÷ø
x x 23. (1) Area of curvilinear trapezoid
2
or ò y(t )dt = x y ( x ) + ò ty (t )dt
0 0
Solutions 187
x Integrating, we get
OABCO = ò f ( x )dx according to question æ1ö
ln | f ( x) - 1 | + ln f ç ÷ - 1 = c
0 èxø
x æ æ1ö ö
n +1 Þ f ( x ) - 1) ç f ç ÷ - 1 ÷ = ec = k
ò f ( x)dx µ { f ( x)} è èxø ø
0 x æ1ö æ1ö
n +1 Þ f ( x) f ç ÷ - f ( x) - f ç ÷ + 1 = k
or ò f ( x) dx = k{ f ( x)} è xø èxø
0 Y
Þ ( f (1))2 - f (1) - f (1) + 1 = k
x)
y = f( B (substituting x = 1)
C Þ k =1 (Q f (1) = 2, given)
æ1ö æ1ö
Þ f ( x) f ç ÷ = f ( x) + fç ÷
è xø è xø
A
X
O x Þ f ( x) = 1 ± x n (From functional equation)
Where k is constant of proportionality. Since f(1) = 2, we must have f(x) = 1 + xn
Differentiating both sides w.r.t. x, Since f(5) = 26, 26 = 1 + 5n Þ n = 2
f ( x ) = k (n + 1)( f ( x )) n f '( x ) \ f ( x) = 1 + x 2 \ f (6) = 1 + (6)2 = 37
1
or { f ( x)n-1} f '( x) = 25. (1) Here,
k (n + 1)' 2 2 4
Integrating both sides w.r.t. x, f ' ( x ) = 1 + 2 x 2 + × 4 x 4 + × × 6 x 6 + ....¥
3 3 5
{ f ( x )}n x
= +c æd ö
n k (n + 1) = 1 + x ç ( xf ( x)) ÷
è dx ø
{ f (0)}n
Putting x = 0 = 0+ c Þ 0= 0+c Þ f '( x) = 1 + x | xf '( x) + f ( x) |
n
(Q f (0) = 0) Þ (1 - x2 ) f '( x) = 1 + xf ( x)
n
{ f ( x )} x dy x 1
\ = Þ - ×y= 2,
n k (n + 1) dx 1 - x 2 x
n nx
Þ { f ( x)} = k (n + 1) ....... (1)
-x 1
ò 1- x2 dx log|1- x 2 |
I.F. = e = e2 = 1- x2
n
Again putting x = 1 then { f ( x )}n = =1 1
k (n + 1) \ y × 1 - x2 = ò 2
× 1- x2 + C
1- x
(Q f (1) = 1)
From (1), ( f ( x ))n = x or f ( x ) = x1/ n Þ y 1 - x = sin -1 x + C ,
2

Þ (f(10))n = 10 Þ k =1 as f (0) = 0 Þ C = 0
24. (7) Given
sin -1 x
æ1ö æ1ö æ1ö Þy=
x2 f '( x) f ç ÷ - f ( x) f ' ç ÷ = x 2 f '( x) - f ' ç ÷ 1- x2
èxø è xø èxø
é æ 1 ö ù æ 1 ö 3/2 sin -1 x
Þ x 2 f '( x ) ê f ç ÷ - 1ú = f ' ç ÷ [ f ( x) - 1] Þ A=ò dx
ë èxø û èxø 1/ 2
1 - x2
ì æ1ö ü 1 æ1ö
Þ f '( x) í f ç ÷ - 1ý = 2 f ' ç ÷ { f ( x ) - 1} p
î èxø þ x è xø p/ 3 æ t 2 ö 2 1 é p2 p2 ù
æ 1 öæ 1 ö = ò t dt = ç ÷ = ê - ú
f ' ç ÷ç - 2 ÷ p/ 6 ç 2 ÷ p 2 4 36
f '( x) è ø ë û
x
+ è øè
x ø 6
Þ =0
f ( x) - 1 æ1ö \[4 A] = 1.
f ç ÷ -1
èxø
188 MATHEMATICS

CHAPTER
26 Vector Algebra

r r
1. (b) a ( x ) and b ( x) are collinear if and only if a b g
Þ cosec 2 + cosec2 + cosec2 = 2
2 2 2
cos x = x . Now let f ( x ) = x - cos x , then
which is not possible as
f '( x ) = 1 + sin x ³ 0
a ß g
cosec2 ³ 1, cosec2 ³ 1, cosec2 ³ 1.
Þ f ( x) is increasing and hence f (x) = 0 for a 2 2 2
unique value of x. So the vectors cannot be coplanar.
p p 3. (b) a, b, c are distinct non-negative numbers
For x ³ , f ( x ) > 0 and x < , f ( x) < 0 .
3 6 and the vectors ai$ + a $j + ck$ , $i + k$ and
Thus cos x = x , for a unique value of r
ci$ + c $j + bk are coplanar..
æp p ö
x, x Î ç , ÷ .
è6 3 ø a a c a a c–a
r r r \ 1 0 1 =0Þ 1 0 0
2. (d) Suppose that a , b , c are coplanar..
c c b c c b–c
cos a 1 1
Operating C3 ® C3 – C1
Þ 1 cos b 1 =0 Expanding along R2, we get
1 1 cos g a c–a

c b – c = c (c – a) – a (b – c) = 0
applying R2 ¾¾ ® R2 - R1 and R3 ¾¾ ® R1
Þ c2 – ac – ab + ac = 0
cos a 1 1
Þ c2 = ab Þ a, c, b are in G.P.
or 1 - cos a cos b - 1 1 =0 \ c is the G.M. of a and b.
1 - cos a 0 cos g - 1 4. (c) In DABC, let AD is angle bisector of angle A.
A
or cos a(cos b - 1)(cos g - 1)
q q
-(1 - cos a)(cos g - 1) - (1 - cos a)(cos b - 1) = 0 a b
2 2
Dividing through out by
(1 - cos a )(1 - cos ß)(1 - cos g ) ; we get B ak D bk C
cos a 1 1 \ BD = ak , DC = bk \ BC = (a + b)k
+ + =0
1 - cos a 1 - cos b 1 - cos g Applying cosine formula, we have
-(1 - cos a) + 1 1 1 ( AB)2 + ( AC )2 - ( BC ) 2
Þ + + =0 cos q =
(1 - cos a) (1 - cos b) (1 - cos g ) 2( AB)( AC )
1 1 1 a 2 + b 2 - ( a + b) 2 k 2
Þ -1 + + + =0
1 - cos a 1 - cos b 1 - cos g = ...(1)
2ab
1 1 1
Þ + + =1
1 - cos a 1 - cos b 1 - cos g
Solutions 189
Also in DADC and DABD uur uur 1 uur uur 1
Þ a . b - 2. = l Þ a . b = l + .
q b 2 + ( AD)2 - b 2 k 2 a 2 + ( AD)2 - a 2 k 2 4 2
cos = =
2 2b AD 2a AD Again,
uur uur uur uur uur uur uur uur uur
Þ ( AD) 2 = ab(1 - k 2 ) b - 2 c = l a Þ b.b - 2b .c = l b .a
ïì a + b - 2ab cos q ïü
2 2 uur uur æ 1ö
= ab í1 - ý [from (1)] Þ 16 - 2 b . c = l ç l + ÷
îï (a + b)2 þï è 2ø
4a 2 b2 cos 2 q / 2 2ab cos q / 2 uur uur 2
= Þ AD = Þ b.c = 8- l - l .
( a + b) 2 ( a + b) 2 4
r r r r
uuur (ab + b a) ab æ a b ö Furthermore,
\ AD = ± =± ç + ÷ uur uur uur uur uur uur uur uur uur
(a + b) (a + b) è a b ø b - 2c = l a Þ b.c - 2 c.c = l a.c
ab 2
=± ( aˆ + bˆ) Þ 8 - l - l - 2(1) = l æç 1 ö÷
(a + b)
2 4 è 4ø
uuur
uuur AD (aˆ + bˆ)
\ AD = =± Þ l 2 + l - 12 = 0 Þ l = -4, 3.
AD 2cos q / 2 ur r r
uur uur uur uur uur uur 7. (b) Let the triangle be PQR with sides p, q, r
5. (a) Let A B = a and AD = b and AC = c ur r r r r
uur uur uur Let p = 3(aˆ ´ b ) and q = b - (aˆ. b) aˆ
when a , b and c are non-collinear coplanar ur r r r r
vectors. \ p.q = 3(aˆ ´ b).{b - (aˆ.b) aˆ}
uuur uuur uuur r r rr r uur
DB = AB - AC = a - b . = 3[aˆ b b] - 3(aˆ.b)[aˆ b aˆ ] = 0
uur uuuur uur uur uur uur uur uur uur p
Now, DB. AB = ( a - b ).( a ) = a . a - b . a Þ ÐR = .
2
ur r r
2 c 2 - a 2 - b 2 3a 2 + b 2 - c 2 Also | p |= 3(aˆ ´ b) = 3 | aˆ | | b | sin q
a 2 - ab cos q = a - =
2 2 r
= 3 | b | sin q ,
é a 2 + b2 - c 2 ù r
ê\ In DABC , cos(p - q ) = ú q is the angle between â and b and | aˆ |= 1
êë 2ab úû r r r r r
D C and | q |= {b - ( aˆ.b)aˆ}.{b - ((aˆ.b ) aˆ}
r r r
= | b |2 -2(aˆ.b)2 + (aˆ.b)2 (aˆ )2
®
b ® r r r
c
= | b |2 + | b |2 cos 2 q. - 2 | b |2 cos 2 q
q r
p–q = | b | sin q
A ® B
a
uur uur uur R
6. (a) Given | a | = 1, | c | = 1 and | b | = 4 ®
q
uur uur 1 1 ®
p
Þ a . c = 1.1. = .
4 4
uur uur uur uur uur uur uur Q P
Again, b - 2 c = l a Þ a . b - 2 a . c = l a 2 ®
r
190 MATHEMATICS
ur ur
| p| 10. (c) Vol. of paralleopiped formed by
r = tan P Þ P = p
| p| r r
\ r = 3 . But
|q| |q| 3 u = $i + a$j + $
k , v = $j + ak$, w
µ = ai$+ $
k is
é pù r r ur
1 a 1
êëQ ÐR = 2 úû V = [u v w] = 0 1 a
p p æ 1 ö a 0 1
\ P = = tan -1( 3) , Q = = tan -1 ç ÷
3 6 è 3ø
= 1(1 - 0) - a(0 - a 2 ) + 1(0 - a) = 1 + a 3 - a
p
and R = = cot -1 (0) . dV
2 For V to be min =0
p da
[Note that as soon as we get R = , only one
2 1
p Þ 3a 2 - 1 = 0 Þ a=± .
option has the value of an angle = cot -1 (0) , 3
2 11. (b) We observe that
hence (b) is the correct answer]. rr
r r rr r r æ b.a ö r r r r r r
8. (c) For the vector a and b to be inclined at an a.b1 = a.b - ç r 2 ÷ a.a = a.b - a.b = 0
rr è|a| ø
obtuse angle, we must have a. b < 0 for all
rr r ur
x Î (0, ¥) rr r æ r c.a r c.b r ö
a.c 2 = a. ç c - r a - uur 1 b1 ÷
Þ c (log 2 x)2 - 12 + 6c log 2 x < 0 for all çè | a |2 | b1 |2 ÷ø
x Î (0, ¥) rr r ur
r r r a.c r 2 c.b r r
Þ cy 2 + 6cy - 12 < 0 for all y Î R , = a.c - c. r | a | - uur 1 (a.b1 )
where y = log 2 x | a |2 | b1 |2
rr rr rr
Þ c < 0 and 36c 2 + 48c < 0 , = a.c - a.c - 0 = 0 [ Q a.b1 = 0 ]
(using ax 2 + bx + c < 0, "x Î R if a < 0 and D < 0 ) rr r ur
r r r æ r c.a r c.b r ö
æ 4 ö and b1 .c 2 = b1. ç c - r a - uur 1 b1 ÷
Þ c < 0 and c (3c + 4) < 0 Þ c Î ç - , 0÷ . çè | a |2 | b1 |2 ÷ø
è 3 ø
ur r r r ur r r ur
9. (b) p . q = ab + bc + ca r r (c.a)(b .a ) c.b r r
= b1.c - r 1
- uur 1 b1.b1
= a 2 + b 2 + c 2 b 2 + c 2 + a 2 cos q | a |2 | b1 |2
r r r r r
ab + bc + ca = b1.c - 0 - b1.c = 0 (Using b1.a = 0 )
Þ cos q = . rr r uur r r
(a 2 + b2 + c 2 ) Hence a.b1 = a.c2 = b1.c 2 = 0 .
Þ Now (a - b)2 + (b - c )2 + (c - a) 2 ³ 0 r uur r
Þ (a, b1, c 2 ) is a set of orthogonal vectors.
a 2 + b 2 + c 2 ³ ab + bc + ca
ab + bc + ca 12. (d) Let xi$ + y $j + zk$ be the required unit
Þ £ 1.
a 2 + b2 + c 2 vector.
Also Since a$ is perpendicular to (2$i - $j + 2k$ ) .
(a + b + c )2 = a 2 + b 2 + c 2 + 2(ab + bc + ca) ³ 0 \ 2x – y + 2z = 0 ......... (i)
$ $ $
Since vector xi + y j + zk is coplanar with the
ab + bc + ca
Þ ³ -1/ 2 vector $i + $j - k$ and 2$i + 2 $j - k$ .
a 2 + b2 + c 2
1 \ xi$ + y $j + zk$
Þ - £ cos q £ 1 Þ q Î[0, 2p / 3].
2 = p ( $i + $j - k$ ) + q ( 2$i + 2 $j - k$ ),
Solutions 191
where p and q are some scalars. 1 r r r r r r r
= (5a - 3b - 4a - b - a + b ) = -b
Þ xi$ + y $j + zk$ 3
= ( p + 2q)i$ + ( p + 2q ) $j - ( p + q )k$ r 1 r r 1 r r
and y = ( r + s ) = ( -5a ) = - a
Þ x = p + 2q, y = p + 2q, z = – p – q 5 5
r
Now from equation (i), Angle between xr and y , i.e.,
2p + 4q – p – 2q – 2p – 2q = 0 r r r r
x×y a ×b
Þ –p=0Þp=0 cos q = r r = r r ...(3)
\ x = 2q, y = 2q, z = –q. | x || y | | a || b |
From (1) and (2),
Since vector xi$ + y $j + zk$ is a unit vector,, r 25 r r r 43 r r
therefore | a |= a × b and | b |= a ×b
19 19
| xi$ + y $j + zk$ | = 1 Þ x + y + z = 1
2 2 2 r r 25 ´ 43 r r
\ | a || b |= × a ×b
Þ x2 + y2 + z2 = 1 Þ 4q2 + 4q2 + q2 = 1 19
æ 19 ö
1 q = cos -1 ç ÷
Þ 9q2 = 1 Þ q = ± è 5 43 ø
3 r r
1 2 2 1 rrr r r r
When q = , then x = , y = , z = – . 15. (d) Let d × a = (cos y )[a b c ] = -d × (b + c )
3 3 3 3 r r r r
1 2 2 1 [as d × (a + b + c ) = 0]
When q = – , then x = – , y = – , z = . r r r
3 3 3 3 d × (b + c ) .
Þ cos y = - r r r ...(1)
2$ 2 $ 1 $ [a b c ]
Here required unit vector is i + j - k r r r
3 3 3 d × (a + b )
Similarly, sin x = - r r r ...(2)
2$ 2 $ 1 $ [ a b c ]
or - i - j + k . r r r
3 3 3 d × (a + c )
13. (b) Let A be the first term and D be the common 2=- rrr . ...(3)
[a b c ]
difference of the corresponding AP. Then,
Adding above equations, we get
1 1
= A + ( p - 1) D, = A + ( q - 1) D , sin x + cos y + 2 = 0
a b
Þ sin x + cos y = -2
1
= A + (r - 1) D Þ sin x = -1, cos y = -1
c
Þ x = (4n - 1)p / 2, y = (2n + 1); n Î Z
Þ a -1 (q - r ) + b-1 (r - p) + c -1 ( p - q) = 0
r r r r x = (4n - 1)p / 2, y = (2n + 1) p .
Þ v × u = 0 Þ u ^ v.
r r Since we want minimum value of x2 + y2,
Hence u and v are orthogonal vectors.
p 2
r r
14. (b) We have p × q = 0 so x = - , y = ±p Þ x 2 + y 2 = 5p .
2 4
r r r r
r
Þ (5a - 3b ) × (-a - 2b ) = 0 r
r 16. (d) As angle between a and b is obtuse,
r r r
Þ 6 | b |2 -5 | a |2 -7a × b = 0 ...(1) r r
r r a ×b < 0
Also r × s = 0
r r r r Þ (2l2iˆ + 4lˆj + kˆ) × (7iˆ - 2 ˆj + lkˆ) < 0
Þ (- 4a - b ) (-a + b ) = 0
r r r r Þ 14l 2 - 8l + l < 0 Þ l(2l - 1) < 0
Þ 4 | a |2 - | b |2 -3a × b = 0 ...(2)
1
Þ0<l< ...(1)
Now xr = 1 ( pr + rr + sr ) 2
3
192 MATHEMATICS
r aˆ
Angle between b and k̂ is acute and less than 18. (3) C B
p bˆ
.
6 b̂
r r
b × kˆ = | b | × | kˆ | cos q 60°
O A
a
Þ l = 53 + l 1× cos q2 uuur uuur
From the figure, OA = aˆ, OC = bˆ
l p p uuur uuur uuur
Þ cos q = \ q< Þ cos q > cos
53 + l 2 6 6 OA + OC = OB = aˆ + bˆ .

3 l 3 Now | aˆ + bˆ |= 1 \ DOAB is equilateral.


Þ cos q > Þ >
2 53 + l 2 2 OA2 + OC 2 - AC 2
\ In DAOC, cos120° =
2
Þ 4l 2 - 3(53 + l 2 ) > 0 Þ l > 159 2OA × OC

Þ l < - 159 ...(2)


1 1 + 1 - AC 2 r r
From Eqs. (1) and (2), l = f \ Domain of l is \- = \ AC = | a - b |= 3 = k .
null set. 2 2
Þ k = 3.
17. (c) Given,
r r r r 19. (3) ( xˆ + yˆ + zˆ)2 ³ 0
cos q = (a ´ iˆ) × (b ´ iˆ) + (a ´ ˆj ) × (b ´ ˆj )
r r Þ 3 + 2 S xˆ. yˆ ³ 0 Þ 2 S xˆ. yˆ ³ -3
+(a ´ kˆ) × (b ´ kˆ) ...(1)
2 2 2
Consider, Now, xˆ + yˆ + yˆ + zˆ + zˆ + xˆ
r r r r r r
(a ´ iˆ) × (b ´ iˆ) = [(a ´ iˆ)b iˆ] = ((a ´ iˆ) ´ b ) × iˆ = 6 + 2 S xˆ. yˆ ³ 6 + ( -3)
2 2 2
r r r r Þ xˆ + yˆ + yˆ + zˆ + zˆ + xˆ ³ 3 .
((a × b )iˆ) - ((iˆ × b )a )iˆ = (a × b)(iˆ × iˆ) - (iˆ × bˆ)(a × iˆ) r
r r rr 20. (3) F = aaˆ + bbˆ + gcˆ
= a × b - a1b1 .
r r r r r r
Similarly, (a ´ ˆj ) × (b ´ ˆj ) = a × b - a2b2 r Fˆ × (bˆ ´ cˆ)
\ F × (bˆ ´ cˆ) = a[aˆ bˆ cˆ] \ a =
r r r r r r [aˆ bˆ cˆ]
and (a ´ kˆ)(b ´ kˆ) = a × b - a3b3
\ From Eq. (i), we get Now, bˆ ´ cˆ = bˆ ´ (aˆ ´ bˆ) = (bˆ × bˆ)aˆ - (bˆ × aˆ )bˆ
r r 1
cos q = 3a × b - (a1b1 + a2b2 + a3b3 ) = aˆ - bˆ
3
r r r r
= 3a × b - a × b ˆ ˆ] = (aˆ ´ bˆ)2 = | aˆ |2 | bˆ |2 - (aˆ × bˆ)2
r r [aˆ bc
a ×b r r r r 1
Þ r r = 2 a × b Þ | a || b |= . 1 8
| a || b | 2 = 1- =
r r 9 9
Now, use AM ³ GM on | a |, | b |
r r 1 9ì 1 ü 9 3
|a| +|b | r r r r 2 \a = í Fˆ × aˆ - Fˆ × bˆ ý \ k1 = , k2 =
\ ³ (| a | × | b |) 2 Þ| a | + | b | ³ 8î 3 þ 8 8
2 2
r r 12 3
Þ| a|+|b |³ 2 . Þ k1 + k2 = = \ 2(k1 + k 2 ) = 3.
8 2
Solutions 193
r r are non-coplanar and which is a vector parallel
21. (5) a . b = 0 Þ x1 + x2 + x3 = 0
to ar
We have to obtain the number of integral solution r r r r r r r r
of this equation Þ Coefficient of Þ A ´ a = -2[b c d ](a ´ a ) = 0
r r
x 0 in ( x -3 + x -2 + x -1 + x 0 + x + x 2 )3 Þ| A ´ a |= 0 .
3 n +1
æ 1 + x + x 2 + x3 + x 4 + x5 ö r r æ é 0 -1/ 2ù ö r
0 24. (2) vn +1 - vn = ç ê ÷ v0
= Coeff. of x in ç ÷ è ë1/ 2 0 úû ø
è x3 ø
r r é 0 -1/ 2 ù r
= Coeff. of x 9 in (1 - x 6 )3 (1 - x ) -3 v2 - v1 = ê v0
ë1/ 2 0 úû
= 11C9 - 3. 5C3 = 25 . 3
r r é 0 -1/ 2 ù r
uuur uuur v3 - v2 = ê v0
22. (2) AB = 2iˆ + ˆj + kˆ, AC = (t + 1)iˆ + 0 ˆj - kˆ ë1/ 2 0 úû
n
iˆ ˆj kˆ r r é 0 -1/ 2 ù r
uuur uuur vn - vn -1 = ê v0
AB ´ AC = 2 1 1 ë1/ 2 0 úû
t + 1 0 -1 Adding all the equations,
r r r
= -iˆ + (t + 3) ˆj - (t + 1)kˆ vn - v0 = ( A + A2 + A3 + .....An )v0
where
= 1 + (t + 3)2 + (t + 1)2 = 2t 2 + 8t + 11 .
é 0 -1/ 2 ù r r
1 uuur uuur A=ê ú Þ vn = ( I + A + A2 + ....)v0 .
Area of DABC = | AB ´ AC | ë1/ 2 0 û
2
25. (7) Each side subtends an angle of 2p/8 = p/4
1 at the centre of the octagon. Let 'O' be the centre
= 2t 2 + 8t + 1
2 of the octagon and r the radius of the circum-
1 2 circle of the octagon. Therefore
2
Let f (t ) = D = (2t + 8t + 1) uuur uuur æ pör
4 OA1 ´ OA2 = ç r 2 sin ÷ n
f '(t ) = 0 Þ t = -2 è 4ø
r
At t = –2, f '' (t) > 0. where n is the vector perpendicular to the plane
So D is minimum at t = –2. of the polygon such that from the side of nr , the
r r r r r r r r points A1, A2, A3, ...., An are in counterclock sense.
23. (0 ) Let V1 = (a ´ b ) ´ (c ´ d ) = (a ´ b ) ´ p
Hence
(say)
r r r r r r r r r r r r r r uuur uuur æ pö r
= (a × p)b - (b × p)a = a × (c ´ d )b - b × (c ´ d )a OA2 ´ OA3 = ç r 2 sin ÷ n
r r r r r r r r è 4ø
= [a c d ]b - [b c d ]a ...(1) uuur uuur pör
æ
r r r r r r r r OA3 ´ OA4 = ç r 2 sin ÷ n , ..... etc.
Similarly V2 = (a ´ c ) ´ (d ´ b ) = q ´ (d ´ b ) è 4ø
r r r r r r r r Therefore
= [a c b ]d - [a c d ]b ...(2)
r r r r r r r r 7 uuur uuur æ pör
And V3 = (a ´ d ) ´ (b ´ c ) = (a ´ d ) ´ r å (OA j ´ OA j +1 ) = 7 çè r 2 sin 4 ÷ø n
r r r r rrr r j =1
= [a b c ]d - [d b c ]a ...(3) uuur uuur
r r r r rr r r r r r r = 7(OA1 ´ OA2 ) .
\ A = V1 + V2 + V3 = -2[b c d ]a ¹ 0 as b , c , d
194 MATHEMATICS

CHAPTER
27 Three Dimensional Geometry

1. (b) The given equations are 3l + m + 5n = 0...(i) Þ cos 2q = 0 or cos2q = – 1


and 6mn - 2nl + 5lm = 0 ...(ii) Þ 2q = p/2 or 2q = p
From (i), we have m = –3l – 5n. Putting
m = –3l – 5n in (ii), we get p ép pù
Þ q = p/4 or q = Þq= ê , ú
6(-3l - 5n )n - 2nl + 5l (-3l - 5n) = 0 2 ë 4 2û
3. (b) Let OA and OB be two lines with DC's l1,
Þ (n + l )(2n + l ) = 0
m1, n1 and l2, m2, n2. Let OA = OB = 1. Then
Þ either l = –n or l = –2n. co-ordinates of A and B are (l1, m1, n1) and
If l = –n, then putting l = –n in (i), (l2, m2, n2) respectively. Let OC Z be the
we obtain m = –2n. bisector of ÐAOB such that C is the mid-
If l = –2n, then putting l = –2n in (i),
point of AB and so its co-ordinates are
we obtain m = n.
Thus, the direction ratios of two lines are æ l1 + l2 m1 + m2 n1 + n2 ö
–n, –2n, n and –2n, n, n i.e., 1, 2, –1 and –2, 1, 1. ç 2 , 2
,
2 ÷ø
è
Hence, the direction cosines are
l1 + l2 m1 + m2 n1 + n2
1 2 -1 -2 1 1 \ DR's of OC are , ,
, , or , , . The angle q 2 2 2
6 6 6 6 6 6
\ We have
between the lines is given by
2 2 2
-2 -1 -1 æ l + l ö æ m + m2 ö æ n1 + n2 ö
cos q =
1
´ + ´ +
2
´ =
1 1 OC = ç 1 2 ÷ + ç 1 ÷ +ç ÷
6 6 6 6 6 6 6 è 2 ø è 2 ø è 2 ø

1
æ -1 ö = (l12 + m12 + n12 ) + (l22 + m22 + n22 ) + 2(l1l2 + m1m2 + n1n2 )
Þ q = cos -1 ç ÷ . 2
è 6 ø
1
= 2 + 2 cos q [Q cos q = l1l2 + m1m2 + n1n2 ]
2. (c) It makes q with x and y-axes. 2

l = cosq, m = cosq, n = cos (p – 2q) 1 æqö


= 2(1 + cos q) = cos ç ÷ .
we have l2 + m2 + n2 = 1 2 è2ø

Þ cos2q + cos2q + cos2 (p – 2q) = 1 l1 + l2 m1 + m2 n1 + n2


\ DCs of OC are 2(OC ) , 2(OC ) , 2(OC )
Þ 2 cos2q + (–cos2q)2 = 1

Þ 2 cos2q – 1 + cos22q = 0 l1 + l2 m + m2 n +n
i.e., , 1 , 1 2
2 cos q / 2 2 cos q / 2 2 cos q / 2
Þ cos2q – [1 + cos2q] = 0
Solutions 195

Y B æ l1 + l2 m1 + m2 n1 + n2 ö
ç 2 , ,
2 ÷ø
è 2 a(cos q + 3) + (b 2)sin q + c(cos q - 3)
C A =
A(l , m , n )
1 1 1 (cos q + 3) 2 + 2sin 2 q
a 2 + b 2 + c2
O +(cos q - 3)2
X' X
E æ l1 - l2 m1 - m2 n1 - n2 ö
ç 2 , 2
,
2 ÷ø
è
D
(- l2 , - m2 , - n2 ) (a + c) cos q + b 2 sin q + (a - c) 3
=
Y' a 2 + b 2 + c2 2 + 6
4. (d) The given lines are
y + 3 z -1 In order that cos a in independent of q
x -1 = = =s ..........(i)
-l l a + c = 0 and b = 0

z-2
and 2 x = y - 1 = =t .........(ii) 2a 3 3 p
-1 \ cos a = = Þa=
a 2 ×2 2 2 6
The lines are coplanar, if
6. (c) Given one vertex A(7, 2, 4) and line
0 - ( -1) -1 - 3 -2 - ( -1) x + 6 y + 10 z + 14
= =
1 -l l =0 5 3 8
1 General point on above line,
1 -1
2
B º (5l - 6, 3l - 10, 8l - 14)
Direction ratios of line AB are
1 -5 -1 < 5l - 13, 3l - 12, 8l - 18 >
C2 ® C2 + C3 ; 1 0 l =0 Direction ratios of line BC are <5, 3, 8>
1
0 -1 p
2 Since, angle between AB and BC is .
4

l
Þ 5( -1 - ) = 0 Þ l = -2 p (5l - 3)5 + 3(3l - 12) + 8(8l - 18)
2 cos =
4 (5l - 13) 2 + (3l - 12) 2
5. (a) Both the lines pass through origin. 52 + 32 + 8 2 ×
+(8l - 18)2
Line L1 is parallel to the vector
Squaring and solving, we have l = 3, 2.
r
V1 = (cos q + 3)ˆi + ( 2 sin q)ˆj + (cos q - 3)kˆ Hence, equation of lines are
and L2 is parallel to the vector x-7 y-2 z-4
r = = and
V2 = aˆi + bˆj + ckˆ 2 -3 6

x-7 y-2 z-4


r r = = .
V1 × V2 3 6 2
\ cos a = r r
| V1 || V2 |
196 MATHEMATICS

1 ˆ
7. (d) D= ( j + l kˆ) ´ (iˆ + lkˆ) z
2
C
1 1
= - kˆ + l iˆ + lˆj = 2l 2 + 1
2 2
P
9 1 33 y
Þ £ (2l 2 + 1) £ O B
4 4 4
A
Þ 4 £ l 2 £ 16 Þ 2 £| l |£ 4. x
8. (c) The line has +ve and equal direction
1 1 1 h k l
cosines, these are , , or direction x + y + z = p or hx + ky + lz = p 2
3 3 3 p p p
ratios are 1, 1, 1. Also the lines passes through P
æ p2 ö æ p2 ö æ p2 ö
(2, – 1, 2). B ç 0, , 0 ÷ C ç 0, 0, ÷
\ çç
A , 0, 0 ÷ , ç ÷ , ç
\ Equation of line is ÷ k l ÷ø
è h ø è ø è
x - 2 y +1 z - 2
= = = l (say) 2 2 2
1 1 1 Now, Area of DABC , D = Axy + Ayz + Azx

Let Q (l + 2, l - 1, l + 2) be a point on this line


2
where, Axy is area of projection of DABC on xy
where it meets the plane
2x + y + z = 9 plane = area of DAOB
Then Q must satisfy the eqn of plane
i.e. 2(l + 2) + l - 1 + l + 2 = 9 Þ l = 1 p2 / h 0 1
1 2 p4
\ Q has coordinates (3, 0, 3) Now, Axy = 0 p /k 1 =
2 2 | hk |
Hence the length of line segment 0 0 1

= (2 - 3)2 + (-1 - 0)2 + (2 - 3)2 = 3


p4 p4
9. (d) Here OP = h + k + l = p 2 2 2 Similarly, Ayz = and Azx =
2 | kl | 2 | lh |
\ DRs of OP are :
p5
h k l \ D 2 = Axy
2
+ A2yz + Azx
2 ,
D=
, , 2hkl
h2 + k 2 + l 2 h2 + k 2 + l2 h2 + k 2 + l2 10. (b) A plane containing the given lines is
2 x + 3 y + 4 z - 6 + l ( x + y + z - 3) = 0 … (i)
h k l
or , ,
p p p This plane is perpendicular to plane z = 0
if 4 + l = 0 Þ l = –4
Since OP is normal to the plane, therefore,
So, the equation (i) becomes
equation of plane is
–2 x - y + 6 = 0 Þ 2 x + y - 6 = 0 … (ii)
Solutions 197

Equation of the projection will be the line of where a.1 + b.2 + c.3 = 0
intersection of plane (2) and the plane z = 0. If i.e., a + 2b + 3c = 0 ....... (ii)
the line has d.c. proportional to l, m, n then Since the plane (i) parallel to the line
2l + m = 0 and n = 0 x y z
Þ l : m : n = 1 : –2 : 0. Obviously (0, 6, 0) is a = =
1 1 4
point on both the planes, hence lies on the line
\ a.1 + b.1 + c.4 = 0
as well.
i.e., a + b + 4c = 0 ....... (iii)
x y -6 z From (ii) and (iii),
\ Equation of the line is = =
1 -2 0
a b c
11. (d) Let the eqn of variable plane be = = = k (let)
8 - 3 3 - 4 1- 2
x y z \ a = 5k, b = –k, c = –k
+ + = 1 which meets the axes at
a b c On putting the value of a, b and c in equation (i),
A (a, 0, 0), B (0, b, 0) and C (0. 0, c). 5(x – 1) – (y – 2) – (z – 3) = 0
Þ 5x – y – z = 0 ....... (iv)
æ a b cö when x = 1, y = 0 and z = 5; then
\ Centroid of D ABC is ç , , ÷
è 3 3 3ø L.H.S. of equation (iv) = 5x – y – 2
and it satisfies the relation =5×1–0–5
=0
1 1 1 9
9 9
+ + =k Þ + 2 + 2 =k = R.H.S. of equation (iv)
2 2 2 2
x y z a b c Hence coordinates of the point (1, 0, 5) satisfy
the equation plane represented by equations (iv),
1 1 1 k Therefore the plane passes through the point
Þ
2
+ 2+ 2 = ...(i)
a b z 9 (1,0,5).
Also given that the distance of plane 13. (c) The position vectors of two given points

x y z are a = iˆ - ˆj + 3kˆ and b = 3iˆ + 3 ˆj + 3kˆ and


+ + = 1 from (0, 0, 0) is 1 unit.
a b c the equation of the given plane is

1 r = (5iˆ + 2 ˆj - 7kˆ) + 9 = 0 or r × n + d = 0
Þ =1
1 1 1 We have
+ +
a2 b2 c2 a × n + d = (iˆ - ˆj + 3kˆ) × (5iˆ + 2 ˆj - 7kˆ) + 9
= 5 – 2 – 21 + 9 < 0
1 1 1 and
Þ + 2 + 2 =1 ...(ii)
a2 b c
b × n + d = (3iˆ + 3 ˆj + 3kˆ) × (5iˆ + 2 ˆj - 7kˆ) + 9
k = 15 + 6 – 21 + 9 > 0
From (i) and (ii), we get = 1 i.e. k = 9
9 So, the points a and b are on the opposite
12. (b) Equation of the plane containing the line sides of the plane.

x -1 y - 2 z - 3 14. (c) 2 x 2 - 2 y 2 + 4 z 2 + 6 xz + 2 yz + 3xy = 0


= = is
1 2 3
or 2 x 2 + x(6 z + 3 y ) - 2 y 2 + 4 z 2 + 2 yz = 0
a (x – 1) + b (y – 2) + c (z – 3) = 0 ....... (i)
198 MATHEMATICS

s2 : ax + by + cz = 0
-(6 z + 3 y) ± 36 z 2 + 9 y 2 + 36 yz - 8( -2 y 2 + 4 z 2 + 2 yz )
\x = s3 : a2x + b2y + c2z = 0
4

1 1 1
-(6 z + 3 y) ± (2 x + 5 y)2
\x = a b c
4 D=
a2 b2 c2
-(6 z + 3 y ) ± (2 z + 5 y )
\x = So, for unique solution, D ¹ 0
4
Þ D = (a – b) (b – c) (c – a) ¹ 0
or 2 x - y + 2 z = 0, x + 2 y + 2 z = 0
Þ a ¹ b, b ¹ c, c ¹ a
\ Angle between planes rr
17. (a) The equation of plane is r. a = 5
(2)(1) + ( -1)(2) + (2)(2) r r r r
q = cos -1 Q r -b + r-c =4
(2) + ( -1) 2 + (2) 2 (1) 2 + (2) 2 + (2) 2
2 r
Þ sum of distances of a point (r ) from two
r r
fixed points with position vector b and c is
æ4ö
= cos -1 ç ÷ constant.
è9ø
Þ such points lies on ellipsoid.
15. (b) PR : PQ = 1: 3 Þ 3PR = PQ r r
Now points with position vector b and c
r r
1 2 satisfies the equation of plane r . a = 5, then
P R Q r r r r
b . a = 5 and c . a = 5
(–3, 1, 1) (3, 4, 2)

Þ 3PR = PR + RQ Þ 2 PR = RQ
Plane r × a = 5
Therefore, PR : RQ = 1 : 2. Hence B(b) C(c)

æ -6 + 3 2 + 4 2 + 2 ö æ 4ö
R =ç , , ÷ = ç -1, 2, ÷
è 1+ 2 3 3 ø è 3ø
The normal to the required plane is Area in the plane constitutes an ellipse
r r
PQ = (6, 3, 1). Hence, the equation of the Distance between b and c
required plane is = 2 × (semi major axis) × e = 14
æ 4ö
6( x + 1) + 3( y - 2) + 1ç z - ÷ = 0 2ae = 14 ...(i)
è 3ø
Sum of distance = constant = major axis = 4
Þ 18 x + 9 y + 3z - 4 = 0
2a = 4 ...(ii)
16. (c) s1 is perpendicular to (iˆ + ˆj + kˆ)
From eqn (i) and (ii)
s2 is perpendicular to (aiˆ + bjˆ + ckˆ)
and s3 is perpendicular to 14 1
e= Þ b = a (1 - e2 ) = (semi
(a 2 iˆ + b2 ˆj + c 2 kˆ) 4 2
Then, the planes are minor axis)
s1 : x + y + z = 0
Solutions 199

Area of ellipse = p.a.b. 20. (7)


21. (7) A plane containing line of intersection of
1
= p.2. = 2 p » 4.443 the given planes is
2
x - y - z - 4 + l ( x + y + 2 z - 4) = 0
x - 0 y +1 z - 0 i.e., (l + 1) x + (l - 1) y + (2l - 1) z - 4(l + 1) = 0
18. (3) L1 : = = =l
1 1 1 vector normal to it

L2 :
x +1 y - 0 z - 0
= = =m
V = (l + 1)iˆ + (l - 1) ˆj + (2l - 1) kˆ
2 1 1 Now the vector along the line of intersection
Hence any point on L1 and L2 can be of the planes
(l, l - 1, l) and (2m - 1, m, m) , respectively.. 2x + 3 y + z - 1 = 0
According to the question, and x + 3 y + 2 z - 2 = 0 is given by
2m - 1 - l m - l + 1 m - l
= = ˆj kˆ
2 1 2 iˆ
On solving, we get m = 1 and l = 3 n = 2 3 1 = 3(iˆ - ˆj + kˆ)
1 3 2
\ A = (3, 2, 3) B = (1, 1, 1) \ AB = 3.
19. (2) The equation of any plane passing through As n is parallel to the plane (i), therefore,
given line is :
n ×V = 0
( x + y + 2z - 3) + l(2x + 3y + 4z - 4) =
(l + 1) - (l - 1) + (2l - 1) = 0

Þ (1 + 2l)x + (1 + 3l) y + (2 + 4l)z - (3 + 4l) = 0 -1


Þ 2 + 2l - 1 = 0 Þ l =
2
....(i)
If this plane is parallel to z-axis whose direction Hence, the required plane is
cosines are 0, 0, 1; then the normal to the plane x 3y
- - 2z - 2 = 0
will be perpendicular to z-axis 2 2
\ (1 + 2l)(0) + (1 + 3l)(0) + (2 + 4l)(1) = 0 Þ x - 3 y - 4z - 4 = 0
1 Hence, | A + B + C - 4 |= 7
Þl=-
2 22. (7) Under the given conditions the possible
Put in eq (i), the required plane is situation is f(–2) = 2; f(0) = 3; f(1) = 1.

1 {where f(–2) = 1 is false, f (0) ¹ 2 is true and


(x + y + 2z - 3) - (2x + 3y + 4z - 4) = 0
2 f (1) ¹ 1 is false}. The triangle formed is with
Þ y+2 =0 ...(ii)
vertices
\ S.D. = distance of any point say (0, 0, 0) on A(–2, 1, 0), B(2, 1, 3) and O(0, 0, 0)

2
z-axis from plane (ii) = =2 iˆ ˆj kˆ
(1) 2 1
Area of DAOB = 2 1 3
2
-2 1 0
200 MATHEMATICS

1 1
= | -3iˆ - 6 ˆj + 4 kˆ |= 61 square units Þ [bc + bl, (c - 2at ) - cb - cbl ]2
2 2
k = k a (-bl )(ba + bl)(a - at 2 ) - (ab - abl )
= ; so k = 61.
2
23. (6) The required plane is of the form Þ [bc + blc - 2ablt - cb - cbl )2
( x + 2 y + 3z - 4) + l(2 x + y - z + 5) = 0
= -kbal(ba + abl - ablt 2 - ab - abl)
whose normal is (1 + 2l, 2 + l, 3 - l). This
Þ 4a 2 b2 l 2 t 2 = -kbal (-ablt 2 )
plane is perpendicular to the plane
5 x + 3 y + 6 z + 8 = 0. So we have Þ 4a 2 b 2 l 2 t 2 = k 2b 2 a 2 l 2 t 2 Þ k = 4.
5(1 + 2l) + 3(2 + l) + 6(3 - l) = 0 25. (4) | x |£ 8 Þ x Î [-8, 8] similarly for y and z.
This represent a cube of side 16 units with
-29
Þ 7 l = -29 Þ l = centre at origin.
7
Now, -8 £ x + y + z £ 8 gives space between
Therefore, the required plane is
two panes namely
29 Z X'
( x + 2 y + 3 z - 4) - (2 x + y - z + 5) = 0
7
Þ 51x + 15 y - 50 z + 173 = 0 (0,0,8)

Comparing this with ax + by + cz + 173 = 0 we


(0,–8,0) O (0,8,0)
get a = 51, b = 15, c = –50 Y' Y
so that, b - 9(a + c) = 15 - 9 = 6.
(0,0,–8)
24. (4) Any point on parabola z2 = 4ax, y = 0 is (8,0,0)
given by Q(at2, 0, 2at)
Now equation of line joining P(a, b, c) and Q
X Z'
(at2, 0, 2at) is given by x + y + z + 8 = 0 and x + y + z – 8 = 0 subject
x-a y -b z-c to the limits of x, y, z Î [-8, 8]
= = = l (say)
a - at 2 b - 0 c - 2at
æ1 1ö
Þ x = a + l(a - at 2 ); y = b + bl; z = c + l; This will take out ç + ÷ volume of the cube
è4 4ø
z = c + l(c - at ) by given condition point Q
1 3
lies on (bz - cy )2 = k a (b - y )(bx - ay ) Þ The required volume = ´ (16) = 2018.
2
Z
t
2 Þ t = 2048 Þ =4
(at ,0,2at) 512
Q B(0,b,0)
Y
,0) O
(a,0 A
L(a,b,0)
Solutions 201
CHAPTER
28 Probability-2

1. (c) Total number of functions from A to B is


1
(20)10. = ( pn - 2 + pn -1 )
2
The number of non-decreasing function is the
number of non-negative integral solutions of the Now
equation 1 1
p n + p n -1 = pn -1 + pn - 2
2 2
x1 + x2 +¼+ x20 = 10 = 29 C10
(Here xi is the pre-image of number i, i = 1, 2, 3, 1
= p n - 2 + p n -3
....,20) 2
29 1
C19 = p n -3 + p n -4
Þ the probability = 2
(20)10 .................................
2. (c) When two dice is thrown then sample .................................
space has 6 × 6 = 36 elements so n(S) = 36. 1
Now consider the event of getting 9 is (3, 6), = p 2 + p1
(4, 5), (5, 4) and (6, 3). 2
So probability of getting 9 when two dice is Since
thrown is 4/36 = 1/9. 1
If Sanchita starts the game then the probability p1 = P ( T ) =
2
that she wins is
and p 2 = P ( ( T Ç T ) È H )
æ 1 ö æ 8 öæ 8 öæ 1 ö
ç 9 ÷ + ç 9 ÷ç 9 ÷ç 9 ÷ +..... ¥ = = P ( T ) P (T ) + P ( H)
è ø è øè øè ø
1 1 1 1 3
=
´ + =
9 = 1 ´ 81 = 9 2 2 2 4
64 9 17 17 We have
1-
81 1 1 3 1
And if Raj starts the game then probability pn + pn -1 = p2 + p1 = + = 1
2 2 4 4
that Sanchita wins the game is 1– 9/17 = 8/17 Therefore
3. (d) As usual H denotes head and T denotes
tail so that 2 1 1
pn - = - p n -1
3 3 2
1
P (H) = = P (T) 2
2 1æ 2ö
= - ç p n -1 - ÷ = æç - ö÷ æç p n - 2 - ö÷
1 2
Let En denote the event that the score is n. One 2è 3ø è 2ø è 3ø
can easily see that
En = (En–2 Ç H) È (En–1 Ç T) æ 1ö æ
3

Therefore = ç - ÷ ç pn -3 - ÷ .....
Pn = P(En) è 2ø è 3ø
= P(En–2 Ç H) + P(En–1 Ç T) n -1
= P(En–2) P(H) + P(En–1)P(T) æ -1 ö æ 2ö
=ç ÷ ç p1 - 3 ÷
è 2ø è ø
202 MATHEMATICS

n -1 5 4 4 2 4 3 5 3 3 5 4 3
æ 1ö æ1 2ö = ´ ´ + ´ ´ + ´ ´ + ´ +
= ç- ÷ ç - ÷ 7 7 7 7 7 7 7 7 7 7 7 7
è 2ø è2 3ø
Hence 80 + 24 + 45 + 60 209
= =
n -1 n 7´ 7´ 7 343
2 æ 1ö æ 1 ö 2 1æ 1 ö
pn = +ç- ÷ ç- ÷ = + ç- ÷ 6. (d)
3 è 2ø è 6ø 3 3è 2ø
7. (b) The graph of y = 16 x 2 + 8(a + 5)
n +1 n
2 + ( -1) x - 7a - 5 is strictly above the x-axis
=
3.2n Þ y>0 " x Î R
4. (c) The number of determinants formed = 16.
Observe that the determinant is non-zero when Þ 16 x 2 + 8(a + 5) x - 7a - 5 > 0 " x Î R
exactly once (–1) appears as shown The above inequality holds if discriminant < 0
1 1 [Q coefficient of x 2 > 0 ]
= 2 Þ 4 ways
-1 1
Þ 64(a + 5)2 -4.16( -7 a - 5) < 0
Similarly the determinant is non-zero when (–1)
is used exactly three times as shown Þ a 2 + 17 a + 30 < 0
-1 -1 Þ ( a + 2 ) (a + 15) < 0
= -2 Þ 4 ways.
-1 1 Þ - 15 < a < - 2
So non-zero determinant can be obtained in 8
ways. Similarly determinant will be zero in 8 + +
determinants. S
–15 – –2
1
Þ P(E) = Given -20 £ a £ 0 and favourable cases
2
5. (b) Let E1, E2 and E3 be the events of the critics -15 < a < -2
giving favourable remarks. Then \ Required probability
5 4 3 length of interval (-15, - 2) - 2 - (-15) 13
P ( E1 ) = , P ( E 2 ) = and P ( E 3 ) = = = =
7 7 7 length of interval (-20, 0) 0 - (-20) 20
Let E be the event that majority reviewed 8. (a) Let G, C, K, and A are events where
favourably. Therefore G = Guess answer, C = Copy answer, K = Know
E = ( E1 Ç E 2 Ç E3 ) È ( E1 Ç E 2 Ç E3 ) the answer, and A = Answer correctly
P(G) = Probability that the Candidate Guess the
È ( E1 Ç E 2 Ç E 3 ) È ( E1 Ç E 2 Ç E3 ) answer = 1/3 (given)
P(C) = Probability that the Candidate Copy the
Hence P(E) answer = 1/6 (given)
= P ( E1 ) P ( E 2 ) P ( E 3 ) + P ( E1 ) P ( E 2 ) P ( E 3 ) P(K) = Probability that the Candidate Know the
answer = ?
+ P ( E1 ) P ( E 2 ) P ( E 3 ) + P ( E1 ) P ( E 2 ) P ( E 3 ) Now, G,C and K are mutually exclusive and
exhaustive events
5 4 æ 3ö æ 5ö 4 3 5 Therefore, P(G) + P(C) + P(K) = 1
= ´ ´ ç1 - ÷ + ç1 - ÷ ´ ´ +
7 7 è 7ø è 7ø 7 7 7 P(K) = 1 – 1/3 – 1/6 = 1/2.
Say G(Candidate guesses) has occurred. As,
æ 4ö 3 5 4 3 there are four choices out of which only one is
´ ç1 - ÷ ´ + ´ ´
è 7ø 7 7 7 7 correct, then the probability that the candidate
made a guess is 1/4.
Solutions 203
P(A/G) = 1/4 11. (c) Let E1 and E2 be the events of the boy
P( A/C) = 1/8 ( given) then watching DOORDARSHAN and TEN SPORTS,
P( A/K) = Answer correctly that the candidate respectively. It is given that
know = 1
1 4
Applying Baye’s Theorem we have, P ( E1 ) = and P ( E 2 ) =
P(K/A) = [ P(K).P(A/K)]/[P(G) × P(A/G) + P(C) × 5 5
P(A/C) + P(K) × P(A/K) = 24/29. Let E be the event of the boy falls asleep. Again
9. (b) Let the sides be x, y, l – (x + y) by hypothesis
Since the triangle will be formed when sum of 3 1
two sides is larger than the third. P ( E / E1 ) = and P ( E / E 2 ) =
4 4
Y Now,
B E = E Ç ( E1 È E2 ) = ( E1 Ç E ) È ( E 2 Ç E )
y so that,
P ( E ) = P ( E1 ) P ( E / E1 ) + P ( E 2 ) P ( E / E 2 )
Q P By Bayes' theorem
l /2
l P ( E1 ) P ( E / E1 )
P ( E1 / E ) =
P ( E1 ) P ( E / E1 ) + P ( E 2 ) P ( E / E 2 )

R A (1/ 5 ) ´ ( 3/ 4 ) 3
X = =
O l /2 l (1/ 5) ´ ( 3 / 4 ) + ( 4 / 5 ) ´ (1/ 4 ) 7
i.e. l – y > y, l – x > x and x + y > l – (x + y) 12. (c) Here, Pn = apn, n > 1
and P0 = 1 – ap (1 + p + p2 + ...)
l l l
Þ 0< y< , 0 < x < and < (x + y) < l Consider the following events :
2 2 2 Ej = There are j children in the family;
DPQR 1 j = 0, 1, 2,....., n
Hence required probability = = A = There are exactly k boys in the family
DOAB 4
We have, P(Ej) = pj = apj; j = 0, 1, 2, ...., n
(as OQ = 1/2 OB)
10. (b) Let x and y be the two quantities. When the æ A ö jC
sum of two non-negative quantities is fixed, the Þ Pç ÷ = k , j ³ k
ç Ej ÷ 2j
product will be maximum when they are equal. è ø
So, the greatest product = xy = 10000
¥
U (A Ç E j )
where x = y = 100
Now, A =
3 j= k
Now, xy ³ ´10000
4
æ ¥ ö
Þ xy ³ 7500 Þ x ( 200 - x ) ³ 7500 Þ P (A) = P ç U A Ç E j
ç j= k
( ) ÷÷
è ø
Þ x 2 - 200x + 7500 £ 0
Þ ( x - 50 )( x - 150 ) £ 0 Þ 50 £ x £ 150 ¥ ¥ æAö
So, favourable number of ways = 101
\ P (A) = å P ( A Ç E j ) = å P ( E j ) P çç E ÷
÷
j= k j= k è jø
Total number of ways = 201
101 ¥ æ j Ck ö ¥ j
æpö j
So, required probability =
201
= å ç j ÷ å çè 2 ÷ø . Ck
ap j
ç ÷ = a
j= k è 2 ø j= k
Hence, (b) is the correct answer.
204 MATHEMATICS

¥ k+r published) = P(X = 1) + P(X = 2)


æpö
=aå k +r
Cr ç ÷ 2 0
è2ø æ 11 öæ 13 ö æ 11 ö æ 13 ö
j= k = 2C1 ç ÷ç ÷ + 2C2 ç ÷ ç ÷
è 24 øè 24 ø è 24 ø è 24 ø
k ¥ r
æpö æpö
P (A) = a ç ÷
è2ø
å k +r
Cr ç ÷
è2ø = 2´
11 13 æ 11 ö
´ +ç ÷ =
407
2
r=0 24 24 è 24 ø 576
We have, k+rCr = (–1)r–(k+1)Cr
1
æpö
¥ k
æ pö
r 15. (b) Here, p = ,n=8
P ( A ) = a ç ÷ å ( ) Cr ç - ÷
- k +1 2
\
è 2 ø r=0 è 2ø
1 1
\ q = 1 – p = 1- =
k -( k +1) k k +1 2 2
æpö æ pö æpö 2
= a ç ÷ ç1 - ÷ = aç ÷ . 8
è2ø è 2ø è 2 ø ( 2 - p )k +1 æ1 1ö
\ The binomial distribution is ç + ÷
13. (d) The probability of showing an even number è2 2ø
in a throw Also, |x – 4| < 2 Þ –2 < x – 4 < 2 Þ 2 < x < 6
3 1 \ p(|x – 4| < 2) = p(x = 2) + p(x = 3)
= = + p(x = 4) + p(x = 5) + p(x = 6)
6 2
2 6 3 5 4 4
\ Required probability æ1ö æ1ö æ1ö æ1ö æ1ö æ1ö
= 8C2 ç ÷ ç ÷ + 8C3 ç ÷ ç ÷ + 8C4 ç ÷ ç ÷
1 æ1ö
2n
æ1ö
3
æ1ö
2n - 2 è2ø è2ø è 2ø è 2ø è 2ø è 2ø
= 2n +1C1. . ç ÷ + 2n +1
C3. ç ÷ .ç ÷
2 è2ø 5 3 6 2
è2ø è2ø æ1ö æ1ö æ1ö æ1ö
+ 8C5 ç ÷ ç ÷ + 8C6 ç ÷ ç ÷
æ1ö
2n +1 è2ø è2ø è2ø è 2ø
2n +1
+.... + C2n +1 ç ÷
8
è2ø C2 + 8C3 + 8C4 + 8C5 + 8C6
=
2n +1 28
= ( 2n +1
C1 + 2n +1
C3 + ..... + 2n +1 æ 1ö
C2n +1 ç ÷
è 2ø ) 238 119
= =
2n +1
256 128
æ1ö 1 16. (a) If both have all 4 cards of the same color,
= 22n. ç ÷ =
è2ø 2 then there are two possibilities at the end.
14. (a) Let G = Event of good book Posibility 1: Ravi holds 4 red cards and
G' = Event of not a good book Rashmi 4 black cards.
E = Event of publication Probability of this possibility = P(Ravi
Then picks red in 1st pick 'AND' Rashmi picks
E = ( G È G ') Ç E = ( G Ç E ) È ( G 'Ç E ) black in 1st pick 'AND' Ravi picks red in 2nd
pick' AND 'Rashmi pick black in 2nd pick')
2 1
Now, P ( E / G ) = ; P ( E / G ') = All 4 picks are independent
3 4
2 2 1 1 1
1 = ´ ´ ´ =
P ( G ) = = P (G ') 4 5 4 5 100
2 Posibility 2: Ravi holds 4 black cards and
1 2 1 1 11 Rashmi 4 red cards similarly probability
Therefore, P ( E ) =
´ + ´ =
2 3 2 4 24 1
=
Further, X denotes the number of books 100
published. Then P (at least one book will be
Solutions 205
Hence, required probability [A wins his serve then B wins his serve or A
loses his serve then B also loses his serve]
1 1 2
P= + = So, probability that ‘A’ wins the game after n
100 100 100 deuces
1 n
=
= 0.02 = 2% æ 5ö 2 1
50 = ç ÷ . . [After nth deuce, A serves and
è 9ø 3 3
17. (a) We have 30 males and 20 females.
wins then B serves and loses]
30 3
P(males) = = \ Required probability of ‘A’ Winning the
50 5 game
20 2 ¥ n
P(females) = = æ 5ö 2 1 1 2 1
50 5 = å çè 9 ÷ø . 3 . 3 = . =
5 9 2 = 0.50
n= 0 1-
9
19. (0.25) Let l be the length of the chord AB of
the given circle of radius a and r be the distance
of the mid point D of the chord from the centre C,
3/5 2/5
then r = a cos q and l = 2a sin q . According to
given condition :
Male Female
1/2 1/2 4/5 1/5

Disease No disease Disease No disease


+ve not +ve not +ve not +ve not
C
+ve +ve +ve +ve a
4/5 1/5 1/5 4/5 4/5 1/5 1/5 4/5
(1) (2) (3) (4) (5) (6) (7) (8) q
r
A B
(1) + ( 3)
Required probability =
(1) + ( 3) + ( 5) + ( 7 ) D

3 1 4 3 1 1 2 5
´ ´ + ´ ´ (2a) < 2a sin q < (2a )
= 5 2 5 5 2 5 3 6
3 1 4 3 1 1 2 1 4 2 4 1
´ ´ + ´ ´ + ´ ´ + ´ ´ 2 5
5 2 5 5 2 5 5 5 5 5 5 5 11 5
Þ < sin q < < cos q <
3 6 Þ 6 3
15
11 5
50 = 75 Þ a <r < a
= 15 16 107 6 3
+
50 125 \ The given condition is satisfied if the mid
point of the chord lies within the region
18. (0.50) Let us assume that A wins after n
deuces, n = 0, 1, 2, 3 ........... 11
between the concentric circles of radii a
2 2 1 1 5 6
Probability of a deuce = . + . =
3 3 3 3 9 5
and a.
3
206 MATHEMATICS
Hence, the required probability
dP( A)
2 2
For maximum probability; =0
æ 5 ö æ 11 ö dx
pç a÷ - p ç a÷
21- x x-3
è 3 ø è 6 ø
=
1 1 é æ 1ö æ 1ö ù
= = 0.25 Þ- êç ÷ ln 2 - ç ÷ ln2 ú = 0
pa2 4 9 ê è 2ø è 2ø ú
ë û
20. (0.60) Let A denotes the event that the
runner succeeds exactly 3 times out of five and Þ x = 12
B denotes the event that the runner succeeds 2
on the first trial. Also, d P ( A)
dx 2
æ B ö P ( B Ç A)
Pç ÷ = x -3 21- x
èAø P ( A) 1 é æ 1ö æ 1ö ù
=- êç ÷ (ln2)2 + ç ÷ (ln2)2 ú < 0
9 ê è 2ø è 2ø ú
But P ( B Ç A ) = P (clearing succeeding in the ë û
first trial and exactly once in two other trials) \ P(A) is maximum when x = 12 Þ k = 6
P (4C2 p2 (1 – p2)) = 6p2 (1 – p)2 22. (2) Clearly all the solutions of f(x) = x are also
and P(A) = 5C3 p3 (1 – p)2 = 10p3 (1 – p)2 solutions of f(f(x)) = x. First, we solve f(x) = x
f(x) = x Þ x2 – 3x + 3 = x
2
æ B ö 6p (1 - p )
2
3 Þ x2 – 4x + 3 = 0
Thus, P ç ÷ = = = 0.60 Þ (x – 1) (x – 3) = 0
è A ø 10p3 (1 - p )2 5
Þ x = 1, 3
21. (6) Let x shell are fixed at point I. Define the Therefore x = 1, 3 are also solutions of f(f(x)) = x.
following events We want to seek if there are any more solutions
8 of f(f(x)) = x other than 1 and 3
E1 : The target is at point I Þ P(E1 ) = f(f(x)) = x Þ f(x2 – 3x + 3) = x
9 Þ (x2 – 3x + 3)2 – 3(x2 – 3x + 3) + 3 = 0
1 Þ x4 – 6x3 + 12x2 – 9x + 3 = 0
E 2 : The target is at point II Þ P ( E2 ) = Þ (x2 – 4x + 3)(x2 – 2x + 1) = 0
9
Þ (x – 1) (x – 3) (x – 1)2 = 0 Þ x = 1, 3
A : The target is hit
In this case we have no additional solutions.
The target will be hit if at least one shell hits the
Therefore the probability that x satisfies equation
target.
P ( A / E1 ) = 1 – None of the shells hit when the 2
f(f(x)) = x is . Therefore m = 2.
9
x
æ 1ö 23. (0) Let E1, E2, E3, E4, E5 and E6 be the events
target is at point I = 1 - ç ÷ and of occurrence of 1, 2, 3, 4, 5 and 6 on the dice
è 2ø
respectively and let E be the event of getting a
21- x sum of numbers equal to 9.
æ 1ö
P ( A / E2 ) = 1 - ç ÷ 1- k 1 + 2k 1- k
è 2ø \ P ( E1 ) = ; P (E2 ) = ; P ( E3 ) =
6 6 6
8 é æ 1 ö x ù 1 é æ 1 ö 21- x ù
\ P (A) = ê 1- ç ÷ ú + ê 1 - ú 1+ k 1 - 2k 1+ k
9 ëê è 2ø ûú 9 êë çè 2 ÷ø ûú
P ( E4 ) = ; P ( E5 ) = ; P ( E6 ) =
6 6 6
x -3 21- x
1 é æ 1ö æ 1ö ù
and
1
£ P (E) £
2
= 1- ê ç ÷ +ç ÷ ú 9 9
9 êë è 2ø è 2ø úû
Solutions 207
Then, E º {(3, 6), (6, 3), (4, 5), (5, 4)} 6 æ 1ö æ 1 öæ 1 öæ 1 öæ 1 öæ 1 ö
Hence, P(E) = P(E3E6) + P(E6E3) + P(E4E5) + Õ çç1 - j2 ÷÷ = çè1 - 22 ÷ç 1 - ÷ç1 - ÷ç1 - ÷ç1 - ÷
øè 23 øè 2 4 øè 25 øè 26 ø
j = 2è ø
P(E5E4)
= P(E3) P(E6) + P(E6) P(E3) + P(E4) P(E5) + P(E5) éæ 1 ö æ 1 ö æ 1 ö æ 1 ö æ 1 ö ù
= êç 1 - ÷ ç 1 - ÷ ç 1 - ÷ ç 1 - ÷ ç 1 - ÷ ú
P(E4) ëè 2 ø è 3 ø è 4 ø è 5 ø è 6 ø û
= 2P(E3) P(E6) + 2P(E4) P(E5) éæ 1 öæ 1 ö æ 1 ö æ 1 ö æ 1 ö ù
´ êç1 + ÷ç1 + ÷ ç1 + ÷ ç1 + ÷ ç1 + ÷ ú
[since E1, E2, E3, E4, E5 and E6 are independent] ëè 2 øè 3 ø è 4 ø è 5 ø è 6 ø û

æ 1 - k öæ 1 + k ö æ 1 + k öæ 1 - 2k ö æ 1 2 3 4 5 öæ 3 4 5 6 7 ö
= 2ç ÷ç ÷ + 2ç ÷ç ÷ = ç ´ ´ ´ ´ ÷ç ´ ´ ´ ´ ÷
è 6 øè 6 ø è 6 øè 6 ø è 2 3 4 5 6 øè 2 3 4 5 6 ø

1 1 7 7 p
= [2 - k - 3k 2 ] = ´ = =
18 6 2 12 q
Therefore, q – p = 12 – 7 = 5
1 2
Since, £ P (E) £ 25. (3) Let S be the sample space, then
9 9 n(S) = Total number of determinants that can be
1 1 é 2 made with 0 and 1 = 2 × 2 × 2 × 2 = 16
Þ £ 2 - k - 3k 2 ù £
9 18 ë û 9 a b
Q , each element can be replaced by two
Þ c d
2 £ 2 - k - 3k 2 £ 4
2 types
Þ 2 £ 2 - k - 3k 2 and 2 - k - 3k £ 4 i.e., 0 and 1
æ 1ö and let E be the event that the determinant made
Þ 3k ç k + ÷ £ 0 and 3k2 + k + 2 > 0 is non-negative.
è 3ø Also, E' be the event that the determinant is
1 negative.
Þ - £ k £ 0 and k Î R
3 ïì 1 1 0 1 0 1 ïü
\ E' = í , , ý
1
- £k £0 îï 1 0 1 1 1 0 ïþ
\
3 \ P(E') = 3
Hence, integral value of k is 0.
n ( E ') 3
24. (5) The sportsman's chance of missing when then P(E') = n S = 16
r = ja is ( )
Hence, the required probability,
a2 1
1- =1 - ( j = 2,3, 4,5, 6 ) 3 13 m
j2 a 2 j 2
P(E) = 1 – P(E') = 1 - = = [given]
The animal escapes when the sportsman misses 16 16 n
in all the five shots. Therefore the probability of Þ m = 13 and n = 16, then n – m = 3
animal escaping to jungle is
208 MATHEMATICS

CHAPTER
29 Properties of Triangle

1. (c) Sum of the roots of the equation is given by


= (c1 + c2 )2 - 2c1c2 (1 + cos 2 A)
c(a + b) a+b
sin A + sin B = = = (2b cos A) 2 - 2(b2 - a 2 )(2cos 2 A)
c2 c
sin A + sin B = 4b 2 cos 2 A - 4b 2 cos 2 A + 4 a 2 cos 2 A
= sin C = 1 = 4a2 cos2A.
sin C
5. (b) a, b, c are sides of a triangle
Þ the triangle is right angled.
2. (c) If D is the diameter of the circumscribed \ a + b > c, b + c > a, c + a > b
circle of D ABC, then a = D sin A, b = D sin B, \ a > | b – c |, b > | c – a |, c > |a – b | square and
c = D sin C add

a 3 + b3 + c 3
=7Þ
D3 ( å sin A) = 7
3 a2 + b2 + c2 < 2 (ab + bc + ca)
Þ a2 + b2 + c2 + 2 (ab + bc + ac)
\
å sin3 A å sin3 A < 4 (ab + bc + ca)
\ D=37. (a + b + c )2
Since no side of triangle can exceed the diameter Þ <4ÞP<4
of the circle, the maximum possible value of a ab + bc + ca
is 3 7 . Again (a - b)2 + (b - c )2 + (c - a) 2 ³ 0
3. (b) Let the altitudes from A, B, C be p, q, r
respectively. Then, (a + b + c )2
Þ ³3Þ P³3
p = b sin C, q = c sin A, r = a sin B ab + bc + ca
\ p : q : r = b sin C : c sin A : a sin B \ 3 £ P < 4 or P Î[3, 4)
6. (a) We have,
A
b+c
c b ³ bc = l 2 Þ b + c ³ 2l
2
p a b c b+c
Now, = = =
B a C sin A sin B sin C sin B + sin C
= sin B sin C : sin C sin A : sin A sin B
1 1 1 a b+c
= : : Þ =
A A B+C B-C
sin A sin B sin C 2 sin cos 2 sin cos
\ sin A, sin B, sin C are in A.P. 2 2 2 2

b2 + c2 - a 2 A
4. (d) cos A = (b + c )sin
2bc Þa= 2
æ B -Cö
Þ c 2 - (2b cos A)c + b2 - a 2 = 0 . cos ç
è 2 ÷ø
It is a quadratic in c, whose roots are c1 and c2,
B-C
so c1 + c2 = 2b cos A and c1c2 = b2 - a 2 Q 0 £ cos £ 1 and
2
\ c12 + c22 - 2c1c2 cos 2 A A
b + c ³ 2l Þ a ³ 2l sin .
2
Solutions 209

1 2 Clearly a and b < 1 but c > 1 as sin a > 0 and


7. (c) L.H.S. = (a (b + c – a) + b2 (c + a – b) + c2 cos a > 0
2
(a + b – c) \ c is the greatest side and greatest angle is C
1 a2 + b 2 - c 2
= (a (b2 + c2 – a2) + b (c2 + a2 – b2) \ cos C =
2 2ab
+ c (a2 + b2 – c2))
sin 2 a + cos 2 a - 1 - sin a cos a 1
1 = =-
= (2abc cos A + 2abc cos B + 2abc cos C) 2 sin a cos a 2
2
\ C = 120°
æ A B Cö
= abc ç1 + 4 sin sin sin ÷ 11. (d) sin2 A + sin2 C = 1001 sin2 B
è 2 2 2ø
Þ a2 + c2 = 1001 b2 (using sine rule)
æ A B Cö
= 4RD ç1 + 4 sin sin sin ÷ .
è 2 2 2ø 2 (tan A + tan C) × tan 2 B
Now,
8. (b) a tan q + b sec q = c tan A + tan B + tan C

Þ b2 sec 2 q = (c - a tan q)2 2 (tan A + tan C) × tan 2 B


=
tan A × tan B × tan C
Þ b 2 (1 + tan 2 q) = c 2 - 2ca tan q + a 2 tan 2 q
æ cot A + cot C ö
Þ (a 2 - b2 ) tan 2 q - 2ca tan q + c 2 - b2 = 0 ...(i) = 2ç ÷ø
è cot B
Roots of equation (i) are tan a and tan b , where
2 (cos A sin C + sin A cos C)
a and b are the two angles of the triangle. = sin B
sin A × sin C × cos B
2ca
We have tan a + tan b = 2 and
a - b2 2 sin (p - B) × sin B 2 sin 2 B
= =
c 2 - b2 sin A sin C cos B sin A sin C cos B
tan a. tan b =
a 2 - b2 2 ´ 2b2 2 ´ 2b 2 2 ´ 2b 2 1
2ca = = 2 2 2
= 2
=
2ac × cos B a + c - b 1000b 250
\ tan(a + b) = a 2 - b 2 = 2ca
sin A sin B sin C c b a
c2 - b2 a 2 - c2 12. (b) + + = + +
1- 2 c sin B c b ab ac bc
a - b2
a sin B sin C c b a
3p + + + +
\ tan æç p - ö÷ =
2ca or =
Þ a 2 - c 2 = 2ca bc c b ab ac bc
è 4 ø a 2 - c2
sin B sin C c b
9. (b) We know that A + B + C = 180° or + = +
Þ A + C – B = 180° – 2B c b ab ac

é1 ù b sin B + c sin C c 2 + b2
Now 2ac sin ê ( A - B + C ) ú or =
ë2 û bc abc
= 2ac sin (90° - B ) b2 + c2
or a=
= 2ac cos b sin B + c sin C

2ac (a 2 + c 2 - b2 ) b (2R sin B) + c (2R sin C)


B= = a 2 + c2 - b2 = =2R
2ac b sin B + c sin C
10. (c) Let a = sin a, b = cos a and p
Þ ÐA =
c = 1 + sin a cos a 2
210 MATHEMATICS
13. (a) A a 2 + b 2 - c2 1
cos C = = Þ c= 6
2ab 8
4p D p 2p 4p
b 16. (a) ÐA = , ÐB = , ÐC =
7 7 7
p
(a2 – b2) (b2 – c2) (c2 – a2)
B C æ
a b2 öæ c2 öæ a2 ö
= a2b2c2 çç 1 - 2 ÷ç 1 - 2 ÷ç 1 - 2 ÷
1 1 è a ÷ç
øè b ÷ç
øè c ÷ø
D= ab = p (4p) Þ ab = 4p2
2 2 = a2b2c2
Also, a2 + b2 = c2 = 16p2
\ (a – b)2 = a2 + b2 – 2ab = 8p2 æ 2p öæ 4p öæ p ö
Also, (a + b)2 = a2 + b2 + 2ab = 24p2 ç sin 2 ÷ç sin 2 ÷ç sin 2 ÷
7 1- 7 1- 7
ç1 - ÷ç ÷ç ÷
A-B a-b C 1 ç 2 p ÷ç 2 2p ÷ç 2 4p ÷
tan = cot = ´1 sin sin sin
2 a+b 2 3 è 7 øè 7 øè 7 ø

A-B = a2b2c2
or = 30° or A – B = 60° 17. (a) Since 4 sin A cos B = 1, so A and B can not
2
BD AD p
14. (a) In DBAD, = be
sin q sin 60° 2

CD AD p p
In DCAD, = [As if B = , then cos B = 0 and if A = , then
sin (75° - q) sin 45° 2 2
tan A is not defined]
BD CD sin 45°
Þ sin 60° = p p
sin q sin (75° - q) so, C = ÞB= -A
2 2
sin q BD sin 60° 1
Þ = =
sin (75° - q) CD sin 45° 6 æp ö
Þ 4sin A cos ç - A ÷ = 1
A è2 ø

q
1 1 p p
75°–q Þ sin 2 A = Þ sin A = Þ A = Þ B =
4 2 6 3
p p p
so angles are , , which are in A.P..
60° 60°+ q 45° 6 3 2
B C
1 D 3 b 2(b / a) p+b
18. (b) tan a = , tan 2a = =
A - Bö 2æ a 1 - (b / a) 2
a
1 - tan ç ÷
è 2 ø = 31
15. (c) cos (A – B) = 2ba p+b P
æ A - B ö 32 Þ =
1 + tan 2 ç ÷ 2
a -b 2 a
è 2 ø p
2ba 2 - a 2 b + b3
æA -Bö 1 Þ =p
Þ tan ç ÷ = a2 - b2
è 2 ø 3 7
a b
æA -Bö a -b C 1 b( a 2 + b 2 ) a
tan ç ÷= cot Þ cos C = Þ p=
è 2 ø a+b 2 8 (a 2 - b 2 ) O a
Solutions 211
19. (c) d = h cot 30° – h cot 60° and time = 3 min. 21. (4)
A
h(cot 30° - cot 60°)
\ Speed = per minute
3 x y z

c b

60° 30°
B C
d a/3 D a/3 E a/3

It will travel distance h cot 60° in a AD


= ...(1)
h cot 60° ´ 3 3sin x sin B
= 1.5minute 2a AE
h(cot 30° - cot 60°) =
20. (d) Let x and y be the heights of the flagstaffs 3sin(x + y) sin B
at P and Q respectively Dividing Eq. (1) by Eq. (2), we get ...(2)
Then, sin(x + y) AD
= ...(3)
x 2sin x AE
AP = x cot 60° = , AQ = y cot 30° = y 3
3 2a AD
= ...(4)
3sin (y + z) sin C
y
BP = x cot 45° = x, BQ = y cot 60° = a AE
3 =
3sin z sin C
x
Þ AB = BP - AP = x - [Q AB = 30 m] Dividing Eq. (5) by Eq. (4), we get ...(5)
3 sin(y + z) AE
=
Þ 30 3 = ( 3 - 1) x Þ x = 15(3 + 3) 2sin z AD
Multiplying Eq. (3) and Eq. (6), we get ...(6)
S sin (x + y) sin(y + z)
\ =4
sin x × sin z
R 22. (3) The given equation is
4 sin A sin B + 4 sin B sin C + 4 sin C sin A = 9
Þ 2 cos (A – B) – 2 cos (A + B) + 2 cos (B – C)
x –2 cos (B + C) + 2 cos (C – A) – 2 cos (C + A) = 9
y or 2[cos(A – B) + cos(B – C) + cos(C – A)]
3
= 9 – 2 (cos A + cos B + cos C) ³ 9 – 2 × = 6
60° 45° 2
30° 60°
or cos (A – B) + cos (B – C) + cos (C – A) ³ 3
Q B 30 m A P But cos (A – B) £ 1, cos (B – C) £ 1,
cos (C – A) £ 1
æ 1 ö or cos (A – B) = 1, cos (B – C) = 1,
Similarly, 30 = y ç 3 - ÷ Þ y = 15 3 cos (C – A) = 1
è 3ø
Thus, A = B = C, i.e., triangle ABC is an
y equilateral triangle.
so that PQ = BP + BQ = x +
3 Hence, D = 3.

= 15(3 + 3) + 15 = (60 + 15 3)m


212 MATHEMATICS
23. (2)
A
B a(b + c) sec × ID
ID ID 2
x = =
AD 2bc cos A 2abc
P
x b+c 2
Q 3 c
Now AI = AB = c = b + c
x 4 ID BD ac a
b+c
C b A
Let BP = PQ = QC = x ID ID a
\ = =
In DABP, using cosine rule AD AI + ID a + b + c
9 = c2 + x2 – 2cx cos B ID IE IF a + b + c
\ + + = =1
c AD BE CF a + b + c
But cos B =
3x
A B
c2 \ a(b + c) sec ID + b(a + c) sec IE
Þ 2
9= x + ...(1) 2 2
3 C
Similarly using cosine rule is DACQ, we get + c(a + b) sec IF = 2abc
2
b2 \ k= 2
16 = x2 + ...(2)
3 25. (7)
b 2 + c2 B
Adding (1) and (2), we get 25 = 2x2 +
3
(3x)2
\ 25 = 2x2 + \ 25 = 2x2 + 3x2 100 m
3
\ x2 = 5 \ BC = 3x = 3 5
A 30° 60°
A d D x C
100 100
In DCBD, tan 60° = Þ x=
24. (2) x 3
F E
I 100
In DACB, tan30° =
x+d
x + d = 100 3 .
B D C 100 200 200 3
Þ d = 100 3 - = = m
3 3 3
@unacademyplusdiscounts_link

https://telegram.me/Unacademyplusdiscounts_link

https://telegram.me/Unacademyplusdiscounts_link

https://telegram.me/Unacademyplusdiscounts_link

Join Us Now For all study Materials for free

You might also like